Download as pdf or txt
Download as pdf or txt
You are on page 1of 494

Tests.

com Format Comparison Chart


PDF
Features & Benefits Online Format
Format

New Questions / Updates YES. With the Online Format, get new question & updates in real time. NO
YES. Optionally test with instant prompts for incorrect answers &
Study Mode NO
explanations of correct ones.
Section / Topic Selection YES. Choose whole test or practice by the section. NO

YES. Optionally simulate actual test conditions. TestSIM™ is easy and flexible
NO
to use.

YES. With TestNOTES™, you have option to type notes for any question for
NO
instant reinforcement & later review.

YES. Our Focus Flash Cards™ online study system allows you to customize
NO
study by time, section(s) and more. Create/share your own too.

Focus Question Format YES. Focus on 1 question at a time. Example. NO


Customize Testing YES. Set the number of questions and time limit OR simulate your actual
NO
Preferences exam. Example.
Instant Scoring & Review YES. The Online format automatically scores your testing. NO
Reports / History / Stats YES. View results and track progress in real time. Example. NO
YES. Only the online test system format will provide you access your tests
Future Testing Needs NO
and test history from anywhere for life.
YES. Our practice exam kit was written by an expert with strong experience,
Expert Written by Leaders
education and credentials. Our test prep is so good we have a 1% return rate YES
in Test Preparation
compared to an 8% industry avg. 1 million+ tests served.
YES. Our materials are a personal assessment tool and study source
Get an Edge YES
which compliments other study materials and may give you an edge.
YES. Registered Users get service & support to pass the first time. Our
Support to Succeed YES
authors, peer reviewers, editors and tech support are on your team.
Convenience & YES. Only the Online Format system scores, reports and customizes your
NO
Study Features testing automatically and has bonus flash cards to use, create and share.

Login for the Online Format


Or Access Free Sample Questions Online

Copyright © 2015 - 2023 Tests.com LLC


_________________________________________

Practice Test Kit for the NBDHE


Dental Hygienist Exam
__________________________________________

Published by Tests.com LLC


PO Box 232
Lititz, PA 17543
www.Tests.com

ISBN: 978-1-938967-17-7

Copyright © 2013-2023 Tests.com LLC


All rights reserved. No part of this publication may be reproduced, distributed or transmitted in any
form or by any means without the prior written permission of Tests.com LLC. Tests.com’s copyright
does not extend to the illustrations. Published in electronic format in the United States of America.

About the Author

Sharon Boyd, RDH, is a registered dental hygienist and dental educator who has worked in dentistry for
over ten years. She has worked in general, pediatric and cosmetic dentistry settings. As a professional
dental writer, Sharon provides educational material to several continuing education programs and is the
head continuing education curriculum writer for an American Dental Association CERP provider. Sharon
graduated with honors from the Tyler Junior College Dental Hygiene Program in Tyler, Texas in 2001
with an A.A.S. degree.

Abby Altergott, RDH, has been a registered dental hygienist for over eight years. She has worked in the
specialties of oral surgery, periodontics and orthodontics. She received her associate’s degree in dental
hygiene from the Milwaukee Technical College.

Copyright © 2015 - 2023 Tests.com LLC


Table of Contents
Review of the NBDHE Dental Hygienist Exam ............................................................................................. 1

Practice Exam Questions

Section 1 – Scientific Basis for Dental Hygiene Practice .................................................................. 7


Section 2 – Provision of Clinical Dental Hygiene Practice .............................................................. 32
Section 3 – Community Dental Health Testlet ............................................................................... 97
Section 4 – Case Testlets

Case 1 .............................................................................................................................. 110


Case 2 .............................................................................................................................. 113
Case 3 .............................................................................................................................. 116
Case 4 .............................................................................................................................. 119
Case 5 .............................................................................................................................. 122
Case 6 .............................................................................................................................. 125
Case 7 .............................................................................................................................. 128
Case 8 .............................................................................................................................. 131
Case 9 .............................................................................................................................. 134
Case 10 ............................................................................................................................ 137
Case 11 ............................................................................................................................ 140
Case 12 ............................................................................................................................ 143
Case 13 ............................................................................................................................ 146
Case 14 ............................................................................................................................ 149
Case 15 ............................................................................................................................ 152
Case 16 ............................................................................................................................ 155
Case 17 ............................................................................................................................ 158
Case 18 ............................................................................................................................ 161
Case 19 ............................................................................................................................ 165
Case 20 ............................................................................................................................ 169
Case 21 ............................................................................................................................ 172
Case 22 ............................................................................................................................ 174
Case 23 ............................................................................................................................ 176
Case 24 ............................................................................................................................ 178
Case 25 ............................................................................................................................ 180

Practice Exam Answers ............................................................................................................................ 183

Practice Exam Questions with Answers .................................................................................................. 239

Test Preparation and Test Taking Tips ................................................................................................... 477

Bubble Sheet ............................................................................................................................................ 480

© 2015 - 2023 Tests.com


Review of the NBDHE Dental Hygienist Exam

About the NBDHE


The licensing of dental hygienists in the United States is regulated by the individual states. A license
issued within a state is only applicable within that state. Typically, each state has a board of dentistry
that is an agency authorized by the state legislature to regulate the profession of dentistry within state
borders. Licensure requirements vary between the states but all require passing an exam to obtain
authorization to practice as a dental hygienist. Currently, all states, and the District of Columbia, Puerto
Rico, Guam and the Virgin Islands, recognize exam results from the National Board of Dental Hygiene
Exam (NBDHE).
The agency responsible for the development and administration of the NBDHE is the Joint Council
on National Dental Examinations (JCNDE). This commission is made up of 15 members representing
dental schools, dental practices, state dental examination boards, dental hygienists, dental students,
and the public. Dental hygienists are consulted by the JCNDE for the examination.
The NBDHE assists state boards in qualifying dental hygienists who seek licensure in their
jurisdiction. The exam assesses a hygienist’s ability to understand relevant information including basic
biomedical, dental and dental hygiene sciences and the ability to apply the information into a problem-
solving context.
In addition to taking the NBDHE, each dental hygiene student must also take and pass a clinical
examination approved by their region or state’s board of dental examiners.

Who May Take The Exam?

Only qualified candidates can sit for the NBDHE. Qualified applicants who may take the NBDHE are
as follows:

1. Dental Hygiene Students

The dental hygiene program where the applicant attends should include at least two
academic years of full-time instruction. The applicant must take the exam within four
months of completing their dental hygiene education. Curriculum must include general
education, biomedical sciences, dental sciences and dental hygiene sciences. For more
information on accreditation, visit www.ada.org.

Graduates of dental hygiene programs should be competent in the following areas:

- Providing dental hygiene care


- Comprehensive collection of patient data
- Analysis of assessment findings

TOP © 2015 - 2023 Tests.com


1
- Establishment of a dental hygiene care plan
- Provision of patient-centered and evidence-based care
- Identify goals of dental hygiene care plan
- Complete and accurate document recording
- Be proficient in English

2. Dental students
3. Dentists
4. Re-examination applicants - Applicants that have not previously passed the NBDHE: a minimum
of 90 days should lapse between exam attempts. If the applicant fails after 3 attempts, they
must wait a minimum of 1 year before applying for re-examination a 4th time. Applicants must
pass the NDBHE within 5 attempts or 5 years, whichever comes first.

Exam Content

The NBDHE consists of 350 multiple-choice questions. Question formats include paired true/false,
completion, direct question, cause-and-effect and case studies. The test context also utilizes matching
questions, ordering questions and questions with multiple correct answers.
There are 2 components to the NBDHE: (1) Discipline Based and (2) Dental Hygiene Patient Cases.
The discipline-based component consists of 200 questions on the following three areas:

1. Scientific Basis for Dental Hygiene Practice

- Anatomical sciences
- Physiology
- Biochemistry and Nutrition
- Microbiology and Immunology
- Pathology
- Pharmacology

2. Provision of Clinical Dental Hygiene Services

- Assessing Patient characteristics


- Obtaining and Interpreting Radiographs
- Planning and managing dental hygiene care
- Performing periodontal procedures
- Using preventive agents
- Providing supportive treatment services
- Professional responsibility

TOP © 2015 - 2023 Tests.com


2
3. Community Health/Research Principles

- Promoting health and preventing disease within groups


- Participating in community programs
- Analyzing scientific literature, understanding statistical concepts and applying research
results

The patient cases component includes 12-15 patient cases of adult or child patients, including the
patient’s health history, dental chart, radiographs and clinical photographs. Each NBDHE exam contains
at least one geriatric, periodontal, pediatric, special needs and medically compromised patient. The
patient cases will address the following knowledge and skill areas:

- Assessing patient characteristics


- Obtaining and interpreting radiographs
- Planning and managing dental hygiene care
- Performing periodontal procedures
- Using preventive agents
- Providing supportive treatment service
- Professional responsibility

Exam Time

The exam lasts for approximately 9 hours, with 2 sessions separated by a one-hour scheduled break.
It is a computer-based exam. The first session focuses on discipline-based items, with 200 questions and
3.5 hours allotted for the completion of this section. Following the one-hour break, the second session
begins, with 150 patient case study items and a 4-hour completion time. There are also optional tutorial
and post-exam surveys included on the date of the exam. The breakdown of the test day is as follows:

- Optional Tutorial: 15 minutes


- First Session: Discipline-based items; 200 items 3.5 hours
- Optional scheduled break 1 hour
- Second Session: Patient Case items; 150 items 4 hours
- Optional Post-examination Survey 15 minutes
- Total: 9 hours

Application to Take the Exam

Applicants may apply for the NBDHE exam up to six months prior to their desired testing dates. All
applicants must obtain a DENTPIN, which is their identification number and allows applicants to have
confidential transmission of their test results. DENTPINs are available from the ADA at www.ada.org and

TOP © 2015 - 2023 Tests.com


3
should be obtained before filing an exam application.
After applying for the exam you are eligible for six months to test. Failing to take the exam during
this six month period where you are eligible will result in the surrender of your application fee. You will
need to resubmit an application and new fee in order to take the examination on a later date.
If you do not arrive at your examination time, or you cancel the test less than 24 hours before the
scheduled time, the examination fees will not be refunded.
Application may be made electronically on www.ada.org. The test is available to take at Pearson
VUE testing centers year-round, and the test date and time is made at your preference. There are fees
to take the exam. The fee includes sending your final score to up to three dental hygiene licensing
jurisdictions and your dental hygiene program director. There are fees for additional reports. A list of
state boards can be found at www.dentalboards.org.

Preparing for the Exam

In order to be prepared for the NBDHE exam, you should create a study plan and follow it. Ideally
you will start studying at least three months ahead of time, but many people begin studying even
sooner. Even experienced dental hygienists can fail the NBDHE exam so it is critical that you create a
plan and follow it. Your plan should begin months before the test and should continue through the day
of the test.
List out the topics that you need to study and spread them throughout your weekly study time,
being sure to cover them all. Review these multiple times a week (every day, if possible) up until the
day before your exam.
There are many study resources available as you prepare for the NBDHE exam. Start with the
NBDHE Candidate Handbook available through the JCNDE site. The handbook provides important,
detailed information about the test that you must know and understand.
It is best to take the Tests.com practice test before you start studying to assess your strengths and
weaknesses and to evaluate how well you know the material. Taking a practice test should be one part
of your total study plan, including the review of reference books and text books, class notes, study
guides, flashcards and test prep courses. You can use the bubble sheet at the end of the test to track
your answers. Print numerous copies of the bubble sheet so you can take the test more than once. You
will want to mix some test taking into your study habits to vary your activities to keep the process
interesting and to more effectively commit the subject matter to memory. You also want to practice
your test taking skills. Use the online TestSIM platform to simulate the actual test or to form smaller
tests that fit your schedule. Limit yourself to a certain time period for a certain number of questions
randomly selected. You can do this more than once throughout your preparation period. Finally, a few
days prior to the actual test, retake this practice test to evaluate your grasp of the material.
The following are more study and test taking tips.

• Study the topics that you are weaker in. Most dental hygiene students know what areas they
find the context difficult to grasp. You may find time to review areas that you have a good grasp

TOP © 2015 - 2023 Tests.com


4
of knowledge, but time should be optimized when it comes to preparing for the national board
exam.
• Form a study group with other students. Exchange flash cards and mock test questions to get a
different standpoint on how questions can be worded.
• Read through board review textbooks, and highlight the areas in the book that you need to
study more. Go through and read aloud to yourself, or copy the facts down on paper, to
improve your memory. Writing has been a proven method of memory aid.

Prior to the day of the exam, you should locate your testing center, and if possible, drive to it so that
you will know your way.
It is not advisable to cram the night before the exam. If you do not know the subjects the day before
the test, then staying up late cramming all evening will not help. In fact it will cause you to not get
enough sleep and could cause you to become easily distracted and not be able to focus during the test.
Enjoy a pleasant day to yourself, as much as possible.

On the Day of Exam

You should arrive 30 minutes before your scheduled examination time. Exams are administered at
Pearson VUE Test Centers across the U.S. You will need two forms of identification - one must be
government issued with a photograph and signature (such as a driver’s license or passport). The
secondary form of identification only requires a signature, such as a debit card or library card.
You will not be allowed to bring any personal items back into the testing area. This includes items
such as:
- Cellular phones
- Food and drink
- Books or study materials
- Dental models, instruments or materials
- Paper, calculating devices or other measuring devices
- Electronic devices such as an iPod or headphones
- Purses, wallets, or other bags
- Highlighters, pens or pencils
- Dictionaries
- Coats, gloves or jackets
- Good luck charms
- Medicinal items
- Watches or timing devices (the test will be computerized and has a
count-down clock)
- Magnification devices
Sit for a time before the test starts and clear your head. Do not let anxiety take control of your
thoughts. Be confident and even keeled. When the test starts, start working diligently, keeping track of

TOP © 2015 - 2023 Tests.com


5
the time. It is important to maintain a good attitude before and during the test. Here are some more
test taking tips:

• Read every answer before selecting the one you think is correct. Many answers may have an
answer that seems correct listed before the true correct answer.
• If the question is difficult to understand, focus on the keywords that you do understand in the
context of the question. Using those words, determine whether or not any of the answers
provided are even relevant to the information that you are able to process.
• Eliminate the wrong answers first, and reason with yourself why they are incorrect.
• Pay attention to qualifying words in the question such as if, only, except, not.”
• Watch the clock. For the first session there are 200 questions and 3.5 hours allotted. That means
you have approximately 1 minute per question.
• Get plenty of sleep. Yes, this probably sounds difficult to do since you will most likely be nervous
the night before. Take a walk in the evening to burn some energy and clear your mind before
you go to bed.
• Eat well and drink plenty of water. Have a high-protein breakfast that will help control your
hunger before the scheduled break time. Keep fruit handy to have when you get a chance. It’s
healthy and acts as brain fuel.
• Don’t stress. Thousands of hygienists take the NBDHE every year and pass. The main focus of
the exam is to make sure you have learned what you studied in dental hygiene school so that
you can take it and apply it in the work force. It sets the bar for dental hygiene programs so that
they will have educated their students in a proper manner.

Passing the Exam

The minimum passing grade of the NDBHE is 75, with score ranges from 49-99. Applicant reports do
not show a score but simply display PASS or FAIL. The passing score is based on the number of correct
answers and the norm-based conversion scale of the exam. Wrong-answer questions are not penalized
in the final scoring tabulation. So, when you are taking the test, you should answer every question,
whether or not you know the correct answer. Results are received about 3-4 weeks after the exam by
postal mail. The JCNDE does not accept calls or email inquiries for results.
While all 50 states recognize the NBDHE exam, some states may place a restriction on obtaining
licensure to if the exam was taken in the past 5 or 10 years. This is relevant to dental hygiene
professionals that are moving to other states of residence and want to obtain licensure in that state.

We hope this practice test kit improves your understanding of the subject matter and gives you the test-
taking skills necessary to do well. Best of luck on the test and with your career as a dental hygienist.

Sharon Boyd, RDH

TOP © 2015 - 2023 Tests.com


6
Practice Test for the NBDHE Dental Hygienist Exam – Section 1
Scientific Basis for Dental Hygiene Practice

1. During swallowing, the hyoid bone moves downward. During swallowing, the larynx elevates.

a. Both statements are true


b. Both statements are false
c. The first statement is true, the second statement is false
d. The first statement is false, the second statement is true

2. Which nervous system produces the fight-or-flight reaction?

a. Parasympathetic
b. Sympathetic
c. Peripheral
d. Central

3. A patient is experiencing symptoms related to Bell’s palsy. Which cranial nerve is suffering from
a motor deficit?

a. Facial
b. Trigeminal
c. Trochlear
d. Hypoglossal

4. What is mobilized in and out of the blood during bone remodeling?

a. Canaliculi
b. Osteoclasts
c. Calcium
d. Osteoblasts

5. Which of the following cranial nerves pass through the foramen ovale of the sphenoid bone?

a. Trigeminal mandibular
b. Nasopalatine
c. Trigeminal maxillary
d. Abducent

TOP © 2015 - 2023 Tests.com


7
6. The maxillary branch of the trigeminal nerve passes through the ______.

a. foramen spinosum
b. superior orbital fissure
c. foramen ovale
d. foramen rotundum

7. Which cranial nerve is the longest and extends to the trunk of the body?

a. Hypoglossal
b. Vagus
c. Facial
d. Trigeminal

8. List the following digestive system structures in order of their function during digestion of a
meal.

_____ a. Stomach
_____ b. Small intestine
_____ c. Pharynx
_____ d. Large intestine
_____ e. Esophagus

9. List the following components of a muscle fiber in order of the LARGEST component to the
SMALLEST component.

_____ a. Myofilaments
_____ b. Muscle fascicle
_____ c. Muscle
_____ d. Myofibril
_____ e. Myofiber

10. The trigeminal ganglion of the fifth (V) trigeminal nerve has three divisions. Which of the
following is NOT a division of the trigeminal ganglion?

a. Opthalmic
b. Mandibular
c. Optic
d. Maxillary

TOP © 2015 - 2023 Tests.com


8
11. Which three of the following are lymphatic organs?

a. Tonsils
b. Pituitary gland
c. Spleen
d. Thymus
e. Kidney
f. Brain

12. Order the following anatomical structures of the respiratory tract as they appear and function in
the processing of inhalation.

_____ a. Trachea
_____ b. Alveoli
_____ c. Alveolar duct
_____ d. Primary bronchi
_____ e. Bronchiole

13. What parallels the venous blood vessels and returns filtered fluids to the bloodstream?

a. The endocrine system


b. The lymphatic system
c. The arterial blood vessel system
d. The circulatory system

14. What type of cell-to-cell attachments are found in enamel-forming ameloblasts?

a. Hemidesmosome
b. Tight junctions
c. Desmosomes
d. Gap junctions

15. What is the homogenous, translucent gel enclosed in a cell by the cell membrane?

a. Lipid droplets
b. Cytoplasm
c. Lysosome
d. Nucleus

TOP © 2015 - 2023 Tests.com


9
16. Which type of epithelia lines the oral cavity?

a. Stratisfied squamous
b. Simple squamous
c. Stratified columnar
d. Simple columnar

17. MATCH the following dental anatomical structures with the permanent tooth to which they
belong.

____Cusp of Carbelli a. Maxillary second premolar


____Two cusps that are approximately the same length b. Mandibular second premolar
____Long central developmental groove c. Maxillary first molar
____“H” or “y” groove patterns int he occlusa surface d. Mandibular first molar
____Greater mesiodistal than faciolingual width e. Maxillary first premolar

18. MATCH the parts of the tongue with their description.

___Lingual papillae a. The side of the tongue


___Sulcus terminalis b. irregular mass of tonsillar tissue on the posterior dorsal surface
___Foramen cecum c. elevated structures of specialized mucosa
___Lateral surface d. pitlike depression in the center of sulcus terminalis
___Lingual tonsil e. v-shaped groove on the posterior dorsal surface of the tongue

19. Which TWO are folds of tissue in the inferior to ventral surfaces of the tongue?

a. Labial frenum
b. Sublingual caruncle
c. Sublingual fold
d. Lingual frenum

20. Which cranial bone is the single bone located in the most posterior portion of the skull?

a. Mandible
b. Lambdiodal suture
c. Parietal
d. Occipital

TOP © 2015 - 2023 Tests.com


10
21. Which muscle of mastication is superficially located and one of the strongest muscles?

a. Medial pterygoid
b. Masseter
c. Temporalis
d. Lateral pterygoid

22. Which tooth has a mesial root concavity that is not as prominent?

a. Mandibular second molar


b. Mandibular first molar
c. Maxillary first molar
d. Maxillary first premolar

23. Which tooth has a furcation that is in the apical third to 1/2 of the tooth?

a. Mandibular first premolar


b. Mandibular second premolar
c. Maxillary first premolar
d. Maxillary second premolar

24. Which two of the following are functions of the stomach?

a. Storage of ingested food until it is emptied into the small intestine


b. Stimulated by PANS
c. Secretion of HC1 and enzymes that initiate digestion
d. Inhibited by SANS

25. Which two of the following are branches that diverge from the maxillary artery?

a. Incisive artery
b. Posterior auricular
c. Ascending palatine artery
d. Inferior alveolar

26. A patient’s mandibular first permanent molar’s mesiobuccal groove is in a mesial relationship to
the mesiobuccal cusp of the maxillary first permanent molar. What classification of occlusion is
this?

a. Class I
b. Class II
c. Class III
d. Class IV

TOP © 2015 - 2023 Tests.com


11
27. Which of the following teeth has a long lingual root that usually extends beyond the lingual
surface of the crown?

a. Mandibular first molar


b. Maxillary first molar
c. Mandibular second molar
d. Maxillary second molar

28. Which tooth may have a lingual groove extending from the crown to the root?

a. Maxillary central incisors


b. Mandibular central incisors
c. Mandibular lateral incisors
d. Maxillary lateral incisors

29. Which two of the following food sources is an example of a disaccharide sugar structure?

a. Apple
b. Honey
c. Banana
d. Carrots

30. What is the recommended fiber intake each day for an adult?

a. 0-10 grams
b. 10-20 grams
c. 20-30 grams
d. 20-50 grams

31. A healthy blood glucose level range is __________ mg/100mL blood.

a. 50-100
b. 70-120
c. 90-150
d. 100-175

32. Which complex carbohydrate is made up of the plant form of glucose?

a. Cellulose
b. Glycogen
c. Starch
d. Pectin

TOP © 2015 - 2023 Tests.com


12
33. Decalcification and demineralization of tooth enamel begins at what pH?

a. 3.3
b. 5.5
c. 6.6
d. 7.7

34. What sweetener is found in plants and can interfere with the metabolism of s. Mutans?

a. Glucose
b. Sorbitol
c. Xylitol
d. Mannitol

35. What is the recommended daily allowance for digestible carbohydrates?

a. 50-100 g/day
b. 130 g/day
c. 100-150 g/day
d. 175 g/day

36. Carbohydrates are an energy source for the growth and repair of periodontal tissues. A firm
texture of complex carbohydrates may increase circulation in gingival tissue.

a. Both statements are true


b. The first statement is true, the second statement is false
c. The first statement is false, the second statement is true
d. Both statements are false

37. Yogurt is not recommended for lactose intolerant patients. Lactose intolerance may be due to a
congenital absence of lactase.

a. Both statements are true


b. The first statement is true, the second statement is false
c. The first statement is false, the second statement is true
d. Both statements are false

38. Which leukocyte is associated with acute inflammation?

a. Neutrophils
b. Basophils
c. Eosinophils
d. Mast cells

TOP © 2015 - 2023 Tests.com


13
39. If an antibody is produced by an individual and passed on to another, such as a mother to her
child, what type of immunity is this?

a. Naturally active
b. Naturally passive
c. Artificially active
d. Artificially passive

40. MATCH the following symptoms to the appropriate disease condition.

___ Fifth’s Disease a. Fever, swollen parotid gland


___ Shingles b. Red, lacy rash
___ Herpes c. clear, fluid-like blisters on the lips
___ Syphilis d. Fever, itching, and red rash on one side of the body
___ Mumps e. Small, round nodule on the lip or tongue

41. Which Hepatitis virus is transmitted through a fecal-oral route?

a. A
b. B
c. C
d. D

42. Which three of the following symptoms are signs found in a child that may have
hypothyroidism?

a. Enlarged tongue
b. Delayed tooth eruption
c. Enlarged lips
d. Periodontal disease

43. Which disease of the bone marrow exhibits abnormally low levels of white blood cells (WBCs)?

a. Neutropenia
b. Leukopenia
c. Leukemia
d. Multiple myeloma

TOP © 2015 - 2023 Tests.com


14
44. What is a common side effect of radiation and or chemotherapy that causes painful
inflammation and ulceration of mucous membranes?

a. Candida infections
b. Osteoradionecrosis
c. Sore throat
d. Mucositis

45. What microorganism is resistant to high temperatures and disinfectants?

a. Glycocalyx
b. Gram-positive bacteria
c. Endospore
d. Capsules

46. Which of the following microbes cannot perform or may be destroyed in the presence of
oxygen?

a. Microaerophiles
b. Aerobes
c. Facultative anaerobes
d. Anaerobes

47. Infection transmission that involves touching a contaminated object is considered:

a. Direct contact
b. Nondirect contact
c. Vehicle
d. Passive

48. An infection that affects an immunocompromised person is considered:

a. Chronic
b. Acute
c. Passive
d. Opportunistic

49. What are the smallest and simplest infectious agents?

a. Endospores
b. Viruses
c. Gram-negative bacteria
d. Fungi

TOP © 2015 - 2023 Tests.com


15
50. Which two conditions are examples of vector-borne infections?

a. Rabies
b. Lyme disease
c. West Nile virus
d. Giardia

51. What is a natural chemotherapeutic compound that is produced by a bacterium and will inhibit
or kill bacteria?

a. Antibiotic
b. Allergy
c. Immunity
d. Vaccination

52. A highly infectious condition that leaves pustules and itchy yellow scabs around the mouth and
face is called _______.

a. Abscess
b. Impetigo
c. Human Papillomavirus
d. Acne

53. What two symptoms are associated with liver disorders?

a. Increased saliva production


b. Xerostomia
c. Jaundice
d. Sunken eyes

54. Which two Hepatitis viral strains are spread through sexual contact?

a. Hep A
b. Hep B
c. Hep C
d. Hep E

TOP © 2015 - 2023 Tests.com


16
55. Which of the following is NOT a sign of leukemia?

a. Abnormal bruising
b. Spontaneous sulcular bleeding
c. Pallor of the gingiva
d. Red gingival tissues

56. What bone disease is most common in children and involves abnormal proliferation of fibrous
connective tissues?

a. Fibrous dysplasia
b. Osteoporosis
c. Paget’s disease
d. Neoplasia

57. The most common form of oral cancer is:

a. Basal cell carcinoma


b. Squamous cell carcinoma
c. Malignant melanoma
d. Pleomorphic adenoma

58. Which cardiovascular disease is associated with periodontal disease, placing an emphasis on
improving oral health in an attempt to reduce the risk of disease?

a. Cerebrovascular disease
b. Hypertension
c. Congestive heart failure
d. Coronary artery atherosclerosis

59. Local anesthetics containing vasoconstrictors should not be administered to asthma patients. It
also should not be used for patients that suffer from emphysema.

a. Both statements are true


b. Both statements are false
c. The first statement is true, the second statement is false
d. The first statement is false, the second statement is true

TOP © 2015 - 2023 Tests.com


17
60. What type of seizure exhibits symptoms of jerking motions and no loss of consciousness?

a. Status epilepticus
b. Petit mal
c. Grand mal
d. None of the above

61. What disease exhibits itself in a classic “butterfly” pattern rash over the patient’s face?

a. Phemphigus vulgaris
b. Hodgkin’s lymphoma
c. Scleroderma
d. Lupus

62. Wedge-shaped lesions along the cervical margin of the teeth are referred to as _________.

a. Erosion
b. Abrasion
c. Abfraction
d. Attrition

63. A condition of the oral tissues due to hormonal changes during pregnancy is called ___________.

a. Pyogenic granuloma
b. Sialolith
c. Gingival hyperplasia
d. Inflammatory papillary hyperplasia

64. A discoloration of the oral mucosa that may have a radiopaque appearance.

a. Traumatic neuroma
b. Amalgam tattoo
c. Necrotizing sialometaplasia
d. Melanoma

65. Which cyst exhibits bluish swelling around a newly erupting crown, most common with the
eruption of primary teeth?

a. Odontogenic keratocyst
b. Dentigerous cyst
c. Nonodontogenic cysts
d. Eruption cyst

TOP © 2015 - 2023 Tests.com


18
66. Supernumerary teeth usually exhibit microdontia. They are most commonly found in the
maxilla.

a. Both statements are true


b. Both statements are false
c. The first statement is true, the second statement is false
d. The first statement is false, the second statement is true

67. Which of the following is an exophytic area of enamel on the root surface?

a. Dens in dente
b. Enamel pearl
c. Enamel hypoplasia
d. Gemination

68. Dental fluorosis causing damage to tooth development occurs approximately at what ages?

a. 0-12 months
b. 12 months - 12 years
c. 6 months - 5 years
d. 5 years - 12 years

69. Which two anomalies may involve the buccal mucosa?

a. Linea alba
b. Leukoedema
c. Lingual variscosities
d. Ranula

70. An asymptomatic cluster of yellow lobules of sebaceous glands is known as what?

a. Median rhomboid glossitis


b. Ranula
c. Fordyce granules
d. Lingual thyroid nodule

71. Which two anomalies involve the desquamation of filiform lingual papillae?

a. Geographic tongue
b. Fissured tongue
c. Black hairy tongue
d. Benign migratory glossitis

TOP © 2015 - 2023 Tests.com


19
72. What is the term used to describe a sharp bend or curve in the root of a formed tooth?

a. Concrescence
b. Dilaceration
c. Fusion
d. Gemination

73. What two medications are contraindicated for children due to the risk of developing Reye’s
syndrome?

a. Acetaminophen
b. Aspirin
c. Ipubrophen
d. Bayer

74. Which pharmacological abbreviation is for medicine to be taken “twice daily”?

a. ud
b. tid
c. bid
d. prn

75. The SANS enables the body to maintain bodily function during normal activities. The PANS
enables the body to function during emergencies.

a. Both statements are true


b. Both statements are false
c. The first statement is true, the second statement is false
d. The first statement is false, the second statement is true

76. Which two of the following are typical effects or reactions while using Adrenergic medications?

a. Vasoconstriction
b. Bronchodilation
c. CNS depression
d. Lowered blood pressure

TOP © 2015 - 2023 Tests.com


20
77. What is the pharmaceutical dose that produces 50% of the maximum response in 50% of
subjects?

a. Lethal dose
b. Log dose
c. Effective dose
d. Duration

78. Which of the following is NOT an anti-inflammatory drug (NSAIDs)?

a. Diazepam
b. Ibuprophen
c. Naproxen
d. Diflunisal

79. Which of the following is TRUE about local anesthetic agents?

a. They should be non-potent.


b. They should be irreversible.
c. They should have a rapid onset.
d. They have a short shelf life.
e. Both c and d only.

80. Which of the following are advantages of nitrous-oxide use in the dental office?

a. Nonflammable
b. May be used on children
c. Rapid onset and recovery
d. All of the above

81. What sedative medications provide a wide therapeutic index and are safer than barbiturates?

a. Analgesics
b. NSAIDs
c. Salicylates
d. Benzodiazepines

82. Which drug used in the dental office should be avoided during treatment of a pregnant patient?

a. NSAIDs
b. Epinephrine
c. Amoxicillin
d. All of the above

TOP © 2015 - 2023 Tests.com


21
83. Which medication is known to cause gingival hyperplasia?

a. Dilantin
b. Diflucan
c. Dyazide
d. Neurontin

84. What is the primary site for drug metabolism?

a. Pancreas
b. Liver
c. Small intestine
d. Cardiovascular system

85. What cardiovascular agents cause vasodilation of the blood vessels?

a. Antiarrhythmic drugs
b. Antihypertensive drugs
c. Calcium channel blockers
d. Anticoagulants

86. Nystatin is used to treat:

a. Oral candidiasis
b. Periodontal infections
c. Herpes simplex
d. Dental abscesses

87. The small elevated tissue opposite of the maxillary second molar:

a. Parotid duct
b. Parotid gland
c. Parotid papilla
d. Mucobuccal fold

88. The facial portion of the alveolus of the canine:

a. Periodontal ligaments
b. Alveolus
c. Alveolar process
d. Canine eminence

TOP © 2015 - 2023 Tests.com


22
89. Abnormal variations on the lingual aspect of the mandibular arch:

a. Tori
b. Exostoses
c. Tuberosity
d. Retromolar pad

90. Which branch of the trigeminal nerve provides nerve supply to the maxillary posterior
lingual periodontium?

a. Middle superior alveolar nerve


b. Greater palatine nerve
c. Nasopalatine nerve
d. Inferior alveolar nerve

91. When normal shedding of epithelium of the filiform lingual papilla does not occur, It produces
thick layers of dead cells and keratin builds on the tongue surface:

a. Geographic tongue
b. Halitosis
c. Black hairy tongue
d. Melanin pigmentation

92. The retention of saliva in a minor salivary gland results in a:

a. Mucocele
b. Ranula
c. Nicotinic stomatitis
d. Xerostomia

93. What is the name of the papillae that are lined along the anterior side of the sulcus terminals on
the body of the tongue?

a. Fungiform
b. Circumvallate
c. Foliate
d. Filiform

TOP © 2015 - 2023 Tests.com


23
94. Which gland is located at the junction of the larynx and the trachea and can be palpated?

a. Parathyroid
b. Sublingual salivary
c. Sternocleidomastoid
d. Thyroid

95. What is a function of the small intestine?

a. Absorb nutrients and minerals from food


b. Absorb water from indigestible food
c. Breakdown and digest food
d. Reabsorb fluids and process waste products

96. What is the drug-nutrient interaction of amoxicillin?

a. Metabolism is inhibited with grapefruit juice.


b. It cannot be taken with food as its effectiveness decreases.
c. It may alter intestinal flora, resulting in malabsorption of nutrients.
d. It cannot be taken with antacids because its effectiveness decreases.

97. Tuberculosis is transmitted by what means?

a. Direct contact with blood


b. Direct contact with saliva
c. Direct contact with blood or saliva
d. Airborne droplets

98. What are two examples of fecal-oral transmission?

a. Contaminated water supply


b. Mosquitos
c. Ticks
d. Inadequate hand washing

99. All of the following are monosaccharides except:

a. Glucose
b. Fructose
c. Galactose
d. Maltose

TOP © 2015 - 2023 Tests.com


24
100. A blood pressure reading of 148/90 is considered:

a. Normal
b. Prehypertension
c. Hypertension - stage 1
d. Hypertension - stage 2

101. On which tooth does the Cusp of Carabelli normally appear?

a. Maxillary first molar


b. Maxillary second molar
c. Mandibular first molar
d. Mandibular second molar

102. An extra chromosome associated with Down syndrome:

a. Trisomy 23
b. Trisomy 21
c. Trisomy 18
d. Trisomy 13

103. When B-cell lymphocytes divide and form plasma cells, they produce:

a. Immunoglobulins
b. Immunogens
c. Monocyte
d. Macrophage

104. What teeth are nonsuccedaneous?

a. All primary canine teeth


b. All permanent premolar teeth
c. All permanent molar teeth
d. All permeant anterior teeth

105. Which teeth have a trifurcation?

a. Maxillary molars
b. Mandibular molars
c. Maxillary premolar
d. Mandibular premolars

TOP © 2015 - 2023 Tests.com


25
106. Which teeth have the greatest depth of curvature of the CEJ on the mesial surface?

a. Mandibular Lateral incisors


b. Maxillary canine
c. Mandibular centrals
d. Maxillary centrals

107. What sterilization technique uses oxidation to sterilize dental instruments?

a. Autoclave
b. Unsaturated chemical vapor
c. Dry Heat
d. Ultrasonic

108. A patient that has a mild systemic disease and may or may not have special considerations in
regards to his or her dental management, would be considered what type of ASA Classification?

a. I
b. II
c. III
d. IV

109. When a tooth loses tooth structure due to a chemical agent:

a. Attrition
b. Abrasion
c. Hypoplasia
d. Erosion

110. What is the vertical overlap of the maxillary and mandibular incisors considered?

a. Overjet
b. Overbite
c. Crossbite
d. Openbite

111. What is the dominant bacterium in the initial plaque stage?

a. Gram positive streptococcus


b. Gram negative streptococcus
c. Spirochetes
d. Gram negative rods

TOP © 2015 - 2023 Tests.com


26
112. At what pH is a root demineralized?

a. 4.5-5.5
b. 5.5-5.9
c. 6.0-6.7
d. 6.5-7.4

113. Tobacco is considered what type of stain?

a. Intrinsic
b. Extrinsic
c. Exogenous
d. Endogenous

114. Which of the following conditions would respond to therapeutic diagnosis?

a. Bone cyst
b. Angular cheilitis
c. Fordyce granules
d. Paget disease

115. A dense, nonmineralized complex mass of bacteria that forms a gel like matrix:

a. Acquired pellicle
b. Biofilm
c. Demineralization
d. Plaque

116. Which of the following is not a major component of saliva that helps protect against dental
caries?

a. Calcium
b. Phosphate
c. Iron
d. Fluoride

117. Which nerve innervates the tooth pulp on the mandible?

a. Mental
b. Long Buccal
c. Posterior Superior Alveolar
d. Inferior Alveolar

TOP © 2015 - 2023 Tests.com


27
118. Which of the following cranial nerves is not a sensory nerve?

a. Trochlear
b. Olfactory
c. Optic
d. Vestibulocochlear

119. Which of the following is not a major salivary gland?

a. Submandibular
b. Sublingual
c. Parotid
d. Wharton

120. In what order does the blood flow from the heart to the lungs? Put the following in the correct
order.

a. Right Atrium
b. Right Ventricle
c. Tricuspid Valve
d. Superior and Inferior Vena Cava
e. Lungs
f. Pulmonary Artery

121. Drugs with a high-first past effect have a smaller oral to parental dose ratio.

a. True
b. False, they have a larger parental dose ratio.
c. False, they have the same parental dose ratio.
d. False, drugs do not have a high-first past effect.

122. What is the study of how a drug is absorbed, distributed, metabolized and excreted by the body?

a. Pharmacology
b. Pharmacokinetics
c. Pharmacodynamics
d. Pharmaceuticals

TOP © 2015 - 2023 Tests.com


28
123. Liver disease or previous or current substance abuse may impair the metabolism of a drug.

a. True
b. False, only current substance use affects the metabolism of a drug.
c. False, only previous substance abuse affects the metabolism of a drug.
d. False, liver disease does not affect the metabolism of a drug.

124. The _______ stage of development is when the first signs of an arrangement of cells occur.

a. Bell
b. Cap
c. Bud
d. Embryotic

125. Which of the following is not a type of tissue found in the body?

a. Connective
b. Endoplasmic
c. Muscle
d. Nerve

126. All of the following are part of the gingival fiber group except:

a. Interradicular
b. Dentogingival
c. Alveologingival
d. Circumferential

127. This vitamin is the catalyst for synthesis of blood clotting:

a. Vitamin A
b. Vitamin E
c. Vitamin D
d. Vitamin K

128. A deficiency in Vitamin B3 (Niacin) can lead to what disease?

a. Pellagra
b. Scurvy
c. Enamel Mottling
d. Sickle Cell Anemia

TOP © 2015 - 2023 Tests.com


29
129. Which of the following is not considered a form of calcium?

a. Cheese
b. Peas
c. Tofu
d. Kale

130. What palpation method is used during an extra oral exam that uses the finger and thumbs of
both hands?

a. Bilateral palpation
b. Digital palpation
c. Bidigital palpation
d. Bimanual palpation

131. Subluxation is when the condyle of the temporal mandibular joint glides beyond the eminence
preventing the jaw from closing. Subluxation can be caused by long dental procedures.

a. Both statements are true


b. Both statements are false
c. First statement is true. Second statement is false.
d. First statement is false. Second statement is true.

132. Periodontal disease can be more severe in patients that smoke when they are deficient in this
vitamin:

a. Vitamin A
b. Vitamin B3
c. Vitamin B6
d. Vitamin C

133. What happens when the globular process fails to fuse with the maxillary process during the first
trimester of pregnancy?

a. Cleft Lip
b. Cleft Palate
c. Cleft Nasal
d. Cleft Mandible

TOP © 2015 - 2023 Tests.com


30
134. Cementum contains collagen fibers. In teeth, Sharpey’s fibers are the terminal ends of the
principal fibers that insert into the cementum and into the periosteum of the avelor bone.

a. Both statements are true.


b. Both statements are false.
c. First statement is true. Second statement is false.
d. First statement is false. Second statement is true.

135. What plane divides the body into left and right halves?

a. Axial
b. Parasagittal
c. Sagittal
d. Transverse

136. What is the smallest permanent tooth in the mouth?

a. Mandibular central incisors


b. Maxillary canine tooth
c. Mandibular lateral incisor
d. Maxillary lateral incisor

137. What is the arrow pointing to?

a. Lamina dura
b. Cancellous bone
c. Periodontal ligament
d. Cementum

138. During swallowing, the esophageal phase is under voluntary neuromuscular control. The
pharyngeal phase is under involuntary neuromuscular control.

a. Both statements are true.


b. Both statements are false.
c. First statement is true. Second statement is false.
d. First statement is false. Second statement is true.

TOP © 2015 - 2023 Tests.com


31
139. Which two organs are endocrine glands?

a. Kidney
b. Ovaries
c. Thyroid
d. Ureters

140. A psuedopocket is produced by edematous gingival tissue. The junction epithelium does not
migrate when there is psuedopocketing.

a. Both statements are true.


b. Both statements are false.
c. First statement is true. Second statement is false.
d. First statement is false. Second statement is true.

141. A patient with a Class I occlusion would have a facial profile that is mesognathic. Mesognathic is
considered to be a normal profile.

a. Both statements are true.


b. Both statements are false.
c. First statement is true. Second statement is false.
d. First statement is false. Second statement is true.

Practice Test for the NBDHE Dental Hygienist Exam – Section 2


Provision of Clinical Dental Hygiene Practice

142. What information source determines the general health of the patient?

a. Clinical interview with the patient


b. Medical history form
c. Dental health history form
d. Letter from patient’s primary care physician

TOP © 2015 - 2023 Tests.com


32
143. A patient leaves the medication portion of her health history form blank. Upon being seated the
hygienist reviews the form with her and asks if she takes any medications or supplements. The
patient states that she takes birth control pills but she did not list them on the medication
portion because it is irrelevant to her dental treatment. The hygienist informs her that it should
be included, because birth control pills _________.

a. may interact with local anesthetic agents


b. may become less effective if given antibiotics
c. could cause gingival overgrowth
d. change the oral flora

144. Medical history forms should be updated:

a. Annually
b. Biannually
c. Every 3 years
d. At each visit

145. A healthy adult’s pulse rate should be between ______ beats per minute.

a. 120-160
b. 80-130
c. 50-120
d. 60-100

146. Which two of the following are reasons that require antibiotic premedication?

a. Systemic lupus erythematosis


b. Joint replacement less than 2 years ago
c. Joint replacement up to 5 years ago
d. Type II diabetes

147. Which of the following is NOT an essential part of an initial extraoral examination?

a. Palpation of the masseter and temporalis muscles


b. Inspection of the lips for pigmentations
c. Inspection of the ventral surfaces of the tongue
d. Inspection for facial symmetry

TOP © 2015 - 2023 Tests.com


33
148. Which of the following conditions are inspected upon examination of the parotid gland? Select
all answers that apply.

a. Hardness
b. Mobility
c. Swelling
d. Enlargement

149. Which lymph nodes are assessed through palpitation with the patient’s head tilted forward?

a. Deep cervical
b. Accessory
c. Auricular
d. Occipital

150. Which of the following appearances in gingiva indicates a disease condition?

a. Loss of gingival stippling


b. Pigmentation of gingiva
c. Pink, stippled gingiva
d. Firm, knife-edged papilla

151. What area should be palpated to determine if a blockage of sialoliths is present?

a. Torus
b. Wharton’s duct
c. Ranula
d. Exostosis

152. Which two locations in the mouth are high-risk areas for oral cancer?

a. Dorsal surface of tongue


b. Ventral surface of tongue
c. Lateral surface of tongue
d. Buccal mucosa

153. What condition of the mouth is caused by the Epstein-Barr virus?

a. Hairy leukoplakia
b. Black hairy tongue
c. Macroglossia
d. Median rhomboid glossitis

TOP © 2015 - 2023 Tests.com


34
154. MATCH the following classifications of decay with their location using G.V. Black’s Classification:

____ Class I a. Proximal surfaces, including incisal edges


____ Class II b. Proximal surfaces of anterior teeth
____ Class III c. Gingival 1/3 of facial or lingual surfaces
____ Class IV d. Occlusal and lingual pit and fissure
____ Class V e. Cusp tips
____ Class VI f. Proximal surfaces of posterior teeth

155. Which two of the following endogenous intrinsic stains exhibit white lesions?

a. Tetracycline
b. Amoxicillin
c. Fluorosis
d. Trauma

156. Which type of restorative overhang comprises 1/3 to 1/2 of the interproximal embrasure space
of a tooth?

a. Type I
b. Type II
c. Type III
d. Type IV

157. Which of the following is NOT a caries risk factor?

a. Xerostomia
b. Radiation therapy
c. Low income
d. Fewer than 15 teeth

158. Which two of the following are signs of chronic gingival inflammation?

a. Tough, fibrous gingiva


b. Soft, spongy gingiva
c. Blunted interdental papilla
d. Swollen interdental papilla

TOP © 2015 - 2023 Tests.com


35
159. During a periodontal screening and reporting (PSR), the deepest measurement in one quadrant
extends only onto the first clear band of the periodontal probe, with calculus present and
bleeding upon probing. What would the proper PSR score be for this quadrant?

a. 0
b. 1
c. 2
d. 3

160. Which classification of tooth mobility exists when the tooth is depressible?

a. Class III
b. Class II
c. Class V
d. Class IV

161. Which plaque score indicates gross soft debris within gingival pockets and/or margins?

a. 0
b. 3
c. 2
d. 4

162. Which classification of furcation involvement indicates through and through involvement of
bone loss between the roots, with an opening covered by the gingiva?

a. Class IV
b. Class II
c. Class III
d. Class I

163. What is the vibration felt in the tooth during occlusion?

a. Fremitus
b. Tribismus
c. Furcation
d. Percussion

TOP © 2015 - 2023 Tests.com


36
164. Which of the following is not a factor that affects the setting of gypsum products?

a. Temperature of water
b. Exposure to light
c. Amount of water used
d. Humidity

165. What is the level of marginal gingiva apical to the cementoenamel junction that can be
measured with a periodontal probe?

a. Clinical attachment level


b. Recession
c. Gingival pocket
d. Periodontal pocket

166. Which bacteria is the main etiological cause for the initiation of coronal lesions?

a. Staphylococcus
b. Lactobacilli
c. Streptococcus mutans
d. Sucrose

167. In regards to radiation wavelengths, select the two appropriate answers regarding wave
penetration.

a. Longer wavelengths are less frequent and produce less penetrating x-rays
b. Shorter wavelengths are more frequent and produce more penetrating x-rays
c. Longer wavelengths are more frequent and produce more penetrating x-rays
d. Shorter wavelengths are less frequent and produce less penetrating x-rays

168. When an x-ray photon intermingles with water and oxygen molecules in body tissue, the result
is ____________.

a. Compton effect
b. Photoelectric effect
c. Scatter radiation
d. Free radicals

TOP © 2015 - 2023 Tests.com


37
169. What is the safest area to stand in relation to a primary x-ray beam about to be exposed on a
patient?

a. 4 feet away at a 90-degree angle from the beam


b. 5 feet away at a 180-degree angle from the beam
c. 6 feet away at a 90-135-degree angle from the beam
d. 3 feet away at a 90-135-degree angle from the beam

170. What is the maximum permissible dose of radiation that a dental professional should receive per
year?

a. 0.5 rem
b. 0.05 rem
c. 5.0 rem
d. 5.0 Sv

171. What is the lethal dose of radiation that would cause death in 50% of the population within 30
days?

a. 2.0 Sv
b. 20.0 rem
c. LD 30/50
d. LD 50/30

172. Which portion of the x-ray tubehead performs collimation?

a. PID
b. Aluminum filter
c. Focusing cup
d. Target

173. Regulations and federal law restrict the size of PID and tubehead devices. Which of the following
options is NOT accurate?

a. Aluminum filters must be no less than 1.5 mm thick for exposures below 70kVp
b. Circular tubeheads must be no less than 2 3/4” in diameter
c. Rectangular tubeheads must be no less than 1 3/8 by 1 3/4” in diameter
d. PID lengths must be no less than 6”

TOP © 2015 - 2023 Tests.com


38
174. _____________ would result in a better contrast of grey shades in the final radiograph.

a. Decreasing mA
b. Increasing exposure time
c. Increasing kVp
d. Increasing mA

175. What radiology technique places the x-ray beam perpendicular to the imaginary line between
the tooth and the film?

a. Bisecting technique
b. Parallel technique
c. XCP technique
d. None of the above

176. What angulation is important to direct between interproximal areas of the teeth so as not to
cause overlapping images on the film?

a. Horizontal
b. Vertical
c. Bisecting
d. Paralleling

177. When attempting to locate the position of a foreign substance or pathology, an x-ray film is
taken and then retaken at a _______ change toward the mesial. If the object moves distally on
the film, then the object is located on the _______ side of the teeth.

a. 40-degree; buccal
b. 40-degree; lingual
c. 20-degree; buccal
d. 20-degree; lingual

178. A _________ object to film distance and ________ source to film distance decreases
magnification of the image.

a. Shorter; Shorter
b. Longer; Longer
c. Shorter; Longer
d. Longer; Shorter

TOP © 2015 - 2023 Tests.com


39
179. Identify the radiolucency extending through the mandible.

a. Hylohyoid ridge
b. Mandibular canal
c. External oblique ridge
d. Inferior border of mandible

180. Identify the radiopaque area.

a. Cementum
b. PDL
c. Lamina Dura
d. CEJ

181. Identify the bilateral radiolucencies.

TOP © 2015 - 2023 Tests.com


40
a. Hyperostotic bone
b. Alveolar crest
c. Trabecular bone
d. Mental foramen

182. Identify the localized interproximal radiopacities:

a. Subgingival calculus
b. Alveolar crest
c. Cervical burnout
d. Cementoenamel junction

TOP © 2015 - 2023 Tests.com


41
183. Identify the radiopaque structure.

a. Maxillary nasal sinus border


b. Zygomatic arch
c. Nasal sinus
d. Ghost of jewelry worn during x-ray exposure

184. Identify the anatomical structure:

a. Anterior foramen
b. Lingual foramen
c. Apical foramen
d. Trabecular bone

TOP © 2015 - 2023 Tests.com


42
185. Identify the following radiopaque anatomical structure:

a. Median suture
b. Nasopalatine foramen
c. Anterior nasal spine
d. Superior foramina

186. Which of the following statements is NOT accurate regarding the following radiograph?

a. The film was placed into the mouth backward


b. Inaccurate horizontal angulation was used
c. The film is mounted upside down
d. The film is positioned at the proper posterior position for a molar PA

TOP © 2015 - 2023 Tests.com


43
187. MATCH the stage of anesthesia with the conditions that it results in

___ Stage I a. Unconscious, regular respiration and pulse


___ Stage II b. Conscious, reduced pain
___ Stage III c. Respiratory arrest
___ Stage IV d. Loss of consciousness, irregular respirations

188. Which of the following conditions would be a contraindication for Nitrous Oxide use in a
patient?

a. Cerebral palsy
b. Strong gag reflex
c. Cardiovascular disease
d. Multiple sclerosis

189. What injection will anesthetize the upper central incisors and canines?

a. Infraorbital nerve
b. Anterior superior alveolar nerve
c. Middle superior alveolar nerve
d. Greater palatine

190. Which injection would anesthetize the palatal tissue of maxillary anterior teeth?

a. Nasopalatine
b. Greater palatine
c. Infiltration
d. Lingual block

191. Local anesthetic causes cell permeability to decrease, preventing ______ from entering into the
nerve cell.

a. chloride
b. calcium
c. sodium
d. potassium

TOP © 2015 - 2023 Tests.com


44
192. What systemic condition will result in pale mucosa, angular cheilosis and glossitis?

a. Hemophelia
b. Leukemia
c. Sickle-cell disease
d. Anemia

193. When treating a patient with a new cardiac pacemaker, which of the following precautions
should be taken?

a. Avoid the use of piezoelectric scalers


b. Avoid the use of magnetostrictive ultrasonic scalers
c. Avoid the use of all electronic scalers
d. None of the above

194. Which condition would be treated with calcium channel blockers, causing possible gingiva
hyperplasia?

a. Muscular Dystrophy
b. Anemia
c. Cardiovascular disease
d. Diabetes

195. Which benign cancer originates in the epithelium?

a. Papilloma
b. Carcinoma
c. Sarcoma
d. Lipoma

196. What should be done prior to administering a breath to the unconscious patient that is not
breathing?

a. Check the airway for obstruction


b. Listen for a breath
c. Check for circulation
d. Administer chest compressions

TOP © 2015 - 2023 Tests.com


45
197. A patient experiences anaphylaxis during a dental procedure. Which of the following is NOT an
appropriate care method for this patient?

a. Terminate the treatment and activate EMS


b. Administer epinephrine
c. Place the patient in an upright position
d. Place the patient in a supine position

198. A diabetic patient arrives having just taken their insulin, but only ate a very light lunch due to
having a dental procedure scheduled. During the appointment the patient begins experiencing
headache and shakiness. What should the practitioner do?

a. Administer nitrous oxide


b. Wait until the patient feels better to continue the treatment
c. Ask the patient to take more insulin
d. Have the patient drink a soda

199. A patient who reclines in the chair for treatment soon begins to convulse and loses
consciousness. What should the practitioner do first?

a. Sit the patient up


b. Place a prop in the mouth to prevent laceration of the tongue
c. Remove equipment from within reach of the patient
d. Activate EMS

200. Which of the following bodily fluids is not considered to be a contamination source?

a. Urine
b. Saliva
c. Sweat
d. None of the above. All are considered sources of contamination.

201. Face masks should be changed:

a. Whenever visibly soiled


b. At least twice per day
c. As often as gloves are changed
d. After each patient

TOP © 2015 - 2023 Tests.com


46
202. What type of solution is shown to break down latex in gloves?

a. Alcohol-based hand sanitizer


b. Antimicrobial soap
c. Petroleum based creams
d. Lotion

203. Which type of sterilization machine is able to perform a complete sterilization cycle in less than
10 minutes?

a. Statims
b. Chemical vapor
c. Steam autoclave
d. Cold sterilization

204. When presenting a treatment plan to a patient, which of the following is NOT appropriate for
the dental hygienist to do upon assessing the patient?

a. Explain risks associated with not completing the treatment


b. Present a dental hygiene treatment plan and inform the patient of how many
appointments are needed
c. Prioritize patient’s periodontal health needs
d. Present a restorative treatment plan and inform the patient of how many appointments
are needed

205. A patient is informed of their dental treatment plan. The plan is then signed by the dentist,
hygienist and patient. This document becomes a part of the patient’s permanent file and is
called __________________.

a. Informed consent
b. Documented consent
c. Treatment plan
d. Treatment agreement

206. Which of the following is an essential part of a patient’s needs assessment documented by the
dental hygienist?

a. General assessment
b. Periodontal assessment
c. Dentition assessment
d. All of the above

TOP © 2015 - 2023 Tests.com


47
207. Factors that determine the prognosis of a completed dental hygiene treatment plan may NOT be
influenced by:

a. Quality of existing restorations


b. Malocclusion
c. The patient’s financial situation
d. Degree of attachment loss

208. Notes regarding the treatment plan, if documented in the margin of a patient’s file, should be
written in _______.

a. Pencil
b. Blue ink
c. Typed
d. Whited out and then documented in the body of the chart with ink.

209. A new patient scheduled for a comprehensive exam and prophylaxis states on her medical
history form that she has undergone a knee replacement in the last six months. She has not
recently spoken with her primary care physician or surgeon. Which of the following would be an
appropriate part of the care plan?

a. Perform the exam and then a prophylaxis if there is a low plaque index
b. Perform the exam and then reschedule the prophylaxis to allow for antibiotic
premedication
c. Do not perform any exam and reschedule the patient to allow for antibiotic
premedication
d. Call her primary care physician to see if it is ok to perform a prophylaxis

210. A patient presents with stage II Hypertension and a blood pressure rate of 165/110. The blood
pressure is then rechecked five minutes later and measures at the same level. What is the next
step of the treatment assessment or plan that should be performed in this case?

a. Perform a prophylaxis, with a written medical referral


b. Refer patient to their medical doctor for a consultation prior to prophylaxis
c. Review history of blood pressure and if it has been low at previous visits then a
prophylaxis is appropriate
d. Emergency medical referral, no dental care whatsoever

211. Digital palpation should not be used on ___________.

a. Hard palate
b. Alveolar ridges
c. Soft palate
d. Buccal mucosa

TOP © 2015 - 2023 Tests.com


48
212. Assessing a treatment plan should be based on using all of the following areas EXCEPT:

a. Eliminating disease
b. Affordability
c. Slowing disease progression
d. Maintaining and promoting overall health

213. Case presentations should:

a. Not include the length of appointments


b. Be based only on the practitioner’s diagnosis without patient preference in regard to
therapy
c. Allow the patient to decline care
d. List causative factors described in scientific, professional terminology

214. A new patient arrives for their dental appointment. Which of the following is most appropriate
in regard to assessing the patient?

a. Take necessary x-rays; hygienist performs intraoral and extraoral exams; perform
prophylaxis if gum health is stable
b. Take necessary x-rays; hygienist performs intraoral exam and periodontal charting; begin
prophylaxis if appropriate
c. Patient has dental needs assessed by the dentist; take necessary x-rays; hygienist
performs intraoral and extraoral exam and prophylaxis
d. Hygienist performs intraoral and periodontal exam; prophylaxis; dentist performs needs
assessment; take necessary x-rays

215. A recall patient comes in for his scheduled biannual prophylaxis. During the appointment the
RDH notices that #30 has moderate bone loss and class II mobility. There are no other significant
findings and the patient is rescheduled for their next preventive care appointment with no
mention made of #30. At this appointment the tooth has 2 more mm of bone loss and class III
mobility. Which of the following statements is correct?

a. The RDH may be guilty of supervised neglect


b. The RDH should place Arestin after scaling #30 and then have the patient back for an
exam
c. The patient should have a periodontal scaling and root planing procedure
d. The patient should have a full mouth debridement

TOP © 2015 - 2023 Tests.com


49
216. A patient with no systemic health risks receives nonsurgical periodontal therapy. One year later
they exhibit good oral health with no bleeding on probing. The patient’s next recare
appointment should be scheduled in:

a. 3 months
b. 6 months
c. 14 months
d. Every 3-4 months for the duration of their care

217. Prognosis of a patient’s periodontal health does NOT include:

a. Race
b. Socioeconomic status
c. Systemic health
d. Age

218. A temporary crown placed on a tooth following an RCT procedure is marked in the dental chart
as:

a. Solid blue area marked with “SSC”


b. Crown restoration marked with a “T”
c. Hashed crown in blue pencil
d. Red crown

219. A diastema between #8 and 9 is marked by:

a. One vertical red line between 8/9


b. Two vertical red lines between 8/9
c. One “v” drawn between 8/9
d. Two horizontal red lines between 8/0

220. A patient presents with rampant caries. The RDH discusses risk factors such as diet, medication
and other areas that may cause concern. Which of the following steps is NOT part of the ideal
care plan for this patient?

a. More frequent bitewing radiographs


b. 6 month recare appointments
c. Home fluoride supplementation
d. Modification of oral hygiene practices

TOP © 2015 - 2023 Tests.com


50
221. Which of the following is NOT associated with a higher rate of tooth decay?

a. GERD
b. Frequent soda exposure
c. Xerostomia
d. Not brushing with fluoride toothpaste

222. Which diagnostic method of decay may NOT be appropriate for coronal smooth surface lesions?

a. Bitewing x-rays
b. DIAGNOdent
c. QFL
d. Clinical examination

223. Early childhood caries may NOT be associated with:

a. Breastfeeding
b. Use of bottled water in bottle preparation
c. Frequent juice or milk consumption
d. Putting child to bed with spill proof cup of milk

224. An enlargement of the gingiva in a coronal direction with no apical migration of the junctional
epithelium is called:

a. Clinical attachment level


b. Mucogingival defect
c. Pseudopocket
d. Periodontal pocket

225. What classification of furcation involvement exists when the periodontal probe can be placed
into the furcation but not extend through to the other side?

a. Class I
b. Class II
c. Class III
d. Class IV

226. When may a patient be referred to a periodontist?

a. When the hygienist decides the SCRP needs are too advanced
b. When the hygienist sees no improvement in SCRP therapy
c. When the dentist decides other therapies are not responding adequately
d. When the patient has not complied with the hygiene treatment plan of the RDH

TOP © 2015 - 2023 Tests.com


51
227. Which brushing method is a good first method for introducing to young children but not for
adults?

a. Bass method
b. Fones method
c. Charter’s method
d. Modified Stillman’s method

228. Which floss is the best option for patients with a tight contact area?

a. Waxed
b. Unwaxed
c. Tufted
d. Toothpick

229. Which device would be most efficient at removing plaque from an interproximal concavity area
with recession?

a. Superfloss
b. Electric toothbrush
c. Manual toothbrush
d. Interdental brush

230. Rubber cup polishing should be used on a selective basis to remove:

a. Black line stain


b. Dental fluorosis
c. Amelogenesis imperfecta discoloration
d. Extrinsic tetracycline staining

231. Which stain has an etiology of ingestion of disturbance of the ameloblasts?

a. Orange
b. Brown
c. Dentinogenesis imperfecta
d. Hypoplasia

TOP © 2015 - 2023 Tests.com


52
232. A patient has heavy brown stain from tobacco use. Which of the following conditions would
contraindicate the use for an air polisher?

a. Diabetes
b. High blood pressure
c. Pacemaker
d. Ovesity

233. Which mouthwash has an extended release that produces a therapeutic effect against bacteria?

a. Fluoride rinses
b. Phenol-containing rinses with essential oils
c. Chlorhexidine gluconate
d. Hydrogen peroxide

234. Failing to remove dental prosthesis for routine cleansing may be harmful to the patient because
it can result in:

a. Excess stain buildup


b. Advanced bone loss
c. Weakening of the appliance
d. Candida albicans infections

235. When scaling an upper posterior implant, which of the following curette tip materials would
NOT be appropriate?

a. Stainless steel
b. Plastic
c. Graphite
d. Nylon

236. Which type of electronic scaler uses air-pressure to create mechanical vibrations?

a. Magnetostrictive
b. Sonic
c. Piezoelectric
d. Air-polisher

TOP © 2015 - 2023 Tests.com


53
237. Which of the following Gracey area-specific curettes would be appropriate to use on the mesial
of posterior teeth?

a. 3/4
b. 7/8
c. 11/12
d. 13/14

238. Which ultrasonic scaler has active surfaces only on the lateral edges?

a. Piezoelectric
b. Magnetostrictive
c. Sonic
d. Ultrasonic

239. Which type of pocket is generated by an apical migration of the junctional epithelium?

a. Periodontal
b. Gingival
c. Suprabony
d. False

240. Which type of bony defect has the worst prognosis?

a. Two-wall defect
b. Three-wall defect
c. One-wall defect
d. All have the same prognosis

241. Which of the following conditions is reversible and does not cause loss of attachment fibers on
the cementum?

a. Mild periodontal disease


b. Gingivitis
c. Moderate periodontal disease
d. All periodontal diseases

242. Which chronic disease appears as an erosive white area on the gingival mucosa?

a. Desquamative gingivitis
b. Pemphigoid
c. Lichen planus
d. ANUG

TOP © 2015 - 2023 Tests.com


54
243. Which condition can be a result of an allergic reaction?

a. Pemphigus vulgaris
b. Necrotizing ulcerative gingivitis
c. Aggressive periodontitis
d. Desquamative gingivitis

244. A patient presents with 5-7 mm pockets with red inflammation of the soft tissue. What
classification of disease is present?

a. Aggressive periodontitis
b. Moderate chronic periodontitis
c. Advanced chronic periodontitis
d. Slight chronic periodontitis

245. What periodontal condition has gingival tissues that appear normal, with little plaque, and
usually occurs around first permanent molars and the permanent incisors?

a. Localized chronic periodontitis


b. Aggressive periodontitis
c. Juvenile periodontitis
d. Rapid progressive periodontitis

246. What periodontal condition appears as punched out papillae covered in a pseudomembrane?

a. Periodontal abscess
b. Necrotizing ulcerative periodontitis
c. Prepubertal periodontitis
d. Advanced chronic periodontitis

247. Removal of diseased pocket epithelium during instrumentation is called _________.

a. Curettage
b. Debridement
c. Scaling
d. Root planing

TOP © 2015 - 2023 Tests.com


55
248. The American Academy of General dentistry now advises that subgingival antibiotic therapy be
administered in periodontal pockets of _____ deep.

a. 3-5 mm
b. 4-6 mm
c. 5-6 mm
d. 7-8 mm

249. Which site-specific antibiotic works over a two-week period?

a. Minocycline
b. Doxycycline
c. Tetracycline
d. Chlorhexidine

250. What surgical intervention corrects bony defects to reshape the alveolar process?

a. Mucogingival surgery
b. Osseous surgery
c. Excisional surgery
d. Periodontal flap surgery

251. What incisional surgery exposes underlying bone and removes diseased tissues, including
calculus removal?

a. Excisional surgery
b. Mucogingival surgery
c. Periodongal flap surgery
d. Periodontal surgery

252. MATCH the following types of bone grafts with the source of the bone:

___ Autograft a. Cadaver bone


___ Allograft b. Synthetic bone
___ Xenograft c. Bone taken from the patient
___ Alloplast d. Cow or pig bone

TOP © 2015 - 2023 Tests.com


56
253. What procedure involves removal of half of the tooth, including a portion of both the crown and
root?

a. Root bisection
b. Root resection
c. Root hemisection
d. Apicoectomy

254. What is the material of choice for dental implant fabrication?

a. Stainless steel
b. Ceramic
c. Porcelain
d. Titanium

255. Approximately _____ of bacteria that causes maloder of the breath originates in the oropharynx
region.

a. 10%
b. 25%
c. 90%
d. 100%

256. What is the most important risk factor related to periodontal disease?

a. Genetic predisposition
b. Diabetes
c. Plaque index
d. Tobacco use

257. Which hormonal condition has NOT been proven to increase symptoms of gingivitis?

a. Menstruation
b. Puberty
c. Pregnancy
d. Use of oral contraceptives

258. Most periodontal pathogens are __________ and _________.

a. gram negative, nonmotile


b. gram negative, motile
c. gram positive, nonmotile
d. gram positive, motile

TOP © 2015 - 2023 Tests.com


57
259. Which of the following clinical symptoms is a sign of occlusal trauma?

a. Widening of the PDL


b. Wear facets
c. Tilted teeth
d. All of the above

260. The first fibers lost during periodontal disease are the periodontal ligament fibers. Transseptal
fibers become involved at the approach of moderate bone loss.

a. Both statements are true


b. Both statements are false
c. The first statement is true, the second statement is false
d. The first statement is false, the second statement is true

261. A narrow, slit like cleft in the marginal gingiva is called a ________ cleft.

a. Stillman’s
b. McCall’s
c. Bass
d. Recession

262. What is the most reliable sign to detect inflammation in the periodontal pocket?

a. Erythema
b. Edema
c. Bleeding on probing
d. McCall’s festoons

263. What is the first sign on a radiographic assessment that there is a periodontal breakdown?

a. Horizontal bone loss


b. Vertical bone loss
c. Enlarged PDL
d. Fuzziness of the alveolar crests

TOP © 2015 - 2023 Tests.com


58
264. Which fluoride should not be applied to people with composite or porcelain restorations?

a. Varnish
b. Acidulated phosphate
c. Neutral sodium
d. No fluoride should be used

265. Fluoride varnish contains ______% of ______ fluoride.

a. 5; sodium
b. 2; sodium
c. 2 neutral
d. 0.4; stannous

266. What is the optimal level of fluoride in a city water supply?

a. 0.1 ppm
b. 0.01 ppm
c. 1 ppm
d. 10 ppm

267. A lethal dose of fluoride is how many grams?

a. 4g
b. 5-10g
c. 15-20g
d. Over 25g

268. What is the toxic dose of fluoride per kg for a child?

a. 1mg
b. 5mg
c. 8mg
d. 10mg

269. Which of the following is NOT true in regard to fluoride?

a. It may cause birth defects


b. Topical application does not cause fluorosis
c. Fluorosis can occur in the bone
d. Toxicity may cause cardiac arrest

TOP © 2015 - 2023 Tests.com


59
270. When placing sealants, acid etch should be left on the surface of the tooth for ___________.

a. 10-15 seconds
b. 15-60 seconds
c. 60-90 seconds
d. 2 minutes

271. Self-cured sealants usually set within _______.

a. 30 seconds
b. 45 seconds
c. 2 minutes
d. 5 minutes

272. Which is NOT appropriate for tooth preparation when placing a sealant?

a. Air polishing
b. Rubber cup polish with prophy paste
c. Rubber cup polish with pumice
d. Prophy brush polish with pumice

273. If a tooth becomes wet due to saliva contact during sealant preparation, what is the next correct
step?

a. Use air/water syringe to re-rinse and dry the tooth


b. Use air/water syringe to dry the tooth
c. Re-etch for the full amount of time
d. Re-etch for a shorter amount of time

274. Which is NOT true in regard to sealant material?

a. It may be clear or white


b. It may be tinted
c. It may consist of filled resins
d. It must be gently dabbed on the tooth.

275. Light curing typically occurs within ________.

a. 15-seconds
b. 20-30 seconds
c. 45-60 seconds
d. 90-seconds

TOP © 2015 - 2023 Tests.com


60
276. Which type of sealant material will wear down naturally and does not require occlusal
adjustment?

a. Filled resin
b. Unfilled resin
c. Tinted
d. Autopolymer

277. Which type of sealant will release fluoride, protecting the tooth by aiding in remineralization?

a. Calcium phosphate
b. Hydrophobic
c. Unfilled resin
d. No sealants release fluoride

278. Which nerve fibers are thought to conduct rapid hypersensitivity?

a. A-delta
b. B-delta
c. C-delta
d. D-delta

279. Polishing the teeth for at least ____ with pumice may remove 3-4 micrometers of enamel.

a. 1 minute
b. 3 minutes
c. 5 minutes
d. 7 minutes

280. Bristle brushes may be used for stain removal on:

a. Occlusal surfaces
b. Gingival margins
c. Exposed dentin
d. Facial surfaces

281. Once an amalgam filling is placed, polishing must wait _______ before performed on the tooth.

a. 30 minutes
b. 4 hours
c. 24 hours
d. 48 hours

TOP © 2015 - 2023 Tests.com


61
282. Glass ionomers are typically placed for Class V restorations and may seep fluoride out of the
final restoration for how long after placement?

a. 6 months
b. 12 months
c. 2 years
d. 5 years

283. What type of cement is widely used in dentistry for bonding, temporary fillings and temporary
cement?

a. Glass ionomer
b. Zinc phosphate
c. Zinc polycarboxylate
d. Zinc oxide and eugenol

284. What is a common material used for impression and is an irreversible hydrocolloid?

a. Silicone
b. Agar hydrocolloid
c. Alginate hydrocolloid
d. Polysulfide

285. What controls the set time for an alginate impression?

a. Temperature of the water


b. Temperature of the room
c. Age of the material
d. All alginate sets at the same rate

286. What model material is used to make casts and dies of the teeth?

a. Alginate
b. Plaster
c. Stone
d. Die Stone

TOP © 2015 - 2023 Tests.com


62
287. Study models typically set within __________.

a. 30 minutes
b. 45-60 minutes
c. 1 1/2 − 2 hours
d. 4 hours

288. When mixing gypsum products for study models, what ratio of powder to water is used?

a. 1:1
b. 2:3
c. 3:2
d. 2:1

289. What is defined as the absence of prudent care that a patient should receive in the dental office?

a. Malpractice
b. Negligence
c. Fraud
d. Intentional torts

290. Training dental personnel to perform dental hygiene duties is known as ___________.

a. Preceptorship
b. Extended duties
c. Fraud
d. On the job training

291. Which of the following is NOT true regarding HIPAA?

a. Electronic health information is included in the privacy rules


b. Patient’s have access to their medical records
c. Group health plans cannot deny an application based on health status
d. It does not protect workers who lose their jobs

292. What level of care would periodontal treatment be considered to be a part of?

a. Tertiary level of prevention


b. Primary level of prevention
c. Secondary level of prevention
d. Moderate level of prevention

TOP © 2015 - 2023 Tests.com


63
293. What two answers describe the deliberate misdirection of the truth to deceive a person such as
a patient?

a. Defamation
b. Fraud
c. Misrepresentation
d. Malpractice

294. When a patient is given a treatment plan and chooses not to have the treatment completed, it is
called:

a. Informed refusal
b. Informed consent
c. Refusal of treatment
d. Neglect

295. A signature on the treatment plan agreeing to undergo the proposed dental treatment is called
______.

a. Documented treatment plan


b. A legal contract
c. Informed consent
d. Documentation

296. What act is the actual infliction of harm to a person?

a. Assault
b. Battery
c. Negligence
d. Unintentional tort

297. What type of contract does not express all of the terms in words?

a. Implied contract
b. Express contract
c. Legal contract
d. Partial contract

TOP © 2015 - 2023 Tests.com


64
298. What type of defamation is written?

a. Slander
b. Personal slander
c. Libel
d. Misrepresentation

299. Contract law is a form of ______ law.

a. Civil
b. Tort
c. Criminal
d. Judicial

300. What judicial measure is used to determine whether a dental provider has exercised reasonable
care to his or her patient?

a. Negligent practitioner measure


b. Reasonably prudent practitioner measure
c. Malpractice measure
d. Licensed practitioner measure

301. What type of consent is given in writing or verbally?

a. Informed refusal
b. Informed consent
c. Implied consent
d. Expressed consent

302. In most states a malpractice suit must be filed within ______ of the time from when the
wrongful act was committed.

a. 1 year
b. 2 years
c. 5 years
d. 10 years

303. Which government agency oversees the use of MSDS sheets?

a. OSHA
b. HIPAA
c. CDC
d. ADA

TOP © 2015 - 2023 Tests.com


65
304. According to HIPAA, all of the following statements are correct EXCEPT:

a. Patients must have access to their records upon request within at least 30 days
b. An office may charge the patient a fee for duplicating their records
c. Patients may file complaints concerning their privacy to the US Dept. of Health and
Human Services
d. At least one staff member in each office must be trained on HIPAA protocols

305. Which factor is one of four that would make a dental hygienist liable for negligence?

a. Not undertaking the patient for care


b. Failing to breach a duty owed to the patient
c. Harm to the patient must be proved
d. Harm does not have to be related to breach of duty

306. Which two considerations can spare a dentist from judgment related to negligence?

a. Patient knowingly failing to inform the dentist of a heart condition


b. Patient not taking antibiotic premedication prescribed by their physician
c. Informed consent signed by both the patient and the doctor
d. The patient having all procedures completed by the dental hygienist

307. Parental consent must be:

a. Given for minors


b. Given for individuals that are not mentally competent
c. Written or verbal
d. All of the above

308. A patient that is experiencing angina should be given _________.

a. Tylenol
b. Nitroglycerin
c. Aspirin
d. NSAIDS

TOP © 2015 - 2023 Tests.com


66
309. What bacterium is used when testing the effectiveness of an autoclave?

a. Bacillus stearothermophilus
b. S. Mutans
c. Lactobacillus
d. Actinomyces

310. All of the following statements, except one, accurately describe the advantages of dry heat
sterilization. Which statement is not accurate as far as dry heat sterilization?

a. It does not corrode instruments.


b. It is used for instruments that cannot be autoclaved.
c. It is affective on oils and powders.
d. It is economical.

311. During CPR on an infant, what artery is used to take a pulse?

a. Carotid
b. Brachial
c. Radial
d. Femoral

312. What is a Naber’s probe used to measure?

a. Abfraction
b. Recession
c. Furcation
d. Periodontal pockets

313. Which tooth surface is least susceptible to plaque formation?

a. Maxillary palatal surfaces


b. Maxillary buccal posterior
c. Proximal surfaces
d. Mandibular lingual anterior

314. What are the two most common areas for supraginvial calculus?

a. Maxillary lingual anterior and mandibular buccal posterior


b. Maxillary buccal posterior and maxillary lingual posterior
c. Maxillary buccal posterior and mandibular lingual anterior
d. Maxillary lingual posterior and mandibular lingual anterior

TOP © 2015 - 2023 Tests.com


67
315. What type of toothbrush should be recommended to a patient with 3rd molars, rotated teeth
and furcation involvement?

a. Rubber tip
b. End-tufted
c. Interdental
d. Sulcabrush

316. What is the most widely used and accepted tooth brushing technique?

a. Charters
b. Stillmans
c. Bass
d. Fones

317. What is the active ingredient in anti-calculus toothpaste?

a. Stannous fluoride
b. Sodium fluoride
c. Potassium nitrate
d. Pyrophosphate

318. All of the following are reasons why the use of hydrogen peroxide is contraindicated in regular
use, except:

a. Sponginess of gingiva
b. Root hypersensitivity
c. Staining
d. Demineralization

319. What is the recommended immersion time of instruments using the liquid chemical
glutaraldehyde as the solution?

a. 10 hours
b. 30 minutes
c. 5 hours
d. 1 hour

TOP © 2015 - 2023 Tests.com


68
320. The facial profile of a Class II occlusion is mesognathic. The facial profile of a Class I occlusion is
retrognathic.

a. Both statements are true


b. Both statements are false.
c. First state statement is true. Second statement is false.
d. First statement is false. Second statement is true.

321. When the alveolar bone fuses directly to the root surface of a tooth:

a. Ankylosis
b. Resportion
c. Periodontal disease
d. Infraverted

322. What is the term used for the phenomenon or natural tendency of teeth to move out toward the
lips?

a. Distal drift
b. Mesial drift
c. Lingual drift
d. Buccal drift

323. Which statement about bioburden is not accurate?

a. It is bacteria on a contaminated instrument.


b. Saliva is not considered bioburden.
c. Blood on an instrument is considered bioburden.
d. Saliva on a reusable prophy angle is considered bioburden.

324. Which of the following is not considered a semi-critical instrument in regards to infection
control?

a. Dental bur
b. Dental mirror
c. Amalgam condenser
d. Headpiece

TOP © 2015 - 2023 Tests.com


69
325. As a cleaning agent, alcohol is effective against all of the following, except:

a. Bacterial spores
b. Gram-positive bacteria
c. Hepatitis B
d. Fungus

326. Where should intermediate disinfectants be used?

a. Dental instruments
b. Floor
c. Dental chair
d. Hand piece

327. When looking at a patient’s mouth from a lateral view, the cusp of the mandibular posterior
teeth and the incisal edge of anterior teeth form a(n) __________.

a. Centric occlusion
b. Ideal occlusion
c. Malocclusion
d. Curve of Spee

328. Which of the following may cause an anterior open bite?

a. Thumb sucking
b. Tongue trust
c. Ankyloglossia
d. All of the above

329. A tooth that is labial to its normal position is considered to be ____________.

a. Labialverted
b. Lingualverted
c. Infraverted
d. Buccoverted

330. What tooth is used to determine occlusal classification if there are no first molars present?

a. Second molars
b. Canine
c. Laterals
d. Permolars

TOP © 2015 - 2023 Tests.com


70
331. In terms of ergonomics, what is the correct sequence of attention when a dental hygienist
establishes her working position? Put the following in the correct order.

a. The patient in a dental chair


b. Dental hygienist’s position in relation to the dental chair
c. Dominant hand
d. The dental chair light
e. Non-Dominant hand

332. Which of the following is not considered part of the neutral body position?

a. Head tilted 1-15 degrees


b. Tip of patients nose above the clinician’s elbow
c. Flat feet on the floor
d. Shoulders relaxed

333. What is the correct clock seated position for the right/left hand operator?

a. 8:00-1:00/11:00-4:00
b. 9:00-2:00/12:00-5:00
c. 7:00-12:00/10:00-3:00
d. 6:00-11:00/9:00-2:00

334. When an operatory uses incorrect ergonomics, he or she is at risk for what type of injury?

a. Musculoskeletal
b. Muscle
c. Skeletal
d. Nerve

335. There are a higher percentage of microbes in supragingival plaque. There is a lower percentage
of microbes in subgingival plaque.

a. Both statements are true.


b. Bother statements are false.
c. First statement is true. Second statement is false.
d. First statement is false. Second statement is true.

TOP © 2015 - 2023 Tests.com


71
336. What is the duration of an acidic attack in the mouth when a person eats or drinks acidic foods?

a. 5 minutes
b. 20 minutes
c. 1 hour
d. 30 minutes

337. What ethnicity has a higher prevalence of periodontal disease?

a. Caucasian
b. Hispanic
c. African-American
d. Pacific Islander

338. How can the endocrine system affect the periodontal health of a patient?

a. Heart disease
b. Diabetes mellitus
c. Decrease in liver function
d. It does not affect periodontal health.

339. Which of the following procedures can be conducted on a patient taking bisphosphonates?

a. Periodontal surgery
b. Root canal
c. Amalgam filling
d. Extraction
e. All of the above

340. A person that is HIV positive is more vulnerable to what type of oral disease?

a. Aggressive necrotizing periodontal disease


b. Angular chelitis
c. Cavities
d. Oral Cancer

341. Which of the following is not a systemic risk factor for periodontal disease?

a. Diabetes type II
b. Coronary heart disease
c. Geographic tongue
d. Asthma

TOP © 2015 - 2023 Tests.com


72
342. Which type of tissue provides a pathway from the mouth to the vascular system for bacteria?

a. Inflamed
b. Pedunculated
c. Keratinized
d. Ulcerated

343. What is the ideal angle of instrumentation for calculus removal?

a. 60-80 degrees
b. 0-40 degrees
c. 20-50 degrees
d. 40-60 degrees

344. The two types of activation when instrumenting include wrist and digital. Digital activation is
only used when removing calculus.

a. Both statements are true.


b. Both statements are false.
c. First statement is true. Second statement is false.
d. First statement is false. Second statement is true.

345. All of the following factor in stroke selection, except:

a. Position and size of gingiva


b. Pocket depth
c. Size and shape of instrument
d. Patient position

346. What type of stroke is used on the facial and lingual surfaces of anterior teeth?

a. Oblique
b. Horizontal
c. Vertical
d. Proximal

TOP © 2015 - 2023 Tests.com


73
347. When cleaning a denture, it is recommended that a cylinder shaped denture brush be used. A
regular toothbrush can also be used.

a. Both statements are true.


b. Both statements are false.
c. First statement is true. Second statement is false.
d. First statement is false. Second statement is true.

348. All of the following are advantages of using alkaline hypochlorite except:

a. Available as a liquid and a solid


b. Broad spectrum of antimicrobial activity
c. Removes biofilm with organic matter present
d. Removes stains

349. Which of the following is considered a secondary prevention of disease control?

a. Fluoride
b. Calculus removal
c. Mechanical/chemical biofilm control
d. Nutrition discipline

350. In what order is bacteria introduced into the caries process?

a. S. mutans, lactobacilli, actinomyces


b. S. mutans, actinomyces, lactobacilli
c. Actinomyces, lactobacilli, S. mutans
d. Lactobacilli, S. mutans, actinomyces

351. Toothbrush abrasion can cause recession. Bacterium does not cause recession.

a. Both statements are true


b. Both statements are false
c. First statement is true. Second statement is false.
d. First statement is false. Second statement is true.

352. What drug is used to treat acute necrotizing ulcerative gingivitis?

a. Penicillin
b. Tetracycline
c. Metronidazole
d. Clindamycin

TOP © 2015 - 2023 Tests.com


74
353. The two main types of exudate are serous and purulent. Both contain white blood cells.

a. Both statements are true.


b. Both statements are false.
c. First statement is true. Second statement is false.
d. First statement is false. Second statement is true.

354. What happens when a drug has a high first past effect?

a. The patient requires a smaller dose.


b. The patient requires a larger dose.
c. The patient should not be given the medication.
d. The patient should seek medical attention.

355. What is the teratogenic effect?

a. Relationship between maternal drug use and congenital abnormalities


b. Hypersensitivity response to a particular drug
c. Dose related reaction to a particular drug
d. Dose related reaction that is not part of a desired therapeutic outcome

356. What is contraindicated for a patient that is under or has been under the influence of cocaine in
the last 24 hours?

a. X-rays
b. Antibiotics
c. Epinephrine
d. Nothing in contraindicated

357. What does a patient known to be an intravenous drug user need to do for the dental treatment
of an infection?

a. Rinse with an anti-microbial rinse prior to dental treatment.


b. Take a premed of antibiotics.
c. Schedule an appointment when they are not using drugs.
d. Dental treatment is not advisable.

TOP © 2015 - 2023 Tests.com


75
358. Which premed cannot be given to a patient that is allergic to penicillin?

a. Clindamycin
b. Azithromycin
c. Amoxicillin
d. Cephalexin

359. Which is not a contraindication of nitrous oxide?

a. COPD
b. Pregnancy
c. Cystic Fibrosis
d. HIV

360. What is camouflage therapy?

a. Moving teeth orthodontically to mask malocclusion due to poor skeletal formation


b. A skeletal surgery to help alleviate malocclusion
c. A surgery that reduces the amount of chin projection
d. A surgery that requires the mouth to be wired shut in the desire occlusion

361. Which is not a common manifestation of orthodontic treatment?

a. Decalcification
b. Aphthous ulcers
c. Root resorption
d. Gingival hypoplasia

362. How does an injury occur during the x-ray process?

a. X-rays are absorbed by tissue resulting in biological damage.


b. Radiation is scattered and is absorbed by bone.
c. X-rays are absorbed by tissue causing a chemical burn.
d. No injury occurs.

363. When talking with a new dental assistant about x-rays, you discuss the principal that exposure
should be kept to a minimum and the concept of ALARA. What does the acronym ALARA stand
for?

a. As Low As Reasonably Acquired


b. As Long As Reasonably Acquired
c. As Low As Reasonably Achievable
d. As Long as Reasonably Achievable

TOP © 2015 - 2023 Tests.com


76
364. Your dentist asks you to take a periapical x-ray of a particular tooth. When exposing the x-ray,
you notice that the film is light, with little detail. What is the cause and correction?

a. The film is overexposed and the settings need to be checked and decreased.
b. Incorrect horizontal angulation and direct central ray through interproximal spaces.
c. The film was placed in the mouth backwards and the white side needs to face the PID.
d. The film was underexposed and the settings need to be checked and increased.

365. #8 needs to be infiltrated for scaling and root planing. Where is the topical anesthetic placed?

a. Near the mesial of the canine tooth, for a field block


b. In the vestibule, for a posterior superior alveolar nerve block
c. In the vestibule, for an anterior superior alveolar nerve block
d. In the vestibule, near the isolated nerve branch

366. When starting a new job in a dental office, you notice that the staff does not use surface barriers
and uses a chlorine based surface disinfectant. What is a disadvantage of using this type of
disinfectant?

a. Diluted solutions need to be prepared daily


b. Leaves a residual film
c. Unstable at high temperatures
d. Poor cleaning capabilities

367. What type of radiography is best for examining the temporomandibular joint?

a. Panoramic
b. Bitewing
c. Cephalometric
d. None

368. The dentist wants you to take a panoramic x-ray to evaluate a joint two dimensionally. What
type of x-ray is the panoramic?

a. Intraoral
b. Extraoral
c. Interoral
d. Periapical

TOP © 2015 - 2023 Tests.com


77
369. When taking a full-mouth series of x-rays, a dental assistant, who is shadowing you, asks why
you have to step out of the room for each individual x-ray taken. How should you respond?

a. To protect yourself from all types of radiation exposure


b. To protect yourself from scatter radiation
c. To be overly cautious, you don’t always have to leave the room
d. To press the button, which just happens to be outside of the operatory

370. A chair button, which is used when positioning a patient in a dental chair, does not have a plastic
barrier. What type of surface is the chair button considered?

a. Splash surface
b. Transfer surface
c. Spatter surface
d. Touch surface

371. What is the correct personal protective equipment you should wear when cleaning a treatment
room after use?

a. Latex gloves, goggles and mask


b. Goggles, mask and scrubs, face shield
c. Mask, goggles protective gown and rubber gloves
d. Non-latex gloves, protective gown, mask and goggles

372. After recording all of the periodontal pocket numbers, you check each tooth for mobility. On
#29, there is moderate mobility. What do you record in the chart?

a. The number 1
b. The number 2
c. The number 3
d. The number 4

373. When performing an intraoral exam, a dentist begins evaluating a patient’s bite. She notices that
the posterior teeth are touching, but the anterior lack vertical overlap. What is this referred to?

a. Cross-bite
b. Open-bite
c. Overjet
d. Crowding

TOP © 2015 - 2023 Tests.com


78
374. What type of material is alginate?

a. Irreversible Hydrocolloid
b. Reversible Hydrocolloid
c. Elastomeric
d. Polysulfide

375. If a patient has an allergic reaction to sealant material, what type of allergy do they have?

a. Polymers
b. Acrylate
c. Fluoride
d. Phosphoric acid

376. Once sealant material has been placed and checked for retention, you notice that one area of
sealant has a small bubble. What is the cause and solution?

a. Moisture contamination - need better isolation during placement of sealant.


b. Applied too much sealant material - sealant must be removed and replaced.
c. Vigorous scrubbing during application - add more sealant. Etch again if necessary.
d. Applied too much sealant material - needs to have bite adjusted.

377. What is the correct order of links in regards to the chain of infection?

a. Infectious agent, reservoir, mode of transmission, portal of entry and susceptible host
b. Infectious agent, reservoir, mode of transmission, portal of exit and susceptible host
c. Infectious agent, reservoir, portal of exit, mode of transmission, portal of entry and
susceptible host
d. Infectious agent, reservoir, mode of transmission and susceptible host

Refer to the following scenario to answer questions 378 - 380

A 55-year-old woman presents for her 1:00 pm dental treatment appointment. When discussing her
medical history, she states that she has been diagnosed with Type 2 diabetes and is taking an oral
medication for her diabetes.

378. What should you ask first?

a. Do you have an insulin pen with you?


b. When was your last meal?
c. When did you take your medication?
d. Do you have an inhaler?

TOP © 2015 - 2023 Tests.com


79
379. The woman states that she took her oral medication, but did not eat anything before her
appointment. You also notice that she is anxious and sweating. What is the patient
experiencing?

a. Asthma Attack
b. Hyperglycemia
c. Hypoglycemia
d. Hyperventilation

380. What should you do in this case?

a. Give the women a concentrated form of a sugar packet, such as cake icing.
b. Ask for the insulin pen and administer it yourself.
c. Ask for the inhaler, as she is hyperventilating.
d. Monitor and record vital signs.

Refer to the following scenario to answer questions 381 - 384.

You are setting up the sterilization room in the morning, before patients arrive. One of your tasks
includes filling the ultrasonic cleaner and performing a test to make sure the ultrasonic cleaner is
working.

381. How do you do this?

a. Fill the chamber, turn on the ultrasonic cleaner and listen for suspicious noises.
b. Do not fill the chamber, but turn on the ultrasonic and listen for suspicious noises.
c. Fill the chamber, put instruments inside the chamber, run the ultrasonic and use a piece
of aluminum foil to detect areas of tiny pebbling.
d. Fill the chamber, run the ultrasonic and use a piece of aluminum foil to detect areas
without the tiny pebbling

382. Your next job in the sterilization room is performing the daily testing on the steam autoclave.
Your autoclave is set to run 15 minutes. What temperature does the autoclave need to reach in
order to be effective in sterilization?

a. 250 degrees Fahrenheit


b. 273 degrees Fahrenheit
c. 273 degrees Celsius
d. 230 degrees Fahrenheit

TOP © 2015 - 2023 Tests.com


80
383. After performing the daily test on the steam autoclave, you are to run a weekly test on the
autoclave. Where do you place the biological indicator in the steam autoclave?

a. In the sterilizer near the top and run the sterilizer without instruments
b. In the sterilizer near the center and run the sterilizer without instruments
c. In the sterilizer near the bottom and run the sterilizer with instruments
d. In the sterilizer near the center and run the sterilizer with instruments

384. Once you finish the setup on the sterilization room, you get a phone call regarding the failure of
the autoclave’s weekly biological tests. What should you look for?

a. How many packages are in the autoclave at time of sterilization


b. Monitor cycle for correct temperature and time
c. Monitor cycle for correct temperature, pressure and time
d. Check for proper separation of packages

Refer to the following scenario to answer questions 385 - 388.

A 68 year-old male presents with swelling in his upper left area. The patient states that the swelling
started overnight and has progressively worsened throughout the day. He is in significant pain.

385. You are instructed to take an intraoral picture of the tooth. What is the purpose of taking a
picture with an intraoral camera?

a. Provides magnification to better evaluate and educate the patient


b. Allows for easier access to areas that are difficult to view
c. Both a and b
d. None of the above

TOP © 2015 - 2023 Tests.com


81
386. The dentist has assessed the tooth through x-rays and a series of testing. She has concluded that
#14, using the Universal Numbering System, needs a root canal. How many roots does tooth #14
usually have?

a. 4
b. 3
c. 2
d. 1

387. The dentist is going to do the root canal on #14 and will need to anesthetize the area. The
dentist asked you to place a topical anesthetic in the areas of injection. How long should the
topical anesthetic remain on the site of injection?

a. 15 seconds to 2 minutes
b. 10 seconds to 3 minutes
c. 1 minute to 5 minutes
d. 5 seconds to 1 minute

388. The dentist asked you to assemble the anesthetic syringe with septocaine. What needs to be
locked in on the syringe in order to allow aspiration?

a. Piston
b. Barrel
c. Needle adaptor
d. Harpoon

389. When x-rays of a patient are taken, who owns the images?

a. The patient. It is part of his dental record.


b. The patient, if he paid for them.
c. The dentist. It is part of the dentist’s dental records.
d. The insurance company, if it paid for them.

390. The dental office you work at is doing their yearly testing on all dental x-ray equipment. The
dentist puts you in charge of managing the testing of the equipment. Though you can perform
in-office testing on all of the x-ray equipment, which of the following needs to be done by a
qualified technician?

a. Test of the output of x-rays


b. Calibration
c. Testing kilo voltage
d. Testing focal point spot

TOP © 2015 - 2023 Tests.com


82
391. What is the main concern involving infection control in radiography?

a. Cross-contamination
b. Splatter contamination
c. Transfer surface contamination
d. Saliva contamination

392. A patient notes on a health history that he is allergic to latex. He also states that he breaks out in
a rash around his mouth when latex gloves are used. What class of allergic reaction is this?

a. Type I
b. Type II
c. Type III
d. Type IV

393. A dental practice is to create a latex free dental office to avoid allergic reactions in those
patients allergic to latex. Patients with latex allergies do not need to be scheduled for treatment
at any particular time of the day.

a. Both statements are true.


b. Both statements are false.
c. First statement is true. Second statement is false.
d. First statement is false. Second statement is true.

394. A 24-year-old woman presents to discuss teeth whitening options. After discussing options with
the patient, the dentist decides that the at-home treatment involving custom trays and peroxide
gel is the best option. What are some adverse effects that should be discussed with this patient?

a. Thermal sensitivity and tissue irritation


b. Tissue irritation and pressure sensitivity
c. Pressure sensitivity and thermal sensitivity
d. Chewing sensitivity and thermal sensitivity

395. You are updating a patient record by recording all of their previous restorations. You
accidentally write “composite filling” instead of “amalgam filling”. When correcting an entry in a
patient chart, you should:

a. Use a black pen and make a single line through the incorrect entry
b. Use a red pen and make a single line through the incorrect entry
c. Use a black pen and make a single line through the incorrect entry. Also, initial and date
d. Use a red pen and make a single line through the incorrect entry. Also, initial and date

TOP © 2015 - 2023 Tests.com


83
396. Which of the following is not a risk factor of periodontal disease?

a. Smoking
b. Diabetes
c. HIV/AIDS
d. Gingivitis

397. A 63-year-old woman comes into the office and requests all of her current x-rays. She is going to
another office and would like to take them with her. What is the first step that should be done
when giving a patient their x-rays?

a. The patient must give a 24 hour notice.


b. The patient needs to sign a release form.
c. The patient needs to sign a waiver of liability form.
d. The patient needs to sign a release form and give a 24-hour notice.

398. You are asked to duplicate x-rays for a patient transferring to another office. You have not
transferred to digital radiography and must duplicate the x-ray film manually. After going
through the necessary steps of duplicating and processing the x-rays, you notice that the images
are dark and cannot be used diagnostically. What most likely is the error that occurred?

a. You did not duplicate the films in a dark room. They are light sensitive.
b. You did not turn on the duplicating machine and produce the white light in order to
transfer the image.
c. When placing the duplicating paper on the x-rays, the emulsion side was facing up
instead of facing down towards the x-rays.
d. You did not process the films in the processor.

399. When duplicating x-rays, you need to be in a dark room. What type of infection control is
needed?

a. No PPE is needed
b. Latex gloves
c. Mask
d. Rubber gloves

TOP © 2015 - 2023 Tests.com


84
Refer to the following scenario to answer questions 400 - 401.

A 9-year-old child presents for a recall examination and prophylaxis with the dentist. The child’s
mother is concerned that the child is missing a permanent tooth, as he lost the baby tooth, but has
not had a permanent tooth erupt in its place.

400. The dentist asks for a panoramic film. Looking at the panoramic film, is the child missing a
permanent tooth?

a. Yes, he is missing #7
b. Yes, he is missing number #21
c. Yes, He is missing #30
d. No, he is not missing any permanent teeth

401. After the x-ray is processed, you notice that the x-ray is dark, especially around the areas of the
mandible? What is the error that was made?

a. The exposure time needs to be increased.


b. The exposure time, kVp and mA all need to be increased.
c. The kVp needs to be decreased.
d. The exposure time, kVp and mA all need to be decreased.

TOP © 2015 - 2023 Tests.com


85
402. All of the following patients have contraindications that would preclude the use of an air
polisher EXCEPT one. Which one is the exception?

a. A patient with exposed cementum


b. A patient on a sodium restricted diet
c. A patient with hypotension
d. A patient with respiratory issues

403. A 55-year-old woman presents for a comprehensive exam with x-rays. After going over the
health history with your patient, you begin to take her vital signs. After taking your patient’s
pulse, you begin to take the blood pressure with a stethoscope and manual blood pressure cuff.
The patient’s reading is 142/82. What does the first number mean in a blood pressure reading?

a. Systolic
b. Diastolic
c. Korotkoff sounds
d. Blood pressure

404. When a procedure is completed and you are disinfecting your operatory for the next patient,
what is done with the used needles from the procedure?

a. Capped and placed in a puncture resistant, red container label for hazardous material
b. Capped and placed in a puncture resistant, red container label for biohazard material
c. Uncapped and placed in a puncture resistant, blue container label for hazardous material
d. Uncapped and placed in a puncture resistant, blue container label for biohazard material

405. Who creates the guidelines for all hazardous and infectious material in the dental office?

a. OSHA
b. CDC
c. ADA
d. HCS

406. You are about to set up the dental operatory for your first patient of the day. What should you
do before the patient is seated to reduce bacterial contamination of the dental unit waterline?

a. Suction 1 full cup of water before the first patient of the day
b. Flush waterlines for several minutes
c. Flush the waterline for 20-30 seconds
d. Nothing needs to be done, the patient can be seated

TOP © 2015 - 2023 Tests.com


86
407. The office is going to conduct a test on the waterlines to ensure the safety of their patients.
When should this type of testing be completed?

a. Before scheduled maintenance, no flushing of the water lines needed


b. After scheduled maintenance, flush the water lines for at least 2 minutes before taking a
water sample
c. After schedule maintenance, no flushing of the water lines needed
d. Before scheduled maintenance, flush the water lines for at least 2 minutes before taking
a water sample

408. How are safety data sheets for chemicals used in a dental office created?

a. The dental hygienist in charge is to create all safety data sheets.


b. The manufacturer of a chemical provides all safety data sheets.
c. The dentist creates the safety data sheets.
d. Both a and c only

409. Your dental office is considering upgrading to digital radiography and using phosphorus plates.
What type of imaging system utilizes phosphorus plates?

a. Direct
b. Indirect
c. Digitized
d. Semi-direct

410. You have taken an x-ray and need to develop it manually. What are the five steps you need to
take after placing the x-ray in a developer?

a. Rinse film, place in fixer, time film in fixer, wash film, dry film
b. Place in fixer, time films in fixer, rinse film, wash film, dry film
c. Wash film, place in fixer, time film in fixer, rinse film, dry film
d. Dry film, rinse film, place in fixer, time film in fixer, wash film

411. After manually processing a periapical x-ray, you notice that the film is light. What could be the
cause of a light image when you are manually processing an x-ray?

a. The developing solution is too hot


b. The developing solution is too cold
c. Excessive developing time
d. Concentrated developer solution

TOP © 2015 - 2023 Tests.com


87
412. The dentist requests that you take 4 bitewings on an 11-year-old patient. However, the size 2 x-
ray is too long and is causing the patient to gag. What size or type of film would you try next?

a. Size 2
b. Size 1
c. Size 0
d. Occlusal

413. After exposing x-rays, you are going to process the films in a daylight loader. What is the first
step that you need to take, after washing and drying your hands, to ensure proper infection
control?

a. Place sterile gloves in the bottom of daylight loader


b. Place contaminated films in the bottom of the daylight loader
c. Place a paper towel in the bottom of the daylight loader
d. Use a cup to collect foil from opened x-rays

414. After finishing a scaling and root planing procedure, a patient is sat into the upright position by
the dentist. As the patient begins to stand up, she falls to the floor, losing consciousness. You
are able to catch the patient and direct her to a chair. The patient comes to quickly, but is
nervous about what just happened. What do you tell the patient?

a. The patient has experienced postural hypotension.


b. The patient has experienced syncope.
c. The patient has experienced angina.
d. The patient has experience a seizure.

415. As it pertains to the administration of a local anesthetic, a larger gauge needle will have _______
deflection as it advances through tissue as compared to a smaller gauge needle.

a. less
b. more
c. the same
d. None of the above.

416. If there are large bubbles in the anesthetic cartridge, can you use it?

a. Yes, it is the result of gas during manufacturing.


b. Yes, the cartridge was frozen.
c. No, it is the result of gas during manufacturing.
d. No, the cartridge was frozen.

TOP © 2015 - 2023 Tests.com


88
417. When a patient has a hard time opening her mouth after receiving anesthetic during a dental
procedure, what is the patient experiencing?

a. Paresthesia
b. Anaphylaxis
c. Trismus
d. Trachyphlaxis

418. When placing anesthetic in an area of infection, the onset is _______ and may require ________
anesthetic.

a. Faster, more
b. Faster, less
c. Slower, more
d. Slower, less

419. Vasoconstrictors are added to local anesthetic for all of the following reasons, except:

a. It constricts blood vessels.


b. It allows for a longer working time.
c. It increases blood flow.
d. It inhibits the conduction of nerves.

420. What abnormality would a dental professional observe if a patient had too much fluoride during
the formation of adult teeth?

a. Dental caries
b. Decreased amount of enamel
c. Macrodontia
d. Dental fluorosis

421. A patient has a bulla. What can that be described as?

a. Loss of epithelium
b. Blister smaller than 5mm
c. Circumscribed area of color change
d. Blister larger than 5mm

TOP © 2015 - 2023 Tests.com


89
422. What type of implant is most commonly used in dentistry?

a. Endosseous
b. Transosteal
c. Subperiosteal
d. Edentulous

423. Match the active ingredient to the corresponding mouthwash:

a. Cetylpyridinium Chloride ____ Peridex


b. Quaternary Ammonium Compounds ____ Listerine
c. Chlorhexidine ____ Crest Pro-Health
d. Essential Oils ____ Scope

424. What percentage of plaque is made up of water?

a. 50%
b. 60%
c. 70%
d. 80%

425. Match the type/color of stain with the cause:

a. Black-line ___ Chlorhexidine


b. Green/Yellow/Orange ___ Iron compounds in saliva
c. Blue/Green ___ Chromogenic bacteria
d. Yellow-Brown ___ Industrial Worker/Copper

426. Which curette would you use to clean subgingival on the mesial, facial and lingual surface of the
posterior teeth?

a. Gracey 11/12
b. Gracey 13/14
c. Gracey 15/16
d. Gracey 17/18

427. Which of the following conditions is considered ASA III?

a. Epilepsy
b. Pregnancy
c. Insulin-dependent diabetes
d. Kidney failure

TOP © 2015 - 2023 Tests.com


90
428. The extra space created by primary canines and molars for the permanent teeth to erupt is
called:

a. Leeway space
b. Primate Space
c. Primary Space
d. Alignment Space

429. If the cone on the x-ray machine is ________ in length, the radiation absorbed dose would
________.

a. Decreased, decrease
b. Increased, stay the same
c. Increased, decrease
d. Decrease, stay the same

430. What causes reticulation on a developed film?

a. Air bubbles in the developer solution or water bath


b. Static electricity in the air
c. Light leakage in the dark room
d. A temperature change between the developer solution and the water bath

431. After a patient has been diagnosed with periodontal disease by a dentist and a dental hygienist
is instructed by the dentist to complete a scaling and root planning procedure on the patient,
what type of supervision is required by the dentist?

a. General supervision
b. Indirect supervision
c. Direct supervision
d. No supervision

432. When polishing with a rubber cup, an irregular shaped abrasive agent will be ______ abrasive as
compared to an abrasive agent with a regular shape.

a. less
b. more
c. equally
d. None of the above.

TOP © 2015 - 2023 Tests.com


91
433. Match the services provided by a dental hygienist with the correct term:

a. Primary Prevention ____ Discussing oral health with the patient


b. Secondary Prevention ____ Removing calculus
c. Tertiary Prevention ____ Administering a fluoride treatment
d. Educational Services ____ Scaling and Root planing
e. Therapeutic Services ____ Periodontal therapy

434. When a doctor performs an exam at a 6-month recall, this can be considered __________
supervision.

a. Indirect
b. Direct
c. General
d. Redirect

435. When you notice that a patient has perimylolsis, what disease could be a contributing factor?

a. Anorexia nervosa
b. Bulimia nervosa
c. Bipolar
d. Schizophrenia

436. A patient states that he had a stroke and was hospitalized two months ago. How long does this
patient need to wait before receiving any non-emergency dental care?

a. 1 month
b. 3 months
c. 4 months
d. No wait is needed.

437. All of the following are dental related risk factors for a pregnant woman except one. Which one
is the exception?

a. Gingivitis
b. Granuloma
c. Aphthous ulcer
d. Cavity

TOP © 2015 - 2023 Tests.com


92
438. Can in-office fluoride treatments be given to an expectant mother?

a. Yes, fluoride is recommended.


b. No, it is never recommended.
c. Yes, but not in the third trimester.
d. Yes, but not in the first trimester.

439. What is it called when the tongue adheres to the floor of the mouth?

a. Anodontia
b. Gemination
c. Circumvallate papilla
d. Ankyloglosia

440. When taking a bite-wing x-ray, you notice that the 2nd molar root tips are not in the x-ray. This is
due to an abnormal curve in the roots of the tooth. What is this abnormality called?

a. Dentinogenesis
b. Dilaceration
c. Hypodontia
d. Gemination

441. What type of drug is Amlodipine Besylate (Norvasc) and what possible dental conditions may
occur when taking this drug?

a. Calcium channel blocker, gingival overgrowth


b. Calcium channel blocker, hypersensitivity
c. Beta-blocker, gingival over-growth
d. Beta-blocker, hypersensitivity

442. What type of retention is used when pit and fissure sealants are placed?

a. Chemical
b. Mechanical
c. Bonded
d. Fused

443. Which is not a common filler for composite material?

a. Quartz
b. Silicates
c. Glass
d. Tin

TOP © 2015 - 2023 Tests.com


93
444. When polishing a composite material, what should never be used?

a. Coarse pumice
b. Tin oxide
c. Diamond
d. Zinc oxide

445. Which of the following is not an active ingredient in desensitizing toothpaste?

a. Potassium nitrate
b. Sodium citrate
c. Carbamide peroxide
d. Strontium chloride

446. How many days should a pre-medicated patient wait between a scaling and root planning
appointment?

a. 5 days
b. 7 days
c. 10 days
d. 14 days

447. What foramen is most often mistaken as a periapical abscess?

a. Mental
b. Incisive
c. Anterior palatine
d. Nasopalatine

448. What medication does not cause xerostomia?

a. Benadryl
b. Delsym
c. Depakote
d. Pepto-Bismol

449. ___________ is caused by altering the neural pathways that stimulate the salivary glands.

a. Sialolithiasis
b. Xerostomia
c. Anesthetic neurotoxicity
d. Salivary gland cancer

TOP © 2015 - 2023 Tests.com


94
450. Which two answers describe bruxism?

a. Pathological form of attrition


b. Loss of tooth structure from mastication
c. Habitual grinding of teeth
d. Loss of tooth structure from acid

451. What tooth is missing a filling on this x-ray?

a. Maxillary first molar


b. Mandibular first molar
c. Maxillary first premolar
d. Maxillary second premolar

452. What is abnormal about the tooth in this picture?

a. Fusion
b. Gemination
c. Primary bicuspid
d. Heart Tooth

453. What is the ultimate goal of scaling and root planing?

a. Decrease the colonization of bacteria


b. Gain connective tissue and reduce the size of a pocket
c. Heal gingivitis
d. Alleviate bleeding around the teeth

TOP © 2015 - 2023 Tests.com


95
454. Which two answers are true regarding Kaposi’s sarcoma?

a. It often affects people with immune deficiencies, such as HIV.


b. It is a virus.
c. It is a bacterial infection.
d. It only affects males.

455. Is it recommended that smokers use alcohol based mouth rinses?

a. Yes, to help control the bacteria in their oral cavity.


b. Yes, to prevent periodontal disease.
c. No, it puts the patient at higher risk of cancer.
d. No, they should not use any form of mouth rinse.

456. What type of surgery may be necessary if the periodontal pockets are too deep for debridement
and scaling, and root planing is not advised?

a. Gingivectomy
b. Periodontal flap
c. Gingival graft
d. Gingivoplasty

457. When a patient has severe attrition and the cusps of molars are worn down exposing the
underlying dentin, what are the areas of wear called?

a. Decay
b. Abrasion
c. Erosion
d. Facets

458. In regards to this picture, the supragingival calculus clearly extends below the gumline forming
subcalculus. Subgingival calculus is formed by gingival cervical fluid.

a. Both statements are true.


b. Both statements are false.
c. First statement is true. Second statement is false
d. First statement is false. Second statement is true.

TOP © 2015 - 2023 Tests.com


96
Practice Test for the NBDHE Dental Hygienist Exam – Section 3
Community Dental Health Testlet

459. ORDER the following in accordance with Maslow’s Hierarchy of Needs as they relate to
identifying human needs and addressing them through personal or community involvement
programs.

______ a. Safety and Security


______ b. Self
______ c. Self Esteem
______ d. Physiological Needs
______ e. Actualization
______ f. Social Needs

460. A team of dental professionals assesses the dental needs of the community and begins to form
the next step of their intervention plan to promote measures that will address these needs.
After the initial assessment, what step comes next in a community program planning routine?

a. Program financing
b. Program formulation
c. Program Implementation
d. Evaluation of effectiveness

461. What is the study of the distribution and determinants of disease in a population?

a. Control study
b. Longitudinal study
c. Epidemiology
d. Pathogenic analysis

462. What type of epidemiology study focuses on testing a theory or hypothesis?

a. Analytical study
b. Descriptive study
c. Convenience
d. Cross-sectional

TOP © 2015 - 2023 Tests.com


97
463. A dental hygiene program wants to conduct a study on a new antimicrobial agent for patients
with advanced periodontal disease symptoms following their scaling and root planing
procedures. What type of study is the LEAST valid due to the makeup of the group being
studied?

a. Randomized
b. Non-randomized
c. Convenience
d. Longitudinal

464. A toothpaste company wants to test the efficacy of a new product. What type of study is the
most valid to prove their hypothesis?

a. Longitudinal
b. Randomized
c. Double-blind
d. Cross-sectional

465. A hypothesis that was tested using a statistical test shows a p-value of 0.04. Which statement is
correct?

a. There was a significant statistical result


b. The statistical result was insignificant
c. The difference occurred by chance
d. The statistic is factual

466. What type of test compares actual results versus independent results?

a. T-test
b. Student’s t test
c. Chi square test
d. Significance level

467. Which of the following are NOT criteria for a good index?

a. Clear
b. Simple
c. Valid
d. Quantity

TOP © 2015 - 2023 Tests.com


98
468. An index is used to study the conditions of gingivitis in test subjects. What type of indices is
being used?

a. Irreversible index
b. Reversible index
c. Reliable index
d. Randomized index

469. When conducting a double-blind study on periodontal disease patients, the examiners record
periodontal probing depths at each visit. What is necessary scoring for these measurements to
be precise?

a. Calibration
b. Standardized probing
c. Dependent variable
d. Independent variable

470. What index is used to measure past and present decay experiences in a population?

a. GI
b. RCI
c. DEFT
d. DMFT

471. What index is used to detect early symptoms of gingivitis?

a. GI
b. PDI
c. SBI
d. DEFT

472. A hygienist is using the PDI to measure the presence and severity of periodontal disease in one of
her patients. How does she determine the PDI score?

a. PDI = total score of teeth examined divided by number of teeth examined


b. PDI = total number of teeth examined divided by the total score of teeth examined
c. PDI = total score of teeth examined by number of teeth present
d. PDI= total number of teeth examined by number of teeth present

TOP © 2015 - 2023 Tests.com


99
473. A dental school aims to measure the periodontal needs of a group of patients. They decide to
choose an index that was developed by the World Health Organization to measure needs in
children and adolescents in one group, and another group of adults over 20 years of age. Which
index was selected?

a. PI
b. PDI
c. CPITN
d. OHI-S

474. Which teeth are the Ramfjord teeth that are used to evaluate different teeth per sextant for the
use in indices?

a. 3, 9, 14, 18, 24, 30


b. 3, 8, 14, 19, 24, 30
c. 3, 8, 15, 20, 25, 31
d. 3, 9, 15, 19, 25, 31

475. When tabulating a PSR recording, a hygienist finds a code 4 on one of the teeth in the current
sextant of the patient’s mouth. What is the next step?

a. Record a 4 on that tooth and continue recording the codes on the other teeth in the
sextant
b. Record a 4 and examine the remaining teeth to record the next lowest number
c. Record a 4 and examine the remaining teeth in the sextant for a lower number
d. Record a 4 and do not examine the remaining teeth in the sextant

476. Upon inspection, the hygienist assigns a plaque index score of 2 to the patient. This score
indicates:

a. Gross soft debris within gingival pockets or margins


b. Film adheres to attached gingiva
c. Moderate soft debris within the gingival margin
d. Fair oral hygiene

477. Disclosing solution is necessary for use in which indices?

a. PHP
b. PI
c. OHI-S
d. All of the above

TOP © 2015 - 2023 Tests.com


100
478. What program was founded in 1997 as a way to meet the needs of children whose families
exceeded the income requirements for Medicaid but were still unable to pay for private medical
insurance?

a. CHAMPUS
b. CHIP
c. Medicare
d. IHS

479. Which of the first amendments to social security involved providing insurance for individuals
over the age of 65, but offers little dental coverage?

a. Title XIX Medicaid


b. Title XXI Medicaid
c. Title XVIII Medicare
d. Title X Medicare

480. What type of dental insurance plan is also called a fee-for-service plan?

a. Indemnity
b. Point-of-Service
c. PPO
d. DMO

481. Which of the following is NOT a category of community oral health research?

a. Epidemiologic research
b. Clinical trials
c. Evaluation of community programs
d. Research in habits

482. What is a group observed over time to determine the natural progression of a disease?

a. Case-control
b. Cohort
c. Cross-sectional
d. Subjects

TOP © 2015 - 2023 Tests.com


101
483. Approximately what percentage of adults has experienced root caries?

a. 15%
b. 25%
c. 30%
d. 55%

484. What is the most common chronic childhood disease?

a. Asthma
b. Hay fever
c. Chicken pox
d. Caries

485. Which teeth are most frequently affected by caries?

a. First molars
b. Second molars
c. Both first and second molars
d. First and second bicuspids

486. What percentage of adults has experienced dental decay?

a. 25%
b. 73%
c. 85%
d. 95%

487. Cleft lip and palates are among the most common congenital malformations in US births. Which
of the following is NOT associated with clefts?

a. Spontaneous abortion
b. Maternal fever
c. Drug consumption
d. Morning sickness

488. A patient exhibits white opaque areas on a tooth but it does not involve more than 30-40% of
the tooth. What classification of fluorosis is present?

a. 4
b. 2
c. 1
d. 3

TOP © 2015 - 2023 Tests.com


102
489. A community based sealant program trains dental professionals to perform sealants in a school-
based program. Studies show that children who participate in these programs for the next 2-5
years will experience _______ fewer new decayed pit and fissure areas.

a. 25%
b. 60%
c. 85%
d. 100%

490. If a natural water supply contains a fluoridation level over ________ then the water system will
undergo a defluoridation program prior to the delivery to residents.

a. 1.8ppm
b. 5ppm
c. 2-4 ppm
d. 3.7ppm

491. Which cancer has a higher death rate?

a. Ovarian
b. Prostate
c. Oral
d. Lung

492. A convenience study involves participants that are closely located to the examiner. It is
considered very reliable as it involves the general population.

a. Both statements are true


b. Both statements are false
c. The first statement is true, the second statement is false
d. The first statement is false, the second statement is true

493. A dental indices must be able to measure consistently at different times. This is called:

a. Validity
b. Reliability
c. Quantifiable
d. Clear

TOP © 2015 - 2023 Tests.com


103
494. How many teeth are evaluated in the DMFT index?

a. 20
b. 32
c. 28
d. 24

495. What index is used to measure the past and present caries experience of a population?

a. deft
b. DMFT
c. RCI
d. DTI

496. How many teeth are evaluated in the deft index?

a. 20
b. 8
c. 28
d. 32

497. Which area is NOT measured during an SBI reading?

a. mesial
b. occlusal
c. distal
d. lingual

498. Based on the periodontal treatment needs scale, a patient or community that needs instruction,
root debridement, elimination of plaque and has retentive margins on restorations or crowns
would be classified as what?

a. I
b. II
c. III
d. O

499. What index is used to measure oral cleanliness by visual inspection for surface debris?

a. OHI-S
b. PI
c. PHP
d. PTN

TOP © 2015 - 2023 Tests.com


104
500. What publication includes a comprehensive list of disease prevention and health promotion
objectives?

a. A National Call to Action to Promote Oral Health


b. Oral health in America
c. Healthy People 2020
d. Indian Health Services

501. Which type of study requires two or more measures for a non-experimental outcome of data?

a. Descriptive
b. Case control
c. Cross-sectional
d. Cohort

502. The difference in a clinical trial and an epidemiologic survey is that an epidemiologic survey will:

a. Target naturally occurring samples of a population


b. Use small groups
c. Gathers data from trials
d. Uses methods that are valid and reliable

503. Which of the following is not a type of sampling technique?

a. Strategic
b. Random
c. Systemic
d. Judgmental

504. What sampling technique is most commonly used in research?

a. Stratified
b. Systemic
c. Random
d. Convenience

TOP © 2015 - 2023 Tests.com


105
505. When there is constant presence of a disease within a given area, it is considered to be
__________.

a. Epidemic
b. Pandemic
c. Endemic
d. Epidemiologic

506. In a research study, what group does not receive treatment or intervention?

a. Experimental
b. Control
c. Discrete
d. Sample

507. All of the following, except one, are levels of government that provide community dental health
care. Which is the exception?

a. State
b. Federal
c. Regional
d. International

508. What form of research helps answer the who, what and where questions concerning disease?

a. Experimental Studies
b. Descriptive Studies
c. Incidence Study
d. Prevalence Study

509. When calculating research data, what is the term used to describe the most frequently occurring
value in the data?

a. Mode
b. Mean
c. Median
d. Measure

TOP © 2015 - 2023 Tests.com


106
510. Which of the following is an accurate statement on the nature of non-parametric inferential
statistics?

a. They have a fixed number of parameters.


b. They are most useful for data measured on a quantitative scale.
c. The data it deals with is more qualitative.
d. The parameters are fixed by the model and do not grow based on the data.

511. In statistics, the p-value describes:

a. Statistical significance
b. Different means
c. Variability in the sample groups
d. How many subjects are needed to provide significance

512. The following items represent the stages of learning. Put them in the correct order.

a. Awareness
b. Unawareness
c. Habit
d. Self-interest
e. Involvement
f. Action

513. What type of study involves repeated observations of the same population over a long period of
time?

a. Retrospective
b. Prospective
c. Analytical
d. Longitudinal

514. If you accept a null hypothesis, you are saying that, statistically, there are no significant
differences between population groups. It is tested by a statistical hypothesis test.

a. Both statements are true.


b. First statement is true. Second statement is false.
c. Second statement is false. First statement is true.
d. Both statements are false.

TOP © 2015 - 2023 Tests.com


107
515. Informed consent is an ongoing process. It is often discussed in a group setting.

a. Both statements are true.


b. Both statements are false.
c. First statement is true. Second statement is false.
d. First statement is false. Second statement is true.

516. What is considered discrete data?

a. Numeric variables
b. Total time
c. Subcategories
d. Observations

517. All of the following, except one, are advantages of doing interviews in research. Which is not an
advantage of doing interviews?

a. Flexibility
b. Time
c. Complete data collected
d. Questions can be clarified

518. A dental hygienist employed in a nursing home can provide routine dental care. They hold a
special license to perform these duties.

a. Both statements are true.


b. Both statements are false.
c. First statement is true. Second statement is false.
d. First statement is false. Second statement is true.

519. When using the Type III examination method, you use a tongue depressor. When using the Type
IV examination method, you would use a mouth mirror.

a. Both statements are true.


b. Both statements are false.
c. First statement is true. Second statement is false.
d. First statement is false. Second statement is true.

TOP © 2015 - 2023 Tests.com


108
520. What type of motivation practice is used when there is the promise of a reward?

a. External
b. Internal
c. Formal
d. Informal

521. Medicare provides insurance to low income families. It provides both medical and dental care.

a. Both statements are true.


b. Both statements are false.
c. First statement is true. Second statement is false.
d. First statement is false. Second statement is true.

522. The human papilloma virus presents with a stem like base, similar to that of a mushroom. This
can be described as ____________.

a. Pedunculated
b. Papule
c. Sessile
d. Papillary

TOP © 2015 - 2023 Tests.com


109
Practice Test for the NBDHE Dental Hygienist Exam – Section 4
Case 1 Testlet
Age: 11 Dental History:
Sex: M This patient has not seen a dentist in 2 years as
Height: 4’11” his family moved into the area and then
Weight:110 underwent a change in dental insurance. His
B/P: 115/78 mother reports that now they have found a
dental office that they like she will bring him
Chief Complaint: every 6 months for preventive care
None. This patient is being brought to the office appointments.
for a routine dental evaluation and cleaning.
Social History:
Medical History: This patient enjoys sporting activities and plays
This patient suffers from seasonal allergies. He soccer regularly. He recently signed up to be on
has not ever had an asthma flare-up but his a flag football team at his school. He often
doctor has prescribed asthma medication to drinks sports drinks during games, but drinks
manage his cough after outdoor activities mostly water during practice.
during the cooler months.

Current Medications:
Albuterol nebulizer PRN
Claritin 5mg qd

TOP © 2015 - 2023 Tests.com


110
523. What should the hygienist recommend to this patient in regard to how his asthma medication
relates to his oral health?

a. The patient should be taking the medication daily


b. Advise the patient to rinse with fluoride daily
c. Recommend the patient clean their mouth after the use of albuterol
d. Keep an asthma inhaler handy during his dental appointments

524. Upon inspection of the oral cavity, the patient appears to have a 3mm oval lesion just inside the
lower lip. It is flat and raw. The patient states that he had one 3 or 4 months ago as well. It is
most likely a(n):

a. Syphillic chancre
b. Apthous ulcer
c. HPV lesion
d. Apical abscess

525. The patient does not have any sealants on his permanent teeth. At this time, what teeth should
the hygienist recommend placing sealants on?

a. 2, 3, 14, 15, 18, 30, 31


b. 3, 14, 18, 19, 30, 31
c. 3, 14, 19, 30
d. 3, 4, 5, 12, 13, 14, 19, 20, 21, 28, 29, 30

526. When discussing the findings on the x-ray, the patient asks about the dark area on the distal of
#K. What should you tell him this looks like?

a. Root resorption
b. Interproximal caries
c. A failed amalgam filling
d. Periapical abscess

527. Based on the findings of tooth #K, what would be the most important piece of oral hygiene
education that should be taught at this visit?

a. The importance of maintenance visits


b. Fones method of toothbrushing
c. Sugar intake
d. Flossing technique

TOP © 2015 - 2023 Tests.com


111
528. Upon discussing the patient’s extracurricular activities, it is discovered that he does not wear
any type of tooth protection device during practice, but he does wear a stock guard during
games. What is the best advice that the hygienist could give to the patient?

a. Suggest the patient wear a guard during all practices as well as games
b. Suggest the patient only wear a custom guard during games
c. Suggest the patient continue what he is doing until a custom guard can be fabricated
d. Recommend the patient not wear a guard unless it is a custom made guard

529. In regards to the patient consuming sports drinks, what type of nutritional counseling advice is
most appropriate?

a. Discuss the hydration benefits of sports drinks vs. water


b. Continue drinking lots of water during practices. Drink only one sports drink during the
game and drink only water afterward.
c. Substitute all water with sport drinks
d. Discuss types of sports drinks and which one is a more healthy option

530. The patient’s mother states she is concerned that some of her son’s teeth are not coming in
straight with the other teeth. The hygienist should:

a. Discuss the occlusion and then give the patient’s mother a card for the orthodontist
b. Discuss the occlusion with the patient’s mother and then report findings to the dentist
c. Have the dentist discuss the occlusion with the patient’s mother
d. Tell the patient’s mother that her son’s occlusion will be evaluated at the next
appointment to see if it has changed

531. The patient reports occasional soreness in the posterior regions of the lower arch. What is the
likely cause?

a. Decalcification
b. Caries
c. Eruption of 12-year molars
d. Use of whitening toothpaste

532. The patient asks how many baby teeth he has left to lose. The hygienist’s correct response
should be ______.

a. 5
b. 4
c. 7
d. 6

TOP © 2015 - 2023 Tests.com


112
Practice Test for the NBDHE Dental Hygienist Exam
Case 2 Testlet
Age: 9 Current Medications:
Sex: F None
Height: 4’4”
Weight: 68 Dental History:
B/P: 109/72 Patient has been coming into the office since
she was 3 for routine preventive care
Chief Complaint: appointments every 6 months. She has no
Patient complains of a broken tooth in her history of dental decay and had sealants placed
upper right posterior region. Her mother on her 6-year molars during the last visit.
reports that she has complained of it hurting for
the past 2 days. Social History:
Patient is in 3rd grade at a local private school.
Medical History: She enjoys reading and drawing.
The patient was hospitalized 7 months ago for a
broken arm when she fell off of a horse.

TOP © 2015 - 2023 Tests.com


113
533. What is the most likely explanation for the patient’s chief complaint of a broken tooth in the
upper right posterior?

a. Her 1st molar has developed large mesiodistal decay


b. A primary molar that had decay is resorbing but has not yet exfoliated
c. Her 2nd bicuspid has developed a fracture
d. #2 is only partially erupted through the gums

534. How many lower molars are shown on the patient’s x-ray?

a. 6
b. 2
c. 8
d. 10

535. Upon inspection of #19, the tooth appears to have fractured and then developed decay. Which
of the following would NOT be a risk factor that could have contributed to this condition?

a. The patient’s horseback injury


b. The patient chewing on pencils and pens at school
c. Hyperfluorosis
d. Bubble gum chewing

536. The dentist prescribes bitewing radiographs to check the interproximal areas of the primary and
permanent molars. How many films would this patient need?

a. 2 vertical BWX
b. 2 horizontal BWX
c. 4 vertical BWX
d. 4 horizontal BWX

537. The patient’s mother complains that the patient’s adult incisors are much more yellow than the
rest of her teeth. What is a reasonable explanation?

a. Enamel fluorosis
b. Stain from a vitamin supplement
c. Natural coloration
d. Tea stain

TOP © 2015 - 2023 Tests.com


114
538. The patient exhibits some grayish intrinsic stain on teeth 7-10. What is the likely cause?

a. Fluorosis
b. Food pigment
c. Tetracycline
d. Genetic

539. The patient shows some moderate marginal inflammation and plaque accumulation. What type
of toothbrush should be recommended?

a. Medium bristled
b. Hard bristled electric
c. Soft bristled manual
d. Any of the above

540. While reviewing brushing techniques, the patient exhibits the method that she uses. She
brushes with the bristles aimed toward the gingival margin and then rolls the brush toward the
occlusal surface. What method is this?

a. Fones
b. Roll
c. Leonard
d. Charters

541. During oral hygiene instruction, the hygienist discloses the patient’s teeth with an agent that
stains new and mature plaque in red and blue tints. What type of agent does she use?

a. Erythrosin
b. Disclosing tablet
c. Two-tone disclosing solution
d. Bismarck Brown

542. The patient lives in a rural area and has a well as her water source. There is no natural
fluoridation of the water in the area. The hygienist recommends a weekly supplemental dosage
of fluoride. What is the proper dosage for the patient’s age?

a. 1.0mg
b. 0.5mg
c. 0.01 mg
d. 0.25mg

TOP © 2015 - 2023 Tests.com


115
Practice Test for the NBDHE Dental Hygienist Exam
Case 3 Testlet
Age: 8 Current Medications:
Sex: M None. Patient just completed a round of
Height: 4’ 4” amoxicillin.
Weight: 70
B/P: 110/78 Dental History:
Patient has not been seen in the office for 20
Chief Complaint: months. The patient’s mother states she had to
Patient’s mother is concerned with upper go back to work full-time and was not able to
anterior crowding and wonders if he needs to make an appointment before now.
have braces.
Social History:
Medical History: The patient is currently enrolled in 2nd grade at
This patient just completed a round of the local public elementary school. He has a pet
antibiotics for an inner ear infection. He is also dog named “Buddy.” He is enrolled in a Boy
allergic to latex. Scout group and enjoys camping on the
weekends with his den.

TOP © 2015 - 2023 Tests.com


116
543. What type of gloves should the hygienist use for treating this particular patient?

a. Standard examination gloves


b. Rubber gloves
c. Nitrile gloves
d. Surgical gloves

544. Based on the appearance of the patient’s radiographs, what permanent tooth appears to be
rotated?

a. 20
b. 7
c. 31
d. 10

545. The patient appears to exhibit good oral hygiene habits, but upon scaling interproximal areas
there is some moderate bleeding. What would be the best oral hygiene counseling to address at
this visit?

a. Fones method
b. Bass method
c. Antiseptic mouthwash
d. Flossing

546. The patient has no existing restorative work. In addition to the prophylaxis and fluoride
application during today’s visit, what other preventive care procedure would help prevent decay
of the permanent molars?

a. Fluoride varnish
b. Sealants
c. Bitewings
d. Exam

547. Which tooth best exhibits the amelogenesis phase of morphodifferentiation?

a. #2
b. #16
c. #11
d. #19

TOP © 2015 - 2023 Tests.com


117
548. Which tooth best demonstrates the appearance of a tooth in the appositional stage of tooth
formation?

a. #4
b. #6
c. #16
d. #14

549. In addition to the patient having mild interproximal bleeding during scaling, the patient also has
moderate marginal calculus on the lingual of the lower anterior teeth. What would be an
appropriate recall schedule for this patient?

a. 3 months
b. 6 months
c. 12 months
d. No longer than 24 months

550. The patient’s mother exhibits concern of her son’s crowding. When the hygienist checked the
patient’s occlusion, the mesiobuccal groove of his mandibular first molar aligned with the
mesiobuccal cusp of his maxillary first molar. He also exhibited a small overbite and crowding of
the anterior teeth. What classification of occlusion is evident?

a. Class III malocclusion


b. Normal occlusion
c. Class II malocclusion division I
d. Class I malocclusion

551. The patient admits to not bringing his toothpaste or floss with him in his backpack on overnight
Boy Scout camping trips. What oral hygiene method may be useful for him to care for his teeth
on overnight hikes?

a. Dry toothbrushing
b. Mouthwash
c. Rinsing his mouth with water
d. Brushing his teeth as soon as he gets home

552. How many permanent teeth have erupted into the patient’s mouth?

a. 12
b. 14
c. 16
d. 10

TOP © 2015 - 2023 Tests.com


118
Practice Test for the NBDHE Dental Hygienist Exam
Case 4 Testlet
Age: 9 Dental History:
Sex: M This patient is a mouth breather and has a
Height: 4’ 6” tongue thrust. He reportedly sucked his thumb
Weight: 85 until he was 4.
B/P: 119/79
Social History:
Chief Complaint: The patient is in 4th grade at the local public
The patient’s mother reports that he has elementary school. His parents are divorced and
halitosis. he spends the weekends at his father’s house,
which lives in the next town. He has begun
Medical History: learning how to use a skateboard, and his
Patient’s mother reports he tends to have mother states he will not wear a mouth guard
allergies year-round when he goes to his father’s house with the
skateboard.
Current Medications:
Claritin 5mg qd

TOP © 2015 - 2023 Tests.com


119
553. The patient has a congenitally missing tooth. Which tooth is it?

a. 16
b. 6
c. 18
d. 32

554. Upon inspecting the patient’s occlusion, there is a mesial relationship of the mesiobuccal groove
of the mandibular first molar to the mesiobuccal cusp of the maxillary first molar. What
classification of occlusion is present?

a. Class I malocclusion
b. Class III malocclusion
c. Normal relationship
d. Class II malocclusion

555. The patient’s mother exhibits concern regarding breath malodor. Microorganisms in what area
of the mouth produce malodor?

a. Tonsils
b. Tongue
c. Periodontal pockets
d. Postnasal drip

556. Which device is appropriate for cleaning the tongue?

a. Tongue scraper
b. Manual toothbrush
c. Electric toothbrush
d. All of the above

557. The patient states that he cannot brush his tongue well because the bristles on his toothbrush
are all splayed out. Which of the following is NOT an area of education that the hygienist should
address?

a. Selecting the appropriate brand of toothbrush


b. Modifying the patient’s brushing technique
c. Changing the toothbrush out every 2-3 months
d. Selecting the appropriate texture of toothbrush bristle

TOP © 2015 - 2023 Tests.com


120
558. When preparing the fluoride tray for a gel application, what is the maximum amount of gel that
should be placed in the fluoride tray?

a. 4ml
b. 8ml
c. 2ml
d. 12ml

559. The patient exhibits a tongue thrust. What oral habit may be related to this anomaly?

a. Chewing pencils at school


b. Thumb sucking
c. Mouth breathing
d. Using a water bottle

560. When selecting a fluoride to place in the tray for the patient application, she chooses a form that
is one which should not be used on patients that have composite restorations. What type of
fluoride did she choose?

a. Neutral sodium
b. Acidulated phosphate
c. Stannous
d. Fluoride varnish

561. The patient has a small, 2mm, round, purple lesion just inside of the buccal mucosa. It is most
likely a(n) ____________.

a. mucocele
b. ranula
c. lipoma
d. hemangioma

562. The patient says he does not wear a mouth guard when using his skateboard because the one he
has does not fit well. Which type of guard would provide an appropriate fit?

a. Mouth-formed protectors
b. Custom-made protectors
c. Stock protectors
d. Rigid

TOP © 2015 - 2023 Tests.com


121
Practice Test for the NBDHE Dental Hygienist Exam
Case 5 Testlet
Age: 72 Current Medications:
Sex: F Daily multivitamin
Height: 5’2” Fosamax 70mg once weekly
Weight: 109
B/P: 110/70 Dental History:
The patient’s previous dentist retired 2 years
Chief Complaint: The patient is experiencing ago. She has not had any dental problems until
some bleeding during brushing around her the recent bleeding, which caused her to go
29x31 3-unit bridge. She also reports a bad ahead and find a new dentist. She wears an
taste in her mouth and her tongue feels as if it upper and lower partial denture which seem to
is burning. not be fitting well as they used to.

Medical History: Social History:


This patient is fairly healthy. She takes a daily The patient is active in her church and conducts
multivitamin and a regular osteoporosis a ladies study at her house each week. She
medication. During her last physical exam her enjoys walking her dog and takes the Active
doctor mentioned she was slightly anemic, but Older Adults class twice a week at the YMCA.
the vitamin supplement has helped to bring her
iron levels back to where they ought to be.

TOP © 2015 - 2023 Tests.com


122
563. The patient is taking medication to control her osteoporosis. Which of the following contributes
to an increased risk of osteoporosis?

a. High intake of vitamin D


b. High body weight
c. Of African decent
d. Smoking
e. Both b and c only

564. During initial head and neck examination, the hygienist notices inflammation and cracking at the
corners of the lips, which extend onto the facial skin. What condition has these classic
symptoms?

a. Angular cheilitis
b. Yeast infection
c. Fordyce granules
d. Chapped lips

565. In addition to the inflammation and cracking at the corners of the patient’s lips, the hygienist
also notices her tongue is devoid of filiform papillae. What condition does the patient likely
have?

a. Extended wear of her partial


b. Geographic tongue
c. Iron deficiency anemia
d. Neutropenia

566. During oral hygiene education, the hygienist should advise the patient to:

a. Remove her partial denture and brush the underlying tissue at least once a day
b. Brush her partial denture with a separate denture brush
c. Remove her appliance overnight
d. All of the above
e. Both a and c only

567. The patient has limited dexterity, which makes it difficult to clean along her fixed bridge with
anything other than a toothbrush. What oral care appliance would help with plaque control in
this area?

a. Floss threader
b. Toothpick
c. Mouthwash
d. Water flosser

TOP © 2015 - 2023 Tests.com


123
568. Tooth #22 exhibits some discoloration along the gingival margin on the distal portion of the
tooth. It is white and brown in color, localized to that particular area. Which statement is correct
regarding this area?

a. The hygienist should use selective polishing to remove the stain


b. It is due to tetracycline intake
c. It is due to embedded plaque biofilm that has caused chromogenic bacteria
d. The discoloration is intrinsic and due to dental fluorosis, therefore it cannot be removed

569. According to the patient’s periodontal probing depths, the patient exhibits _________________.

a. Generalized moderate bone loss


b. Localized moderate bone loss
c. Generalized severe bone loss
d. Localized severe bone loss

570. What might occur to the diseased pocket epithelium during subgingival scaling the distal of #22?

a. Irrigation
b. Incidental curettage
c. Debridement
d. Calculus removal

571. When probing #17, the tooth exhibited severe involvement with through and through visibility.
What grade of furcation involvement does this tooth have?

a. Grade III
b. Grade I
c. Grade IV
d. Grade II

572. Tooth #14 responds to pressure by having moderate mobility of about 2mm in a buccolingual
direction. What class of mobility is present?

a. Class II
b. Class I
c. Class III
d. Class IV

TOP © 2015 - 2023 Tests.com


124
Practice Test for the NBDHE Dental Hygienist Exam
Case 6 Testlet
Age: 68 Current Medications:
Sex: F Aricept 10 mg/day
Height: 5’1” Risperdal 4mg/day
Weight: 110
B/P: 109/69 Dental History:
The patient states at her old home she was
Chief Complaint: seeing a dentist who was going to make a new
“I’m having a hard time eating food, because I partial denture for her to wear on her lower
don’t have enough teeth in my bottom jaw.” teeth. Her old partial denture has broken.

Medical History: Social History:


Patient tripped on her front porch 6 months The patient recently moved in with her son and
ago, broke her leg, and shattered her elbow. his wife. She enjoys her own private living
She is still undergoing physical therapy once a quarters and kitchenette, while still being
week in addition to her exercises at home. The connected to the main house.
patient is experiencing early stages of
dementia.

TOP © 2015 - 2023 Tests.com


125
573. Tooth #14 does not have any teeth to occlude with. What will the result likely be?

a. Advanced wear
b. Furcation involvement
c. Mobility
d. Supraeruption

574. What medical conditions are the patient’s prescriptions designed for?

a. Blood pressure
b. Dementia
c. Osteoporosis
d. Cholesterol

575. Which tooth has had root canal therapy performed on it?

a. 27
b. 26
c. 22
d. 28

576. The patient has a white line-shaped lesion inside of the buccal mucosa. It is bilateral and
keratinized. What is this most appropriate name for this lesion?

a. Leukoedema
b. Linea alba
c. Leukoplakia
d. Keratosis

577. Which bridge would be the easiest for the patient to floss?

a. 2x6
b. 7x10
c. 12x14
d. They would all be equally easy to floss

578. Which of the following teeth has probing depths that reflect moderate bone loss?

a. #7
b. #13
c. #10
d. #22

TOP © 2015 - 2023 Tests.com


126
579. If the distal of tooth #31 measures 3mm from the cementoenamel junction to the top of the
marginal gingiva and has a 4mm periodontal pocket, what is the total measured bone loss?

a. 3mm
b. 4mm
c. 7mm
d. 8+mm

580. Before the patient can have a new lower partial denture made, what nutritional counseling
should the hygienist provide?

a. Advise the patient to eat a soft diet that includes nutrient-dense food
b. Encourage the patient to have a liquid diet and consume nutritional shakes or smoothies
c. Advise the patient to remove her partial denture at night so it will last longer
d. Advise the patient to have implants placed so she can function as normal as possible

581. The incisal edges of teeth #7 and 11 are flat and the exposed tooth enamel below the crown is
exposed. What is the likely cause?

a. Enamel erosion
b. Toothbrush abrasion
c. Attritional wear
d. Tongue thrust

582. The patient reports having some pain in her hand following her fall. All of the following would
NOT be an appropriate modified brushing method for this patient?

a. Use of an electric toothbrush


b. Using the Fones brushing method
c. Placing a large handle on the brush, such as a bike handle
d. Having someone help her brush

TOP © 2015 - 2023 Tests.com


127
Practice Test for the NBDHE Dental Hygienist Exam
Case 7 Testlet
Age: 47 Current Medications:
Sex: M None
Height:: 5’9”
Weight: 161 Dental History:
B/P: 140/89 The patient’s last dental appointment was over
7 years ago for treatment on a root canal. His
Chief Complaint: teeth have not been cleaned in over 15 years.
“One of my top right teeth hurts. I have not
been to the dentist in over 7 years and need to Social History:
have them cleaned.” The patient is a truck driver. He is away from
home some, but is at home most evenings. His
Medical History: favorite drink is Mountain Dew and he admits
Patient had his last physical 3 years ago. He is to drinking it 3 or 4 times a day for the caffeine.
allergic to penicillin.

TOP © 2015 - 2023 Tests.com


128
583. What is the radiopacity evident on the lower right border of the mandible?

a. Ghost from jewelry worn during x-ray


b. Ghost of cervical vertebrae
c. Hyoid bone
d. Clavicle

584. Based on the patient’s social history and the recurrent caries rate evident on his x-rays, what
would be the most important aspect of counseling at today’s visit?

a. Oral hygiene instruction


b. More frequent preventive care appointments
c. Nutritional counseling
d. Oral cancer screening

585. What is the projection on the distal surface of #3?

a. Amalgam overhang
b. Calculus deposit
c. Caries
d. None of the above

586. What is the radiolucency near the apex of #7?

a. Incisive fossa
b. Incisive foramen
c. Nasopalatine foramen
d. Nasopalatine cyst

587. What classification of periodontal disease would this patient fall under?

a. Localized aggressive periodontitis


b. Generalized chronic severe periodontitis
c. Aggressive moderate periodontitis
d. Localized chronic moderate gingivitis

588. What type of crown is placed on #19?

a. Gold
b. Stainless Steel
c. Silver
d. Porcelain fused to metal

TOP © 2015 - 2023 Tests.com


129
589. Which of the following is a long-term goal of nonsurgical periodontal therapy?

a. Elimination of pockets
b. Removal of hard and soft deposits
c. Definitive biofilm control instruction
d. Control of systemic risk factors

590. The hygienist prepares a solution for subgingival irrigation after the scaling and root planing
procedure. Which of the following solutions is NOT a commonly used irrigant?

a. Chlorhexidine digluconate
b. Stannous Fluoride
c. Listerine
d. Povidone-iodine

591. When selecting the appropriate ultrasonic insert to scale the distal concavity of #3, which
ultrasonic tip would be the most effective to use?

a. Universal
b. Thin
c. Beavertail
d. After Five

592. Which curette would the hygienist choose to scale the distal area of #14?

a. 11/12
b. 15/16
c. 7/8
d. 13/14

TOP © 2015 - 2023 Tests.com


130
Practice Test for the NBDHE Dental Hygienist Exam
Case 8 Testlet
Age: 37 Current Medications:
Sex: M None
Height: 6’0”
Weight: 173 Dental History:
B/P: 124/ 85 The patient’s last dental visit was 8 years ago to
place the crown on tooth #19 following a root
Chief Complaint: canal. Most of his fillings are 10-20 years old.
“My kids tell me that my breath stinks. I brush The patient does not floss because he finds it
my teeth well every day, but I think I just need difficult to fit his hands into his mouth.
to have them cleaned.”
Social History:
Medical History: The patient is a shift supervisor at an oil
The patient saw his doctor last month when he refinery. He works alternating day and night
came down with the flu. The only surgery the
patient has ever had was his appendectomy 21
years ago.
shifts.

TOP © 2015 - 2023 Tests.com


131
593. As the patient demonstrates his flossing technique, which technique is improper and would
need to be addressed through oral hygiene education?

a. Wrapping the floss in a “c” shape around each tooth


b. Sliding below the gingival margin, moving up and down
c. Using unwaxed floss
d. Snapping the floss through the contacts

594. When demonstrating his tooth brushing method, the patient positions the bristles 45 degrees to
the long axis of the tooth, toward the occlusal surface of the tooth. What brushing method is
this?

a. Charter’s
b. Fones
c. Roll
d. Bass

595. The hygienist then demonstrates a brushing method that places the bristles at the attached
gingiva and sweeps them up apically, then sweeps over the gingiva and the tooth. What method
is this?

a. Combination
b. Leonard
c. Roll
d. Stillman’s

596. What type of restoration is placed on tooth #18?

a. Amalgam
b. Composite
c. Porcelain onlay
d. Gold inlay

597. The patient has a green stain on the facial surface of the upper anterior teeth. Which statement
is true in regard to this type of stain?

a. The stain is intrinsic and due to chromogenic bacteria


b. The stain is extrinsic and due to chromogenic bacteria
c. The stain is intrinsic and of unknown etiology
d. The stain is extrinsic and due to food stain such as tea

TOP © 2015 - 2023 Tests.com


132
598. The patient has supragingival calculus that covers more than 1/3 but less than 2/3 of the
exposed teeth. What calculus rating does the hygienist assign in the patient’s record?

a. 0
b. 1
c. 2
d. 3

599. When the patient is probing the periodontal pockets, she uses a probe with millimeters marked
at 1, 2, 3, 5, 7 and deeper. What type of probe is this?

a. Michigan
b. Williams
c. Color Coded PSR
d. Nabor

600. During the appointment, the hygienist needs to sharpen a different scaler for use on a particular
tooth. The face of an instrument should be at what angle to the sharpening stone?

a. 110 degrees
b. 45 degrees
c. 90 degrees
d. 180 degrees

601. The dentist wants to place a site-specific therapy at the distal of #16 that uses a drug implanted
in a cord that will be placed and then removed 7-10 days later. What drug is this?

a. Arestin
b. Periostat
c. Doxycycline
d. Tetracycline

602. The patient is assigned a plaque index of 2.1. This is considered:

a. Excellent
b. Good
c. Fair
d. Poor

TOP © 2015 - 2023 Tests.com


133
Practice Test for the NBDHE Dental Hygienist Exam
Case 9 Testlet
Age: 46
Sex: M advised controlling it through diet and exercise.
Height: 6’4”
Weight: 205 Current Medications:
B/P: 134/91 None

Chief Complaint: Dental History:


“I’m packing food between a couple of teeth on The patient has only had one filling, and he says
the lower right side of my mouth. It gets very it is over 10 years old.
tender and the gums are swollen in that area.”
Social History:
Medical History: This patient is a postal carrier. He has been with
The patient has no health concerns. He sees his the USPS for 18 years, 16 of those have been on
physician every year or two for check-ups. His walking the same route. He and his wife’s child
blood pressure was mentioned as being slightly just graduated from high school. The patient
elevated at his last health check, but his doctor dips chewing tobacco a few times each day
.

TOP © 2015 - 2023 Tests.com


134
603. Patients that use spit tobacco are prone to develop _________.

a. Black hairy tongue


b. Squamous cell carcinoma
c. Basal cell carcinoma
d. Pleomorphic adenoma

604. During tobacco cessation education, the hygienist tells the patient that chewing tobacco is
associated with oral cancers. She also tells the patient that the oral lesions usually disappear
after chewing tobacco is discontinued.

a. Both statements are true


b. Both statements are false
c. The first statement is true; the second statement is false
d. The first statement is false; the second statement is true

605. The patient has an oval-shaped elevation in the midline of the dorsal portion of the tongue,
which is void of papilla. What condition is this known as?

a. Hemangioma
b. Pyogenic granuloma
c. Geographic tongue
d. Median rhomboid glossitis

606. During the scaling and root planing procedure, the hygienist observes that tooth #8 is mobile
more than normal, moving approximately 2 mm side to side, but it is not depressible. What
classification of mobility is this?

a. III
b. II
c. I
d. IV

607. The root on tooth #25 appears shorter than the others. The patient reports having trauma to
that tooth as a teenager. What is this condition known as?

a. Resorption
b. Recession
c. Granuloma
d. Chondroma

TOP © 2015 - 2023 Tests.com


135
608. What type of bone loss is evident radiographically on the mesial of tooth #30?

a. Infrabony pocket
b. Horizontal bone loss
c. Vertical bone loss
d. Triangulation

609. When probing, one periodontal pocket has a base of the pocket that is apical to the alveolar
crest. What type of pocket is this?

a. Suprabony pocket
b. Supra-crestal pocket
c. Supra-alveolar pocket
d. Infrabony pocket

610. Which of the following is NOT correct in regard to the histologic changes of the periodontium in
cases of periodontal disease?

a. Vascular proliferation
b. Junctional epithelium protrudes into the connective tissue
c. Tightening of the intercellular spaces in the junctional epithelium
d. Collagenase actively breaks down connective tissue

611. Following the patient’s periodontal scaling and root planing appointments, the dentist has the
hygienist place Arestin (Minocycline hydrochloride) in appropriate areas that have more
advanced stages of disease. This drug has an extended release for up to how many days?

a. 7 days
b. 14 days
c. 21 days
d. 30 days

612. What method of delivery is used for Minocycline hydrochloride (Arestin) into periodontal
pockets?

a. chip
b. capsule
c. liquid
d. Injection

TOP © 2015 - 2023 Tests.com


136
Practice Test for the NBDHE Dental Hygienist Exam
Case 10 Testlet
Age: 32 Current Medications
Sex: F Dilantin 30mg qd
Height: 5’6”
Weight: 132 Dental History:
B/P: 140/78 The patient enjoys going to the dentist. She
brushes her teeth independently each day with
Chief Complaint: fluoride toothpaste.
The patient’s caregiver states that the patient
has broken off several of her teeth. She also has Social History:
heavy bleeding when she brushes around her The patient’s caregiver reports that the patient
lower canines. has not been eating as much as she used to. She
also seems to be losing weight. Otherwise she is
Medical History: participating in normal functions and activities.
Down’s Syndrome She does enjoy drinking juice.
History of seizures- The patient’s last seizure
was 42 days ago.

TOP © 2015 - 2023 Tests.com


137
613. What is the patient taking medication for?

a. Down Syndrome
b. Blood pressure
c. Seizures
d. Cholesterol

614. The patient has a mesh like white lesion on the buccal mucosa that cannot be wiped off.
Between the white lines the tissue appears normal. What is this condition?

a. Lichen Planus
b. White sponge nevus
c. Hyperkeratosis
d. Leukoplakia

615. During the intra and extra oral exam on this patient, which would be typical to find?

a. Short neck
b. Dry, fissured lips
c. Protrusive, tongue
d. All of the above

616. Patient’s with Down syndrome have an anomaly of which chromosome?

a. 18
b. 21
c. 22
d. 23

617. What oral manifestation is NOT found in persons with Down syndrome?

a. Mandibular prognathism
b. Decreased salivary flow
c. Increased risk to periodontal disease
d. Premature tooth eruption

618. The patient’s last seizure was over a month ago. It lasted about 20 seconds and she experienced
a transient loss of consciousness. What type of seizure was this?

a. Generalized tonic-clonic
b. Grand mal
c. Generalized absence
d. Complex partial

TOP © 2015 - 2023 Tests.com


138
619. The hygienist sees plaque film at the gingival margins but no soft debris under the margin. What
plaque score would the patient be given?

a. 0
b. 2
c. 1
d. 3

620. The patient’s oral hygiene has not been a concern in the past, yet the patient has a high rate of
recurrent tooth decay. What is a likely cause?

a. The patient has a high intake of fruit juice


b. The patient cannot floss without assistance
c. Hypercalcification of the enamel
d. Hyperfluorosis

621. Which fluoride would be contraindicated for this patient?

a. Acidulated phosphate fluoride


b. Neutral fluoride
c. Fluoride varnish
d. All fluorides are contraindicated for this patient

622. What would be a likely contributor in regard to the increase of bleeding during brushing around
the lower canine teeth?

a. Localized moderate periodontal disease


b. Rotation of the teeth
c. Irregular tooth brushing
d. Dexterity limitations of the patient

TOP © 2015 - 2023 Tests.com


139
Practice Test for the NBDHE Dental Hygienist Exam
Case 11 Testlet
Age: 49 Dental History:
Sex: M The patient wore a lower partial denture until
Height: 6’3” his accident that damaged some of his lower
Weight: 185 right teeth as well.
B/P: 123/80
Social History:
Chief Complaint: The patient is receiving workers compensation
This is the patient’s first dental visit since a as his primary income source. His wife has gone
work-related accident last spring. He admits back to work part-time at the high-school as a
that early on after the accident he did not do teacher’s aide. The patient is hoping to start
much to take care of his teeth, but he has operating a small vending machine business at
begun being more proactive when it comes to an area highway rest stop. The patient was
his oral health. battling depression after his accident but has
been receiving counseling and is doing much
Medical History: better. He now does many tasks independently
The patient was in a work-related accident, around the house such as washing dishes, doing
which caused blindness in both eyes. the laundry and making coffee.

Current Medications:
Ambien 10mg qd

TOP © 2015 - 2023 Tests.com


140
623. When escorting the patient to the operatory, what method is the best used?

a. Take the patient by the hand, or place his hand on your shoulder
b. Allow the patient to follow your voice
c. Ask the patient to have someone assist him
d. Explain the surroundings such as steps or doorways

624. When discussing home care methods, what is the best approach for a visually impaired patient?

a. Allow the patient to listen as you describe a particular method.


b. Use a “touch, tell, feel” approach.
c. Move the patient’s hands to show them what to do.
d. Use a tell and listen approach.

625. The patient states that he feels a rough area near the gumline on the facial of #8. What is the
likely cause?

a. Exposed dentin due to gum recession


b. Heavy marginal calculus
c. Enamel hypoplasia
d. Margin of his porcelain crown

626. What condition may affect the upper left posterior molars?

a. Supereruption
b. Localized severe bone loss
c. Inflammation of the nasal sinus
d. Mobility

627. What is the vertical radiopaque area near the apex of 8 and 9?

a. Median suture
b. Nasal septum
c. Nasopalatine foramen
d. Superior foramina

628. During subgingival exploration, the hygienist feels an extension on the distal of #2 as she pulls
up with her explorer, but does not feel it as she goes down. What is she likely feeling?

a. CEJ
b. Subgingival calculus
c. Amalgam overhang
d. Crown margin

TOP © 2015 - 2023 Tests.com


141
629. The patient drinks coffee on a frequent basis. What type of stain would be typical?

a. Green stain
b. Black line stain
c. Brown stain
d. Orange stain

630. During the intraoral examination, the patient’s palate is found to have an irregular lesion near
the front of the hard palate that is red and has tissue sloughed away. What is the likely cause?

a. Thermal burn
b. Cyclic neutropenia
c. Pleomorphic adenoma
d. Herpes

631. The patient appears to have elongated papilla of the tongue that is brown and black in
appearance. The hygienist suspects the patient is using what as a mouthwash?

a. Chlorhexidine
b. Listerine
c. Hydrogen peroxide
d. None

632. The patient has an opaque, white lesion on the buccal mucosa on the right side, near the
fractured teeth. What is the likely diagnosis of this lesion?

a. Frictional keratosis
b. Gingival hyperplasia
c. Traumatic ulcer
d. Irritation fibroma

TOP © 2015 - 2023 Tests.com


142
Practice Test for the NBDHE Dental Hygienist Exam
Case 12 Testlet
Age: 12 has done well with symptoms since receiving
Sex: F medication to manage the condition.
Height: 5’3”
Weight: 112 Current Medications:
B/P: 103/59 Concerta 27mg qd

Chief Complaint: Dental History:


“The right side of my mouth hurts. I can’t open The patient has two congenitally missing teeth.
it well since my soccer accident this morning.” Her father was missing the same teeth.

Medical History: Social History:


The patient was born with a unilateral cleft lip The patient is a member of a community
which was repaired during her early childhood. softball league. Last year they advanced all the
She was diagnosed with ADHD 3 years ago, but way to state where they came in 3rd place.

TOP © 2015 - 2023 Tests.com


143
633. What could be causing the discomfort that the patient is experiencing?

a. Bruising from an injury the week before


b. #T preparing to exfoliate
c. Fracture of the jaw
d. Decay of #3

634. What permanent teeth appear to be congenitally missing?

a. #6 and 11
b. #7 and 10
c. #1 and 7
d. #11 and 32

635. What is the opacity on the mesial of #5?

a. Retained primary canine


b. Fractured tooth cusp
c. Retained primary second molar
d. Odontoma

636. Which tooth in the upper left quadrant is rotated?

a. #5
b. #11
c. #12
d. #15

637. Which tooth shows the Amelogenesis stage of tooth development?

a. #31
b. # 16
c. #18
d. #6

638. The patient was born with a bilateral cleft lip on the right side. A cleft in this area occurs when
what processes fail to fuse during fetal development?

a. Frontonasal and stomadeum


b. Frontonasal and maxillary
c. Maxillary and medial nasal
d. Medial nasal and frontonasal

TOP © 2015 - 2023 Tests.com


144
639. The patient asks why her remaining baby teeth have not fallen out yet. At what age do
mandibular second bicuspids typically erupt?

a. 9-10
b. 11-12
c. 13-14
d. 7-8

640. Which of the following is NOT true in regard to ADHD?

a. Treatment may include stimulants


b. May cause irritability and impulsiveness
c. More common in girls
d. Can make it difficult to acquire new skills

641. What could the hygienist suggest as a way to help further protect the patient’s orofacial area
from trauma associated with playing softball?

a. Helmet use
b. Mouthguard
c. Not clenching her teeth
d. Complete her dental treatment timely

642. What is the round circular area around the crown of tooth #29?

a. Pseudocyst
b. Periodontal cyst
c. Dentigerous cyst
d. Primordial cyst

TOP © 2015 - 2023 Tests.com


145
Practice Test for the NBDHE Dental Hygienist Exam
Case 13 Testlet
Age: 29 Current Medications:
Sex: M None
Height: 6’2”
Weight: 178 Dental History: The patient has not been to the
B/P: 121/68 dentist for 2 years. He recently started a new
job and is taking advantage of having dental
Chief Complaint: “My gums bleed when I brush insurance by coming in to have a routine check-
and floss.” up and cleaning. He reports a history of grinding
his teeth. 3 months ago he had an oral piercing
Medical History: placed in the middle of his lower lip.
The patient had a spinal cord injury 6 years ago
when he fell off of an ATV. He is restricted to a Social History:
wheel chair and cannot use his lower limbs. His The patient graduated from college 2 years ago
upper body, including arms and hands, function and is now living independently. He is on a
normally. basketball team that meets once each week to
play at the community recreation center in the
next town.

TOP © 2015 - 2023 Tests.com


146
643. The patient requires some help transferring himself to the operatory chair during this
appointment. During a one person assisted transfer, where is the most appropriate place that a
hygienist should place their hands?

a. Under the patient’s elbows


b. Under the patient’s arms
c. Around the patient’s waist
d. Around the patient’s safety-waist belt

644. Which tooth has a crown still intact and exhibits a periapical radiolucency?

a. #1
b. #14
c. #19
d. #29

645. What is the long, radiolucent line that extends down through each side of the patient’s jaw, as
evident on the x-ray?

a. Mandibular canal
b. Mandibular foramen
c. Border of the mandible
d. External oblique ridge

646. The patient has a bluish, 3cm bulge under the tongue. What is this called?

a. Sialolith
b. Ranula
c. Mucocele
d. Rhomboid glossitis

647. Which tooth has the doctor diagnosed as needing a porcelain crown?

a. #29
b. #30
c. #18
d. #19

TOP © 2015 - 2023 Tests.com


147
648. The patient is showing signs of moderate gum recession that is localized to #24 and 25. What is
the likely cause?

a. Failure to floss
b. Aggressive brushing
c. Chewing fingernails
d. Irritation from oral piercing

649. During the intraoral examination, the hygienist records a healthy probing depth that has plaque
and bleeds. What PSR score would this tooth receive?

a. 0
b. 1
c. 2
d. 3

650. The patient experiences occasional pain near the right Temporomandibular joint. What oral
condition could be contributing to this discomfort?

a. Crepitation
b. Bruxism
c. Acid reflux erosion
d. Failed restorations

651. What would an appropriate therapy be for his TMJD symptoms?

a. Bite splint
b. Sport guard
c. Restoring missing and fractured teeth
d. Correcting his oral care habits

652. During nutritional counseling, the patient states that he is an avid gum chewer. Because this
most likely is contributing to his caries rate, which type of gum would best be advised for this
patient?

a. Sugar-free gum
b. Xylitol containing gum
c. Bubble gum
d. No gum

TOP © 2015 - 2023 Tests.com


148
Practice Test for the NBDHE Dental Hygienist Exam
Case 14 Testlet
Age: 19 Current Medications:
Sex: F Insulin
Height: 5’5” Birth control pills
Weight: 165
B/P: 156/94 —SEE KAPLAN PAGE 364 Dental History: The patient comes for routine
preventive care appointments but has
Chief Complaint: The patient is here for her consistently developed dental caries as the
routine preventive care appointment. She years have gone by.
expresses concern with one tooth in the lower
right posterior that is sensitive to sweet. She Social History: The patient is in her freshman
says she also has some off and on pain on the year of university and is studying journalism.
lower teeth behind her 12 year molars. She recently joined Weight Watchers and is part
of an on-campus student nutritional support
Medical History: group that she found when taking her required
The patient is Type 2 Diabetic. physical education class. She has made an effort
to begin drinking more bottled water rather
than diet sodas when she purchases drinks at
on-campus vending machines.

TOP © 2015 - 2023 Tests.com


149
653. After the patient was seated in the operatory chair her blood pressure was recorded. Based on
the reading of 156/94, what is the most appropriate measure for the hygienist to take?

a. Recheck her in 5 minutes


b. Perform normal treatment and monitor her at the next visit
c. Perform no treatment and refer her for a medical consultation
d. Her blood pressure is normal

654. What classification of hypertension does the patient have?

a. Normal
b. Stage I
c. Stage II
d. Stage III

655. The hygienist asks the patient what her last blood glucose reading was. A normal level will be:

a. 126-240 mg/dL
b. 100-125 mg/dL
c. 70-99 mg/dL
d. Less than 70 mg/dL

656. What should be kept on hand in the dental office should a patient like this have a hypoglycemic
episode?

a. Diet soda
b. Splenda
c. Orange juice
d. Bottled water

657. The difference between Type I and Type II diabetes is that Type II _______.

a. is non-insulin dependent
b. is more serious
c. requires daily insulin injections
d. has a juvenile onset

TOP © 2015 - 2023 Tests.com


150
658. The patient states that she is trying to choose healthier drinks when she is on campus between
classes and meals. What is the healthiest choice given her diabetic condition?

a. Milk
b. Diet Soda
c. Bottled water
d. Tap water

659. During nutritional counseling, the hygienist explains that soda is too erosive to the teeth. The
patient should drink diet soda because it is less erosive to teeth.

a. Both statements are true


b. Both statements are false
c. The first statement is true; the second statement is false
d. The first statement is false; the second statement is true

660. The patient is currently taking birth control pills. What type of medication could cause birth
control pills to fail?

a. Hyperglycemic medications
b. Blood pressure medications
c. Diet pills
d. Antibiotics

661. Because the patient is diabetic, she is more prone to:

a. Increased saliva flow


b. Periodontal disease
c. Enamel hypercalcification
d. Pyogenic granuloma

662. The patient shows mild erosion on the lingual surfaces of the maxillary anterior teeth. What is
the probable cause of this condition?

a. Increased acidic diet


b. Whitening the teeth
c. Bulimia
d. Sucking on lemons

TOP © 2015 - 2023 Tests.com


151
Practice Test for the NBDHE Dental Hygienist Exam
Case 15 Testlet
Age: 34 Current Medications:
Sex: M Procardia (for blood pressure)
Height: 5’ 10”
Weight: 149 Dental History:
B/P: 165/105 The patient has a history of fairly healthy teeth.
Chief Complaint He has only had a few cavities in the past and
has no problems with gingivitis.
Medical History:
The patient is a smoker and smokes Social History:
approximately one pack of cigarettes each day. The patient is in a bowling league and competes
He also is under the care of his physician for every Tuesday night. His son plays little league
high blood pressure. baseball, and he is an assistant coach for the
team.

TOP © 2015 - 2023 Tests.com


152
663. Based on the patient’s blood pressure reading, what is the next appropriate step?

a. Refer the patient for a medical consult


b. Have patient seek immediate medical attention
c. Retake the blood pressure in 5 minutes
d. Perform normal treatment

664. The patient has enlarged, fibrous papillae in the anterior portion of the mouth. What is the likely
cause?

a. Inadequate home care


b. Elevated blood pressure
c. Drug induced
d. Tobacco use

665. Which of the patient’s teeth is supererupted?

a. 2
b. 16
c. 17
d. 32

666. During the tobacco cessation portion of the hygienist’s assessment, she should tell the patient
that smoking is a risk factor for all of the following EXCEPT:

a. Obesity
b. Lung cancer
c. Heart disease
d. Periodontal disease

667. In addition to tobacco placing this patient at a higher risk for oral cancer, what other factors
would place him at a higher risk?

a. Alcohol use
b. HPV
c. Sun exposure
d. All of the above

TOP © 2015 - 2023 Tests.com


153
668. During the oral cancer screening, what portion of the lips is a frequent site of oral cancer and
should be checked by the hygienist?

a. Vermillion zone
b. Philtrum
c. Angular commisures
d. Labial commisures

669. The patient’s hard palate has small red spots that are likely caused by his smoking habit. What is
this called?

a. Stomatitis
b. Ecchymosis
c. Hematoma
d. Sialometaplasia

670. What stage of hypertension is this patient currently classified under?

a. Normal
b. Stage II
c. Stage III
d. Stage I

671. If this patient were to go into cardiac arrest and require CPR, approximately how deep would
chest compressions need to be in order to be effective?

a. 1.5-2cm
b. 6-7.5 cm
c. 2.5-4 in
d. 1.5-2 in

672. If the patient needed to have oxygen administered during their appointment, what color tank
would the oxygen be delivered from?

a. Blue
b. Red
c. Green
d. Orange

TOP © 2015 - 2023 Tests.com


154
Practice Test for the NBDHE Dental Hygienist Exam
Case 16 Testlet
Age: 54 Current Medications:
Sex: M None
Height: 5’11”
Weight: 172 Dental History:
B/P: 131/92 The patient had several dental implants placed
4 years ago in areas where he was missing
Chief Complaint: “I seem to get cold sores very permanent teeth.
frequently.”
Social History:
Medical History The patient is a retired US marine who now
Knee replacement - 4 months ago operates a small welding business out of his
home.

TOP © 2015 - 2023 Tests.com


155
673. The patient complains of frequent cold sores. What virus causes this condition?

a. HPV
b. HSV
c. PPV
d. HIV

674. What preventive method does the hygienist need to use to ensure she does not contract the
virus from the patient?

a. Double glove
b. Avoid use of the ultrasonic
c. Pre procedural mouth rinse
d. None of the above

675. What condition could possibly occur if the hygienist did not use universal precautions and
contracted the herpes virus in her nail bed?

a. Headache
b. Irritability
c. Herpetic whitlow
d. Finger loss

676. Because the patient’s knee was replaced 4 months ago, what is the necessary preventive step
for their dental treatment?

a. None, 3 months have lapsed since her joint replacement


b. Antibiotic premedication one hour before the appointment
c. Antibiotic premedication one day before the appointment
d. No dental treatment until 6 months have lapsed

677. What type of dental implants has the patient had put in place?

a. Intramucosal
b. Endosseous
c. Subperiosteal
d. Endodontic

TOP © 2015 - 2023 Tests.com


156
678. What type of scaler should be used on the dental implants to prevent scratching of the titanium
material?

a. Plastic
b. Stainless steel
c. Carbon steel
d. Metal ultrasonic tip

679. During the extraoral assessment, the TMJ is noted to have a rough, grinding feeling upon
opening and closing. What is this condition called?

a. Deviation
b. Disk degeneration
c. TMJD
d. Crepitus

680. During the extraoral assessment, what should be felt for during palpation of the lymph nodes?

a. Swelling
b. Hardness
c. Enlargement
d. All of the above

681. Which of the following is NOT something that needs to be recorded about an abnormal skin
lesion on the patient’s face?

a. Color
b. Size
c. Date of origin
d. Shape

682. At what point in the life cycle of herpetic ulcerations should treatment be avoided?

a. Initial tingling
b. Active ulceration
c. Dried ulceration
d. Treatment should not occur during any stage of herpetic ulcerations

TOP © 2015 - 2023 Tests.com


157
Practice Test for the NBDHE Dental Hygienist Exam
Case 17 Testlet
Age: 62 Current Medications:
Sex: F Rituxan (for RA)
Height: 5’4”
Weight: 151 Dental History:
B/P: 145/95 The patient has had extensive restorative
treatment over the past decade. She comes in
Chief Complaint: The patient is unhappy with about every 12 months to have her teeth
the way her upper front teeth appear to be cleaned despite the last hygienist’s efforts to
“longer” than the other teeth. She also encourage more frequent appointments.
complains of yellow plaque built up on the
lower teeth. She started using a new toothpaste Social History:
and reports that her teeth are sometimes The patient spends 2 days a week volunteering
sensitive. at a thrift-store that benefits the county
hospital. She would enjoy working more
Medical History: frequently, but she is not physically comfortable
The patient has rheumatoid arthritis. enough to do so. She enjoys reading books and
has a small dog, named “Tito.”

TOP © 2015 - 2023 Tests.com


158
683. What habit might be contributing to the gum recession in the area of the patient’s upper
anterior teeth?

a. Sucking lemons
b. Aggressive flossing
c. Bone health
d. Abrasive brushing

684. To remove the patient’s moderate supragingival calculus, the hygienist selects an electronic
scaler that produces 25,000-50,000 vibrations per second and vibrates in a linear direction. What
type of scaler did she choose?

a. Sonic
b. Ultrasonic piezoelectric
c. Ultrasonic magnetorestrictive
d. Air polisher

685. What would better help prevent moderate calculus deposits on the patient’s teeth?

a. Changing toothbrushes
b. More frequent prophylaxis appointments
c. Using tartar control toothpaste
d. Nothing, it has to do with her metabolism

686. Which tooth is the patient missing?

a. #4
b. #6
c. #3
d. #10

687. What orofacial region may the patient experience some difficulties with due to her RA?

a. TMJD
b. TMJ
c. Delayed eruption
d. Decreased appetite

TOP © 2015 - 2023 Tests.com


159
688. To discourage pain in the TMJ during the dental appointment, what can the hygienist do?

a. Apply a warm compress to the area


b. Have the patient premedicate with an NSAID
c. Use a bite block to help the patient keep her mouth open
d. Take frequent breaks and allows the patient to rest her jaw

689. The patient has a bony hard protuberance in the midline of their palate. What is this called?

a. Torus mandibularis
b. Torus palatinus
c. Odontoma
d. Exostoses

690. When choosing a curette for instrumentation, the hygienist selects a universal curet rather than
an area specific curette. What is the difference between the two types?

a. The lower shank of the universal curette is longer.


b. The face of the area specific curette is smaller.
c. The there are two cutting edges on a universal curette.
d. The toe of the universal curette is rounder.

691. During hand scaling of this patient, the hygienist should have her forearms and hands _______.

a. parallel to the floor


b. slightly lowered below the level of her elbows
c. slightly elevated above her elbows
d. where the arms are parallel to the floor and the hands are perpendicular to the floor

692. During scaling, the hygienist uses a fulcrum rest, which uses a finger from her non-dominant
hand to concentrate lateral pressure against the tooth in order to control the instrument stroke.
What type of fulcrum is this?

a. Basic intraoral
b. Advanced piggy-back
c. Advanced finger-on-finger
d. Advanced stabilized

693. The patient reports having just recovered from shingles a few months ago. What virus causes
shingles?

a. Herpes zoster
b. Hepatitis C
c. Coxsackievirus
d. Epstein-Barr virus

TOP © 2015 - 2023 Tests.com


160
Practice Test for the NBDHE Dental Hygienist Exam
Case 18 Testlet
Age: 56 Dental History:
Sex: F The patient has had relatively healthy teeth in
Height: 5’3” the past 4 years. Prior to that she had a history
Weight: 132 of scaling and root planing associated with
B/P: 125/85 active periodontal disease. She had periodontal
maintenance visits every 3 months, then 4
Chief Complaint: months and has now extended recare visits to
The patient has no concerns at this time. She is every 6 months. The patient missed her last 2
here for her scheduled preventive care appointments due to family reasons.
appointment.
Social History:
Medical History: The patient is a sales manager at a local clothing
The patient is anemic and also has retailer. The patient’s mother was diagnosed
hyperthyroidism. She is under the care of her with cancer 18 months ago and passed away 5
physician and was last seen 4 months ago. months ago. The patient was caring for her
mother and as a result was not able to make it
Current Medications: to all of her dental appointments because she
Iron supplement had already missed so much work.
Synthroid

TOP © 2015 - 2023 Tests.com


161
694. The difference in iron-deficiency anemia and other anemia is that iron-deficiency anemia
requires _____________.

a. B12 supplementation
b. antibiotic premedication
c. blood transfusions
d. only iron supplementation

695. Which of the following is NOT a clinical feature associated with hyperthyroidism that might be
seen in this patient?

a. Large jaws
b. Rapidly accelerated caries
c. Delayed tooth eruption
d. Osteoporosis of the alveolar bone

696. What features may be visible during the extraoral exam of a patient with hyperthyroidism?

a. Sunken eyes
b. Thick hair
c. Weight gain
d. Dry, cracked skin

697. The furcation involvement at tooth #18 is visible clinically. What class of furcation involvement is
this?

a. Class V
b. Class III
c. Class IV
d. Class II

698. What type of film would be necessary to achieve a high quality view of the apex and bone
around #18?

a. Bitewing
b. Periapical
c. Panoramic
d. Digital

TOP © 2015 - 2023 Tests.com


162
699. During the intraoral assessment, the mouth mirror continues to fog up. What options can the
clinician use to prevent this?

a. Ask the patient to breathe through the nose


b. Rub the mirror along the cheek to coat with saliva
c. Use defogger or warm water to rinse the mirror
d. All of the above

700. The hygienist is having a difficult time scaling a piece of subgingival calculus off of the distal of
#27. What portion of the toe on a universal curette should be used for deposit removal?

a. Lower 1/3
b. Lower 2/3
c. Upper 3/4
d. The entire toe

701. The patient requests nitrous oxide use during the remainder of her appointment. At what level is
the average desired effect of nitrous oxide achieved?

a. 10%
b. 60%
c. 35%
d. 55%

702. The patient has continued using a sensitivity toothpaste to prevent hypersensitivity in areas of
recession. Which of the following ingredients is NOT an active ingredient of sensitivity
toothpastes?

a. Potassium nitrate
b. Hydrogen peroxide
c. Calcium hydroxide
d. Sodium citrate

703. The patient asks about irrigating around existing pockets that tend to pack food. How deep
below the gingival margin should the hygienist tell her is safe enough to place a rubber tip
irrigator?

a. Only supragingivally to the margin


b. 2mm below the margin
c. 5mm below the margin
d. As deep as comfortably possible

TOP © 2015 - 2023 Tests.com


163
704. The hygienist decides to use an air-polishing device to remove some of the tenacious stain that
has built up since the patient’s last appointment. Approximately what psi does an air pressured
polisher function at?

a. 40-100
b. 75-150
c. 150-200
d. 175-250

TOP © 2015 - 2023 Tests.com


164
Practice Test for the NBDHE Dental Hygienist Exam
Case 19 Testlet
Age: 16 Current Medications:
Sex: F Albuterol prn
Height: 5’7”
Weight: 145 Dental History:
B/P: 111/76 The patient has not been seen for 2 years. At
her last appointment her teeth were cleaned
Chief Complaint: ”I’m unhappy with the way my and decay was diagnosed as a small DO on #29.
teeth look. I am about to get braces. My
orthodontist told me to have my teeth cleaned Social History:
and cavities filled before he will put the braces The patient is a student at the local high school.
on my teeth.” She plays trumpet in the school band.

Medical History
The patient has asthma. Her last attack was 8
months ago.

TOP © 2015 - 2023 Tests.com


165
705. Which of the patient’s teeth appear to have an incomplete root canal?

a. #19
b. #20
c. #29
d. #30

706. During the extraoral assessment, the patient exhibits a butterfly-shaped rash across the face.
This is a classic feature of what condition?

a. Pemphigus vulgaris
b. Lupus
c. Erythema multiforme
d. Mono

707. The patient states she does not brush along the gumlines because they bleed easily. What is the
likely cause?

a. Irritation due to albuterol use


b. Periodontal disease
c. Marginal inflammation
d. Trauma

708. According to Black’s classification of caries, tooth #9 has what class of decay based on the
appearance of the radiograph?

a. II
b. IV
c. III
d. VI

709. Which of the following appearances of gum tissue evident in the patient’s mouth would be
considered healthy?

a. Soft
b. Coral/pink
c. Shiny with thin epithelial tissue
d. Blunted

TOP © 2015 - 2023 Tests.com


166
710. The patient states that she can feel a small pit in the back of her anterior teeth. What teeth are
capable of having a lingual pit?

a. Maxillary central incisors


b. Mandibular central incisors
c. Maxillary lateral incisors
d. Maxillary canines

711. During the intraoral assessment, the clinician sees one of the teeth has a Cusp of Carabelli.
Which tooth would this be on?

a. Mandibular first molar


b. Maxillary first molar
c. Maxillary second molar
d. Mandibular second molar

712. What should the hygienist tell the patient to do in order to reduce the amount of gingival
bleeding?

a. Use an antiseptic mouth rinse


b. Use an electric toothbrush
c. Brush along the gum margin
d. Rinse with warm salt water

713. What supplement may help the patient maintain healthy enamel during their orthodontic
therapy?

a. Fluoride
b. Proxa-brush
c. Electric toothbrush
d. Antiseptic mouth rinse

714. The patient has green stain on the lingual of the upper anterior teeth, typical of stain found in
persons that smoke illegal substances. What polishing method would be appropriate for the
removal of this stain?

a. Air polishing
b. Fine coarse pumice
c. Rubber cup polishing
d. Selective polishing

TOP © 2015 - 2023 Tests.com


167
715. What should the patient be advised to do after using her albuterol inhaler?

a. Rinse with water


b. Brush his teeth
c. Rest for 5 minutes
d. Do not brush his teeth for 1 hour

TOP © 2015 - 2023 Tests.com


168
Practice Test for the NBDHE Dental Hygienist Exam
Case 20 Testlet
Age: 22 Current Medications:
Sex: F Prenatal vitamin
Height: 5’1”
Weight: 125 Dental History:
B/P: 118/74 The patient has had a few fillings in the past,
but generally possesses good overall oral
Chief Complaint: hygiene
The patient is due for a preventive care
appointment. She also states that she had a Social History:
weird growth on her gums but that it went The patient was an accountant at a credit union
away. However she is finding that she is but decided to be a stay at home mother after
experiencing some irritation on her tongue. giving birth to her child. She is currently doing
some freelance work from her home a couple of
Medical History: days each week.
The patient had a C-section 6 months ago. She
is currently breastfeeding her infant.

TOP © 2015 - 2023 Tests.com


169
716. What is the hormone-induced growth that is likely the cause of the lesion on the patient’s gums
that went away after having her baby?

a. Papillary hyperplasia
b. Fibrous hyperplasia
c. Pregnancy granuloma
d. Peripheral granuloma

717. What condition would cause the patient to have a map-shaped erythematous lesion with a
white border on the dorsal surface of her tongue?

a. Geographic tongue
b. Median rhomboid glossitis
c. Linea alba
d. Elongation of filiform papillae

718. The patient states she has increased sensitivity in her upper anterior teeth since her pregnancy.
Which of the following is the likely cause?

a. Hormonal changes
b. Rampant gum recession
c. Acid erosion
d. Homeopathic toothpaste use

719. The patient states she has had some tissue sloughing and that is why she changed toothpastes.
What was most likely an ingredient in the last toothpaste that would have caused the sloughing?

a. Fluoride
b. SLS
c. Sodium bicarbonate
d. Xylitol

720. During oral hygiene education, the hygienist addresses care of the patient’s newborn child’s
teeth. What information would be accurate?

a. Prolonged nursing age can lead to caries


b. Prolonged nursing age can lead to inadequate jaw growth
c. Cleansing the baby’s mouth after breastfeeding is essential
d. The baby should have his first exam by 6 months of age

TOP © 2015 - 2023 Tests.com


170
721. What should the hygienist recommend to brush the baby’s teeth with, once his first teeth erupt?

a. A washcloth
b. A soft bristled toothbrush with tap water
c. A brush with a bb sized amount of toothpaste
d. The mother’s toothbrush and tap water

722. At what age can she expect her baby to have his first tooth erupt?

a. 6-10 months
b. 8-13 months
c. 11-15 months
d. 12-18 months

723. What age should the patient expect her son to have his first permanent tooth erupt?

a. 5 years
b. 6 years
c. 8 years
d. 12 years

724. During probing, the patient’s UR quadrant has plaque but no calculus, has no pockets deeper
than 2 mm and bleeds on probing at 2 teeth. What PSR score will be assigned?

a. 0
b. 1
c. 2
d. 3

725. How will the hygienist conduct the intraoral examination to areas around the tongue?

a. She will ask the patient to move her tongue to the side
b. She will retract the tongue with her mirror
c. She will hold the tongue with gauze
d. She will feel by palpitation

726. The hygienist assigns the patient a plaque score of 1. What does this mean?

a. There was no plaque


b. There was plaque at the gingival margin
c. There was plaque within the gingival margin
d. There was gross soft debris within the pockets and margins

TOP © 2015 - 2023 Tests.com


171
Practice Test for the NBDHE Dental Hygienist Exam
Case 21 Testlet
Age: 7 Current Medications:
Sex: M None
Height: 4’3”
Weight: 65 Dental History:
B/P: 107/70 Patient just moved to the area and is a new
patient of record. He has not been to a dentist
Chief Complaint: in one year. Previous to the move, he was seen
The parents have noticed that one of the once a year by a general dentist.
permanent teeth has not erupted in the place
of the deciduous tooth. It has been about 4 Social History:
months since the child lost the tooth. Patient is in 1st grade at a local public school.
Loves playing sports and is always outside
Medical History: playing with his brother and sister.
Patient had his adenoids and tonsils removed
when he was 4 years old, due to many throat
and ear infections

TOP © 2015 - 2023 Tests.com


172
727. During the appointment, the patient receives a topical fluoride treatment. Between what ages is
fluoride most effective in preventing cavities?

a. 6 months to 14 years old


b. 6 months to 21 years old
c. 12 months to 16 years old
d. 12 months to 21 years old

728. The dentist proceeds to check on the eruption patterns of the child. When looking at the x-ray, is
the child missing a permanent tooth?

a. Yes, he is missing #7.


b. Yes, he is missing number #21.
c. Yes, He is missing #30.
d. No, he is not missing any permanent teeth.

729. When looking at the same panoramic image, you see a dark, circular area distal to tooth #18.
What are you looking at?

a. Static bone cyst


b. Traumatic bone cyst
c. Tooth bud
d. Abscess

730. The mother is concerned about a permanent tooth missing. Your office does not have a
panoramic x-ray machine. What is the next best type of x-ray to take?

a. Bitewings
b. Periapical
c. Occlusal
d. Full-mouth series

731. The dentist notes the child has excessive crowding and will need to be referred to a specialist.
What specialist would the dentist refer the parents and child to?

a. Orthodontist
b. Periodontist
c. Prosthodontist
d. Endodontist

TOP © 2015 - 2023 Tests.com


173
Practice Test for the NBDHE Dental Hygienist Exam
Case 22 Testlet
Age: 57 Current Medications:
Sex: M Lisinopril q.d. baby aspirin q.d.
Height: 6’0”
Weight: 205 Dental History:
B/P: 128/84 Patient has been going to the dentist on and off
throughout his life. Had scaling and root planing
Chief Complaint: 10 years ago, along with extractions of most of
After having his implants placed 6 months ago, his mandibular molars.
the patient is ready to proceed with his
removable prosthesis. Social History:
Patient has two children and recently became a
Medical History: grandfather for the first time. The patient wants
Smoker since the age of 16, but quite 2 years to be able to smile again, since he hasn’t smiled
ago. since losing his teeth many years ago.

TOP © 2015 - 2023 Tests.com


174
732. Looking at the x-ray, what type of implant was used?

a. Endosteal
b. Osseointergrated
c. Subperiosteal
d. Transosteal

733. Considering the dental history of this patient, what type of prosthesis is the patient likely to be
completing?

a. Denture
b. Partial denture
c. Bridge
d. Crown

734. Looking at the x-ray, what is the radiopaque image in the lower left and right corner?

a. Spine
b. Vertebrae
c. Hyoid bone
d. Mandible

735. All of the following, except one, are instruments used in debridement around dental implants.
Which one is the exception?

a. Plastic resin
b. Graphite
c. Stainless steel
d. Gold-tipped

736. What type of fixed prosthesis is being utilized in quadrant #1?

a. Implant
b. Maryland bridge
c. Cantilever Bridge
d. Crown

TOP © 2015 - 2023 Tests.com


175
Practice Test for the NBDHE Dental Hygienist Exam
Case 23 Testlet
Age: 78 Current Medications:
Sex: F donepezil q.d. galantamine extended release
Height: 5’4” q.d.
Weight: 164
B/P: 132/84 Dental History:
Patient has not been into a dentist in two years.
Chief Complaint: The patient is new to the clinic.
Husband has noticed that some of her teeth are
broken and that he noticed her bad breath Social History:
starting a couple of weeks ago. Before being diagnosed with Alzheimer’s
disease, the patient was active in the
Medical History: community and was part of a gardening club.
Patient was diagnosed with Alzheimer’s disease She raised three kids in their small town.
about 1.5 years ago. The husband has been able
to care for his wife since the diagnosis. She had
been diagnosed with Raynaud’s disease when
she was in her 30’s.

TOP © 2015 - 2023 Tests.com


176
737. The patient is very cooperative and is allowing you to look in her mouth. When trying to do your
intraoral exam, you notice that the patient has tightening of skin around her mouth, making it
hard for her to open wide. What is the cause?

a. Oral cancer
b. Scleroderma
c. Candida infection
d. Herpes virus

738. The patient has been having a hard time brushing her teeth or forgetting to do so altogether.
Since she has many mandibular teeth that are infected beyond repair, what type of appliance
would the dentist recommend to help the patient restore chewing function?

a. Partial Denture
b. Denture
c. Implants
d. Fixed Bridges

739. Which of the following is not a material used in making a partial denture and denture teeth?

a. Acrylic resin
b. Composite resin
c. Porcelain
d. Glass ionomer

740. What is the proper way to clean a removable prosthesis in the dental clinic?

a. Use water and a hard tooth brush


b. Put denture in disinfectant, clean in an ultrasonic and brush with a denture brush
c. Put denture in disinfectant and place in the ultrasonic
d. Place in ultrasonic for 10 minutes and rinse with water

741. All of the following are soft tissue considerations regarding the use of a prosthesis, except:

a. Angular cheilitis
b. Stomatitis
c. Traumatic ulcers
d. Geographic tongue

TOP © 2015 - 2023 Tests.com


177
Practice Test for the NBDHE Dental Hygienist Exam
Case 24 Testlet
Age: 14 Current Medications:
Sex: M None
Height: 5’9”
Weight: 140 Dental History:
B/P: 107/78 Patient has a full set of braces for over a year.
Doesn’t know when he will be getting them
Chief Complaint: removed. Pt had #14 extracted before braces
Parents would like to know if he is ready to get due to decay.
his wisdom teeth out. He is also having some
soreness near his tonsils again. Social History:
Patient is on the junior varsity football team,
Medical History: plays the quarterback position. Sips on sports
Had strep throat 6 weeks ago and finished a full drinks throughout practice and during games.
round of antibiotics prescribed by his doctor.

TOP © 2015 - 2023 Tests.com


178
742. The parents are concerned about the patient’s wisdom teeth, since some of his friends have
been getting them removed. The dentist states that he is not ready to have them removed.
What else do you notice from the panoramic x-ray regarding the patient’s wisdom teeth?

a. The patient has 3 wisdom teeth.


b. All of the patient’s wisdom teeth are extracted.
c. The patient has 2 wisdom teeth.
d. All of the wisdom have decay.

743. When doing your intraoral exam, you notice that the patient has something lodged in the folds
near his tonsils. When the doctor does his exam, he dislodges the white, hard nodule with a
cotton swab and says that it is nothing to worry about. What most likely is the nodule the
dentist dislodged?

a. Candida
b. Tonsil stone
c. Papilloma
d. Fibroma

744. When cleaning around the brackets, you notice that there is a white line that has formed
underneath the brackets on tooth #27. It cannot be removed. What mostly likely is the white
line?

a. Decalcification
b. Hypocalcification
c. Linea Alba
d. Fluorosis

745. What type of brushing technique should one use with braces?

a. Stillmans
b. Bass
c. Charters
d. Modified Stillmans

746. A patient should wear an athletic mouth guard when playing sports. However, a patient should
not wear an athletic mouth guard if he has a full set of braces.

a. Both statements are true.


b. Both statements are false.
c. First statement is true. Second statement is false.
d. First statement is false. Second statement is true.

TOP © 2015 - 2023 Tests.com


179
Practice Test for the NBDHE Dental Hygienist Exam
Case 25 Testlet
Age: 32 Current Medications:
Sex: F Prenatal vitamins, folic acid
Height: 5’7”
Weight: 145 Dental History:
B/P: 122/82 Pt routinely comes to the dentist every 6
months and has not had periodontal issues in
Chief Complaint: the past.
“My gums are bleeding more than usual”
Social History:
Medical History: This is the patient’s second child. Has a 2-year-
Pt is 3 months pregnant. Patient is currently old son at home. Patient is a stay-at-home
having morning sickness and is vomiting every mother.
morning.

TOP © 2015 - 2023 Tests.com


180
747. When completing this periodontal evaluation, you notice generalized bleeding upon probing,
but there are not any pockets above the score of three. What most likely is the cause of this
condition?

a. Hormones
b. Poor oral hygiene
c. Trauma
d. Periodontal disease

748. The patient states that she has been vomiting every morning due to the pregnancy. What should
you tell her to do about her tooth brushing?

a. Brush and floss your teeth immediately.


b. Wait until nighttime to brush and floss.
c. Only rinse your mouth, do not brush or floss your teeth until the 2nd trimester.
d. Wait 30 minutes after vomiting to brush and floss your teeth.

749. If a pregnant patient needed scaling and root planing, when would be the best time to perform
this procedure?

a. You should not perform this procedure while a patient is pregnant


b. 1st trimester
c. 2nd trimester
d. 3rd trimester

750. When you begin to do the intra oral exam, you notice a lesion on the lower lip, which is shown in
the photo. The patient states that she had it for about 5 days, that she gets them periodically
and that each one lasts about 2 weeks. What is this lesion?

a. Herpes lesion
b. Aphthous ulcer
c. Stomatitis
d. Linea alba

TOP © 2015 - 2023 Tests.com


181
751. All of the following are known triggers for an aphthous ulcer, except:

a. Hormones
b. Stress
c. Vitamin C
d. Injury

TOP © 2015 - 2023 Tests.com


182
Practice Test for the NBDHE Dental Hygienist Exam
Answers with Explanations

1. d - During swallowing, the hyoid bone moves upward as the larynx elevates. After moving upward,
the hyoid bone moves forward then returns to its starting position.

2. b - The sympathetic nervous system releases norepinephrine to raise the heart rate and elevate
blood pressure, triggering the fight or flight reaction in a stressful situation.

3. a - Bell’s palsy causes loss of movement to muscles due to lesions in the facial nerve. This condition
is bilateral and usually is only temporary, though it may take several weeks prior to returning to
normal use.

4. c - Calcium is mobilized in and out of the bloodstream during bone remodeling. This allows new
bone formation to occur in areas of the body where it is needed.

5. a - The trigeminal mandibular nerve passes through the foramen ovale of the sphenoid bone.

6. d - The foramen rotundum is the opening that conveys the maxillary branch of the trigeminal
nerve.

7. b - The vagus nerve, or cranial nerve X is the longest of all cranial nerves. It reaches areas of the
head and neck as well as the heart and stomach.

8. c. Pharynx
e. Esophagus
a. Stomach
b. Small intestine
d. Large intestine

9. c. Muscle
b. Muscle fascicle
e. Myofiber
d. Myofibril
a. Myofilaments

10. c - The trigeminal ganglion does not have an optic division. Its divisions include the opthalmic,
mandibular and maxillary, which enter in by the superior orbital fissure, foramen ovale and
foramen rotundum.

11. a, c & d - Tonsils, Spleen, Thymus are all lymphatic organs. The 4th and final lymphatic organ not
listed is the lymph nodes.

TOP © 2015 - 2023 Tests.com


183
12. a. Trachea
d. Primary bronchi
e. Bronchiole
c. Alveolar duct
b. Alveoli

13. b - The lymphatic system filters fluids and returns them to the bloodstream from various body
tissues. It parallels the venous blood vessels.

14. c - Desmosomes are the cell-to-cell attachments in ameloblasts, which are responsible for forming
tooth enamel.

15. b - Cytoplasm is the aqueous, translucent gel that is enclosed in a cell by a cell membrane. It is the
liquid housing that contains other cellular particles and allows them to move about within the cell.

16. a - The oral cavity tissues such as the buccal mucosa and gingival mucosa are lined with stratisfied
squamous epithelia, the shorter, flatter squamous cells found in multiple layers at the surface of
the oral cavity tissues.

17. c. Cusp of Carbelli


a. Two cusps that are approximately the same length
e. Long central developmental groove
b. “H” or “y” groove patterns int he occlusa surface
d. Greater mesiodistal than faciolingual width

18. c. Lingual papillae


e. Sulcus terminalis
d. Foramen cecum
a. Lateral surface
b. Lingual tonsil

19. c & d - The sublingual fold and lingual frenum are both folds of tissue in the floor of the mouth
below the ventral surface of the tongue.

20. d - The occipoital bone is the bone on the posterior lower portion of the skull.

21. b - The masseter is the most superficially located muscle used in mastication and is the strongest
muscle used.

22. a - Mandibular second molars have shallower mesial root concavities that are not as prominent as
the other teeth listed.

23. c - The maxillary first premolar is the only premolar to have two roots, and it’s furcation is toward
the apical 1/3 to 1/2 of the tooth.

24. a & c - The stomach stores ingested food until it is emptied into the small intestine, and also
secretes HC1 and enzymes that initiate digestion.

TOP © 2015 - 2023 Tests.com


184
25. a & d - The incisive artery and inferior alveolar artery both diverge from the maxillary artery.

26. c- Class III occlusion is when the mesiobuccal groove of the mandibular first molar is mesial to the
mesiobuccal cusp of the maxillary first permanent molar.

27. b - The maxillary first molar has three roots, with the lingual root being longer than the others,
extending past the lingual surface of the tooth’s crown.

28. d - Maxillary lateral incisors are the only incisor teeth to sometimes have a palatoradicular groove
that extends from the crown to the root on the lingual surface.

29. a & d - Apple and Carrots are examples of containing disaccharide sugar structures. Select fruits,
vegetables and table sugars are a food source of disaccharide sugars.

30. c - 20-30 grams of fiber each day is the recommended daily adult intake for healthy individuals.

31. b - 70-120 mg/100mL blood is a healthy blood glucose level and is the level that a controlled
diabetic or other healthy individual should measure within.

32. c - Starch is the complex carbohydrate that comes from plant storage in the form of glucose.

33. b - 5.5 pH is the acidic level at which decalcification and demineralization of tooth enamel begins
to occur.

34. c - Xylitol is the sweetener found naturally in plants and can interfere with s.mutans metabolism,
decreasing the demineralization of enamel.

35. b - 130 g/day is the recommended daily allowance for digestible carbohydrates in both healthy
adults and children.

36. a - Both statements are true. Carbohydrates are an energy source for the growth and repair of
periodontal tissues. A firm texture of complex carbohydrates may increase circulation in gingival
tissue. Firm carbohydrates stimulate the gum tissue during mastication.

37. c - Yogurt is recommended for lactose intolerant patients, because the lactose is digested by the
yogurt. It therefore makes a healthy snack and alternative to other dairy products such as ice
cream.

38. a - Neutrophils are associated with acute inflammation by acting against invading pathogens in the
body.

39. b - Naturally passive acquired immunity is when antibodies are passed from one individual to
another. An example of this is when the mother passes her immunity on to her child during
gestation.

TOP © 2015 - 2023 Tests.com


185
40. b. Fifth’s Disease
d. Shingles
c. Herpes
e. Syphillis
a. Mumps

41. a - Hepatitis A is transmitted through fecal-oral contamination, such as in contaminated food or


water due to poor living conditions or lack of hygiene by food preparers.

42. a, b, & c - Enlarged tongue, delayed tooth eruption and enlarged lips are symptoms found in
children suffering from hypothyroidism, as well as possible enamel hypoplasia.

43. b - Leukopenia is abnormally low levels of white blood cells and is a disease of the WBCs and bone
marrow; It should not be confused with leukemia, which is cancer of the bone marrow or WBCs.

44. d - Mucositis is the inflammation and ulceration of mucous membranes that line the digestive
tract, including the mucosa of the oral cavity. It is a common side effect in patients undergoing
radiation therapy or chemotherapy.

45. c - Endospores are resistant to high temperatures and disinfectants, and may remain viable for an
extended amount of time. Endospores are the primary reason for the need of higher levels of
temperature during sterilization processes.

46. d - Anaerobes require an absence of oxygen in order to survive. Other types of microbes may be
able to function without oxygen or with limited oxygen, but anaerobes require ultimate absence of
oxygen in order to survive.

47. b - Nondirect contact is the infection transmission that involves touching a contaminated object,
such as a touching a used instrument without wearing gloves while transporting it to the
sterilization area.

48. d - Opportunistic infections are infections that affect immunocompromised persons such as an
infant or older adult. This is due to the pathogen taking an “opportunity” when the person’s
immune system is compromised. Anyone may suffer from passive, acute or chronic infections.

49. b - Viruses are the smallest and simplest infectious agent. They contain only RNA or DNA strands.

50. b & c - Lyme disease and West Nile virus are both examples of vector-borne infections that are
spread through an infected carrier such as a mosquito or tick and transferred to the host.

51. a - Antibiotics such as penicillin or amoxicillin are chemotherapeutic compounds that will inhibit or
kill bacteria and is produced by a bacterium.

52. b - Impetigo is a highly infectious condition that can be due to staph or strep infections, leaving
pustules and itchy yellow scabs around the mouth and face. It is highly contagious.

53. b & c - Xerostomia (dry mouth) and jaundice (yellowing of the skin) are both symptoms associated
with liver disorders such as Hepatitis.

TOP © 2015 - 2023 Tests.com


186
54. b & c - Both Hepatitis B and C are transmitted through sexual contact, as is Hep D (not listed).

55. c - Pallor of the gingiva is not a symptom of leukemia. Instead, gingiva will appear red and tender
to the touch, as well as exhibit bruising and bleeding.

56. a - Fibrous dysplasia is a bone disease which affects children and involves two types: Monostotic
and Polyostotic. Both cause abnormal proliferation of fibrous connective tissues.

57. b - Squamous cell carcinoma is the most common oral cancer, attributing for approximately 90% of
all oral cancer cases.

58. d - Coronary artery atherosclerosis is associated with periodontal disease. Decreasing periodontal
disease symptoms through preventative and maintenance care will reduce the patient’s risk
factors.

59. c - The first statement is true; the second statement is false. Local anesthetics containing
vasoconstrictors should not be given to asthma patients because it may cause reduced oxygen
flow in the body, but they may be administered to patients with emphysema.

60. b - Petit mal seizures are simple, with jerking motions and no loss of consciousness. The patient is
aware of their surroundings and is able to recount the episode without loss of memory.

61. d - Lupus is a chronic disease that displays a “butterfly” pattern over the patient’s face.

62. c - Abfractions are the wedge-shaped lesion along the cervical margin of the teeth, usually due to
fatigue or flexing of the tooth. The excess flexing causes enamel to chip off of the tooth in this
area, exposing the dentin.

63. a - Pyogenic granulomas is a reactive tissue hyperplasia related to hormonal changes, usually
associated with pregnancy. This condition may also be called “pregnancy tumors.”

64. b - Amalgam tattoos are bluish discolorations of the oral mucosa due to leaking of particles from
an amalgam filling directly into the mucosa. Therefore, they may appear radiopaque on a
radiograph.

65. d - Eruption cysts usually involve primary tooth eruption and exhibit bluish tissue with swelling
around the erupting tooth, hence the term “eruption cyst.”

66. a - Both statements are true. Supernumerary teeth typically exhibit microdontia (small in size) and
are in the maxilla, typically distal to the 3rd molars or between the upper central incisors.

67. b - Enamel pearls are projections of an exophytic area of enamel on the surface of a root. The area
appears as a small round “pearl” of enamel on the surface where enamel isn’t normally found.

68. c - 6 months to 5 years is the age range where dental fluorosis causes damage to the developing
teeth. At this point in age all permanent teeth are developing and at increased susceptibility to
excess fluoride exposure.

TOP © 2015 - 2023 Tests.com


187
69. a & b - Linea alba and leukoedema may both involve the buccal mucosa. A ranula is in the floor of
the mouth while lingual variscosities are on the tongue.

70. c - Fordyce granules are asymptomatic clusters of yellow lobules involving sebaceous glands within
the oral cavity.

71. a & d - Geographic tongue and benign migratory glossitis are the same condition, which involves
the desquamation filiform lingual papillae. They appear as red, raw areas on the tongue with a
well-defined border which frequently changes in shape.

72. b - Dilaceration is the sharp bend or curve in the root of a tooth, it can make root canal therapy or
extractions difficult to complete.

73. b & d - Aspirin and Bayer (brand-name) are contraindicated for pediatric use due to the risk of
developing Reye’s syndrome in children.

74. c - The pharmacological abbreviation “bid” refers to medicine that should be taken twice per day.
(“Bi” twice “d” day)

75. b - Both statements are false. The PANS enables normal activity function while the SANS enables
emergency functions.

76. a & b - Vasoconstriction and bronchodilation are typical effects due to the use of Adrenergic
medications. While the blood vessels constrict, the bronchial system can dilate.

77. c - Effective dosages are the amount of a prescription drug dose that produce 50% of the
maximum response of the expected outcome in 50% of the subjects taking the medication.

78. a - Diazepam is not an anti-inflammatory drug (NSAID). Valium is the brand name for diazepam and
is used as an antianxiety medication.

79. c - Local anesthetics should have a rapid onset, to allow the dentist to perform treatment within a
timely manner for the patient. They should also have a long shelf life, be reversible and potent.

80. d - All of the above are correct in regards to the advantages of nitrous-oxide use. It is non-
flammable, may be used on patients of all ages and has a rapid onset and recovery time.

81. d - Benzodiazepines provide a wide therapeutic index and are much safer than barbiturates for
routine prescription use within the dental office.

82. a - NSAIDs should be avoided when treating a pregnant patient, but the other medications are
typically safe on the use of pregnant patients. Always check with the patient’s primary care
practitioner if a drug substitution is needed to include an NSAID.

83. a - Dilantin (phenytoin) is known to contribute to gingival overgrowth (hyperplasia). These areas
are typically fibrous along the papillary and marginal gingiva.

TOP © 2015 - 2023 Tests.com


188
84. b - The liver is the primary site for drug metabolism. It is broken down in this area and then
distributed throughout the rest of the body for its purposed treatment.

85. c - Calcium channel blockers vasodilate the blood vessels to relax resistance. This decreases blood
pressure and allows blood to flow easier throughout the cardiovascular system.

86. a - Oral candidiasis. Nystatin is a fungicidal drug commonly used to treat fungal infections such as
oral candidiasis (yeast).

87. c - The parotid papilla is a tissue mass that protects the opening of the parotid duct of the parotid
salivary gland. This duct is also called the Stenson’s duct.

88. d – The canine eminence is part of the alveolus and maxillary alveolar process. It is especially
prominent on the maxilla.

89. a - Tori are found bi-lateral on the lingual area of the mandibular arch. It is a developmental
growth of normal bone that often appears lobulated or nodular.

90. b - The greater palatine nerve is a branch off of the maxillary nerve, which is a branch from the
trigeminal nerve.

91. c - Black hair tongue is an accumulation of dead cells on the tongue that become stained by such
things as food, tobacco, medicine and certain oral bacteria.

92. a – Mucocele is a result of a saliva duct that retains saliva and forms a small sack. A ranula is
similar, but affects the submandibular gland.

93. b - Circumvallate lingual papillae are a linear line of 10-14 mushroom shaped papillae anterior of
the foramen cecum.

94. d - The thyroid is located inferior to the thyroid cartilage. Parathryrode glands are located in the
same region, but cannot be palpated.

95. a - 90% of digestion and the absorption of food happens in the small intestine. The other 10%
takes place in the stomach and large intestine.

96. c - Amoxicillin is an antibiotic that is used to treat an assortment of bacterial infections. It can alter
the flora of the intestine and cause a deficiency in the absorption of essential nutrients.

97. d - Tuberculosis is carried in airborne particles called droplet nuclei, that can be generated when a
person coughs, sneezes, shouts or sings.

98. a & d - Fecal-oral transmission is associated with organisms that affect the digestive systems. The
microorganisms enter the body through ingesting contaminated water or food.

99. d - Maltose is considered a disaccharide, which contains two glucose units. Monosaccharaides
contain one glucose unit.

TOP © 2015 - 2023 Tests.com


189
100. d - A blood pressure 120/80 is considered normal. A person with a reading of 148/90 would be
considered hypertensive - stage 2. Hypertension - stage 2 is a reading of 140/90 or higher.

101. a - The Cusp of Carabelli is normally found on the mesiopalatal line angle of the maxillary first
molar. It is hereditary and not found on all patients.

102. b - Trisomy 21 is associated with Down syndrome. It means that instead of each cell having two
copies of the chromosome 21, it has three. Trisomy 18 is associated with Edwards syndrome and
Trisomy 13 is associated with Patau syndrome.

103. a - When the plasma cells mature, they produce antibodies, which consist of IgA, IgE, IgD, IgG and
IgM. Each plasma cell produces a specific antibody and is in response to an immunogen.

104. c – Nonsuccedaneous means that the teeth do not replace primary teeth. Permanent molar teeth
are the only ones that do not follow primary teeth.

105. a - Maxillary molars are the only teeth that have a trifurcation, due to having three roots.
Mandibular molars have a bifurcation, because they only have two roots.

106. d - Not only do the central incisors have the greatest depth of curvature on the mesial, they also
have the largest height of contour for both labial and lingual surfaces.

107. b - When sterilizing instruments using this technique, the substances combine with oxygen. Timing
starts when the temperature reaches its peak.

108. b - A person that has a systemic disease, but may need medication or have other medical
considerations for their appointment, would be classified as Type II.

109. d - A chemical agent, such as soda or acid reflux, could cause the loss of tooth structure. When
tooth structure, such as enamel is lost, you cannot get it back.

110. b - The maxillary teeth completely cover the mandibular teeth when the jaw is in the biting
position. Overjet occurs when the maxillary teeth flare out from the mandibular teeth.

111. a - In the initial phase, gram positive streptococcus is dominant. However, once the plaque is
established, the dominant bacterium becomes gram negative rods and spirochetes.

112. a - Root structure is more likely to demineralize because of a high pH. Enamel demineralizes at a
pH of 4.5-5.5. This makes the root surface more susceptible to decay.

113. c - Exogenous stains originate from the outside of a tooth. Endogenous stains, such
as tetracycline or tooth trauma, originate from inside the tooth.

114. b – Angular cheilitis is a fungal infection that can be treated with an anti-fungal medication. The
other conditions cannot be treated with medications. They will not respond to therapeutic
therapies.

TOP © 2015 - 2023 Tests.com


190
115. b - Dental biofilm is a yellow mass of bacteria that when uncontrolled, can cause dental caries or
periodontal infections.

116. c - Iron is not a major component of saliva. However, there are trace amounts of the mineral that
are found in saliva. A lack of iron in your diet could cause: angular cheilosis, burning tongue, or
glossitis

117. d - The IA nerve innervates the pulp of the teeth. The long buccal and the mental nerve bring
supply to the surrounding tissues and tongue.

118. a - The Trochlear nerve is a motor nerve, specifically with the eye. It has purely somatic motor
function. It is the smallest cranial nerve.

119. d - Warton’s is a duct that is part of the submandibular gland. It is an opening for saliva and is not a
major salivary gland.

120. Order: d, a, c, b, f, e
Blood flows from the heart to the lungs in the following order:
Superior and Inferior Vena Cava
Right Atrium
Tricuspid Valve
Right Ventricle
Pulmonary Artery
Lungs

121. b - Drugs with a high-first past effect have a LARGER oral to parental dose ratio.

122. b - Pharmacokinetics is the movement of a drug in your body, once it is administered.


Pharmacodynamics is the study of effects the drug has on your body

123. a - Metabolism will be impaired in relation to the severity of the damage to the liver. A
nonfunctioning liver could result in possible toxicity. The metabolism of drugs by means of enzyme
reactions, is mostly completed in the liver.

124. b - The cap stage is the second stage in tooth development. The first is the bud stage where the
formation begins. The last stage is the bell stage, where the tissue form and the tooth shape is
established.

125. b - Endoplasmic or endoplasmic reticulum is found throughout a cell and connected to the nucleus.
It is not a tissue in the body, but is an important part of cell.

126. a - Interradicular fibers are not part of the gingival fiber group. It is actually a reference to the
fibers from the cementum and between the roots of teeth to the adjacent bone.

127. d - Vitamin K is the catalyst for blood clotting. It is derived from food sources in our gut.

128. a - Pellagra is caused by a chronic lack of the Vitamin B3. It can cause diarrhea, dermatitis and can
lead to mental disturbance.

TOP © 2015 - 2023 Tests.com


191
129. b - Peas are rich in vitamin B. Cheese, tofu and kale are all excellent sources of calcium and should
be considered when a patient is under nutritional counseling.

130. c - Bidigital palpation is accomplished by using your finger and thumb of both hands. This
technique is used to compress and palpate tissue during an extra oral exam.

131. a - Subluxation is the clinical term for dislocation. This can be caused by long dental procedures,
which stretch the ligaments around the joint, making it easy for the condyle to dislocate. It is
important to inform your patients that they should restrict large movements after long dental
procedures.

132. d - Vitamin C helps the lining of the gums stay healthy. When a person, especially a smoker, is
deficient in this vitamin, the lining is susceptible to bacterial infections, potentially leading to
periodontal disease.

133. a - Cleft lip begins to develop between the 4th and 7th weeks of the first trimester. It becomes
evident during the second month of pregnancy.

134. a - Collagen fibers become mineralized and incorporated into the cementum. In teeth, the fibers
that are the terminal ends of the principal fibers, that insert into the cementum and into the
periosteum of the avelor bone are called Sharpey’s fibers.

135. c - The sagittal plane runs down the midline of our body, creating the left and right side. The
Parasagittal plane means any plane that runs parallel to the sagittal plane.

136. a - The mandibular central incisors are the smallest permanent tooth in the oral cavity. They are
the smallest due to the fact that they have a very slight concavity with labial surfaces that are
smooth.

137. b - Cancellous bone is spongy looking and is the inner or central part of the bone. It is part of the
alveolar bone that supports the tooth in both the maxilla and mandible.

138. d - Both pharyngeal and esophageal phases of swallowing are under involuntary neuromuscular
control. They are both considered involuntary reflex of muscles that allow us to swallow our food
fast and easy, causing little harm to surrounding organs.

139. Both b and c. The endocrine glands secrete their product or hormones directly into the blood,
rather than using ducts. The thyroid and ovaries both secrete hormones. The kidneys and ureters
are part of the urinary system.

140. a - A psuedopocket is due to gingival enlargement and does not involve the migration of
connective tissue attachment and other supporting periodontal tissues.

141. a - Mesognathic is when your jaw slightly protrudes in its resting state. This is considered a normal
facial profile.

142. b - Medical history forms are part of the paperwork that patients will fill out upon their initial visit

TOP © 2015 - 2023 Tests.com


192
with the dental office. These forms contain information that determines their general health,
allowing dental care providers to determine whether or not there may need to be special
accommodations regarding their care.

143. b - Birth control pills may become less effective if the patient is placed on a prescription antibiotic
treatment. Even though patients may leave certain areas of their health history form blank it is still
important to review them, to be sure that the patient did not fail to relay any vital information.

144. d - At each visit, patients should review their health history forms and note if any changes have
occurred. This includes surgeries, medications, supplements, or even a change of doctor.

145. d - 60-100 beats per minute is a healthy heart rate for a fully grown adult. Infants are between
120-160, toddlers between 80-130 and children at 70-110 beats per minute.

146. a & b - Both systemic lupus erythematosis and joint replacement within the last 2 years are
reasons requiring antibiotic premedication prior to dental procedures. Other examples of
premedication needs include artificial joint infections, weak immune system and Type I diabetes.

147. c - Inspection of the ventral surfaces of the tongue is a part of the intraoral examination, but not
part of the extraoral examination.

148. a, c & d - Hardness, swelling and enlargement as well as pain are inspected in a patient’s parotid
gland. Mobility is not one of the factors determining the health of the gland, which is inspected
using bilateral palpitation.

149. d - Occipital lymph nodes should be palpated with the patient’s head tilted forward, allowing the
practitioner access to palpate the area around the occipital region of the skull.

150. a - Loss of gingival stippling is a sign of disease like conditions within the mouth. Gingiva should
appear pink and stippled, with a knife-edge papilla. Pigmentation may be normal in patients of
different races.

151. b - Wharton’s duct is the submandibular salivary gland that should be palpated during an intraoral
examination. This gland may become blocked with sialoliths, which would be present on an
occlusal x-ray.

152. b & c - The ventral and lateral surfaces of the tongue are high-risk areas for finding oral cancerous
lesions during an intraoral exam.

153. a - Hairy leukoplakia is caused by the Epstein-Barr virus, which causes hyperkeratosis of the filiform
papilla, leaving a corrugated, white area on the lateral borders of the tongue.

154. d. Class I
f. Class II
b. Class III
a. Class IV
c. Class V
e. Class VI

TOP © 2015 - 2023 Tests.com


193
155. b & c - Amoxicillin and fluorosis can cause white staining in the enamel of the teeth if the person
was exposed during early age when tooth development was still occurring. Fluorosis may also have
brown spots or no white spots at all, and amoxicillin causes possible white opacities.

156. b - Type II overhangs comprise 1/3-1/2 of the interproximal embrasure space. Type I is less than
1/3, Type III is more than 1/2 and there is no type IV overhang.

157. d - Fewer than 15 teeth is not regarded as a caries risk factor, though patients that have less than 9
present teeth are regarded as at a high caries risk.

158. a & c - Tough, fibrous gingiva with blunted interdental papilla are signs of chronic gingival
inflammation. Other signs include dark colored gingiva and widening of embrasures between the
teeth.

159. c - A PSR score of 2 would be appropriate for areas of calculus buildup, bleeding upon probing, but
probing depths only extending into the first clear band of the periodontal probe. If the depth of
the pocket extended into the color section, then the PSR score would be 3.

160. a - Class III mobility (the most severe classification) exists during severe mobility, when a tooth is
depressible.

161. b - A plaque score of 3 is the highest plaque score on the plaque index (PI), indicating gross
amounts of soft plaque debris within and along gingival pockets and margins.

162. c - Class III furcation involvement indicates through and through loss between the trifurcation and
bifurcation of the roots, with the area still covered by gingiva. If the area were visible during
examination, then it would be a class IV furcation involvement.

163. a - Fremitus is the vibration felt in a tooth when the teeth occlude. You can feel it when placing the
finger near the gingival margin of the crown and asking the patient to tap his or her teeth
together.

164. b - Exposure to light is not a factor that affects the setting of gypsum products. However, the
others listed do affect it.

165. b - Recession is the area measured with a periodontal probe from the CEJ to the margin of the
gingiva that is apical to the CEJ. Measuring recession does not determine pocket depth or
attachment.

166. c - Streptococcus mutans, or S. Mutans is the primary bacteria responsible for the initiation of
coronal lesions. As the decay advances, Lactobacilli may progress it to a larger decayed area.

167. a & b - Longer x-rays are fewer in frequency (less waves throughout the beam) and produce a
beam that is less penetrating that shorter x-ray waves. Shorter wavelengths occur at a higher
frequency and produce a much more penetrating x-ray beam. The longer the beam, the weaker
the x-ray wave. The shorter the beam, the stronger the x-ray wave.

TOP © 2015 - 2023 Tests.com


194
168. d - Free radicals result when x-ray photons intermingle with water and oxygen biological molecules
inside of biological molecules, with a highly chance of causing damage to biological cells.

169. c - Dental practitioners should always stand at least six feet away from the primary beam source,
at an angle of 90-135 degrees from the path of the direct beam.

170. c - 5.0 rem (equivalent to 0.05 Sv) is the maximum amount of radiation per year that any dental
professional should be exposed to. For the average person, it is 0.5 rem per year.

171. d - The lethal dose of radiation that can be absorbed and would cause death in 50% of the
population within 30 days is LD 50/30. The latent period would extend between the exposure and
the time that a biological effect took place.

172. a - The position indication device (PID) performs collimation of the x-ray beam to restrict the size
of the beam. The PID is round or rectangular in shape.

173. d - The statement that PID lengths must be no less than 6” is incorrect. PID lengths must actually
be no shorter than 8”, 12” or 16”, depending on the type of beam.

174. c - Increasing the kVp to 75-90 kVp allows for a better contrast of grey shades in the final
radiograph. This helps when diagnosing other tissues such as bone height.

175. a - Bisecting technique placed the x-ray beam perpendicular to the imaginary line between the
long axis of the tooth and the x-ray film placed at an angle to the tooth. It allows the practitioner
to get an accurate image even though the film is not placed parallel to the long axis of the tooth.

176. a - Horizontal angulation that is properly aligned between interproximal areas of the teeth will
allow the beam to go through easily and result in open contacts on the final film. If the horizontal
angulation is inaccurate it will cause overlapping.

177. c - 20-degree changes are best for locating pathology or objects in the mouth that need to be
identified. Moving a beam one way (mesial) with the resulting image moving in the opposite
direction (distal), then the object is on the lingual side of the teeth. “SLOB”: same mesial, opposite
buccal.

178. a - Both shorter object to film and source to film distances will reduce magnification of the image
on the x-ray. Distancing the lengths between object to film or source to film would cause the
image to become magnified on the film.

179. b - The mandibular canal houses nerve tissue and appears as a radiolucent canal extending down
through the mandible.

180. c - The lamina dura is the radiopaque margin of supporting bone structure that houses the socket
of the root, and provides an attachment area for the periodontal ligament.

181. d - The mental foramen is the bilateral round or oval opening in the mandible that allows nerve
tissues to pass through into the supporting alveolar structures.

TOP © 2015 - 2023 Tests.com


195
182. a - Subgingival calculus in large amounts appears as radiopaque extensions of the tooth structure
near or around the CEJ in the interproximal areas on radiographs.

183. b - The zygomatic arch appears as a bony “U” that is visible just superior to the maxillary molars
and is more defined than the nasal sinuses.

184. b - The lingual foramen is the tiny hole at the anterior midline of the mandible and is surrounded
by the genial tubercles.

185. c - The anterior nasal spine is the radiopaque bony structure superior to the median suture.

186. c - The film is mounted the correct way. While the film was positioned in the proper place (the
distal of the first bicuspid is viewable on the film), inaccurate vertical angulation of the tubehead
was used, resulting in interproximal overlap. The film was placed into the mouth backward and
this is evident by the patterned area across the entire film where the x-ray beam came into
contact with the lead foil.

187. b. Stage I
d. Stage II
a. Stage III
c. Stage IV

188. d - Multiple sclerosis is a contraindication for nitrous oxide use, as are people with respiratory
infections, sinus obstructions, epilepsy or emotional instability.

189. b - The anterior superior alveolar nerve injection will anesthetize the upper incisors and canines.
The injection is placed in the mucobuccal fold just mesial to the canine eminence.

190. a - The nasopalatine injection anesthetizes the palatal tissues of the maxillary anterior teeth. The
teeth are not anesthetized, but the hard palatal tissues are.

191. c - Sodium is prevented from entering into nerve cells when local anesthetic is used. This prevents
nerve impulses because the potassium cannot get out of the nerve, and inhibits nerve
depoliarization.

192. d - Anemia in a patient would present itself as painful glossitis, pale gingiva and mucosa, and
angular cheilitis. The condition may cause abnormal bleeding in conjunction with other diseases of
the gingiva.

193. b - It is advised that magnetostrictive ultrasonic scalers should be avoided in the use with people
with cardiac pacemakers, even though there are shields in place to protect the pacemaker.
Piezoelectric scalers have not been shown to interfere with pacemaker devices.

194. c - Cardiovascular disease is often treated with medications that are calcium channel blockers.
These prescription drugs often cause gingival hyperplasia in the oral tissues.

195. a - Papilloma cancer is a benign form that originates in the epithelium tissue, including the skin and
mucous membranes. The malignant form of cancer that affects the epithelium is squamous or

TOP © 2015 - 2023 Tests.com


196
basal cell carcinoma.

196. a - Check the airway for obstruction prior to administering rescue breaths to the patient. Look
inside of the mouth and perform a finger swipe if necessary.

197. c - Patients should not be placed in an upright position, but instead placed in a supine position
(flat). Terminate the treatment and activate EMS, while a team member brings the emergency
office kit containing an Epi-pen (epinephrine) for administration.

198. d - Have the patient drink a soda or juice, or have another sugar-containing snack due to a low
level of blood sugar. After administering the sugar, monitor the patient.

199. c - Removing equipment from within the reach of the patient is the first thing that should be done
for a patient that is experiencing a seizure in the operatory chair. This prevents the patient from
coming into contact with something that could harm them.

200. c - Sweat is not regarded as a contamination source. All other bodily fluids should be considered
contaminated with blood or disease.

201. d - Face masks should be changed after each patient is seen, whether or not it is visibly soiled.

202. c - Dental professionals should not use petroleum based creams because petroleum causes the
breakdown of latex.

203. a - Statim autoclaves are capable of performing complete sterilization cycles in under 10 minutes.
Steam is injected directly into the statim cassette and the temperature is a lower level, which
allows for quicker heating and cooling.

204. d - Dental hygienists are not to diagnose restorative treatment needs, even if they are aware of
what treatment may be completed. The hygienist may choose to discuss possible needs that the
dentist may diagnose or plan, but hygienists are only to present dental hygiene case plans to the
patient.

205. a - Informed consent is when the patient is informed of their dental treatment plan and signs the
plan acknowledging that the plan has been explained to them and the treatment is necessary. If
the patient eventually refuses to have the treatment completed there will still be documentation
that shows the treatment was reviewed with the patient, who should also be told of what possible
problems could arise if the treatment was not completed.

206. d - All of the above are necessary for a dental hygienist to perform a comprehensive needs
assessment on a patient. General items such as medical history, intraoral and extraoral exams,
periodontal assessment and a dentition assessment need to be integrated together so as to
determine what the appropriate method of a treatment should be for the patient.

207. c - A patient’s financial situation does not affect the prognosis of a completed treatment plan.
Other health-related situations such as malocclusion, severe periodontal attachment loss and poor
or failing restorations can encourage the continuation of oral disease within the mouth.

TOP © 2015 - 2023 Tests.com


197
208. b - Using ink is the appropriate method to document notes of any sort within a patient’s file.
White-out should never be used; instead a line should be drawn through any words that were
incorrect, with initials placed at the end of the line. Pencil notes are not legitimate in a legal sense
in regard to documenting patient information.

209. b - The dentist may perform an intraoral exam on the patient to diagnose any care needs, but the
patient should not have a prophylactic procedure until she has been premedicated with an
antibiotic at least one hour prior to the appointment. Premedication is necessary in patients that
have had a recent joint replacement up until two years prior to receiving dental care.

210. b - Patients with stage II Hypertension symptoms may only receive noninvasive care such as an
exam and x-rays, but invasive procedures should be preceded by a medical consultation with the
patient’s primary care physician. Only elective emergency care should be provided for this patient.

211. c - Palpitation of the soft palate is not recommended during intraoral exams as it could trigger a
gag reflex in the patient.

212. b - While affordability is a large concern of most patients, it should not be the determining factor
when presenting an effective treatment plan needs assessment to the patient. The goal of the
dental provider is to eliminate dental disease or slow its progression, while maintaining and
promoting oral and systemic levels of health.

213. c - Every patient has the right to decline their treatment plan, but it should be documented so in
the patient’s file. The case presentation should review in layman’s terminology what treatment is
needed, why, what results will occur if treatment is not completed and explain options to the
patient so that they can co-plan their treatment.

214. c - New patients should always be seen by the dentist and have the dentist diagnose x-rays to be
taken prior to any procedures such as a prophylaxis are performed on the patient. Patients of
record that have not been seen by the dentist in over a year should see the dentist prior to the
completion of any procedures.

215. a - The RDH may be guilty of supervised neglect because she failed to inform the patient of an area
or tooth that was suffering from periodontal disease conditions. As a result the patient took no
additional oral health care steps, assuming that their oral health was in line. Further attachment
loss now may lead to the loss of the entire tooth because no preventive methods were taken and
the patient was never informed.

216. b - 6-12 month recare appointments are appropriate for the periodontal patient that receives
nonsurgical care and responds well. Initial maintenance visits are performed about every 3
months, but if the patient has made it to a year post procedure and exhibits good general and oral
health then the recare appointments may be performed less frequently.

217. a - Race is not regarded as a risk factor in assessing a patient’s periodontal health. Conditions that
do affect periodontal health include socioeconomic status, age, systemic health conditions,
periodontal disease, malocclusion, appliances worn and any habits such as tobacco or alcohol use.

218. b - Temporary crowns are marked with a “T” for temporary over the type of restoration that was

TOP © 2015 - 2023 Tests.com


198
placed.

219. b - Two vertical red lines placed between teeth #8 and 9 on a dental chart indicate a diastema
between the two teeth.

220. b - 6 month recare appointments may be too infrequent for patients that present with a high
caries rate. More frequent prophylactic appointments (such as every 3 months) can allow for
plaque removal, review of nutritional habits and fluoride application to reduce the decay rate.

221. d - Not brushing with a fluoride toothpaste is not considered a risk factor for caries. Risk factors
are those, which place the patient at an increased risk of tooth decay, as patients with acid reflux,
high soda intake or xerostomia would be. A patient may brush with tap water and be able to keep
their teeth clean and healthy.

222. b - DIAGNOdent diagnosis for decay is not appropriate for use on smooth coronal surface lesions.
It is for use only in suspected pit and fissure lesions on the occlusal surfaces.

223. a - Breastfeeding does not place a child at a higher risk to develop early childhood caries. However,
because teeth do erupt while many children are at a breastfeeding stage, the child’s teeth should
be cleaned immediately after nursing in order to reduce extended exposure to the milk.

224. c - A pseudopocket or “gingival pocket” is an area where the gum tissue is enlarged coronally but
not apically, meaning there is no loss of attachment. It is a pseudo-pocket and does not imply a
sign of bone loss.

225. b - Class II furcation involvement is when the periodontal probe is able to reach into the furcation
area of the tooth that is probed, but it cannot extend through the furcation into the other side.
There may be some slight radiolucency in the furcation area of the radiographs.

226. c - Only the supervising dentist may make referrals to specialists such as a periodontist. The
hygienist is able to assess the health of the patient and provide a hygiene treatment plan to the
patient, but she may not refer him to a specialist unless the doctor states that is what should be
done.

227. b - The Fones method is a good first method for introducing brushing techniques to a young child,
however it is not an efficient method for adults in regard to routine plaque removal. The Fones
method is also called the “circular” method and emphasizes plaque removal on the crown.

228. a - Waxed floss is best for patients with tight interproximal contact areas, because it allows the
floss to slip more easily into the space.

229. d - Interdental brushes such as the Proxabrush are efficient at reaching concavities in the
interproximal areas when it comes to efficient plaque removal. They are also useful for wider
interdental spaces or orthodontic patients.

230. a - Black line stain is the only extrinsic stain listed (there is no such thing as extrinsic tetracycline
staining). Rubber cup polishing should only be used on a selective basis and will only remove
superficial, extrinsic stains.

TOP © 2015 - 2023 Tests.com


199
231. c - Dentinogenesis imperfecta is a white or brown discoloration of one or many teeth due to the
disturbance of ameloblasts during tooth formation.

232. b - Patients with high blood pressure or those on diets that restrict their intake of sodium should
not have air polishers used on them due to the use of sodium bicarbonate powder which is used to
remove the stain.

233. c - Chlorhexidine gluconate is a bactericidal agent that continues to release in the oral cavity
prolonging therapeutic effects against periodontal disease and oral bacteria.

234. d - Candida albicans infections (yeast infections) may occur as a result of a patient failing to
adequately clean their dental appliance on a regular basis. Stain may build up as well, but this is
not harmful to the patient as a yeast infection would be.

235. a - Stainless steel curettes or scalers should not be used on titanium implants as they may cause
scratching on the implant surface, resulting in bacterial accumulation.

236. b - Sonic scalers use air-pressure to cause mechanical vibrations used for debris removal.
Piezoelectric use expansion and contraction of crystals inside a housing unit and magnetostrictive
use magnetic strips. Air-polishers are not electric scalers.

237. c - A Gracey 11/12 would be appropriate to use on the mesial of posterior teeth, as would a Gracey
15/16. A 13/14 would only be used on the distal surfaces. Other curettes listed would be for use in
the anterior teeth.

238. a - Piezoelectric scalers use only the lateral edges as an active surface, because piezo scalers have a
linear tip motion. The face and back are not active.

239. a - Periodontal pockets are the result of loss of junctional epithelium in an apical direction,
indicative of bone loss.

240. c - One-wall bony defects have the worst prognosis. The bone loss occurs interdentally leaving only
one surface remaining. A 3-wall defect has the best prognosis.

241. b - Gingivitis is the initial stage of periodontal disease, but there is no loss of attachment and the
condition is reversible.

242. c - Lichen planus manifests itself as an erosive lichen planus of the gingival mucosa. There may be
erythematous areas interspersed with white striae.

243. d - Desquamative gingivitis may be due to a serious systemic disease, but it can also be caused by
an allergic reaction. It appears as sloughing tissues that leave red, raw areas in the mouth.

244. b - Moderate chronic periodontitis has pocket depths of 5-7 mm. More than 7 is considered
Advanced or aggressive, and less than 5 is only slight periodontitis.

245. c - Juvenile periodontitis normally has no plaque, inflammation or bleeding present. Other

TOP © 2015 - 2023 Tests.com


200
periodontal diseases do. Juvenile periodontitis typically affects the permanent anterior and first
molars, while other periodontal diseases typically begin manifesting themselves in the posterior
teeth.

246. b - Necrotizing ulcerative periodontitis manifests itself as “punched out” papillae and is typical in
HIV-infected patients or people with systemic immune disorders.

247. a - Curettage is the removal of diseased epithelium within the periodontal pocket during
subgingival scaling. This may occur during normal curettage or done purposely.

248. c - 5-6 mm pockets are now part of the new guidelines for the advised depth of periodontal
pockets that should receive antibiotic therapy during periodontal treatment. In the past, patients
would receive periodontal treatment and then possibly be administered subgingival antibiotic
therapy at a six-week evaluation appointment if needed.

249. a - Minocycline (Arestin) inhibits protein synthesis of bacterial cells and is active to work over a
two-week period after administration.

250. b - Osseous surgery reshapes the alveolar process to attain a better physiologic form around the
teeth. This may be done around defects involving periodontal disease.

251. c - Periodontal flap surgery retracts the gingiva so that bone can be modified and diseased tissues
removed. It also allows for residual calculus removal before the flap is sewn back into place.

252. c. Autograft
a. Allograft
d. Xenograft
b. Alloplast

253. c - Root hemisection is the removal of half of a tooth, including portions from both the crown and
the root. This procedure will need to be followed up with endodontic treatment.

254. d - Titanium is the material of choice for the fabrication of implants. The titanium has a high rate of
osseointegration with the bony tissue. Implants may sometimes be coated with ceramic coatings.

255. c - Approximately 90% of malodor bacteria reside in the oropharynx region, such as on the tongue,
periodontal conditions or around the tonsils. Systemic conditions account for the other 10%.

256. d - The use of tobacco products is the most important risk factor related to periodontal disease.
The chemical irritants in tobacco products traumatize the periodontal tissues and inhibit healing.

257. a - Menstruation has not been proven to have an increased risk of gingivitis symptoms. It is
actually thought that gingivitis symptoms decrease during ovulation and pre-menstruation, a time
when hormones typically peak.

258. a - The majority of periodontal pathogens are gram negative and nonmotile, except for one or two
which are motile (but also gram negative).

TOP © 2015 - 2023 Tests.com


201
259. d - All of the above are signs and symptoms of occlusal trauma. Not only does trauma such as
bruxism cause wear facets, but it also widens the periodontal ligament and may cause the teeth to
shift or even become mobile.

260. c - The first statement is true. PDL fibers are the first fibers lost during periodontal disease.
However, transseptal (interdental) fibers are not lost until such severe disease has occurred that
the tooth is about to be lost.

261. a - Stillman’s clefts are slit-like clefts that occur vertically in the marginal gingiva. They are usually
caused by atrophy of the facial gingiva.

262. c - Bleeding on probing is the most accurate sign to diagnose active inflammation in the
periodontal pocket. Healthy gums do not bleed, but unhealthy gums may appear to have no
inflammation or redness (as in smokers).

263. d - Fuzziness of the alveolar crests as evidenced on an x-ray is the first radiographic sign that there
is a breakdown of the periodontium. As further breakdown occurs it will be noticed by vertical or
horizontal bone loss on the dental x-ray.

264. b - Acidulated phosphate fluoride should not be applied to patients with crowns, bridges, veneers
or composite fillings as it may cause a layer of demineralization on the tooth surface.

265. a - Fluoride varnish uses 5% sodium fluoride that adheres onto the tooth surfaces and releases
fluoride ions for 3 to 4 hours following the application.

266. c - 1ppm is the optimal level of fluoride in a municipal water supply. This level will be less in areas
that are warmer, and more in areas that are cooler (due to the effect temperature has on the
amount of water people drink.)

267. b - 5-10g of fluoride taken at one time is the lethal dose of fluoride. For a small child it would be
less, at approximately 0.5-1.0g.

268. b - 5mg/kg is the toxic dose of fluoride for a child. For an adult it is 8 mg/kg. When giving a child
fluoride in the office using a foam tray, no more than 4mg of gel should be placed in the tray.

269. a - Fluoride use does not cause birth defects. Toxicity may cause cardiac arrest, respiratory distress
and other symptoms. Heavy fluoride consumption such as found in natural water supplies can
cause fluorosis of the teeth and bones (but not when delivered topically).

270. b - Depending on manufacturer recommendations, acid etch is typically left on the tooth between
15-60 seconds for proper conditioning of the dental tubules.

271. c - 2 minutes is the most appropriate answer. Typically, self-cured sealant material will set with 1-3
minutes.

272. b - Prophy paste should not be used when preparing a tooth for sealant preparation because the
paste contains fluoride, which will block tubules and prevent sealant retention.

TOP © 2015 - 2023 Tests.com


202
273. d - Re-etching the tooth for a shorter amount of time is enough to re-condition the dental tubules
after salivary contamination during a sealant prep. Simply drying the tooth is not enough because
saliva has come into contact with the pores of the tooth and will disrupt retention of the sealant.

274. d - Sealant material does not need to be dabbed onto the tooth. It may be applied with a syringe
tip or brush, and spread where it is needed to fill in the necessary pits and fissures.

275. b - Sealant material typically cures within 20-30 seconds of exposure to the curing light. Always be
sure to check the sealant for under-curing after the procedure is finished.

276. b - Unfilled resin sealants do not require occlusal adjustment with a finishing bur. They will wear
down naturally. However, filled resin sealants do require occlusal adjustment because it will not
wear down through chewing or occlusal forces.

277. a - Calcium phosphate, or fluoride-releasing type sealants release fluoride during the life of the
sealant and help promote remineralization, decreasing tooth decay.

278. a - A-delta nerve fibers line the pulp and conduct stimuli rapidly. Stimulation comes through the
dentinal tubules and moves odontoblasts, accessing the A-delta nerves at a fast pace.

279. b - Polishing times of at least 3 minutes can cause removal of the enamel, as much as 3-4
micrometers deep. Over time a significant amount of enamel would be removed. Take care using
selective polishing.

280. a - Bristle brushes are designed to use for stain removal in the occlusal surfaces of teeth. They may
be too abrasive to use near the gingival margins or on dentin, and would not work as well as a
rubber cup prophy angle on facial surfaces.

281. c - 24 hours must lapse after an amalgam filling is placed prior to the tooth undergoing polishing,
because the amalgam must continue to set.

282. c - 2 years is the approximate length of time that glass ionomer restorations seep fluoride after
placement. Teeth with glass ionomer restorations also typically have lower s. mutans on the
surface than teeth that do not.

283. d - Zinc oxide and eugenol cement is widely used in dentistry and has many purposes including
impressions, temporary cement or fillings, bonding and even periodontal dressing.

284. c - Alginate hydrocolloid is a commonly used material for dental impressions, especially for the use
of creating study models, whitening trays and retainers. Agar is a reversible hydrocolloid.

285. a - Temperature of the water determines how fast or slow an alginate impression sets. The warmer
the water, the shorter the set time required by the alginate.

286. d - While stone can be used to make casts, die stone is for use on both casts and dies of teeth. It
requires even less water than stone and produces a stronger material than stone.

287. b - Most study models set within 45-60 minutes after being poured. This is approximately how long

TOP © 2015 - 2023 Tests.com


203
it takes for the exothermic reaction to occur and for the material to harden.

288. a - A 1:1 ratio of powder to water is used when gypsum products are being mixed in the laboratory
for study models. Not using enough water or using too much water will make the mixture
inconsistent and unable to manipulate while pouring.

289. b - Negligence is when a professional does not provide prudent care to a patient in the dental
office, but is careless. It differs from Intentional torts in that actual injury or harm was not a
deliberate act; It differs from malpractice in that malpractice fails to perform up to professional
standards which results in injury.

290. a - Preceptorship is when dental personnel are trained to perform dental hygiene duties in an
office setting. This is currently limited to only very few areas and is not widely accepted.

291. d - HIPAA does protect workers who lose their jobs by protecting their health insurance coverage.

292. c - The secondary level of prevention includes periodontal treatment as well as restorations such
as fillings and crowns. This level of prevention is for correcting disease processes that have begun
in the oral cavity.

293. b and c - Fraud and Misrepresentation are essentially the same action, which causes deliberate
misdirection of what is true, such as causing harm to someone or representing oneself falsely.

294. a - Informed refusal is when the patient has been educated and informed of their proposed
treatment plan needs and what will happen if treatment is refused, yet still refuses the treatment.
Document refusals thoroughly in the patient chart.

295. c - Informed consent is when the patient has agreed to and signed the necessary treatment plan,
as well as being informed of the risks, procedures and what will happen if treatment is not
completed.

296. b - Battery is the actual infliction of harm or injury to a patient, and puts them in danger.

297. a - An implied contract is a contract that has some portions of the contract which are not
expressed in terms of words.

298. c - Libel is a defamation of an individual and occurs in a written form. If defamation is verbal, it is
considered slander.

299. a - Civil law encompasses contract law, and addresses parties during the failing or breaking of a
contract by one of the parties.

300. b - The reasonably prudent practitioner measure is used by legal authorities to determine if a
provider has provided reasonable care for their patient or client.

301. d - Expressed consent may be written or verbal, but documenting the consent in writing helps to
protect the practitioner and patient at a greater level.

TOP © 2015 - 2023 Tests.com


204
302. b - 2 years is the typical length of time in which most states require a malpractice suit to be filed
from when the wrongful act was committed.

303. a - OSHA, the Occupational and Safety Health Administration, oversees MSDS sheets within the
dental office. These sheets are to help protect staff should occupational exposure occur.

304. d - All staff members in a dental office must be trained to meet HIPAA regulations concerning
patient privacy policies. Each office must document this training.

305. c - Harm to the patient must be proved, along with 3 other factors- undertaking the patient for
care, breaching a duty owed to the patient, and the breach of duty causing the harm. All 4 factors
must be in place prior to the hygienist being held liable for negligent care.

306. a and b - If a patient knowingly fails to inform their dentist that they have a heart condition, or if
the patient knowingly does not take their antibiotic premedication prescribed by their primary
care physician prior to dental therapy, the dentist may be spared from a judgment of negligence.
In both cases the patient failed to comply with medical consultation or identify medical conditions
that the dentist should have been informed of.

307. d - All of the above are correct in regards to parental consent to the care of their child, whether
they are a minor or not mentally competent. It is best if consent is documented in writing.

308. b - Nitroglycerin is used in prevention or to treat a patient with angina. The medication comes in a
spray or tablet form. It should be placed or sprayed under the tongue.

309. a - Bacillus stearothermophilus is a gram-positive bacterium that is highly resistant to the


sterilization process. It is used in the sterilization ampules to test the effectiveness of the
sterilization process of an autoclave.

310. d - The dry heat sterilization process is long and not economical due to the restrictions on what
can and cannot be used in the process. In order to kill the most resistant spores, a dry-heat
sterilizer must bring its load to 170°C and hold it at that temperature for one hour. This method
uses a lot more energy and time than a steam autoclave. However, this is a great way to sterilize
oils and powders that cannot get wet.

311. b - The brachial artery is used when taking the pulse of an infant. It is located inside the upper arm,
between the elbow and the shoulder

312. c - A Naber’s probe is used to measure the extent of furcation on both the buccal and lingual of a
tooth.

313. a - The maxillary teeth harbor less bacteria than the mandibular teeth. The palatal surfaces are
smooth surfaces and see activity from the tongue, which helps clear off bacteria from the smooth
surfaces.

314. c - The maxillary buccal posterior and the mandibular lingual anterior are the most common areas
of supragingival calculus, because of their location to the parotid, submandibular and sublingual
glands.

TOP © 2015 - 2023 Tests.com


205
315. b - The end-tuft brush has a small, round head with a few tufts of toothbrush bristles. It is designed
to be small to fit in those hard to reach areas, such as 3rd molars and furcations.

316. c - The bass method or modified bass method is the most common and most widely accepted
tooth brushing technique, because it is suitable for everyone to use. Your toothbrush is placed at a
45-degree angle towards your gingival margin. This method cleans the sulcus and the cervical third
area. It can be taught to periodontal maintenance patients and regular recall patients.

317. d - Pyrophosphate is the ingredient in most anti-calculus toothpastes. It is designed to stop the
formation of calcium-phosphates salts from forming on your teeth.

318. c - Staining is not a contraindication of hydrogen peroxide. In fact, hydrogen peroxide is a major
ingredient in teeth whitening products. However, as a mouth rinse, it is contraindicated for regular
use, due to its potential harmful effects of the gingiva and demineralization of the teeth.

319. a - 10 hours is the recommended time of submersion of instruments. You would use this
technique on heat sensitive instruments. All instruments must be rinsed with sterile water prior to
their use.

320. b - Mesognathic is a Class I profile, meaning everything is properly developed and in correct
occlusion. Class II is retrognathic and means the maxilla or mandible is underdeveloped.

321. a - Ankylosis is when the alveolar bone fuses directly with the root surfaces, leaving no area for
ligament space. Resorption of the cementum and root dentin is a result of ankylosis.

322. b - Mesial drift is a natural occurrence that causes our teeth to move toward our lips or to move
mesial. This is what is attributed to causing crowding. It is said to be caused by torsion in the jaw
and deflection of teeth in their tooth sockets.

323. b - Saliva, blood and any other bodily fluid is considered bioburden. The bioburden needs to be
pre-cleaned before the disinfection process. Otherwise, this can cause cross-contamination.

324. a - The dental bur is considered critical in regards to infection control, because it is used to
penetrate soft tissue or bone. Items are also considered critical if they come into contact with
blood and should be sterilized after each use.

325. a - Alcohol is not effective against cleaning bacterial spores. Bacterial spores are eliminated by high
pressures and heat, such as the autoclave.

326. c - The dental chair is considered an intermediate surface, because it is not an area of high
contamination and is able to be sterilized by the autoclave or statim.

327. d - The Curve of Spee, or Spee’s curvature, is a curved line that is formed when a patient is in
occlusion and the occlusal line curves upward towards the terminal molar.

328. d - All listed items can cause an open bite. The open bite can be caused by hereditary factors or by
repeated patterns, such as digit sucking or thrusting of the tongue. It can be corrected through

TOP © 2015 - 2023 Tests.com


206
orthodontics.

329. a - Labialverted teeth lay labial or towards the lips. Buccoverted is in the posterior and lays
towards the buccal or cheek of the mouth.

330. b - Canine teeth are used to determine the occlusion classification when molars are not present.
However, when determining the occlusion classification, both ideally should be observed.

331. The correct order is: b, a, d, e, c


The correct sequence when establishing your working position is:
(1) Dental hygienist’s position in relation to the dental chair
(2) The patient in a dental chair
(3) The dental chair light
(4) Non-Dominant hand
(5) Dominant hand

332. b - The tip of the patient’s nose should be below the clinician’s elbow. This allows for relaxed
shoulders and elbows and prevents fatigue.

333. a - The correct clock seated position for the operator allows for proper ergonomics. It prevents
fatigue and injury.

334. a - Musculoskeletal injuries are when the muscles and the skeletal systems are injured together. It
is caused by repeated incorrect dental hygiene ergonomics and is often seen in the neck, back and
hands/wrists of the operator.

335. b - There is a higher percentage of microbes subgingivally, because of the shift to anaerobic and
gram negative bacteria. Supragingival plaque consists mostly of aerobic bacteria and yeast.

336. b - Immediately after eating or drinking acidic foods, your mouth is attacked by the acid. This
attack lasts for 20 minutes, until our mouth can neutralize the pH.

337. c - African Americans have a higher prevalence of periodontal disease, with Hispanics coming in a
close second. It is also more prevalent in men.

338. b - The endocrine system that is not controlled can cause hyperglycemia or high blood sugar levels.
This can result in an inability to fight infection or impair PMN chemotaxis. Research data suggests
that diabetes is a risk factor for gingivitis and periodontitis.

339. c - Fillings can be done when a patient is on bisphosphonates. However, more invasive procedures
should be avoided. It is due to the fact that bisphosphonates inhibit osteoclast bone resorption or
necrotic tissue. It affects healing and disrupts blood supply.

340. a - HIV positive patients are more susceptible to aggressive necrotizing periodontal disease (NUG
or NUP), because of their lack of immune response and the impairment of the CD4 glycoprotien.

341. c - Geographic tongue is an inflammatory condition of the mucus membranes of the tongue. The
causes of this condition are unknown. It is not a systemic disease nor can it increase your risk

TOP © 2015 - 2023 Tests.com


207
factor for periodontal disease.

342. d - Ulcerated tissue provides a direct pathway to the vascular system and could cause a patient to
go into sepsis.

343. a - 60-80 degrees is ideal for calculus removal. This angulation is in regards to the face of the
instrument to the tooth.

344. c - Wrist action is used for calculus removal and digital action is used when probing, exploring and
using sonic scalers.

345. d - Your patient’s position should not be a factor in the correct stroke selection. The patient should
be repositioned or positioned in the correct way that allows for effective and ergonomically
correct instrumentation.

346. a - The oblique stroke is used on the facial and lingual surfaces of anterior and posterior teeth. It is
not used at the line angles.

347. c - It is recommended that a cylinder shaped denture brush be used on dentures when cleaning
the inner curves. A rectangular shaped denture brush is recommended for the occlusal surfaces. A
regular toothbrush would be an ineffective cleaning option due to its shape.

348. c - Alkaline hypochlorite is inactive in the presence of organic matter. If a surface you wish to
disinfect has organic matter, it needs to be wiped away and then disinfected using the instructions
of the manufacturer.

349. b - Calculus removal is considered secondary in the prevention of disease control. Restorations
would also be considered secondary.

350. a - S. mutans is the bacteria that is found during the developmental stage of the caries process.
Lactobacilli are usually found in the dentin during the process. Actinomyces are opportunistic and
colonize after the initial infection.

351. c - Toothbrush abrasion can cause trauma to the tissue and cause the tissue to retreat. This
exposes the root surface of the tooth and makes it susceptible to decay or sensitivity. In many
cases, bacterium is the cause of recession, as in periodontitis.

352. c - Metronidazole is an antibiotic used to treat bacterial infections. It is the most commonly
prescribed medication for ANUG.

353. a - Both serous and purulent are types of exudate. Serous exudate has a few white blood cells and
purulent contains many white blood cells and also plasma.

354. b - When a patient has a high first past effect, it means that the dose of the medication given was
not large enough to produce an effect. A larger dose must be given.

355. a - Teratogenic effect directly relates to the effects that drug use and medications can have on a
developing baby. Continued use can cause abnormalities. An example of this would be fetal

TOP © 2015 - 2023 Tests.com


208
alcohol syndrome in newborns.

356. c - Cocaine causes an epinephrine release. It would be contraindicated to give more epinephrine to
a patient with high levels of epinephrine already.

357. b - A known intravenous drug user has a high vascular bacterial load and will need to be
premedicated to prevent a further or more serious infection.

358. c - Amoxicillin is part of the penicillin family. It will cause the same allergic reaction as penicillin.

359. d - HIV is not a contraindication for nitrous oxide. Respiratory diseases, such as COPD and cystic
fibrosis, are contraindicated, because nitrous oxide can further respiratory complications.
Pregnancy is contra indicated, because of an elevated risk of spontaneous abortion.

360. a - When there is a skeletal deformity, the use of orthodontics can move teeth to achieve an ideal
occlusion. It is done to mask the deformity and achieve correct occlusion.

361. d - Gingival hypoplasia is the incomplete development of tissue. Gingival hyperplasia is the
enlargement or overgrowth of tissue, caused by plaque accumulation.

362. a - X-rays are absorbed by tissue every time radiation is exposed. Not all of the radiation passes
through the patient onto the film and it can cause chemical changes resulting in biological damage.

363. c -The concept of As Low As Reasonable Achievable states that all exposure to radiation should be
kept to a minimum. This is accomplished by a dental assistant and hygienist using the correct
techniques and equipment.

364. d - The film was underexposed and the settings need to be increased. Decreasing the setting would
result in a dark film. Placing a film backwards does result in a light film, but you would also see a
herring bone pattern from outer layer of the film.

365. d - You are doing an infiltration of a specific tooth, rather than a field block or a nerve block, which
would anesthetize more teeth at one time. In this case, you would place the topical agent in the
vestibule of the specific tooth and nerve branch the dentist plans to work on.

366. a - The chlorine based in-office solutions need to be prepared daily. Chlorine dioxide, part of the
chlorine family, is a poor surface cleaner. However, that is not true for the rest of the chlorine
category.

367. d - It is important to note that radiography is not the best when trying to get an accurate image of
the TMJ joint. That is because there are many bony structures that obstruct the view. It is
recommended that a patient get cone beam imaging to get a more accurate image.

368. b - The panoramic x-ray is considered to be an extraoral x-ray. The film is on the outside of the
mouth and exposed by rotating around the head of the patient. The patient often bites down on a
stick for stability, but nothing else is placed inside of the oral cavity.

369. a - Stepping outside of the operatory while exposing x-rays protects you from all kinds of radiation,

TOP © 2015 - 2023 Tests.com


209
such as primary, leakage and scatter radiation. It is important for the assistant shadowing to
understand that all dental professionals should achieve zero amounts of radiation.

370. d - The patient chair button would be considered a touch surface, considering the office is not
using plastic barriers to protect the button from contamination. It could also be considered a
transfer surface if there was more than one button or key pad in the room.

371. c - You should always wear rubber gloves when handling contaminated materials and using
chemicals. Latex gloves can break down with certain chemicals and can easily be punctured.

372. b - 2 = moderate mobility. Every tooth has mobility, but periodontal disease and bone loss can
make certain teeth more mobile than others. 0 = normal, 1 = slight mobility, 3 = extreme mobility.

373. b - The patient has open-bite. This type of bite in common among thumb suckers. To correct this
type of bite, orthodontics should be considered.

374. a - Irreversible hydrocolloid aka alginate, is used when making preliminary and final impressions. It
can be used to make whitening trays and study models.

375. b - Acrylate resin is the material used in sealants. It can be highly allergic to a patient with this type
of allergy. If skin contact occurs, the area exposed needs to be thoroughly wash and rinsed with
water. Acrylates can penetrate gloves. Phosphoric acid is in the etching material, not the sealant
material.

376. c - The sealant material needs to be carefully dispensed, avoiding bubbles. If one does occur, try to
manipulate the bubbles out of the working area with explorer. If a bubble still does exist, add
sealant material to fill in the gap. If isolation was lost, etch again to add sealant.

377. c - There are six links in the chain of infection. If one link is broken, infection cannot occur. It is
important to understand this chain in regards to infection control throughout the dental office. It is
also important to understand the types of infections you may be exposed to in the dental office.

378. b - Type 2 diabetes is managed through proper food intake and through oral medication. Though
both are important, asking about the patient’s last meal would help you to distinguish between
hyperglycemia and hypoglycemia and allow for proper further action.

379. c - Hypoglycemia is when a patient has low blood sugar. It can occur when a diabetic patient skips
a meal, taking too much insulin without proper food intake or exercising without adjusting their
food intake.

380. a - The patient is hypoglycemic and needs to raise her insulin. Giving the patient concentrated
sugar, such as cake icing, helps the body to absorb the sugar more rapidly into the bloodstream.

381. d - Aluminum foil is used to detect areas where you do not see pebbling. The pebbling should be
evenly marked and throughout the entire submerged piece of foil. You perform this without
instruments to get the most accurate reading.

382. a - When running in a steam autoclave for 15 or 30 minutes, it should always reach the

TOP © 2015 - 2023 Tests.com


210
temperature of 250 degree Fahrenheit. However, when running on shorter cycles or a flash steam,
of three to ten minutes, you will need a temperature of 273 degrees Fahrenheit.

383. d - Placing the indicator in the center of the sterilizer with instruments allows for the most
accurate reading, as the center is the most difficult for the sterilizing agent to reach. Instruments
should be used to closely resemble everyday use.

384. c - To make sure the autoclave is working properly, you need to monitor the temperature,
pressure and time that it takes for each load. Knowing how to correctly load the autoclave and to
get the proper separation is important for the sterilization process. However, when receiving a
failure regarding biological testing, you should evaluate the equipment first for proper readings.

385. c - Both a and b are correct. The intraoral camera is a great tool to use in patient education. It also
allows the dentist to magnify and access areas that would not normally be as visible for evaluation.
It is not meant for diagnosis, but as an aid.

386. b - Generally, the maxillary posterior molars have three roots. Sometimes maxillary posterior teeth
have 4 roots, as these canals are hidden on the facial or lingual side. A dentist would be able to
assess for possible hidden roots during the root canal procedure.

387. a - The topical solution should be placed in the areas of injection for a minimum of 15 seconds to a
maximum of 2 minutes. It is important to note the type of anesthetic you are using and follow the
instructions of placement time.

388. d - The harpoon needs to be locked in or engaged in order for the dentist to pull back on the
thumb ring and aspirate before administering local anesthetic to the necessary site. The harpoon is
part of the rod that pushes the anesthetic out through the needle, also known as the piston.

389. c - Technically speaking, the dentist owns the x-rays even though they are the patient’s images and
they may have paid for them. They are part of a patient’s dental records that should be retained
indefinitely. Even though the dentist owns the images, copies can be made and released with
signed consent to another dentist or to the patient themselves.

390. b - Calibration is done by qualified technician in intervals regulated by each state. There are many
tests a dental assistant can perform to make sure that the equipment is working properly, such as
the output of x-rays, kilo voltage and focal point spot tests.

391. a - Digital radiography’s main infection control problem is cross-contamination of surfaces due to
blood and saliva. It is important to follow the CDC regulations regarding proper handling of the
digital devices.

392. d - Type IV is often related to a contact reaction that involves the immune system. It is the
chemicals used to process the gloves that cause the allergic reaction. The rash appears 48-72 hours
after contact. Type I is the most serious of latex allergies as it involves a reaction to the proteins in
latex. A response occurs within 2-3 minutes of contact. The patient could exhibit coughing,
shortness of breath or anaphylaxis.

393. b - It is impossible to create a latex free dental office, as there is latex in some of the products and

TOP © 2015 - 2023 Tests.com


211
supplies that are use. It is important to understand that latex protein can be present in the air, so it
is important to schedule these patients first thing in the morning, when the proteins are at a
minimum. It is also important that all PPE and exam rooms be latex-free while the patient is in the
office, even if it not the treatment room the patient is seated in.

394. a - The patient may experience thermal sensitivity that involves both hot and cold. It is
recommended that a patient use sensitivity toothpaste after teeth whitening. A patient may also
experience tissue irritation, such as redness. They may also get a gingival “burn” where the tissue
turns white from gel exposure. When this happens it is important that patient understands the
importance of not overfilling the whitening trays with gel.

395. c - Using a black pen with a single line allows another user to read the incorrect entry. Using your
initials and dating the entry allows the reader to know who made the mistake an when that
mistake was corrected.

396. d - Gingivitis is a sign and symptom of periodontal disease, not a risk factor. Gingivitis is
inflammation in the gingival tissue and can often be reversed with good oral care. Smoking,
diabetes and HIV/AIDS are al risks factors that can contribute to periodontal disease. These risks
factors can alter the body’s response in control the bacteria that causes periodontal disease

397. b - Any patient that is asking to have x-rays transferred or duplicated need to sign a release form
before the x-rays can be released. An office should never give out the original x-rays for legal
reasons. However, they can release duplicates or transfer x-rays electronically to another office or
to the patient.

398. c - Often times, when a duplicated image is dark, there are two possible errors: exposure time was
too short or the duplicating paper was placed with the emulsion side up. Placing the emulsion side
down allows for accurate duplication of x-rays. Remember, when duplicating x-rays, it is the
opposite of exposing x-ray film.

399. a - No PPE is needed when duplicating x-rays, as no chemicals are involved. The duplicating
machine should be placed in an area that is considered clean and sterile. It should not be placed in
an area where contaminated chemicals or x-rays may come into contact with the machine.

400. d - He is not missing any permanent teeth. All necessary teeth are developing. Sometimes, when a
primary tooth is lost, a permanent tooth does not erupt right away. It is important to check on
erupting periodically through x-rays and oral exams, as some children may be a case for early
orthodontics.

401. d - The panoramic x-ray was most likely taken at an adult setting and not a child setting. When the
exposure time, kVp and mA are increased, you get a darker image or increased density. By
lowering the exposure time, kVp and mA, you can decrease the density lighten the image. It is
important to know the correct setting for each patient.

402. c - A patient with HYPERtension would be contraindicated for the air polisher. The polishing
solution contains sodium bicarbonate, which could cause ankyloses. It should never be used on a
person with a heart condition.

TOP © 2015 - 2023 Tests.com


212
403. a - The first number is the systolic reading. It reflects the amount of pressure that is required for
the left ventricle of the heart to compress or push oxygenated blood into the blood vessels.
Diastolic is the second number and it reflects the muscle at rest.

404. b - All items should already be capped for infection control. They are then placed in a red container
that is labeled for sharps material. The container should be puncture resistant, closable, leak proof
and labeled with a biohazard symbol.

405. b - The Center for Disease Control and Prevention is the governing body that protects the health
and safety of the people. Occupational Safety and Health Administration is to protect the health
and safety of the workers. It is important to know the difference and which governing body you
are to refer to when having questions about infection control.

406. b - It is recommended that the waterlines be flushed for several minutes prior to the first patient
of the day to prevent biofilm in the waterlines. It is important that you also flush the waterlines
between patients for 20-30 seconds. This helps to eliminate material retracted from the previous
patient.

407. d - When sending out water to be tested, it is important that you flush the waterlines for at least
two minutes prior to taking the sample. This allows for the most accurate sample to be tested. It is
important that you take the sample before scheduled maintenance, so they can properly make
suggestions and corrections.

408. b - The manufacturer of a chemical is required to enclose a safety data sheet with every purchase
and shipment.

409. b - Phosphorus plates are part of indirect imaging system. The phosphor layer is able to store the
energy of the x-ray photons. It is considered indirect, because the phosphorus plates must be
scanned into a computer in order to see the image. With digital sensors, the image is directly
placed on the computer, whether the sensor is wireless or not.

410. a - After placing the x-ray in the developer, you need to rinse the film to remove all residual
developer from the film. Next you place the film in the fixer and time how long it is emerged. Next
you wash the film by using a water bath. Lastly, you dry the film before be used diagnostically.

411. b - One of the reasons the image may be light is that your developer solution is too cold. Some
other causes could be: Inadequate development time, inaccurate timer or thermometer, depleted
or contaminated developer solution. Having the developing solution too hot, excessive developing
time or concentrate developer would cause a dark image.

412. b - You would try Size 1 film, as it is not as long horizontally, but still has the height to get all
necessary tooth and bone structure. Size 0 would work, but it is shorter vertically and horizontally.
This means you might miss needed tooth and bone structure needed for accurate diagnosis.

413. c - The first things that should be done after washing and drying your hands is to place a paper
towel down on the bottom of the daylight loader. This is where you will place a container with
contaminated films, sterile gloves and empty paper cup. This allows for proper infection control
and easy cleanup.

TOP © 2015 - 2023 Tests.com


213
414. b - The patient has experienced syncope, which is the most frequent medical emergency in the
dental office. It happens when there is an imbalance in the distribution of blood to the brain and
to the larger vessels in the body. The possible signs and symptoms include: being in one position
to long, apprehension, fear, the sight of blood, or the sight of certain instruments.

415. a - Deflection is less when using a larger gauge needle. Meaning, it is less likely to bend as it enters
the tissue and allows for better accuracy. A larger gauge needle also allows for easier aspiration
and has a lower chance of needle breakage.

416. d - When there are large bubbles in the cartridge, it means that it was frozen. It is no longer sterile
and cannot be used.

417. c - Trismus occurs when the muscles in the oral cavity spasm, causing limited opening to the oral
cavity. This may cause pain or distress to a patient. However, it is usually temporary, not
permanent.

418. c - When anesthetic is placed in an area of infection, the onset is slower due to less anesthetic
being absorbed. This means that more anesthetic may be required to complete a longer dental
procedure.

419. c - Vasoconstrictors decrease blood flow, not allowing the anesthetic to be absorbed in the blood
stream. Once it is absorbed in systemic circulation, the anesthetic action is stopped.

420. d - Dental fluorosis is when a patient ingests too much fluoride during the formation of enamel.
Though it is a cosmetic concern, fluorosis does not mean the teeth are more susceptible to decay.

421. d - A bulla is a large blister that is under or in the epithelium of the skin or mucosa. If the area is
smaller than 5mm, it would be considered a vesicle.

422. a -This is due to the fact that these are placed to support dental prosthesis such as crowns, bridges
and dentures. These types of implants act as anchors to support the function of the prosthesis.

423. c - Chlorhexidine - Peridex


d - Essential Oils - Listerine
a - Cetylpyridinium Chloride - Crest Pro-Health
b - Quaternary Ammonium Compounds - Scope

424. d - Plaque is 80% water, with the other 20% comprised of bacteria and organic/inorganic solids.

425. d - Yellow-Brown - Chlorhexidine


a - Black-line - Iron compounds in saliva
b - Green/Yellow/Orange - Chromogenic bacteria
c - Blue/Green - Industrial Worker/Copper

426. a - The use of the half-moon Gracey curette is used with a modified-pen grip. Gracey curettes are
site specific due to the lateral offset of 70 degrees relative to the shank.

TOP © 2015 - 2023 Tests.com


214
427. c - Insulin-dependent diabetes would place a patient under an ASA III classification. You can still
treat this patient, but you will need to ask questions regarding their condition. It is also important
to note the role diabetes plays in periodontal health and vice versa.

428. a - Leeway space is created by the primary teeth to allow for the larger permanent teeth to erupt.
On each side of the mandible, the leeway space created is about 2.5mm of room with about
1.5mm on each side of the maxilla.

429. c - When increasing the PID or cone length on the x-ray machine, it decreases the amount of
radiation that a patient absorbs during exposure. A shorter beam means more exposure to the
patient.

430. d - Reticulation is when an arrangement of lines, resembling a net, can be seen on a developed
film. This occurs when there is a large temperature difference between developer solution and
water bath.

431. b - Indirect supervision is required by a dentist for scaling and root planning, especially when
anesthetic is being used on a patient. A doctor must be present in the office when a hygienist is
performing any procedure on a patient.

432. b - The irregular shape of a polishing agent would make it more abrasive to the tooth structure.
Having a more regular shape would make the polishing agent smooth, less abrasive to the enamel.

433. d - Educational Services - Discussing oral health with the patient


b - Secondary Prevention - Removing calculus
a - Primary Prevention - Administering a fluoride treatment
e - Therapeutic Services - Scaling and Root planing
c - Tertiary Prevention - Periodontal therapy

434. c - General supervision is when a doctor gives a diagnosis and authorizes a procedure. Removing
calculus and taking x-rays during a recall appointment is also considered general supervision.

435. b - Bulimia nervosa is an eating disorder where a person binges on high caloric foods and then
purges through vomiting. The acid from repeated vomiting causes erosion, usually found on the
lingual of maxillary anterior teeth.

436. c - A patient should wait at least 6 months to have any non-emergency dental procedures
completed. This is due to the high probability of second stoke during this time. Since he comes
into the office 2 months after his stroke, he has to wait 4 months.

437. c - An aphthous ulcer is not a contagious condition, nor is it a risk factor for a pregnant woman.
Pregnant woman may be more susceptible to getting these ulcers with acid reflux and hormone
changes. Gingivitis, pregnancy granulomas and cavities could pose a risk to a pregnant woman and
baby, due to the bacteria involved. All three should be addressed by a dentist to see if further
treatment is necessary.

438. a - Fluoride is deemed safe and recommended for pregnant patients, due to hormone and
bacterial imbalances.

TOP © 2015 - 2023 Tests.com


215
439. d - Ankyloglossia is when the tip of the tongue is adhered to the floor of the mouth or the anterior
portion of the mandible. This is commonly called “tongue tied”.

440. b - Dilacteration occurs when a root curves or bends. This can make it hard to get periapical or
bite-wing x-rays of a tooth. There is nothing clinically wrong with tooth.

441. a - Calcium channel blockers disrupt the movement of calcium through calcium channel. This can
cause an overgrowth of gingival tissue anywhere in the oral cavity.

442. b - Sealants use a mechanical bond, which means it fills in the space in the grooves of the teeth
and is held there by those spaces. There is not a chemical bond between the sealant material and
the tooth.

443. d - Tin is part of an amalgam filling. Other amalgam fillers include: silver, copper and mercury.
Mercury is used as a binder for all of the materials.

444. a - Coarse Pumice should never be used to polish composite material, because it is too abrasive to
the material. It causes the material to lose gloss. Polishes that use fine particles of uniform shape
should be used.

445. c - Carbamide peroxide is a whitening agent that could cause tooth sensitivity. The desensitizing
agents in toothpaste would help alleviate sensitivity after whitening.

446. c - There should be 10 days between any dental treatment that requires a patient to pre-medicate.
This reduces the emergence of resistant bacteria. If a procedure needs to be done prior to 10 days,
the patient should take a different antibiotic to help stave off the resistant bacteria.

447. a - The mental foramen is the most often mistaken foramen as a periapical abscess due to its
location between the root tips of the mandibular. The angulation of your PID during x-rays and the
foramen location make this look like an abnormality.

448. b - Delsym is an antitussive medication, which is a cough suppressant. Most medications, other
than cough suppressants, can cause dry mouth for a patient.

449. b - Xerostomia is a side effect of many drugs. It is important to note the drugs patients are taking
and educate them on how their oral health can be affected.

450. a and c - Bruxism is very common among kids and adults. Most patients who experience bruxism
do not realize they are clenching or grinding their teeth, because they do it in their sleep.
However, during stressful times, people also clench or grind during the day.

451. c - The tooth that is missing a filling is tooth #12, which is a maxillary first premolar.

452. b - When a tooth takes on a heart shape and looks like two teeth, but the child has the correct
number of primary teeth in the mouth, it is called gemination. Gemination occurs when a tooth
bud tries to divide. When two teeth fuse together, thus creating a shortage of teeth in the mouth,
this is called fusion.

TOP © 2015 - 2023 Tests.com


216
453. b - Though the job of the hygienist is to help heal infection, the ultimate goal is to allow the
connective tissue to reattach, thus creating a smaller pocket.

454. Both a and b - Kaposi’s sarcoma is a cancerous virus, stemming from the herpes family. It is most
often seen in those affected with suppressed immune systems, such as those whom are HIV+ or
have AIDS. This cancer causes abnormal tissue under the skin that is a purple/red color.

455. c - Smokers have a higher risk of oral cancer when they use an alcohol based mouth rinse. This is
due to the drying effect of the alcohol, creates a higher risk environment in the oral cavity. It is
recommended that smokers use a non-alcoholic mouth rinse.

456. b - Periodontal flap surgery is a surgery that is performed by a periodontist. It involves cutting back
the tissue to expose the teeth and bone. This allows for access for debridement.

457. d - Wear facets are often the product of severe bruxism or clenching. These areas are susceptible
to decay and can be sensitive, especially since dentin is exposed. In some cases, these areas can
be filled with composite material to help with sensitivity. It also recommended that the patient
gets a night guard.

458. d - Calculus is dental plaque that has undergone mineralization. Supragingival calculus is formed by
our saliva and subgingival calculus is formed by gingival cervical fluid. The picture shows
supragingival calculus and moderate gingivitis, not subcalculus.

459. d. Physiological Needs


a. Safety and Security
f. Social Needs
c. Self Esteem
e. Actualization
b. Self

460. b - Program formulation is conducted after the initial needs are assessed. The formulation will
precede implementation, financing or evaluation of the said program.

461. c - Epidemiology is when a disease is studied within a population. These studies will focus on how
they relate to specific cases as well as testing theories or hypothesis.

462. a - Analytical studies are a type of epidemiology study method that focuses in testing a theory or
hypothesis regarding the distribution of a disease in a population.

463. b - Non-randomized studies are least valid because the selection of test subjects is not
randomized. The results may be skewed, as the test subjects aren’t randomly assigned,
jeopardizing the validity of the study.

464. c - The double-blind study would be the most valid test to perform. Neither the company or the
test participants know who is in the control or experimental group, which means there is no bias
regarding the data collection.

TOP © 2015 - 2023 Tests.com


217
465. a - If a p-value is less than 0.05 during an experiment, then there was a significant statistical result.
If it was greater than 0.05 then it was insignificant, and if it was equal to 0.05 then the difference
occurred by chance.

466. c - The Chi square test compares actual results versus independent results and provides a
comparison that measures the significance of observed and expected frequencies.

467. d - Quantity does not qualify something as good criteria for an index. Criteria for a good index
includes reliable, valid, simple, clear, objective, acceptable and quantifiable.

468. b - A reversible index is used to study gingivitis because gingivitis is a reversible condition.

469. a - Calibration, or intraexaminer reliability, is a way to record data the same way between
examiners during a test study.

470. d - The DMFT (decayed, missing, filled teeth) index is used to measure caries experiences in a
population. It is an irreversible index.

471. c - The SBI- sulcular bleeding index is used to detect early symptoms of gingivitis. It is a reversible
index and scores from 0-5. The GI index determines the prevalence and severity of gingivitis but it
does not detect early symptoms.

472. a - The PDI score would be tabulated using the score of the teeth examined divided by the number
of teeth that were examined. Scoring would range from 0-1 with no inflammation present to
advanced destruction.

473. c - CPITN is the Community Periodontal Index of Treatment Needs and was developed by the WHO
as an irreversible index to measure a group’s periodontal needs. The CPITN and PSR use the same
scoring criteria.

474. b - The Ramfjord teeth are teeth #’s 3, 8, 14, 19, 24 and 30. There is one tooth per sextant that is
used and measured. If the tooth in that sextant is missing, then there is no number recorded for
that sextant.

475. d - Once a 4 is recorded in a sextant there is no need to examine the rest of the teeth in the
sextant, because a code 4 is the highest coding used, indicating pockets of 6mm or deeper.

476. c - There is moderate soft debris within the gingival margin of the patient’s teeth. Using the PI
plaque scores range from 0-3.

477. a - The PHP (Patient Hygiene Performance) index is a reversible index that uses disclosing solution
to measure the amount of debris present.

478. b - CHIP is the Children’s Health Insurance Program and was an amendment in 1997 to cover
children whose families had too high of an income to qualify for Medicaid coverage.

479. c - Title XVIII Medicare was to provide insurance for people over age 65 to cover medical needs,
although it has very limited dental coverage. This was one of the first amendments to the social

TOP © 2015 - 2023 Tests.com


218
security act of 1935.

480. a - Indemnity plans are also called traditional or fee-for-service plans and are offered by most
insurance providers. Coverage typically range from 50-80% depending on the procedure being
performed

481. d - Research in habits is not a category of community oral health research. The category that was
not listed that is the remaining of the 4 categories is Research in educational techniques and
behavioral science as they relate to oral health education.

482. b - A Cohort is a well-defined group that is watched over time to observe a natural progression of a
condition or disease after an exposure, and no factors are controlled.

483. b - Approximately 25% of adults over the age of 18 have had a case of root caries. The rate
increases with age.

484. d - Dental caries is the most common chronic childhood disease, surpassing both asthma and hay
fever. It is one of the major reasons for hospitalizations in a child.

485. c - Both first and second molars are the teeth most frequently affected by dental caries. These
teeth erupt at a young age and the deep grooves and fissures often develop decay if care is not
adequate.

486. c - Approximately 85% of adults have experienced dental decay. Prevalence of decay increases
with age.

487. d - Morning sickness is not among the risk factors for cleft lip or palate. Other conditions that are
related to clefts include maternal influenza, oxygen deprivation, alcohol consumption and
smoking.

488. b - Class 2 fluorosis is when the fluorosis is more extensive but it does not involve up to 50% of the
tooth surface.

489. b - Nearly 60% of new decay areas on pits and fissures will be reduced for 2-5 years following
school based sealant programs. These type of programs are an efficient method to discourage
decay in school age children.

490. c - Water levels with over 2-4ppm fluoride should undergo defluoridation procedures. 2ppm is a
nonenforceable guideline while 4ppm is the maximum concentration allowed by the EPA.

491. c - Oral cancers have a higher death rate than most popularly discussed cancers. Because many
cancers discussed in the media promote early diagnosis and intervention, these are detected
earlier and receive care. Oral cancer does not receive as much publicity and may not be noticed by
a patient who is not under the regular routine of a dentist.

492. c - A convenience study does involve participants that are close to the examiner, but it is not
considered a reliable study because it does not involve the general population, but rather a group
that is convenient to the examiner.

TOP © 2015 - 2023 Tests.com


219
493. b - Reliability is when the index will measure consistently no matter when the reading is taken
place. Reliability allows different examiners to collect the same data and have the same outcome
between them.

494. c - The DMFT (decayed, missing, filled teeth) index measures 28 teeth, excluding the 3rd molars and
primary teeth.

495. a - The deft index (decayed, exfoliated, filled teeth) is an irreversible index that excludes missing or
unerupted teeth.

496. a - The deft index only evaluates 20 teeth, the primary teeth, in people ages 11 and younger.

497. b - Occlusal surfaces are not recorded in the SBI (sulcular bleeding index). Only distal, mesial, facial
and lingual surfaces are recorded.

498. b - A class II periodontal treatment need would indicate that the patient or community needed
debridement, instruction and has retentive margins. The next (and highest) level also includes
possible periodontal surgical intervention with local anesthesia.

499. a - The OHI-S (Simplified Oral Hygiene Index) examines the tooth surface for any debris or calculus,
and is used to measure oral cleanliness. It is an irreversible index.

500. c - Healthy People 2020 and 2010 editions contain lists of oral health objectives that help establish

501. d - A Cohort study requires at least 2 (or more) measures in order to perform a longitudinal,
prospective or incidence type of study.

502. a - An epidemiologic survey will target naturally occurring samples of a population. Usually a large
sample is used, with general indices that gather data to establish underlying etiologic factors to
help develop hypothesis regarding the condition.

503. a - Strategic is not an example of a sampling technique. There are 6 which include: Random,
Stratified, Systemic, Convenience, Judgmental and Cluster.

504. c - Random sampling is the most commonly used technique, because every subject is selected
independently and all subjects have an equal chance at being selected. It prevents research bias.

505. c - When a disease is endemic, there is a constant presence of the disease in a particular area or
region. An example of this would be malaria, which is localized to the tropical areas of South
America and Africa.

506. b - The control group is a reliable baseline that does not receive the treatment or intervention of
the study. All of the results are compared against this group.

507. c - Regional is not a type or level of governmental agency that has a community dental health
program. The agencies that are part of dental health are: Local, State, Federal and International.

TOP © 2015 - 2023 Tests.com


220
508. b - Descriptive study helps track who is getting a disease, what they are getting and where it is
occurring. It answers the who, what and where questions related to disease. It does not answer
the how/when/why questions. Experimental studies test hypotheses to establish the cause of the
disease.

509. a - The mode is the most frequently occurring value in research data. The median is the halfway
point in a set of research data, usually measured by dividing the set into two equal parts. The
mean is found by adding all of the values/numbers and dividing by the number of values in the set.

510. c - Non-parametric inferential statistics are most useful for data measured on the nominal or
ordinal scale. Nominal or ordinal data is qualitative and although numbers may be involved, they
are derived more subjectively than data associated with quantitative data. Techniques in non-
parametric statistics rely on data that does not belong to any particular distribution. Another
difference between non-parametric and parametric inferential statistics is that the parameters in
non-parametric statistics are more flexible and grow with the training data, where the parameters
are fixed with the model in parametric statistics. The most commonly used non-parametric test is
the chi-squared test, which is used to examine the differences between observed and expected
frequencies.

511. a - In statistics, the p-value shows statistical significance, in that it represents the probability that a
study could have come to a false conclusion. It shows the strength of the evidence about the
population in question. The smaller the p-value, the more significant the findings of the study are
considered. A small p-value (typically < 0.05) indicates strong evidence against the null hypothesis,
so it can be rejected. A large p-value (typically > 0.05) indicates weak evidence against the null
hypothesis, so its rejection fails. A p-value at or close to 0.05 is considered marginal, in that it
could go either way.

512. The correct order is: b, a, d, e, f, c


The stages of learning occur in the following order:
(1) Unawareness
(2) Awareness
(3) Self-interest
(4) Involvement
(5) Action
(6) Habit

513. d - A longitudinal study often spans decades and involves the same subjects. An example of this
type of study would be the decade long study of children that studied the effects of fluoridated
water and caries reduction.

514. a - A null hypothesis means that it is opposite of the hypothesis. The null hypothesis is a default
position in the modern practice of science that there is no relationship between phenomena or
differences between groups. It is tested by using a statistical hypothesis test, which uses a null
hypothesis and alternative hypothesis.

515. c - Informed consent is an ongoing process, but it should not be discussed in a group setting.
Informed consent is often done in the initial stage of treatment and should be completed in a one-
on-one setting.

TOP © 2015 - 2023 Tests.com


221
516. a - Discrete data involves numeric variables, such as the number of patients being examined. It is
only counted in whole terms.

517. b - The biggest disadvantage of doing interviews in research is that it is very time consuming.
However, if time is not an issue, it can be a great way to get data.

518. c - A job of a dental hygienist is to help provide preventative care to those who may not have the
ability or the dexterity to perform oral care themselves. A dental hygienist license is sufficient to
perform these duties in a nursing home facility and no special license is needed.

519. b - In a Type III examination, you use a mouth mirror. Type IV examinations use a tongue
depressor.

520. a - External motivation is described as motivation derived from outside of the individual. Intrinsic
motivation stems from the individual them self.

521. d - Medicare provides medical and dental care, but to persons over the age of 65. Medicaid is
designed to help low income families get the proper medical and dental care they need.

522. a - Peduncle is an elongated stalk of tissue. Sessile is the opposite, meaning there is no stalk, or
lacks a peduncle. Cysts, polyps and tumors can also be pedunculated.

523. c - It should be recommended to the patient that they clean the mouth thoroughly following the
use of albuterol, as many asthma inhalants can cause drying of the mouth as well as higher decay
rates in the teeth. The patient may want to rinse with water and brush following his treatment. An
additional fluoride rinse available over the counter would not be a bad idea, but it isn’t the primary
concern.

524. b - Apthous ulcers or canker sores are typically flat, round or oval, and raw in appearance. Factors
like stress or sunlight can cause them to occur more frequently.

525. c - The 6 year molars, teeth 3, 14, 19 and 30 would be a good place to start with recommending
dental sealants. The 12-year molars have not all fully erupted. While sealants can be placed on
premolars as well, not all premolars have erupted so this is not an option.

526. b - The radiolucency in this area is typical of interproximal caries in a primary tooth. It is near the
proximal areas of #k and #14, but thankfully looks as if the decay has not reached into #14.

527. d - The flossing technique and regular flossing should be introduced and reviewed with the patient
to prevent interproximal decay.

528. a - Suggest the patient wear a guard during all practices as well as games, because orofacial
injuries are not specific to just organized sporting events but may occur at any time a sport is
practiced. A custom guard will be more effective than a stock guard, but wearing any guard at all
for all activities is better than none at all.

529. b - Advise the patient to drink lots of water during practices. If he wants to have a sports drink

TOP © 2015 - 2023 Tests.com


222
during the game, advise him to have only one and then rinse the mouth and drink only water
afterward. Discuss the benefits of water and how frequent long-term exposure of sports drinks
may contribute to decay.

530. b - Hygienists may discuss the occlusion findings with the patient and parent, but they are not
allowed to refer patients to specialists such as orthodontists. The correct procedure would then
follow with the hygienist discussing the findings with the dentist so that the dentist can make the
appropriate referral or diagnose needs.

531. c - The patient’s 12 year molars are coming very close to erupting into the oral cavity. Sporadic
discomfort is normal with tooth eruption, and as the soreness is localized to these 2 areas then
eruption would be the cause.

532. a - The patient has 5 remaining primary teeth. Teeth #A, H, J, M & R are still present in the mouth.

533. b - The patient still has 2 primary molars in this area, with #A appearing to have decayed along as
the roots undergo the natural resorption during the process of exfoliation. The patient should be
encouraged to wiggle the tooth out on her own in the fairly near future.

534. d - There are 10 molars evident in the mandible on the patient’s x-ray. 4 primary molars, her 6-
year molars and the developing 12 year molars and wisdom teeth.

535. c - Hyperfluorosis would not have contributed to an increased risk of fracture and decay. However,
if the patient sustained an injury during her horseback riding accident it could have done some
damage, which could be compounded by the fact that she chews bubble gum and pens at school.
The patient should be encouraged to wear a mouth guard even during horseback riding.

536. b - 2 horizontal BWX would show the interproximal areas of the primary molars and permanent
first molars. Because there are no permanent second molars erupted in the mouth there is no
need for an additional view with a 3rd and 4th film.

537. c - Adult incisors are much more “yellow” than primary incisors, because of the large amount of
dentin inside of the tooth structure. Dentin is naturally yellow and is evident especially when the
incisors erupt next to existing primary teeth.

538. c - Tetracycline stain is an intrinsic stain that manifests itself as a grayish discoloration of the tooth
enamel. The stain is caused from tetracycline ingestion by the mother while pregnant, or if the
child was given the medication at an early age.

539. c - A soft bristled toothbrush should always be used rather than stiffer bristles that are known to
cause conditions like gum recession and abrasion to the enamel.

540. b - The Roll method of brushing starts with the bristles at the gum margin and then rolls the brush
toward the incisal or occlusal surfaces.

541. c - Two-tone disclosing solutions will stain new plaque red and mature plaque blue, that way oral
hygiene instruction can address areas in the mouth that have developed plaque buildup due to
inefficient brushing over time.

TOP © 2015 - 2023 Tests.com


223
542. a - 1.0mg of weekly supplemental fluoride is the proper dosage for people ages 6-16.

543. c - Nitrile gloves would be the appropriate choice to use on a patient with a latex allergy. Another
option that is not listed would include vinyl gloves. The other options listed contain latex and
would cause an allergic reaction.

544. b - According to the patient’s radiographs, #7 is rotated. This is evidenced by the appearance of
the crown facing distally and the margin of the lingual surface visible toward #8.

545. d - Flossing would be the most appropriate choice of oral hygiene instruction for a patient that
exhibits interproximal bleeding, as floss is the only oral hygiene method listed above that would
remove plaque biofilm in this area.

546. b - Placing sealants on the permanent molars (3, 14, 19, 30) would be an effective preventive care
procedure that could prevent pit and fissure caries in these teeth.

547. b - Tooth #16 exhibits the initial enamel formation of a tooth bud. This phase is the amelogenesis
phase of morphodifferentiation where differentiated cells begin to form tooth enamel. The other
3rd molars are not yet visible.

548. a - Tooth #4 best exhibits the appearance of a tooth in the appositional stage of development. The
matrix will soon undergo maturation or calcification.

549. b - The ideal recall schedule for this patient should be 6 months. It is important to explain to the
child’s mother that routine preventive care appointments can help prevent large amounts of
calculus from building up and contributing to inflammation.

550. d - A class I malocclusion is a normal relationship between the molars with versions such as cross-
bites or overjets. The patient’s molars are in a normal class I relationship, but the slight overbite
places it into a class I malocclusion.

551. a - Using a dry toothbrush, or even brushing with water alone can be useful in the removal of
plaque biofilm. The patient should be encouraged to take his toothbrush with him at the very
least, and to use it regardless of whether or not he also has toothpaste packed.

552. a - The patient has 12 permanent teeth that have erupted in the mouth - #3, 7, 8, 9, 10, 14, 19, 23,
24, 25, 26 and 30.

553. c - #18 is a congenitally missing tooth. It is a 12-year molar and should be evident on the x-rays.
Some of the other teeth to choose answers from were 3rd molars, but because the patient is only 9
it would not be possible to know at this time if those teeth were congenitally missing or not.

554. b - A class III malocclusion includes a lower first molar that is in a mesial relationship with the
mesiobuccal groove compared to the mexiobuccal cusp of the maxillary first molar. This type of
bite usually appears as an underbite or a jaw that is too far forward.

555. b - Microorganisms on the tongue and teeth can create malodor as they produce sulfur

TOP © 2015 - 2023 Tests.com


224
compounds or other foul-smelling compounds. 90% of malodor is from the oropharynx region. The
other 10% is due to systemic health.

556. d - All of the above options are appropriate to use on the surface of the tongue to remove odor
causing bacteria.

557. a - The appropriate brand of toothbrush is not applicable. The patient should be educated on the
proper brushing technique so that not too much force is applied, in addition to the fact that the
patient needs to replace his toothbrush every few months or when the bristles become splayed.
Because the patient pushes hard when he brushes, it should be noted that he selects a soft
bristled brush to use.

558. a - A maximum of 2ml per arch, or 4ml total should be placed in a fluoride tray for application. A
toxic dose is 5mg/kg for children.

559. b - Thumb sucking for an extended period of time may have caused the patient to develop a
tongue thrusting habit.

560. b - Acidulated phosphate fluoride should not be used on composite restorations, bridges, crowns
or veneers due to its acidity.

561. d - Hemangiomas are areas that contain a benign proliferation of blood capillaries. They are
typically present at birth or as a result of trauma.

562. b - A custom made mouth guard will fit the most comfortably and snugly against the teeth as
opposed to a stock guard or mouth-formed (boil-and-bite) protector available at the store.

563. d - Smoking is a risk factor associated with osteoporosis. Other factors include low body weight,
low intake of calcium, alcoholism and of white or Asian descent.

564. a - Angular cheilitis is inflammation and cracking in the corners of the mouth. It is most often
caused by a candida albicans infection and may also be due to a nutritional deficiency or denture-
wear.

565. c - In iron deficiency anemia, the tongue is usually devoid of filiform papilla, causing a burning,
painful sensation. The mouth may also have dry cracks in the corners. Systemic symptoms may
include brittle nails, an enlarged spleen, pallor of skin and fatigue.

566. d - All of the above information is important in the daily care of removable prosthesis and the oral
tissues under them.

567. d - A water flosser would provide an effective means of plaque removal for a person with limited
dexterity, under areas such as bridges. Another option available to the patient would include a
proxa-brush.

568. d - Dental fluorosis is an intrinsic stain that causes white to brown colored lesions and affects the
ameloblasts during tooth formation. It cannot be removed.

TOP © 2015 - 2023 Tests.com


225
569. c - The patient has generalized severe bone loss. There are several probing depths throughout the
entire mouth that measure 7mm or deeper.

570. b - Incidental curettage of diseased pocket epithelium may occur during the scaling of subgingival
areas. This occurs on the other working edge of the curette when the active blade is against the
tooth surface.

571. c - Grade IV furcation is a tooth with severe furcation involvement that has through and through
visibility of the furcation. It also includes complete loss of interradicular bone.

572. a - Class II mobility is moderate mobility in a buccolingual direction, but there is no mobility when
the tooth is compressed.

573. d - Supraeruption of the teeth typically occurs when a tooth no longer has another tooth to
occlude with, causing it to erupt further out of the jaw.

574. b - The patient is taking the two prescriptions for dementia. She is taking a lower dosage of Aricept
than its full dosage for advanced Alzheimer’s disease.

575. a - Tooth #27 is the only tooth in the mouth to have had a root canal. This is evidenced by the
radiopacity of the nerve canal on the x-ray. The patient is missing the adjacent teeth, #26 and 28,
as noted on their clinical chart.

576. b - Linea alba is keratinized tissue along the occlusal plane, evident on the buccal mucosa. It is a
single line of thickness and does not require any treatment. It is simply due to occlusion forces
against the cheek.

577. c - The 12x14 bridge would be easiest for the patient to floss, as there are no abutments on either
end and it is placed on a single tooth. This would allow the floss to be brought interproximally at
the end of the bridge and then slid underneath the pontic.

578. b - The deepest probing depth at tooth #13 is 5mm, which is in the moderate range of bone loss.
7mm or deeper is considered severe bone loss, and 3mm or shallower is considered a healthy
pocket.

579. c - The total measured amount of bone loss is 7mm, because the pocket is 4mm deep and there is
already 3mm of recession from the CEJ to the top of the detached periodontal pocket tissue.

580. a - The best nutritional counseling would be to advise the patient to have a soft diet that includes
nutrient dense foods. She may also want to have nutritional shakes, but she needs to be having
some solid food to maintain the health of the remaining teeth and physical functions.

581. c - Attritional wear of the above mentioned crowns can be caused by premature wear of these
teeth during their occlusion against 2 of the only 3 teeth on the lower arch, most likely to the
patient not having other teeth to chew with.

582. b - Using the Fones brushing method would not aid the patient in any way in regards to her
dexterity. However, tips such as placing a large handle on her brush for an easier grasp, using a

TOP © 2015 - 2023 Tests.com


226
large handled electric toothbrush or having someone help her brush would be an effective plaque
removal method for someone with dexterity problems.

583. c - The hyoid bone is the “u” or “j” shaped radiopacity that is evident in this area of all panoramic
x-rays. Depending on the positioning of the patient’s head, the hyoid may overlap onto the jaw on
the final film.

584. c - Nutritional counseling is extremely important for this patient. Based on his social history, he
consumes soda on a high frequency, which is contributing to a high rate of recurrent caries.

585. b - A calculus deposit is evident on the distal of #3, and is most likely visible on the mesial of #3,
although its location at the mesial appears as if it could be an amalgam overhang. Because there
are no restorations on the distal of #3 we know it must be calculus.

586. a - The incisive fossa is a radiolucent area that appears around the apex of the maxillary lateral
incisors. It is due to an area of bone that is less dense than the surrounding areas.

587. b - The patient has generalized, severe bone loss as evident by multiple pockets 5mm deep or
deeper.

588. b - A stainless steel crown is placed on #19. This can be seen through the “SSC” marked on the
dental chart, as well as observing the rough margins on the x-ray, which are typical of temporary
stainless steel crowns.

589. a - Elimination of periodontal pockets is a long-term goal of nonsurgical periodontal therapy. The
other options are short-term goals.

590. c - Listerine is not a commonly used solution for subgingival irrigation following a periodontal
procedure. The most common are 0.12% chlorhexidine, 0.4% stannous fluoride and 0.05%
povidone-iodine.

591. d - An After Five ultrasonic tip is useful for deep areas that have furcation involvement, or
developmental concavities on the tooth.

592. d - A 13/14 curette is for scaling the distal areas of the posterior teeth on both the upper and
lower arches.

593. d - Dental floss should not be snapped through the contacts as it could result in damage to the
interdental papillae.

594. a - Charter’s method involves placing the bristles angled upward against the long axis of the tooth,
toward the occlusal surface of the tooth, then brushing in a short back and forth motion.

595. c - The roll method involves brushing with the bristles at the gingival margin and sweeping them
apically over the gingiva, then sweeping up over the gingiva onto the tooth.

596. b - A composite MO filling is placed on #18. This is evidenced by the smoother contours on the
radiograph, and the encircled area on the patient’s chart.

TOP © 2015 - 2023 Tests.com


227
597. b - Green stains are typically caused by chromogenic bacteria and are extrinsic. Therefore they can
be polished off of the tooth.

598. c - A calculus rating of 2 indicates that supragingival calculus covers more than a third, but less
than two thirds of the exposed tooth surface.

599. b - The Williams probe marks mm measurements at 1, 2, 3, 5, 7, 8, 9 & 10. It is useful for
measuring the periodontium, attached tissue and oral lesions.

600. a - 110 degrees is the proper angulation of the sharpening stone to the face of the instrument.
Then begin sharpening at the heel 1/3 of the working end, then the middle 1/3 and the toe 1/3.

601. d - Tetracycline (actisite) is delivered to specific sites by being impregnated in a cord that is placed
into the pocket and then removed a week later.

602. d - A plaque index of 2.0-3.0 is considered poor.

603. b - Squamous cell carcinoma is common near the site where tobacco is held, in people that use spit
tobacco.

604. a - Both statements are true in regards to chewing tobacco. Oral lesions often disappear after
discontinuing the use of dip. Dip is associated with oral, laryngeal and pharyngeal cancers.

605. d - Median rhomboid glossitis is the atrophy of the central papilla, and typically leaves an oval or
diamond-shape in the middle of the tongue, just before the circumvallate papilla.

606. b - Mobility that is greater than 1mm is at least a class II mobility. Once the tooth is depressible it
becomes class III, therefore this tooth is only a class II.

607. a - Resorption of the root causes a loss of tooth structure as it is destroyed either internally or
externally. This is typically due to trauma or caries in the pulp.

608. c - Vertical bone loss is evident on the mesial of #30. The distal of #30 has horizontal bone loss.

609. d - An infrabony pocket is also called an intrabony, intra-alveolar, or subcrestal pocket, and is when
the base of the pocket is apical to the alveolar crestal bone.

610. c - The intercellular spaces of the junctional epithelium widen as pathogens invade the host
tissues.

611. c - Arestin has an extended-release of 21 days following placement. After placement the area
should not be scaled for 10 days, probed for 30 days or retreated for 3 months.

612. b - Arestin is delivered in a capsule form that is inserted into the periodontal pocket with a syringe.

613. c - Dilantin is a medication used to treat epilepsy (seizures) and control them. The patient has no
other health conditions.

TOP © 2015 - 2023 Tests.com


228
614. a - Lichen Planus is a benign disorder that is typical in middle age person. It is often found on the
buccal mucosa or the dorsal surface of the tongue and it cannot be wiped off.

615. d - All of the above are some of the normal features and manifestations that accompany a person
with Down syndrome.

616. b - Chromosome 21 is the one affected in some or all of body cells in patients with Down
syndrome. It is also called trisomy 21.

617. d - Persons that have Down syndrome typically have delayed eruption of the teeth, and may have
abnormal tooth development.

618. c - Generalized absence seizures are also known as petit mal seizures, and only last about 5-30
seconds. A grand mal (Generalized tonic-clonic) would last 1-3 minutes, while a complex partial
seizure would cause behavioral alterations.

619. c - A plaque score of I indicates that there is film at the gingival margin.

620. a - Nutrutional counseling should address the patient’s high intake of fruit juices, or liquids other
than water frequently throughout the day, as this may contribute to an increased risk of recurrent
decay.

621. a - APF would be contraindicated as the low pH may cause the dissolving of glass restorations.

622. b - The rotation of #22 and 27 make them more susceptible to gingivitis as plaque removal is more
difficult to achieve. The other areas of the mouth do not have problems with bleeding, so brushing
and dexterity are not the causes. Bone loss is not evident on the x-rays.

623. d - The most important thing to do for visually impaired patients is to describe the surroundings or
procedures to them. The patient will let the provider know if he needs assistance.

624. b - Using a “touch, feel, tell” approach is most appropriate for visually challenged patients as it
allows them to experience different sensations and better understand new concepts. Use large
educational models when possible.

625. a - An area where some mild recession is present would allow the dentin to be exposed, and may
feel rougher to the patient. This would only occur at the gumline, below the CEJ.

626. a - Supereruption may occur in these teeth as there are no teeth in the opposite arch to occlude
with. If the patient is able to have a new partial fabricated, then it will help prevent further
supereruption in these teeth.

627. b - The nasal septum is the radiopaque area found here. All other options would appear as
radiolucent.

628. d - Judging by the appearance of the patient’s radiograph, a poorly contoured crown is present in
this area. The hygienist is feeling the distal margin of the crown with her explorer.

TOP © 2015 - 2023 Tests.com


229
629. c - Brown stain is typical in coffee and tea drinkers. However, black line stain is of unknown
etiology. The two are different types of stains.

630. a - A thermal burn due to a hot food is the likely cause of this lesion. Pizza often causes burns in
the roof of the mouth.

631. c - Hydrogen peroxide is a common cause of black hairy tongue. While in smaller dosages (less
frequently) it may not cause black hairy tongue, it can when used regularly.

632. a - Frictional keratosis has occurred due to the chronic rubbing trauma of the buccal tissues against
the fractured teeth, resulting in keratinized tissue. Treating the teeth in the lower right area will
correct this condition. If it were an irritation fibroma the tissue would be the same color of
adjacent tissues.

633. c - The patient has been unable to open her mouth since an accident this morning. The appearance
on the radiograph appears to show a fracture of her right jaw, which will require immediate
medical attention.

634. b - The patient’s maxillary lateral incisors are missing (7 & 10). It is too early to determine whether
or not their 3rd molars are missing or not.

635. a - This opacity is a retained primary canine. This is visible by the location and the shape of the
opacity, which has been pushed out of the way for the permanent canine to erupt.

636. c - Tooth #12 is rotated due to the extra space in the arch, caused by other missing teeth.

637. b - Tooth #16 is the 3rd molar and is just beginning to show up on the radiograph, signaling that the
amelogenesis stage of tooth formation is occurring.

638. c - Failure of the maxillary and medial nasal processes to fuse during fetal development will result
in cleft lip, or if severe enough, a cleft palate.

639. b - Mandibular second premolars typically erupt around the age of 11-12. Maxillary second
bicuspids may erupt a year sooner.

640. c - ADHD is more common in boys. All of the other statements are correct.

641. b - Using a mouth guard (or sport guard) during recreational activities can decrease the rate of
tooth fractures and concussions. Most organized sports already require protective equipment such
as helmets or pads, but many do not require mouth guards.

642. c - Dentigerous cysts are also called follicular cysts. They are always associated with the crown of
an unerupted tooth.

643. d - Patients that require some assistance transferring into and out of their chair for any reason are
best helped by utilizing a safety-waist belt.

TOP © 2015 - 2023 Tests.com


230
644. d - Tooth #29 still has a large portion of the crown intact, and exhibits a periapical radiolucency,
unlike #14 which was the other tooth listed that still had a crown intact.

645. a - The mandibular canal appears as a radiolucent line that runs down each side of the mandible
from the ramus to the mandibular foramen.

646. b - A ranula is a large 1-3cm smooth lesion that is under the tongue and is caused by the blockage
of a salivary stone.

647. c - Tooth #18 is charted as needing to have a porcelain crown placed on it. The other teeth
diagnosed with needing crowns were 3 and 14.

648. d - Oral piercings may cause irritation in the mouth at the area where they are located. The patient
should be advised that his piercing is contributing to gum recession in this area and should be
removed.

649. b - A PSR score of 1 would be a healthy depth of pocket with plaque but no calculus deposits that
bleeds upon probing. If calculus were present, then it would be scored higher.

650. b - A patient that grinds or experiences bruxism can place an additional strain on the TMJ area,
causing joint pain and discomfort.

651. a - A bite splint may help relieve TMJD symptoms by preventing the muscles of mastication from
remaining clenched and strained.

652. b - Xylitol is known to reduce the risk of dental caries, and is found in select chewing gums.
Chewing gum with xylitol can help patients reduce the chance of developing dental decay.
However, it may want to be pointed out to this patient that chewing gum can further irritate his
TMJD.

653. b - The patient’s blood pressure is slightly high, but she can be treated as normal as long as her
blood pressure is monitored at the next few future visits.

654. c - Stage I hypertension is for blood pressure levels at 130-139 over 80-89. Numbers higher than
that will fall under Stage II. This patients BP was 156/94.

655. c - Healthy blood glucose readings are in the 70-99 mg/dL range.

656. c - Orange juice, icing or some other source of sugar should be kept on hand in the dental office
should an emergency arise when a patient happens to have a hypoglycemic episode. This allows
them to quickly get sugar back into their system and stabilize their blood sugar level.

657. a - Type II diabetes is non-insulin dependent and may be controlled by dietary changes, but may
also include insulin injections.

658. d - In most instances it is a better choice to drink tap water over bottled water. This is because
many bottled waters do not contain fluoride or have an unknown amount of fluoride. By drinking
tap water in a municipal area she can know she is getting the proper amount of fluoride. Water is

TOP © 2015 - 2023 Tests.com


231
always a healthier choice over other drinks like soda, sports drinks or juices.

659. c - Both regular soda and diet soda are erosive to the teeth. Diet soda may not have the additional
sugar contributing to it, but the acidic levels of the solution can also contribute to tooth decay.

660. d - Antibiotic therapy may cause birth control medications to have a lapse in coverage. A
contingent method of birth control should be used if antibiotics need to be taken.

661. b - People with diabetes are more susceptible to developing gingivitis and periodontal disease
conditions.

662. c - Bulimia often is evident clinically by the loss of enamel (erosion) on the lingual surfaces up the
upper anterior teeth, due to the stomach acids coming into frequent contact with these teeth. The

663. c - Retake the patient’s blood pressure in 5 minutes to see if it has stabilized. Many people have
elevated blood pressure when visiting the dental office. If it does not go down, do not perform any
dental treatment and refer the patient for a medical consultation.

664. c - Medications such as Procardia can cause gingival overgrowth, which results in fibrous and
enlarged papillae, usually in the anterior regions of the mouth.

665. b - Tooth #16 has supererupted due to the lack of occlusal forces from the tooth in the opposite
arch.

666. a - Smoking is not a risk factor for obesity. It is a risk factor for lung disease, chronic obstructive
lung disease, heart disease, cancers of the head and neck and periodontal disease.

667. d - All of the above factors could place a patient at an increased risk to develop oral cancer.

668. a - The vermillion zone of the lip (the area between the skin of the face and the oral mucosa) is a
common site of oral cancer.

669. a - Nicotine stomatitis is often seen in the hard palate of patients with a smoking habit. It appears
as small red spots across the roof of the mouth.

670. b - Stage II hypertension indicates an individual has a blood pressure reading greater than 140 over
90.

671. d - Adult CPR requires compressions approximately 1.5-2 inches deep.

672. c - Oxygen cylinders are always green.

673. b - The herpes simplex virus (HSV) causes herpetic ulcerations such as “cold sores” and “canker
sores.”

674. d - None of the above are correct. Any patient with a virus or communicable disease must be
treated using universal precautions - precautions that protect the provider and the patient no
matter what the health condition.

TOP © 2015 - 2023 Tests.com


232
675. c - Herpetic whitlow can occur in the nail of a hygienist if universal precautions are not used
around patients with the herpes simplex virus.

676. b - Antibiotic premedication should be used on all patients that have had a joint replacement in
the last 6 months. Premedication is taken one hour prior to their appointment.

677. b - The patient’s implants are endosseous - they extend into the bone. Endosseous implants make
up more than 90% of the dental implants now performed.

678. a - Plastic scalers must be used on dental implants to prevent damage to the implant.

679. d - Crepitus or crepitation is the rough feeling felt by the practitioner as the TMJ is palpated during
opening and closing.

680. d - All of the above should be checked for when palpating the lymph nodes, as well as pain and
fixed position.

681. c - The date of origin is not something that is charted in regards to an abnormal skin lesion. Things
like size, shape, texture and color should all be noted.

682. b - If a patient has an active ulceration that has not dried and crusted over yet, then treatment
should be avoided until it does.

683. d - Abrasive brushing is a bad habit that can cause severe gum recession as well as enamel
abrasion.

684. b - Only piezoelectric scalers move in a linear direction. Sonic and magnetorestrictive move in an
elliptical direction.

685. b - Encouraging the patient to be seen for more frequent prophylactic appointments can prevent
tartar deposits from becoming so large on her teeth.

686. a - Tooth #4, the second bicuspid, is missing. The first molar (#3) has come forward some.

687. b - The temporomandibular joint may have some problematic symptoms due to the patient’s RA,
as RA greatly affects joints.

688. d - Frequent breaks during the appointment can help the patient rest her joint, and prevent
straining it by keeping it open too long.

689. b - Torus palatinus, or tori is a bony protuberance in the roof of the mouth.

690. c - A universal curette has two cutting edges on the working face of the blade. An area specific
curette has only one working edge.

691. a - The clinician’s forearms and hands should be parallel to the floor, level with the elbows in order
to prevent muscle strain.

TOP © 2015 - 2023 Tests.com


233
692. d - Advanced stabilized fulcrums use the non-dominant hand to provide lateral pressure in order to
control instrument strokes against the tooth, to prevent slips or tissue trauma.

693. a - The herpes zoster virus is a reactivation of the varicella zoster virus, which also causes
chickenpox and causes shingles typically in older adults.

694. d - Iron-deficiency anemia only requires iron supplementation for its treatment. Other anemia
types may include needs such as b12 supplementation, blood transfusions, antibiotic therapy, and
bone marrow transplants.

695. c - In hyperthyroidism it is typical to see accelerated tooth eruption rather than delayed eruption.

696. d - Patients with hyperthyroidism typically have increased sweating and moist skin.

697. c - Class IV furcations are the highest level of furcation involvement. They are visible clinically
without probing of the furcation area. Probes will extend through the entire furcation.

698. b - A periapical film would allow the clinician and dentist to have a high-quality view of the apex of
the tooth, and inspect the level of the surrounding bone.

699. d - All of the options listed above are useful to help prevent fogging of the dental mirror during an
appointment.

700. a - Only the lower 1/3 of the toe on a universal curette should be used for deposit removal. The
clinician should reassess their instrumentation to reflect this.

701. c - Approximately 35% nitrous oxide is optimum the desired effect on a dental patient by anxiety
as well as decreasing pain stimulation.

702. b - Hydrogen peroxide is NOT an active ingredient of sensitivity toothpastes. It is often found in
whitening products and may actually contribute to tooth sensitivity.

703. b - Rubber tip irrigators may be safely used by placing them up to 2mm below the margin of the
gums and irrigating periodontal pockets underneath.

704. a - Air polishers best function around 40-100psi, and use a combination of air and powder to
remove the stain from the teeth. A water-powered polisher would use 20-60psi.

705. a - Tooth #19 appears to have an incomplete root canal. There is evidence of a radiopaque
material in a portion of the canals but not the entire cavity.

706. b - Lupus clinically is seen as a butterfly configuration on the face and the nose. It is a chronic
inflammatory autoimmune disease.

707. c - Marginal inflammation due to inadequate plaque removal is a common cause of bleeding,
uncomfortable gums during brushing. The patient should be educated on proper brushing
methods to reduce bleeding and gingival inflammation.

TOP © 2015 - 2023 Tests.com


234
708. c - Class III caries includes proximal areas of the incisors and canines, but does not include the
incisal edge.

709. b - Healthy gums will appear coral pink in color. They are also firm, pointed in contour and have
smooth margins.

710. c - Maxillary lateral incisors may have a natural lingual groove or lingual pit because they typically
have a more prominent cingulum.

711. b - The cusp of Carabelli only develops on the maxillary first molar. It is the 5th lobe and found on
the mesiolingual cusp.

712. c - Brushing along the gum margin is the only way to improve oral hygiene and reduce bleeding
caused by excess plaque. The patient should be informed that it may take several days prior to
seeing an improvement in bleeding.

713. a - Fluoride, when used in conjunction with efficient oral hygiene, can prove to be a valuable
supplement that increases the health of enamel, especially in patients with braces or poor oral
hygiene.

714. d - Selective polishing should be the preferred method of stain removal in all patients. Prolonged
polishing can cause the removal of enamel and tooth structure, in addition to scratching the
surface of the tooth.

715. a - Rinsing with water after the use of albuterol can reduce the risk of tooth staining, taste
alteration, and dental caries.

716. c - A pregnancy granuloma, also called pregnancy tumor or pyogenic granuloma, is commonly seen
in patients that have changes in hormonal levels such as related to pregnancy.

717. a - Geographic tongue, also called benign migratory glossitis, is a map-shaped lesion that changes
shapes and sizes on a person’s tongue. It has a defined border and usually involves the
desquamation of filiform papilla.

718. c - Acid erosion due to excess vomiting or heartburn associated with her pregnancy may have
occurred, which would contribute to tooth sensitivity.

719. b - Sodium laurel sulphate (SLS) commonly irritates oral mucosa and is found in some toothpastes.
It is now usually replaced with lauryl sarcosinate.

720. c - Cleansing the baby’s mouth after breastfeeding or bottle feeding can reduce the prolonged
exposure of milk on the surface of the teeth, thereby decreasing the rate of decay.

721. b - A soft bristled toothbrush and tap water are adequate to clean the baby’s teeth. A child should
not use fluoridated toothpaste until they are old enough to expectorate and rinse.

722. a - Most children typically have their first tooth erupt around 6-10 months of age. The lower

TOP © 2015 - 2023 Tests.com


235
central incisors are typically the first teeth to erupt.

723. b - The first permanent teeth to erupt are usually the lower central incisors or the first molars.
These teeth typically erupt around age 6.

724. b - A PSR of 1 will be assigned to this quadrant. If there was no bleeding or plaque it would score a
0, and if there were tartar present it would be a 2.

725. c - Intraoral assessments around the tongue need to be conducted by gently holding the tongue
with a piece of gauze.

726. b - A plaque score of 1 indicates that there is only plaque film along the gingival margin, but not
within it.

727. a - 6 months to 14 years old is the most effective age range for fluoride, since these are the ages of
tooth development and eruption. By the age of 14 years old all permanent teeth are erupted.

728. d - He is not missing any permanent teeth. All necessary teeth are developing. Sometimes, when a
primary tooth is lost, a permanent tooth does not erupt right away. It is important to check on
eruption patterns periodically through x-rays and oral exams, as some children may be a case for
early orthodontics.

729. c - The dark, circular area that is distal to #18 is a tooth bud. It is specifically tooth #17 developing
into a wisdom tooth. You can see there is a dark, circular area distal to tooth #31 as well.

730. c - Though you could take a periapical, it would be best to take an occlusal image. When teeth are
unerupted after losing a primary tooth, impaction may be a concern. An occlusal image would give
you the best image to see where the tooth is and if it is impacted.

731. a - Orthodontists specialize in the movement of teeth and aligning the jaw. Though all general
dentists are trained to be able to provide orthodontics, most dentists prefer to refer their patients
to the specialists.

732. a - The patient received endosteal implants. They are the most common type of implant placed.
The endosteal implant is surgically placed in the jawbone, whereas a subperiosteal implant is
placed on top of the bone. Transosteal implants are primary used in edentulous patients.
Osseointergration is the process on the bone growing around the implant.

733. b - The patient is receiving a partial denture. The implants will allow for a better, tighter fit for the
patient. Most mandibular partials can be hard to fabricate to where they are comfortable to the
patient. That is why implants are placed, to help with retention.

734. c - The partial image is that of the hyoid bone. The hyoid bone is u-shaped and located in the neck.
This particular bone is there to help support the tongue.

735. c - Stainless steel cannot be used in the debridement around dental implants. The material can
create micro abrasions that can collect bacteria and cause the implant to fail.

TOP © 2015 - 2023 Tests.com


236
736. c - A cantilever bridge is a fixed bridge that is only supported by one adjacent tooth. These types of
bridges are not common, since have been proven to cause damage to the supporting tooth.

737. b - Scleroderma is the thickening and tightening of skin. When this is localized, it causes changes
mostly on the hands and face. Raynaud’s disease, which affects the small blood vessel of the
fingers, toes, ears and nose, is often the first sign of scleroderma.

738. a - A partial denture would most likely be the prosthesis the dentist would recommend. It would
be the easiest way to restore chewing function to the patient considering there are many viable
teeth in the mandibular region to support the prosthesis.

739. d - Glass ionomers are used in prosthesis. Most partials and dentures are made of a resin material.
This makes them easy to clean and longer lasting.

740. b - The proper way to clean a removable appliance is by placing the prosthesis in a disinfectant
solution and then agitating it in the ultrasonic. This is affective in loosening the debris. The use of a
denture brush is recommended, since it is designed not so scratch or dent the resin material of the
prosthesis.

741. d - Geographic tongue is irregular, smooth red patches on the dorsal surface of tongue. The exact
cause of geographic tongue is unknown, but has been linked to a vitamin B deficiency. It is not
relevant to or considered a factor in the use of a prosthesis.

742. a - The patient is congenitally missing #32, which means he has only 3 wisdom teeth. It is not
uncommon for wisdom teeth to be congenitally missing.

743. b - Tonsil stones are accumulations of bacteria that harden in the folds and crypts of the tonsils.
They can be covered by bacteria, which can lead to bad breath. If for some reason they cannot be
dislodged, they may need to dislodge by an ENT specialist.

744. a - Decalcification is high among teenagers, but significantly increases with those having braces.
Decalcification looks like a chalky white spot on the teeth. It is formed due to the accumulation of
plaque and poor oral hygiene.

745. c - The best brushing technique for patients with braces is the Charters technique. This is done by
placing the brush at 45-degree angle where the tooth and the tissue meet. The brush is swept
toward the apex of the tooth in a circular vibrating motion followed by a rolling stroke toward the
occlusal surface

746. c - It is important to protect the teeth, especially during any type of contact sport. Braces or not, a
mouth guard should always be worn. An over-the-counter mouth guard can be purchased or it can
be specifically made for the mouth.

747. a - Hormones during pregnancy are constantly fluctuating and leave your mouth more vulnerable
to plaque and bacteria. The best way to combat the generalized bleeding is exceptional homecare.

748. d - Vomit is highly acidic and needs time to neutralize in your mouth before you brush and floss
your teeth. Otherwise, brushing your teeth that are coated in acid could cause erosion.

TOP © 2015 - 2023 Tests.com


237
749. c - The second trimester is best, because all of the fetus’s vital organs will have been developed
and there is a lower risk of cutting off blood supply. A patient that is in the supine position for long
periods of time can occlude the blood supply to the heart, leading to a loss of consciousness.

750. b - Aphthous ulcers are usually recurrent, small round ulcers where the skin is not tightly bound to
the bone. It is not known what causes these particular ulcerations, just what may trigger them.

751. c - Vitamin C could actually help fight against aphthous ulcers, since vitamin C has been known to
boost the immune system and help aid in the fight against infection.

TOP © 2015 - 2023 Tests.com


238
Questions with Answers
An additional practice exam format to make it easier for you to
reference answers.

Practice Test for the NBDHE Dental Hygienist Exam – Section 1


Scientific Basis for Dental Hygiene Practice

1. During swallowing, the hyoid bone moves downward. During swallowing, the larynx elevates.

a. Both statements are true


b. Both statements are false
c. The first statement is true, the second statement is false
d. The first statement is false, the second statement is true

d - During swallowing, the hyoid bone moves upward as the larynx elevates. After moving upward,
the hyoid bone moves forward then returns to its starting position.

2. Which nervous system produces the fight-or-flight reaction?

a. Parasympathetic
b. Sympathetic
c. Peripheral
d. Central

b - The sympathetic nervous system releases norepinephrine to raise the heart rate and elevate
blood pressure, triggering the fight or flight reaction in a stressful situation.

3. A patient is experiencing symptoms related to Bell’s palsy. Which cranial nerve is suffering from
a motor deficit?

a. Facial
b. Trigeminal
c. Trochlear
d. Hypoglossal

a - Bell’s palsy causes loss of movement to muscles due to lesions in the facial nerve. This condition
is bilateral and usually is only temporary, though it may take several weeks prior to returning to
normal use.

TOP © 2015 - 2023 Tests.com


239
4. What is mobilized in and out of the blood during bone remodeling?

a. Canaliculi
b. Osteoclasts
c. Calcium
d. Osteoblasts

c - Calcium is mobilized in and out of the bloodstream during bone remodeling. This allows new
bone formation to occur in areas of the body where it is needed.

5. Which of the following cranial nerves pass through the foramen ovale of the sphenoid bone?

a. Trigeminal mandibular
b. Nasopalatine
c. Trigeminal maxillary
d. Abducent

a - The trigeminal mandibular nerve passes through the foramen ovale of the sphenoid bone.

6. The maxillary branch of the trigeminal nerve passes through the ______.

a. foramen spinosum
b. superior orbital fissure
c. foramen ovale
d. foramen rotundum

d - The foramen rotundum is the opening that conveys the maxillary branch of the trigeminal
nerve.

7. Which cranial nerve is the longest and extends to the trunk of the body?

a. Hypoglossal
b. Vagus
c. Facial
d. Trigeminal

b - The vagus nerve, or cranial nerve X is the longest of all cranial nerves. It reaches areas of the
head and neck as well as the heart and stomach.

TOP © 2015 - 2023 Tests.com


240
8. List the following digestive system structures in order of their function during digestion of a
meal.

_____ a. Stomach
_____ b. Small intestine
_____ c. Pharynx
_____ d. Large intestine
_____ e. Esophagus

c. Pharynx
e. Esophagus
a. Stomach
b. Small intestine
d. Large intestine

9. List the following components of a muscle fiber in order of the LARGEST component to the
SMALLEST component.

_____ a. Myofilaments
_____ b. Muscle fascicle
_____ c. Muscle
_____ d. Myofibril
_____ e. Myofiber

c. Muscle
b. Muscle fascicle
e. Myofiber
d. Myofibril
a. Myofilaments

10. The trigeminal ganglion of the fifth (V) trigeminal nerve has three divisions. Which of the
following is NOT a division of the trigeminal ganglion?

a. Opthalmic
b. Mandibular
c. Optic
d. Maxillary

c - The trigeminal ganglion does not have an optic division. Its divisions include the opthalmic,
mandibular and maxillary, which enter in by the superior orbital fissure, foramen ovale and
foramen rotundum.

TOP © 2015 - 2023 Tests.com


241
11. Which three of the following are lymphatic organs?

a. Tonsils
b. Pituitary gland
c. Spleen
d. Thymus
e. Kidney
f. Brain

a, c & d - Tonsils, Spleen, Thymus are all lymphatic organs. The 4th and final lymphatic organ not
listed is the lymph nodes.

12. Order the following anatomical structures of the respiratory tract as they appear and function in
the processing of inhalation.

_____ a. Trachea
_____ b. Alveoli
_____ c. Alveolar duct
_____ d. Primary bronchi
_____ e. Bronchiole

a. Trachea
d. Primary bronchi
e. Bronchiole
c. Alveolar duct
b. Alveoli

13. What parallels the venous blood vessels and returns filtered fluids to the bloodstream?

a. The endocrine system


b. The lymphatic system
c. The arterial blood vessel system
d. The circulatory system

b - The lymphatic system filters fluids and returns them to the bloodstream from various body
tissues. It parallels the venous blood vessels.

TOP © 2015 - 2023 Tests.com


242
14. What type of cell-to-cell attachments are found in enamel-forming ameloblasts?

a. Hemidesmosome
b. Tight junctions
c. Desmosomes
d. Gap junctions

c - Desmosomes are the cell-to-cell attachments in ameloblasts, which are responsible for forming
tooth enamel.

15. What is the homogenous, translucent gel enclosed in a cell by the cell membrane?

a. Lipid droplets
b. Cytoplasm
c. Lysosome
d. Nucleus

b - Cytoplasm is the aqueous, translucent gel that is enclosed in a cell by a cell membrane. It is the
liquid housing that contains other cellular particles and allows them to move about within the cell.

16. Which type of epithelia lines the oral cavity?

a. Stratisfied squamous
b. Simple squamous
c. Stratified columnar
d. Simple columnar

a - The oral cavity tissues such as the buccal mucosa and gingival mucosa are lined with stratisfied
squamous epithelia, the shorter, flatter squamous cells found in multiple layers at the surface of
the oral cavity tissues.

17. MATCH the following dental anatomical structures with the permanent tooth to which they
belong.

____Cusp of Carbelli a. Maxillary second premolar


____Two cusps that are approximately the same length b. Mandibular second premolar
____Long central developmental groove c. Maxillary first molar
____“H” or “y” groove patterns int he occlusa surface d. Mandibular first molar
____Greater mesiodistal than faciolingual width e. Maxillary first premolar

c. Cusp of Carbelli
a. Two cusps that are approximately the same length
e. Long central developmental groove
b. “H” or “y” groove patterns int he occlusa surface
d. Greater mesiodistal than faciolingual width

TOP © 2015 - 2023 Tests.com


243
18. MATCH the parts of the tongue with their description.

___Lingual papillae a. The side of the tongue


___Sulcus terminalis b. irregular mass of tonsillar tissue on the posterior dorsal surface
___Foramen cecum c. elevated structures of specialized mucosa
___Lateral surface d. pitlike depression in the center of sulcus terminalis
___Lingual tonsil e. v-shaped groove on the posterior dorsal surface of the tongue

c. Lingual papillae
e. Sulcus terminalis
d. Foramen cecum
a. Lateral surface
b. Lingual tonsil

19. Which TWO are folds of tissue in the inferior to ventral surfaces of the tongue?

a. Labial frenum
b. Sublingual caruncle
c. Sublingual fold
d. Lingual frenum

c & d - The sublingual fold and lingual frenum are both folds of tissue in the floor of the mouth
below the ventral surface of the tongue.

20. Which cranial bone is the single bone located in the most posterior portion of the skull?

a. Mandible
b. Lambdiodal suture
c. Parietal
d. Occipital

d - The occipoital bone is the bone on the posterior lower portion of the skull.

21. Which muscle of mastication is superficially located and one of the strongest muscles?

a. Medial pterygoid
b. Masseter
c. Temporalis
d. Lateral pterygoid

b - The masseter is the most superficially located muscle used in mastication and is the strongest
muscle used.

TOP © 2015 - 2023 Tests.com


244
22. Which tooth has a mesial root concavity that is not as prominent?

a. Mandibular second molar


b. Mandibular first molar
c. Maxillary first molar
d. Maxillary first premolar

a - Mandibular second molars have shallower mesial root concavities that are not as prominent as
the other teeth listed.

23. Which tooth has a furcation that is in the apical third to 1/2 of the tooth?

a. Mandibular first premolar


b. Mandibular second premolar
c. Maxillary first premolar
d. Maxillary second premolar

c - The maxillary first premolar is the only premolar to have two roots, and it’s furcation is toward
the apical 1/3 to 1/2 of the tooth.

24. Which two of the following are functions of the stomach?

a. Storage of ingested food until it is emptied into the small intestine


b. Stimulated by PANS
c. Secretion of HC1 and enzymes that initiate digestion
d. Inhibited by SANS

a & c - The stomach stores ingested food until it is emptied into the small intestine, and also
secretes HC1 and enzymes that initiate digestion.

25. Which two of the following are branches that diverge from the maxillary artery?

a. Incisive artery
b. Posterior auricular
c. Ascending palatine artery
d. Inferior alveolar

a & d - The incisive artery and inferior alveolar artery both diverge from the maxillary artery.

TOP © 2015 - 2023 Tests.com


245
26. A patient’s mandibular first permanent molar’s mesiobuccal groove is in a mesial relationship to
the mesiobuccal cusp of the maxillary first permanent molar. What classification of occlusion is
this?

a. Class I
b. Class II
c. Class III
d. Class IV

c- Class III occlusion is when the mesiobuccal groove of the mandibular first molar is mesial to the
mesiobuccal cusp of the maxillary first permanent molar.

27. Which of the following teeth has a long lingual root that usually extends beyond the lingual
surface of the crown?

a. Mandibular first molar


b. Maxillary first molar
c. Mandibular second molar
d. Maxillary second molar

b - The maxillary first molar has three roots, with the lingual root being longer than the others,
extending past the lingual surface of the tooth’s crown.

28. Which tooth may have a lingual groove extending from the crown to the root?

a. Maxillary central incisors


b. Mandibular central incisors
c. Mandibular lateral incisors
d. Maxillary lateral incisors

d - Maxillary lateral incisors are the only incisor teeth to sometimes have a palatoradicular groove
that extends from the crown to the root on the lingual surface.

29. Which two of the following food sources is an example of a disaccharide sugar structure?

a. Apple
b. Honey
c. Banana
d. Carrots

a & d - Apple and Carrots are examples of containing disaccharide sugar structures. Select fruits,
vegetables and table sugars are a food source of disaccharide sugars.

TOP © 2015 - 2023 Tests.com


246
30. What is the recommended fiber intake each day for an adult?

a. 0-10 grams
b. 10-20 grams
c. 20-30 grams
d. 20-50 grams

c - 20-30 grams of fiber each day is the recommended daily adult intake for healthy individuals.

31. A healthy blood glucose level range is __________ mg/100mL blood.

a. 50-100
b. 70-120
c. 90-150
d. 100-175

b - 70-120 mg/100mL blood is a healthy blood glucose level and is the level that a controlled
diabetic or other healthy individual should measure within.

32. Which complex carbohydrate is made up of the plant form of glucose?

a. Cellulose
b. Glycogen
c. Starch
d. Pectin

c - Starch is the complex carbohydrate that comes from plant storage in the form of glucose.

33. Decalcification and demineralization of tooth enamel begins at what pH?

a. 3.3
b. 5.5
c. 6.6
d. 7.7

b - 5.5 pH is the acidic level at which decalcification and demineralization of tooth enamel begins
to occur.

34. What sweetener is found in plants and can interfere with the metabolism of s. Mutans?

a. Glucose
b. Sorbitol
c. Xylitol
d. Mannitol

TOP © 2015 - 2023 Tests.com


247
c - Xylitol is the sweetener found naturally in plants and can interfere with s.mutans metabolism,
decreasing the demineralization of enamel.

35. What is the recommended daily allowance for digestible carbohydrates?

a. 50-100 g/day
b. 130 g/day
c. 100-150 g/day
d. 175 g/day

b - 130 g/day is the recommended daily allowance for digestible carbohydrates in both healthy
adults and children.

36. Carbohydrates are an energy source for the growth and repair of periodontal tissues. A firm
texture of complex carbohydrates may increase circulation in gingival tissue.

a. Both statements are true


b. The first statement is true, the second statement is false
c. The first statement is false, the second statement is true
d. Both statements are false

a - Both statements are true. Carbohydrates are an energy source for the growth and repair of
periodontal tissues. A firm texture of complex carbohydrates may increase circulation in gingival
tissue. Firm carbohydrates stimulate the gum tissue during mastication.

37. Yogurt is not recommended for lactose intolerant patients. Lactose intolerance may be due to a
congenital absence of lactase.

a. Both statements are true


b. The first statement is true, the second statement is false
c. The first statement is false, the second statement is true
d. Both statements are false

c - Yogurt is recommended for lactose intolerant patients, because the lactose is digested by the
yogurt. It therefore makes a healthy snack and alternative to other dairy products such as ice
cream.

38. Which leukocyte is associated with acute inflammation?

a. Neutrophils
b. Basophils
c. Eosinophils
d. Mast cells

TOP © 2015 - 2023 Tests.com


248
a - Neutrophils are associated with acute inflammation by acting against invading pathogens in the
body.

39. If an antibody is produced by an individual and passed on to another, such as a mother to her
child, what type of immunity is this?

a. Naturally active
b. Naturally passive
c. Artificially active
d. Artificially passive

b - Naturally passive acquired immunity is when antibodies are passed from one individual to
another. An example of this is when the mother passes her immunity on to her child during
gestation.

40. MATCH the following symptoms to the appropriate disease condition.

___ Fifth’s Disease a. Fever, swollen parotid gland


___ Shingles b. Red, lacy rash
___ Herpes c. clear, fluid-like blisters on the lips
___ Syphilis d. Fever, itching, and red rash on one side of the body
___ Mumps e. Small, round nodule on the lip or tongue

b. Fifth’s Disease
d. Shingles
c. Herpes
e. Syphillis
a. Mumps

41. Which Hepatitis virus is transmitted through a fecal-oral route?

a. A
b. B
c. C
d. D

a - Hepatitis A is transmitted through fecal-oral contamination, such as in contaminated food or


water due to poor living conditions or lack of hygiene by food preparers.

TOP © 2015 - 2023 Tests.com


249
42. Which three of the following symptoms are signs found in a child that may have
hypothyroidism?

a. Enlarged tongue
b. Delayed tooth eruption
c. Enlarged lips
d. Periodontal disease

a, b, & c - Enlarged tongue, delayed tooth eruption and enlarged lips are symptoms found in
children suffering from hypothyroidism, as well as possible enamel hypoplasia.

43. Which disease of the bone marrow exhibits abnormally low levels of white blood cells (WBCs)?

a. Neutropenia
b. Leukopenia
c. Leukemia
d. Multiple myeloma

b - Leukopenia is abnormally low levels of white blood cells and is a disease of the WBCs and bone
marrow; It should not be confused with leukemia, which is cancer of the bone marrow or WBCs.

44. What is a common side effect of radiation and or chemotherapy that causes painful
inflammation and ulceration of mucous membranes?

a. Candida infections
b. Osteoradionecrosis
c. Sore throat
d. Mucositis

d - Mucositis is the inflammation and ulceration of mucous membranes that line the digestive
tract, including the mucosa of the oral cavity. It is a common side effect in patients undergoing
radiation therapy or chemotherapy.

45. What microorganism is resistant to high temperatures and disinfectants?

a. Glycocalyx
b. Gram-positive bacteria
c. Endospore
d. Capsules

c - Endospores are resistant to high temperatures and disinfectants, and may remain viable for an
extended amount of time. Endospores are the primary reason for the need of higher levels of
temperature during sterilization processes.

TOP © 2015 - 2023 Tests.com


250
46. Which of the following microbes cannot perform or may be destroyed in the presence of
oxygen?

a. Microaerophiles
b. Aerobes
c. Facultative anaerobes
d. Anaerobes

d - Anaerobes require an absence of oxygen in order to survive. Other types of microbes may be
able to function without oxygen or with limited oxygen, but anaerobes require ultimate absence of
oxygen in order to survive.

47. Infection transmission that involves touching a contaminated object is considered:

a. Direct contact
b. Nondirect contact
c. Vehicle
d. Passive

b - Nondirect contact is the infection transmission that involves touching a contaminated object,
such as a touching a used instrument without wearing gloves while transporting it to the
sterilization area.

48. An infection that affects an immunocompromised person is considered:

a. Chronic
b. Acute
c. Passive
d. Opportunistic

d - Opportunistic infections are infections that affect immunocompromised persons such as an


infant or older adult. This is due to the pathogen taking an “opportunity” when the person’s
immune system is compromised. Anyone may suffer from passive, acute or chronic infections.

49. What are the smallest and simplest infectious agents?

a. Endospores
b. Viruses
c. Gram-negative bacteria
d. Fungi

b - Viruses are the smallest and simplest infectious agent. They contain only RNA or DNA strands.

TOP © 2015 - 2023 Tests.com


251
50. Which two conditions are examples of vector-borne infections?

a. Rabies
b. Lyme disease
c. West Nile virus
d. Giardia

b & c - Lyme disease and West Nile virus are both examples of vector-borne infections that are
spread through an infected carrier such as a mosquito or tick and transferred to the host.

51. What is a natural chemotherapeutic compound that is produced by a bacterium and will inhibit
or kill bacteria?

a. Antibiotic
b. Allergy
c. Immunity
d. Vaccination

a - Antibiotics such as penicillin or amoxicillin are chemotherapeutic compounds that will inhibit or
kill bacteria and is produced by a bacterium.

52. A highly infectious condition that leaves pustules and itchy yellow scabs around the mouth and
face is called _______.

a. Abscess
b. Impetigo
c. Human Papillomavirus
d. Acne

b - Impetigo is a highly infectious condition that can be due to staph or strep infections, leaving
pustules and itchy yellow scabs around the mouth and face. It is highly contagious.

53. What two symptoms are associated with liver disorders?

a. Increased saliva production


b. Xerostomia
c. Jaundice
d. Sunken eyes

b & c - Xerostomia (dry mouth) and jaundice (yellowing of the skin) are both symptoms associated
with liver disorders such as Hepatitis.

TOP © 2015 - 2023 Tests.com


252
54. Which two Hepatitis viral strains are spread through sexual contact?

a. Hep A
b. Hep B
c. Hep C
d. Hep E

b & c - Both Hepatitis B and C are transmitted through sexual contact, as is Hep D (not listed).

55. Which of the following is NOT a sign of leukemia?

a. Abnormal bruising
b. Spontaneous sulcular bleeding
c. Pallor of the gingiva
d. Red gingival tissues

c - Pallor of the gingiva is not a symptom of leukemia. Instead, gingiva will appear red and tender
to the touch, as well as exhibit bruising and bleeding.

56. What bone disease is most common in children and involves abnormal proliferation of fibrous
connective tissues?

a. Fibrous dysplasia
b. Osteoporosis
c. Paget’s disease
d. Neoplasia

a - Fibrous dysplasia is a bone disease which affects children and involves two types: Monostotic
and Polyostotic. Both cause abnormal proliferation of fibrous connective tissues.

57. The most common form of oral cancer is:

a. Basal cell carcinoma


b. Squamous cell carcinoma
c. Malignant melanoma
d. Pleomorphic adenoma

b - Squamous cell carcinoma is the most common oral cancer, attributing for approximately 90% of
all oral cancer cases.

TOP © 2015 - 2023 Tests.com


253
58. Which cardiovascular disease is associated with periodontal disease, placing an emphasis on
improving oral health in an attempt to reduce the risk of disease?

a. Cerebrovascular disease
b. Hypertension
c. Congestive heart failure
d. Coronary artery atherosclerosis

d - Coronary artery atherosclerosis is associated with periodontal disease. Decreasing periodontal


disease symptoms through preventative and maintenance care will reduce the patient’s risk
factors.

59. Local anesthetics containing vasoconstrictors should not be administered to asthma patients. It
also should not be used for patients that suffer from emphysema.

a. Both statements are true


b. Both statements are false
c. The first statement is true, the second statement is false
d. The first statement is false, the second statement is true

c - The first statement is true, the second statement is false. Local anesthetics containing
vasoconstrictors should not be given to asthma patients because it may cause reduced oxygen
flow in the body, but they may be administered to patients with emphysema.

60. What type of seizure exhibits symptoms of jerking motions and no loss of consciousness?

a. Status epilepticus
b. Petit mal
c. Grand mal
d. None of the above

b - Petit mal seizures are simple, with jerking motions and no loss of consciousness. The patient is
aware of their surroundings and is able to recount the episode without loss of memory.

61. What disease exhibits itself in a classic “butterfly” pattern rash over the patient’s face?

a. Phemphigus vulgaris
b. Hodgkin’s lymphoma
c. Scleroderma
d. Lupus

d - Lupus is a chronic disease that displays a “butterfly” pattern over the patient’s face.

TOP © 2015 - 2023 Tests.com


254
62. Wedge-shaped lesions along the cervical margin of the teeth are referred to as _________.

a. Erosion
b. Abrasion
c. Abfraction
d. Attrition

c - Abfractions are the wedge-shaped lesion along the cervical margin of the teeth, usually due to
fatigue or flexing of the tooth. The excess flexing causes enamel to chip off of the tooth in this
area, exposing the dentin.

63. A condition of the oral tissues due to hormonal changes during pregnancy is called ___________.

a. Pyogenic granuloma
b. Sialolith
c. Gingival hyperplasia
d. Inflammatory papillary hyperplasia

a - Pyogenic granulomas is a reactive tissue hyperplasia related to hormonal changes, usually


associated with pregnancy. This condition may also be called “pregnancy tumors.”

64. A discoloration of the oral mucosa that may have a radiopaque appearance.

a. Traumatic neuroma
b. Amalgam tattoo
c. Necrotizing sialometaplasia
d. Melanoma

b - Amalgam tattoos are bluish discolorations of the oral mucosa due to leaking of particles from
an amalgam filling directly into the mucosa. Therefore, they may appear radiopaque on a
radiograph.

65. Which cyst exhibits bluish swelling around a newly erupting crown, most common with the
eruption of primary teeth?

a. Odontogenic keratocyst
b. Dentigerous cyst
c. Nonodontogenic cysts
d. Eruption cyst

d - Eruption cysts usually involve primary tooth eruption and exhibit bluish tissue with swelling
around the erupting tooth, hence the term “eruption cyst.”

TOP © 2015 - 2023 Tests.com


255
66. Supernumerary teeth usually exhibit microdontia. They are most commonly found in the
maxilla.

a. Both statements are true


b. Both statements are false
c. The first statement is true, the second statement is false
d. The first statement is false, the second statement is true

a - Both statements are true. Supernumerary teeth typically exhibit microdontia (small in size) and
are in the maxilla, typically distal to the 3rd molars or between the upper central incisors.

67. Which of the following is an exophytic area of enamel on the root surface?

a. Dens in dente
b. Enamel pearl
c. Enamel hypoplasia
d. Gemination

b - Enamel pearls are projections of an exophytic area of enamel on the surface of a root. The area
appears as a small round “pearl” of enamel on the surface where enamel isn’t normally found.

68. Dental fluorosis causing damage to tooth development occurs approximately at what ages?

a. 0-12 months
b. 12 months - 12 years
c. 6 months - 5 years
d. 5 years - 12 years

c - 6 months to 5 years is the age range where dental fluorosis causes damage to the developing
teeth. At this point in age all permanent teeth are developing and at increased susceptibility to
excess fluoride exposure.

69. Which two anomalies may involve the buccal mucosa?

a. Linea alba
b. Leukoedema
c. Lingual variscosities
d. Ranula

a & b - Linea alba and leukoedema may both involve the buccal mucosa. A ranula is in the floor of
the mouth while lingual variscosities are on the tongue.

TOP © 2015 - 2023 Tests.com


256
70. An asymptomatic cluster of yellow lobules of sebaceous glands is known as what?

a. Median rhomboid glossitis


b. Ranula
c. Fordyce granules
d. Lingual thyroid nodule

c - Fordyce granules are asymptomatic clusters of yellow lobules involving sebaceous glands within
the oral cavity.

71. Which two anomalies involve the desquamation of filiform lingual papillae?

a. Geographic tongue
b. Fissured tongue
c. Black hairy tongue
d. Benign migratory glossitis

a & d - Geographic tongue and benign migratory glossitis are the same condition, which involves
the desquamation filiform lingual papillae. They appear as red, raw areas on the tongue with a
well-defined border which frequently changes in shape.

72. What is the term used to describe a sharp bend or curve in the root of a formed tooth?

a. Concrescence
b. Dilaceration
c. Fusion
d. Gemination

b - Dilaceration is the sharp bend or curve in the root of a tooth, it can make root canal therapy or
extractions difficult to complete.

73. What two medications are contraindicated for children due to the risk of developing Reye’s
syndrome?

a. Acetaminophen
b. Aspirin
c. Ipubrophen
d. Bayer

b & d - Aspirin and Bayer (brand-name) are contraindicated for pediatric use due to the risk of
developing Reye’s syndrome in children.

TOP © 2015 - 2023 Tests.com


257
74. Which pharmacological abbreviation is for medicine to be taken “twice daily”?

a. ud
b. tid
c. bid
d. prn

c - The pharmacological abbreviation “bid” refers to medicine that should be taken twice per day.
(“Bi” twice “d” day)

75. The SANS enables the body to maintain bodily function during normal activities. The PANS
enables the body to function during emergencies.

a. Both statements are true


b. Both statements are false
c. The first statement is true, the second statement is false
d. The first statement is false, the second statement is true

b - Both statements are false. The PANS enables normal activity function while the SANS enables
emergency functions.

76. Which two of the following are typical effects or reactions while using Adrenergic medications?

a. Vasoconstriction
b. Bronchodilation
c. CNS depression
d. Lowered blood pressure

a & b - Vasoconstriction and bronchodilation are typical effects due to the use of Adrenergic
medications. While the blood vessels constrict, the bronchial system can dilate.

77. What is the pharmaceutical dose that produces 50% of the maximum response in 50% of
subjects?

a. Lethal dose
b. Log dose
c. Effective dose
d. Duration

c - Effective dosages are the amount of a prescription drug dose that produce 50% of the
maximum response of the expected outcome in 50% of the subjects taking the medication.

TOP © 2015 - 2023 Tests.com


258
78. Which of the following is NOT an anti-inflammatory drug (NSAIDs)?

a. Diazepam
b. Ibuprophen
c. Naproxen
d. Diflunisal

a - Diazepam is not an anti-inflammatory drug (NSAID). Valium is the brand name for diazepam and
is used as an antianxiety medication.

79. Which of the following is TRUE about local anesthetic agents?

a. They should be non-potent.


b. They should be irreversible.
c. They should have a rapid onset.
d. They have a short shelf life.
e. Both c and d only.

c - Local anesthetics should have a rapid onset, to allow the dentist to perform treatment within a
timely manner for the patient. They should also have a long shelf life, be reversible and potent.

80. Which of the following are advantages of nitrous-oxide use in the dental office?

a. Nonflammable
b. May be used on children
c. Rapid onset and recovery
d. All of the above

d - All of the above are correct in regards to the advantages of nitrous-oxide use. It is non-
flammable, may be used on patients of all ages and has a rapid onset and recovery time.

81. What sedative medications provide a wide therapeutic index and are safer than barbiturates?

a. Analgesics
b. NSAIDs
c. Salicylates
d. Benzodiazepines

d - Benzodiazepines provide a wide therapeutic index and are much safer than barbiturates for
routine prescription use within the dental office.

TOP © 2015 - 2023 Tests.com


259
82. Which drug used in the dental office should be avoided during treatment of a pregnant patient?

a. NSAIDs
b. Epinephrine
c. Amoxicillin
d. All of the above

a - NSAIDs should be avoided when treating a pregnant patient, but the other medications are
typically safe on the use of pregnant patients. Always check with the patient’s primary care
practitioner if a drug substitution is needed to include an NSAID.

83. Which medication is known to cause gingival hyperplasia?

a. Dilantin
b. Diflucan
c. Dyazide
d. Neurontin

a - Dilantin (phenytoin) is known to contribute to gingival overgrowth (hyperplasia). These areas


are typically fibrous along the papillary and marginal gingiva.

84. What is the primary site for drug metabolism?

a. Pancreas
b. Liver
c. Small intestine
d. Cardiovascular system

b - The liver is the primary site for drug metabolism. It is broken down in this area and then
distributed throughout the rest of the body for its purposed treatment.

85. What cardiovascular agents cause vasodilation of the blood vessels?

a. Antiarrhythmic drugs
b. Antihypertensive drugs
c. Calcium channel blockers
d. Anticoagulants

c - Calcium channel blockers vasodilate the blood vessels to relax resistance. This decreases blood
pressure and allows blood to flow easier throughout the cardiovascular system.

TOP © 2015 - 2023 Tests.com


260
86. Nystatin is used to treat:

a. Oral candidiasis
b. Periodontal infections
c. Herpes simplex
d. Dental abscesses

a - Nystatin is a fungicidal drug commonly used to treat fungal infections such as oral candidiasis
(yeast).

87. The small elevated tissue opposite of the maxillary second molar:

a. Parotid duct
b. Parotid gland
c. Parotid papilla
d. Mucobuccal fold

c - The parotid papilla is a tissue mass that protects the opening of the parotid duct of the parotid
salivary gland. This duct is also called the Stenson’s duct.

88. The facial portion of the alveolus of the canine:

a. Periodontal ligaments
b. Alveolus
c. Alveolar process
d. Canine eminence

d - The canine eminence is part of the alveolus and maxillary alveolar process. It is especially
prominent on the maxilla.

89. Abnormal variations on the lingual aspect of the mandibular arch:

a. Tori
b. Exostoses
c. Tuberosity
d. Retromolar pad

a - Tori are found bi-lateral on the lingual area of the mandibular arch. It is a developmental
growth of normal bone that often appears lobulated or nodular.

TOP © 2015 - 2023 Tests.com


261
90. Which branch of the trigeminal nerve provides nerve supply to the maxillary posterior lingual
periodontium?

a. Middle superior alveolar nerve


b. Greater palatine nerve
c. Nasopalatine nerve
d. Inferior alveolar nerve

b - The greater palatine nerve is a branch off of the maxillary nerve, which is a branch from the
trigeminal nerve.

91. When normal shedding of epithelium of the filiform lingual papilla does not occur, it produces
thick layers of dead cells and keratin builds on the tongue surface:

a. Geographic tongue
b. Halitosis
c. Black hairy tongue
d. Melanin pigmentation

c - Black hair tongue is an accumulation of dead cells on the tongue that become stained by such
things as food, tobacco, medicine and certain oral bacteria.

92. The retention of saliva in a minor salivary gland results in a:

a. Mucocele
b. Ranula
c. Nicotinic stomatitis
d. Xerostomia

a - Mucocele is a result of a saliva duct that retains saliva and forms a small sack. A ranula is
similar, but affects the submandibular gland.

93. What is the name of the papillae that are lined along the anterior side of the sulcus terminals on
the body of the tongue?

a. Fungiform
b. Circumvallate
c. Foliate
d. Filiform

b - Circumvallate lingual papillae are a linear line of 10-14 mushroom shaped papillae anterior of
the foramen cecum.

TOP © 2015 - 2023 Tests.com


262
94. Which gland is located at the junction of the larynx and the trachea and can be palpated?

a. Parathyroid
b. Sublingual salivary
c. Sternocleidomastoid
d. Thyroid

d - The thyroid is located inferior to the thyroid cartilage. Parathryrode glands are located in the
same region, but cannot be palpated.

95. What is a function of the small intestine?

a. Absorb nutrients and minerals from food


b. Absorb water from indigestible food
c. Breakdown and digest food
d. Reabsorb fluids and process waste products

a - 90% of digestion and the absorption of food happens in the small intestine. The other 10%
takes place in the stomach and large intestine.

96. What is the drug-nutrient interaction of amoxicillin?

a. Metabolism is inhibited with grapefruit juice.


b. It cannot be taken with food as its effectiveness decreases.
c. It may alter intestinal flora, resulting in malabsorption of nutrients.
d. It cannot be taken with antacids because its effectiveness decreases.

c - Amoxicillin is an antibiotic that is used to treat an assortment of bacterial infections. It can alter
the flora of the intestine and cause a deficiency in the absorption of essential nutrients.

97. Tuberculosis is transmitted by what means?

a. Direct contact with blood


b. Direct contact with saliva
c. Direct contact with blood or saliva
d. Airborne droplets

d - Tuberculosis is carried in airborne particles called droplet nuclei, that can be generated when a
person coughs, sneezes, shouts or sings.

TOP © 2015 - 2023 Tests.com


263
98. What are two examples of fecal-oral transmission?

a. Contaminated water supply


b. Mosquitos
c. Ticks
d. Inadequate hand washing

a & d - Fecal-oral transmission is associated with organisms that affect the digestive systems. The
microorganisms enter the body through ingesting contaminated water or food.

99. All of the following are monosaccharides except:

a. Glucose
b. Fructose
c. Galactose
d. Maltose

d - Maltose is considered a disaccharide, which contains two glucose units. Monosaccharaides


contain one glucose unit.

100. A blood pressure reading of 148/90 is considered:

a. Normal
b. Prehypertension
c. Hypertension - stage 1
d. Hypertension - stage 2

d - A blood pressure 120/80 is considered normal. A person with a reading of 148/90 would be
considered hypertensive - stage 2. Hypertension - stage 2 is a reading of 140/90 or higher.

101. On which tooth does the Cusp of Carabelli normally appear?

a. Maxillary first molar


b. Maxillary second molar
c. Mandibular first molar
d. Mandibular second molar

a - The Cusp of Carabelli is normally found on the mesiopalatal line angle of the maxillary first
molar. It is hereditary and not found on all patients.

TOP © 2015 - 2023 Tests.com


264
102. An extra chromosome associated with Down syndrome:

a. Trisomy 23
b. Trisomy 21
c. Trisomy 18
d. Trisomy 13

b - Trisomy 21 is associated with Down syndrome. It means that instead of each cell having two
copies of the chromosome 21, it has three. Trisomy 18 is associated with Edwards syndrome and
Trisomy 13 is associated with Patau syndrome.

103. When B-cell lymphocytes divide and form plasma cells, they produce:

a. Immunoglobulins
b. Immunogens
c. Monocyte
d. Macrophage

a - When the plasma cells mature, they produce antibodies, which consist of IgA, IgE, IgD, IgG and
IgM. Each plasma cell produces a specific antibody and is in response to an immunogen.

104. What teeth are nonsuccedaneous?

a. All primary canine teeth


b. All permanent premolar teeth
c. All permanent molar teeth
d. All permeant anterior teeth

c - Nonsuccedaneous means that the teeth do not replace primary teeth. Permanent molar teeth
are the only ones that do not follow primary teeth.

105. Which teeth have a trifurcation?

a. Maxillary molars
b. Mandibular molars
c. Maxillary premolar
d. Mandibular premolars

a - Maxillary molars are the only teeth that have a trifurcation, due to having three roots.
Mandibular molars have a bifurcation, because they only have two roots.

TOP © 2015 - 2023 Tests.com


265
106. Which teeth have the greatest depth of curvature of the CEJ on the mesial surface?

a. Mandibular Lateral incisors


b. Maxillary canine
c. Mandibular centrals
d. Maxillary centrals

d - Not only do the central incisors have the greatest depth of curvature on the mesial, they also
have the largest height of contour for both labial and lingual surfaces.

107. What sterilization technique uses oxidation to sterilize dental instruments?

a. Autoclave
b. Unsaturated chemical vapor
c. Dry Heat
d. Ultrasonic

b - When sterilizing instruments using this technique, the substances combine with oxygen. Timing
starts when the temperature reaches its peak.

108. A patient that has a mild systemic disease and may or may not have special considerations in
regards to his or her dental management, would be considered what type of ASA Classification?

a. I
b. II
c. III
d. IV

b - A person that has a systemic disease, but may need medication or have other medical
considerations for their appointment, would be classified as Type II.

109. When a tooth loses tooth structure due to a chemical agent:

a. Attrition
b. Abrasion
c. Hypoplasia
d. Erosion

d - A chemical agent, such as soda or acid reflux, could cause the loss of tooth structure. When
tooth structure, such as enamel is lost, you cannot get it back.

TOP © 2015 - 2023 Tests.com


266
110. What is the vertical overlap of the maxillary and mandibular incisors considered?

a. Overjet
b. Overbite
c. Crossbite
d. Openbite

b - The maxillary teeth completely cover the mandibular teeth when the jaw is in the biting
position. Overjet occurs when the maxillary teeth flare out from the mandibular teeth.

111. What is the dominant bacterium in the initial plaque stage?

a. Gram positive streptococcus


b. Gram negative streptococcus
c. Spirochetes
d. Gram negative rods

a - In the initial phase, gram positive streptococcus is dominant. However, once the plaque is
established, the dominant bacterium becomes gram negative rods and spirochetes.

112. At what pH is a root demineralized?

a. 4.5-5.5
b. 5.5-5.9
c. 6.0-6.7
d. 6.5-7.4

a - Root structure is more likely to demineralize because of a high pH. Enamel demineralizes at a
pH of 4.5-5.5. This makes the root surface more susceptible to decay.

113. Tobacco is considered what type of stain?

a. Intrinsic
b. Extrinsic
c. Exogenous
d. Endogenous

c - Exogenous stains originate from the outside of a tooth. Endogenous stains, such
as tetracycline or tooth trauma, originate from inside the tooth.

TOP © 2015 - 2023 Tests.com


267
114. Which of the following conditions would respond to therapeutic diagnosis?

a. Bone cyst
b. Angular cheilitis
c. Fordyce granules
d. Paget disease

b - Angular cheilitis is a fungal infection that can be treated with an anti-fungal medication. The
other conditions cannot be treated with medications. They will not respond to therapeutic
therapies.

115. A dense, nonmineralized complex mass of bacteria that forms a gel like matrix:

a. Acquired pellicle
b. Biofilm
c. Demineralization
d. Plaque

b - Dental biofilm is a yellow mass of bacteria that when uncontrolled, can cause dental caries or
periodontal infections.

116. Which of the following is not a major component of saliva that helps protect against dental
caries?

a. Calcium
b. Phosphate
c. Iron
d. Fluoride

c - Iron is not a major component of saliva. However, there are trace amounts of the mineral that
are found in saliva. A lack of iron in your diet could cause: angular cheilosis, burning tongue, or
glossitis

117. Which nerve innervates the tooth pulp on the mandible?

a. Mental
b. Long Buccal
c. Posterior Superior Alveolar
d. Inferior Alveolar

d - The IA nerve innervates the pulp of the teeth. The long buccal and the mental nerve bring
supply to the surrounding tissues and tongue.

TOP © 2015 - 2023 Tests.com


268
118. Which of the following cranial nerves is not a sensory nerve?

a. Trochlear
b. Olfactory
c. Optic
d. Vestibulocochlear

a - The Trochlear nerve is a motor nerve, specifically with the eye. It has purely somatic motor
function. It is the smallest cranial nerve.

119. Which of the following is not a major salivary gland?

a. Submandibular
b. Sublingual
c. Parotid
d. Wharton

d - Warton’s is a duct that is part of the submandibular gland. It is an opening for saliva and is not a
major salivary gland.

120. In what order does the blood flow from the heart to the lungs? Put the following in the correct
order.

a. Right Atrium
b. Right Ventricle
c. Tricuspid Valve
d. Superior and Inferior Vena Cava
e. Lungs
f. Pulmonary Artery

Order: d, a, c, b, f, e
Blood flows from the heart to the lungs in the following order:

Superior and Inferior Vena Cava


Right Atrium
Tricuspid Valve
Right Ventricle
Pulmonary Artery
Lungs

121. Drugs with a high-first past effect have a smaller oral to parental dose ratio.

a. True
b. False, they have a larger parental dose ratio.
c. False, they have the same parental dose ratio.
d. False, drugs do not have a high-first past effect.

TOP © 2015 - 2023 Tests.com


269
b - Drugs with a high-first past effect have a LARGER oral to parental dose ratio.

122. What is the study of how a drug is absorbed, distributed, metabolized and excreted by the body?

a. Pharmacology
b. Pharmacokinetics
c. Pharmacodynamics
d. Pharmaceuticals

b - Pharmacokinetics is the movement of a drug in your body, once it is administered.


Pharmacodynamics is the study of effects the drug has on your body

123. Liver disease or previous or current substance abuse may impair the metabolism of a drug.

a. True
b. False, only current substance use affects the metabolism of a drug.
c. False, only previous substance abuse affects the metabolism of a drug.
d. False, liver disease does not affect the metabolism of a drug.

a - Metabolism will be impaired in relation to the severity of the damage to the liver. A
nonfunctioning liver could result in possible toxicity. The metabolism of drugs by means of enzyme
reactions, is mostly completed in the liver.

124. The _______ stage of development is when the first signs of an arrangement of cells occur.

a. Bell
b. Cap
c. Bud
d. Embryotic

b - The cap stage is the second stage in tooth development. The first is the bud stage where the
formation begins. The last stage is the bell stage, where the tissue form and the tooth shape is
established.

125. Which of the following is not a type of tissue found in the body?

a. Connective
b. Endoplasmic
c. Muscle
d. Nerve

b - Endoplasmic or endoplasmic reticulum is found throughout a cell and connected to the nucleus.
It is not a tissue in the body, but is an important part of cell.

TOP © 2015 - 2023 Tests.com


270
126. All of the following are part of the gingival fiber group except:

a. Interradicular
b. Dentogingival
c. Alveologingival
d. Circumferential

a - Interradicular fibers are not part of the gingival fiber group. It is actually a reference to the
fibers from the cementum and between the roots of teeth to the adjacent bone.

127. This vitamin is the catalyst for synthesis of blood clotting:

a. Vitamin A
b. Vitamin E
c. Vitamin D
d. Vitamin K

d - Vitamin K is the catalyst for blood clotting. It is derived from food sources in our gut.

128. A deficiency in Vitamin B3 (Niacin) can lead to what disease?

a. Pellagra
b. Scurvy
c. Enamel Mottling
d. Sickle Cell Anemia

a - Pellagra is caused by a chronic lack of the Vitamin B3. It can cause diarrhea, dermatitis and can
lead to mental disturbance.

129. Which of the following is not considered a form of calcium?

a. Cheese
b. Peas
c. Tofu
d. Kale

b - Peas are rich in vitamin B. Cheese, tofu and kale are all excellent sources of calcium and should
be considered when a patient is under nutritional counseling.

TOP © 2015 - 2023 Tests.com


271
130. What palpation method is used during an extra oral exam that uses the finger and thumbs of
both hands?

a. Bilateral palpation
b. Digital palpation
c. Bidigital palpation
d. Bimanual palpation

c - Bidigital palpation is accomplished by using your finger and thumb of both hands. This
technique is used to compress and palpate tissue during an extra oral exam.

131. Subluxation is when the condyle of the temporal mandibular joint glides beyond the eminence
preventing the jaw from closing. Subluxation can be caused by long dental procedures.

a. Both statements are true


b. Both statements are false
c. First statement is true. Second statement is false.
d. First statement is false. Second statement is true.

a - Subluxation is the clinical term for dislocation. This can be caused by long dental procedures,
which stretch the ligaments around the joint, making it easy for the condyle to dislocate. It is
important to inform your patients that they should restrict large movements after long dental
procedures.

132. Periodontal disease can be more severe in patients that smoke when they are deficient in this
vitamin:

a. Vitamin A
b. Vitamin B3
c. Vitamin B6
d. Vitamin C

d - Vitamin C helps the lining of the gums stay healthy. When a person, especially a smoker, is
deficient in this vitamin, the lining is susceptible to bacterial infections, potentially leading to
periodontal disease.

133. What happens when the globular process fails to fuse with the maxillary process during the first
trimester of pregnancy?

a. Cleft Lip
b. Cleft Palate
c. Cleft Nasal
d. Cleft Mandible

TOP © 2015 - 2023 Tests.com


272
a - Cleft lip begins to develop between the 4th and 7th weeks of the first trimester. It becomes
evident during the second month of pregnancy.

134. Cementum contains collagen fibers. In teeth, Sharpey’s fibers are the terminal ends of the
principal fibers that insert into the cementum and into the periosteum of the avelor bone.

a. Both statements are true.


b. Both statements are false.
c. First statement is true. Second statement is false.
d. First statement is false. Second statement is true.

a - Collagen fibers become mineralized and incorporated into the cementum. In teeth, the fibers
that are the terminal ends of the principal fibers, that insert into the cementum and into the
periosteum of the avelor bone are called Sharpey’s fibers.

135. What plane divides the body into left and right halves?

a. Axial
b. Parasagittal
c. Sagittal
d. Transverse

c - The sagittal plane runs down the midline of our body, creating the left and right side. The
Parasagittal plane means any plane that runs parallel to the sagittal plane.

136. What is the smallest permanent tooth in the mouth?

a. Mandibular central incisors


b. Maxillary canine tooth
c. Mandibular lateral incisor
d. Maxillary lateral incisor

a - The mandibular central incisors are the smallest permanent tooth in the oral cavity. They are
the smallest due to the fact that they have a very slight concavity with labial surfaces that are
smooth.

TOP © 2015 - 2023 Tests.com


273
137. What is the arrow pointing to?

a. Lamina dura
b. Cancellous bone
c. Periodontal ligament
d. Cementum

b - Cancellous bone is spongy looking and is the inner or central part of the bone. It is part of the
alveolar bone that supports the tooth in both the maxilla and mandible.

138. During swallowing, the esophageal phase is under voluntary neuromuscular control. The
pharyngeal phase is under involuntary neuromuscular control.

a. Both statements are true.


b. Both statements are false.
c. First statement is true. Second statement is false.
d. First statement is false. Second statement is true.

d - Both pharyngeal and esophageal phases of swallowing are under involuntary neuromuscular
control. They are both considered involuntary reflex of muscles that allow us to swallow our food
fast and easy, causing little harm to surrounding organs.

139. Which two organs are endocrine glands?

a. Kidney
b. Ovaries
c. Thyroid
d. Ureters

Both b and c. The endocrine glands secrete their product or hormones directly into the blood,
rather than using ducts. The thyroid and ovaries both secrete hormones. The kidneys and ureters
are part of the urinary system.

TOP © 2015 - 2023 Tests.com


274
140. A psuedopocket is produced by edematous gingival tissue. The junction epithelium does not
migrate when there is psuedopocketing.

a. Both statements are true.


b. Both statements are false.
c. First statement is true. Second statement is false.
d. First statement is false. Second statement is true.

a - A psuedopocket is due to gingival enlargement and does not involve the migration of
connective tissue attachment and other supporting periodontal tissues.

141. A patient with a Class I occlusion would have a facial profile that is mesognathic. Mesognathic is
considered to be a normal profile.

a. Both statements are true.


b. Both statements are false.
c. First statement is true. Second statement is false.
d. First statement is false. Second statement is true.

a - Mesognathic is when your jaw slightly protrudes in its resting state. This is considered a normal
facial profile.

Practice Test for the NBDHE Dental Hygienist Exam – Section 2


Provision of Clinical Dental Hygiene Practice

142. What information source determines the general health of the patient?

a. Clinical interview with the patient


b. Medical history form
c. Dental health history form
d. Letter from patient’s primary care physician

b - Medical history forms are part of the paperwork that patients will fill out upon their initial visit
with the dental office. These forms contain information that determines their general health,
allowing dental care providers to determine whether or not there may need to be special
accommodations regarding their care.

TOP © 2015 - 2023 Tests.com


275
143. A patient leaves the medication portion of her health history form blank. Upon being seated the
hygienist reviews the form with her and asks if she takes any medications or supplements. The
patient states that she takes birth control pills but she did not list them on the medication
portion because it is irrelevant to her dental treatment. The hygienist informs her that it should
be included, because birth control pills _________.

a. may interact with local anesthetic agents


b. may become less effective if given antibiotics
c. could cause gingival overgrowth
d. change the oral flora

b - Birth control pills may become less effective if the patient is placed on a prescription antibiotic
treatment. Even though patients may leave certain areas of their health history form blank it is still
important to review them, to be sure that the patient did not fail to relay any vital information.

144. Medical history forms should be updated:

a. Annually
b. Biannually
c. Every 3 years
d. At each visit

d - At each visit, patients should review their health history forms and note if any changes have
occurred. This includes surgeries, medications, supplements, or even a change of doctor.

145. A healthy adult’s pulse rate should be between ______ beats per minute.

a. 120-160
b. 80-130
c. 50-120
d. 60-100

d - 60-100 beats per minute is a healthy heart rate for a fully grown adult. Infants are between
120-160, toddlers between 80-130 and children at 70-110 beats per minute.

146. Which two of the following are reasons that require antibiotic premedication?

a. Systemic lupus erythematosis


b. Joint replacement less than 2 years ago
c. Joint replacement up to 5 years ago
d. Type II diabetes

a & b - Both systemic lupus erythematosis and joint replacement within the last 2 years are
reasons requiring antibiotic premedication prior to dental procedures. Other examples of
premedication needs include artificial joint infections, weak immune system and Type I diabetes.

TOP © 2015 - 2023 Tests.com


276
147. Which of the following is NOT an essential part of an initial extraoral examination?

a. Palpation of the masseter and temporalis muscles


b. Inspection of the lips for pigmentations
c. Inspection of the ventral surfaces of the tongue
d. Inspection for facial symmetry

c - Inspection of the ventral surfaces of the tongue is a part of the intraoral examination, but not
part of the extraoral examination.

148. Which of the following conditions are inspected upon examination of the parotid gland? Select
all answers that apply.

a. Hardness
b. Mobility
c. Swelling
d. Enlargement

a, c & d - Hardness, swelling and enlargement as well as pain are inspected in a patient’s parotid
gland. Mobility is not one of the factors determining the health of the gland, which is inspected
using bilateral palpitation.

149. Which lymph nodes are assessed through palpitation with the patient’s head tilted forward?

a. Deep cervical
b. Accessory
c. Auricular
d. Occipital

d - Occipital lymph nodes should be palpated with the patient’s head tilted forward, allowing the
practitioner access to palpate the area around the occipital region of the skull.

150. Which of the following appearances in gingiva indicates a disease condition?

a. Loss of gingival stippling


b. Pigmentation of gingiva
c. Pink, stippled gingiva
d. Firm, knife-edged papilla

a - Loss of gingival stippling is a sign of disease like conditions within the mouth. Gingiva should
appear pink and stippled, with a knife-edge papilla. Pigmentation may be normal in patients of
different races.

TOP © 2015 - 2023 Tests.com


277
151. What area should be palpated to determine if a blockage of sialoliths is present?

a. Torus
b. Wharton’s duct
c. Ranula
d. Exostosis

b - Wharton’s duct is the submandibular salivary gland that should be palpated during an intraoral
examination. This gland may become blocked with sialoliths, which would be present on an
occlusal x-ray.

152. Which two locations in the mouth are high-risk areas for oral cancer?

a. Dorsal surface of tongue


b. Ventral surface of tongue
c. Lateral surface of tongue
d. Buccal mucosa

b & c - The ventral and lateral surfaces of the tongue are high-risk areas for finding oral cancerous
lesions during an intraoral exam.

153. What condition of the mouth is caused by the Epstein-Barr virus?

a. Hairy leukoplakia
b. Black hairy tongue
c. Macroglossia
d. Median rhomboid glossitis

a - Hairy leukoplakia is caused by the Epstein-Barr virus, which causes hyperkeratosis of the filiform
papilla, leaving a corrugated, white area on the lateral borders of the tongue.

154. MATCH the following classifications of decay with their location using G.V. Black’s Classification:

____ Class I a. Proximal surfaces, including incisal edges


____ Class II b. Proximal surfaces of anterior teeth
____ Class III c. Gingival 1/3 of facial or lingual surfaces
____ Class IV d. Occlusal and lingual pit and fissure
____ Class V e. Cusp tips
____ Class VI f. Proximal surfaces of posterior teeth

TOP © 2015 - 2023 Tests.com


278
d. Class I
f. Class II
b. Class III
a. Class IV
c. Class V
e. Class VI

155. Which two of the following endogenous intrinsic stains exhibit white lesions?

a. Tetracycline
b. Amoxicillin
c. Fluorosis
d. Trauma

b & c - Amoxicillin and fluorosis can cause white staining in the enamel of the teeth if the person
was exposed during early age when tooth development was still occurring. Fluorosis may also have
brown spots or no white spots at all, and amoxicillin causes possible white opacities.

156. Which type of restorative overhang comprises 1/3 to 1/2 of the interproximal embrasure space
of a tooth?

a. Type I
b. Type II
c. Type III
d. Type IV

b - Type II overhangs comprise 1/3-1/2 of the interproximal embrasure space. Type I is less than
1/3, Type III is more than 1/2 and there is no type IV overhang.

157. Which of the following is NOT a caries risk factor?

a. Xerostomia
b. Radiation therapy
c. Low income
d. Fewer than 15 teeth

d - Fewer than 15 teeth is not regarded as a caries risk factor, though patients that have less than 9
present teeth are regarded as at a high caries risk.

TOP © 2015 - 2023 Tests.com


279
158. Which two of the following are signs of chronic gingival inflammation?

a. Tough, fibrous gingiva


b. Soft, spongy gingiva
c. Blunted interdental papilla
d. Swollen interdental papilla

a & c - Tough, fibrous gingiva with blunted interdental papilla are signs of chronic gingival
inflammation. Other signs include dark colored gingiva and widening of embrasures between the
teeth.

159. During a periodontal screening and reporting (PSR), the deepest measurement in one quadrant
extends only onto the first clear band of the periodontal probe, with calculus present and
bleeding upon probing. What would the proper PSR score be for this quadrant?

a. 0
b. 1
c. 2
d. 3

c - A PSR score of 2 would be appropriate for areas of calculus buildup, bleeding upon probing, but
probing depths only extending into the first clear band of the periodontal probe. If the depth of
the pocket extended into the color section, then the PSR score would be 3.

160. Which classification of tooth mobility exists when the tooth is depressible?

a. Class III
b. Class II
c. Class V
d. Class IV

a - Class III mobility (the most severe classification) exists during severe mobility, when a tooth is
depressible.

161. Which plaque score indicates gross soft debris within gingival pockets and/or margins?

a. 0
b. 3
c. 2
d. 4

b - A plaque score of 3 is the highest plaque score on the plaque index (PI), indicating gross
amounts of soft plaque debris within and along gingival pockets and margins.

TOP © 2015 - 2023 Tests.com


280
162. Which classification of furcation involvement indicates through and through involvement of
bone loss between the roots, with an opening covered by the gingiva?

a. Class IV
b. Class II
c. Class III
d. Class I

c - Class III furcation involvement indicates through and through loss between the trifurcation and
bifurcation of the roots, with the area still covered by gingiva. If the area were visible during
examination, then it would be a class IV furcation involvement.

163. What is the vibration felt in the tooth during occlusion?

a. Fremitus
b. Tribismus
c. Furcation
d. Percussion

a - Fremitus is the vibration felt in a tooth when the teeth occlude. You can feel it when placing the
finger near the gingival margin of the crown and asking the patient to tap his or her teeth
together.

164. Which of the following is not a factor that affects the setting of gypsum products?

a. Temperature of water
b. Exposure to light
c. Amount of water used
d. Humidity

b - Exposure to light is not a factor that affects the setting of gypsum products. However, the
others listed do affect it.

165. What is the level of marginal gingiva apical to the cementoenamel junction that can be
measured with a periodontal probe?

a. Clinical attachment level


b. Recession
c. Gingival pocket
d. Periodontal pocket

b - Recession is the area measured with a periodontal probe from the CEJ to the margin of the
gingiva that is apical to the CEJ. Measuring recession does not determine pocket depth or
attachment.

TOP © 2015 - 2023 Tests.com


281
166. Which bacteria is the main etiological cause for the initiation of coronal lesions?

a. Staphylococcus
b. Lactobacilli
c. Streptococcus mutans
d. Sucrose

c - Streptococcus mutans, or S. Mutans is the primary bacteria responsible for the initiation of
coronal lesions. As the decay advances, Lactobacilli may progress it to a larger decayed area.

167. In regards to radiation wavelengths, select the two appropriate answers regarding wave
penetration.

a. Longer wavelengths are less frequent and produce less penetrating x-rays
b. Shorter wavelengths are more frequent and produce more penetrating x-rays
c. Longer wavelengths are more frequent and produce more penetrating x-rays
d. Shorter wavelengths are less frequent and produce less penetrating x-rays

a & b - Longer x-rays are fewer in frequency (less waves throughout the beam) and produce a
beam that is less penetrating that shorter x-ray waves. Shorter wavelengths occur at a higher
frequency and produce a much more penetrating x-ray beam. The longer the beam, the weaker
the x-ray wave. The shorter the beam, the stronger the x-ray wave.

168. When an x-ray photon intermingles with water and oxygen molecules in body tissue, the result
is ____________.

a. Compton effect
b. Photoelectric effect
c. Scatter radiation
d. Free radicals

d - Free radicals result when x-ray photons intermingle with water and oxygen biological molecules
inside of biological molecules, with a highly chance of causing damage to biological cells.

169. What is the safest area to stand in relation to a primary x-ray beam about to be exposed on a
patient?

a. 4 feet away at a 90-degree angle from the beam


b. 5 feet away at a 180-degree angle from the beam
c. 6 feet away at a 90-135-degree angle from the beam
d. 3 feet away at a 90-135-degree angle from the beam

c - Dental practitioners should always stand at least six feet away from the primary beam source,
at an angle of 90-135 degrees from the path of the direct beam.

TOP © 2015 - 2023 Tests.com


282
170. What is the maximum permissible dose of radiation that a dental professional should receive per
year?

a. 0.5 rem
b. 0.05 rem
c. 5.0 rem
d. 5.0 Sv

c - 5.0 rem (equivalent to 0.05 Sv) is the maximum amount of radiation per year that any dental
professional should be exposed to. For the average person, it is 0.5 rem per year.

171. What is the lethal dose of radiation that would cause death in 50% of the population within 30
days?

a. 2.0 Sv
b. 20.0 rem
c. LD 30/50
d. LD 50/30

d - The lethal dose of radiation that can be absorbed and would cause death in 50% of the
population within 30 days is LD 50/30. The latent period would extend between the exposure and
the time that a biological effect took place.

172. Which portion of the x-ray tubehead performs collimation?

a. PID
b. Aluminum filter
c. Focusing cup
d. Target

a - The position indication device (PID) performs collimation of the x-ray beam to restrict the size
of the beam. The PID is round or rectangular in shape.

173. Regulations and federal law restrict the size of PID and tubehead devices. Which of the following
options is NOT accurate?

a. Aluminum filters must be no less than 1.5 mm thick for exposures below 70kVp
b. Circular tubeheads must be no less than 2 3/4” in diameter
c. Rectangular tubeheads must be no less than 1 3/8 by 1 3/4” in diameter
d. PID lengths must be no less than 6”

d - The statement that PID lengths must be no less than 6” is incorrect. PID lengths must actually
be no shorter than 8”, 12” or 16”, depending on the type of beam.

TOP © 2015 - 2023 Tests.com


283
174. _____________ would result in a better contrast of grey shades in the final radiograph.

a. Decreasing mA
b. Increasing exposure time
c. Increasing kVp
d. Increasing mA

c - Increasing the kVp to 75-90 kVp allows for a better contrast of grey shades in the final
radiograph. This helps when diagnosing other tissues such as bone height.

175. What radiology technique places the x-ray beam perpendicular to the imaginary line between
the tooth and the film?

a. Bisecting technique
b. Parallel technique
c. XCP technique
d. None of the above

a - Bisecting technique placed the x-ray beam perpendicular to the imaginary line between the
long axis of the tooth and the x-ray film placed at an angle to the tooth. It allows the practitioner
to get an accurate image even though the film is not placed parallel to the long axis of the tooth.

176. What angulation is important to direct between interproximal areas of the teeth so as not to
cause overlapping images on the film?

a. Horizontal
b. Vertical
c. Bisecting
d. Paralleling

a - Horizontal angulation that is properly aligned between interproximal areas of the teeth will
allow the beam to go through easily and result in open contacts on the final film. If the horizontal
angulation is inaccurate it will cause overlapping.

177. When attempting to locate the position of a foreign substance or pathology, an x-ray film is
taken and then retaken at a _______ change toward the mesial. If the object moves distally on
the film, then the object is located on the _______ side of the teeth.

a. 40-degree; buccal
b. 40-degree; lingual
c. 20-degree; buccal
d. 20-degree; lingual

TOP © 2015 - 2023 Tests.com


284
c - 20-degree changes are best for locating pathology or objects in the mouth that need to be
identified. Moving a beam one way (mesial) with the resulting image moving in the opposite
direction (distal), then the object is on the lingual side of the teeth. “SLOB”: same mesial, opposite
buccal.

178. A _________ object to film distance and ________ source to film distance decreases
magnification of the image.

a. Shorter; Shorter
b. Longer; Longer
c. Shorter; Longer
d. Longer; Shorter

a - Both shorter object to film and source to film distances will reduce magnification of the image
on the x-ray. Distancing the lengths between object to film or source to film would cause the
image to become magnified on the film.

179. Identify the radiolucency extending through the mandible.

a. Hylohyoid ridge
b. Mandibular canal
c. External oblique ridge
d. Inferior border of mandible

b - The mandibular canal houses nerve tissue and appears as a radiolucent canal extending down
through the mandible.

TOP © 2015 - 2023 Tests.com


285
180. Identify the radiopaque area.

a. Cementum
b. PDL
c. Lamina Dura
d. CEJ

c - The lamina dura is the radiopaque margin of supporting bone structure that houses the socket
of the root, and provides an attachment area for the periodontal ligament.

181. Identify the bilateral radiolucencies.

a. Hyperostotic bone
b. Alveolar crest
c. Trabecular bone
d. Mental foramen

TOP © 2015 - 2023 Tests.com


286
d - The mental foramen is the bilateral round or oval opening in the mandible that allows nerve
tissues to pass through into the supporting alveolar structures.

182. Identify the localized interproximal radiopacities:

a. Subgingival calculus
b. Alveolar crest
c. Cervical burnout
d. Cementoenamel junction

a - Subgingival calculus in large amounts appears as radiopaque extensions of the tooth structure
near or around the CEJ in the interproximal areas on radiographs.

TOP © 2015 - 2023 Tests.com


287
183. Identify the radiopaque structure.

a. Maxillary nasal sinus border


b. Zygomatic arch
c. Nasal sinus
d. Ghost of jewelry worn during x-ray exposure

b - The zygomatic arch appears as a bony “U” that is visible just superior to the maxillary molars
and is more defined than the nasal sinuses.

184. Identify the anatomical structure:

a. Anterior foramen
b. Lingual foramen
c. Apical foramen
d. Trabecular bone

TOP © 2015 - 2023 Tests.com


288
b - The lingual foramen is the tiny hole at the anterior midline of the mandible and is surrounded
by the genial tubercles.

185. Identify the following radiopaque anatomical structure:

a. Median suture
b. Nasopalatine foramen
c. Anterior nasal spine
d. Superior foramina

c - The anterior nasal spine is the radiopaque bony structure superior to the median suture.

Continue to the next page.

TOP © 2015 - 2023 Tests.com


289
186. Which of the following statements is NOT accurate regarding the following radiograph?

a. The film was placed into the mouth backward


b. Inaccurate horizontal angulation was used
c. The film is mounted upside down
d. The film is positioned at the proper posterior position for a molar PA

c - The film is mounted the correct way. While the film was positioned in the proper place (the
distal of the first bicuspid is viewable on the film), inaccurate vertical angulation of the tubehead
was used, resulting in interproximal overlap. The film was placed into the mouth backward and
this is evident by the patterned area across the entire film where the x-ray beam came into
contact with the lead foil.

187. MATCH the stage of anesthesia with the conditions that it results in

___ Stage I a. Unconscious, regular respiration and pulse


___ Stage II b. Conscious, reduced pain
___ Stage III c. Respiratory arrest
___ Stage IV d. Loss of consciousness, irregular respirations

b. Stage I
d. Stage II
a. Stage III
c. Stage IV

TOP © 2015 - 2023 Tests.com


290
188. Which of the following conditions would be a contraindication for Nitrous Oxide use in a
patient?

a. Cerebral palsy
b. Strong gag reflex
c. Cardiovascular disease
d. Multiple sclerosis

d - Multiple sclerosis is a contraindication for nitrous oxide use, as are people with respiratory
infections, sinus obstructions, epilepsy or emotional instability.

189. What injection will anesthetize the upper central incisors and canines?

a. Infraorbital nerve
b. Anterior superior alveolar nerve
c. Middle superior alveolar nerve
d. Greater palatine

b - The anterior superior alveolar nerve injection will anesthetize the upper incisors and canines.
The injection is placed in the mucobuccal fold just mesial to the canine eminence.

190. Which injection would anesthetize the palatal tissue of maxillary anterior teeth?

a. Nasopalatine
b. Greater palatine
c. Infiltration
d. Lingual block

a - The nasopalatine injection anesthetizes the palatal tissues of the maxillary anterior teeth. The
teeth are not anesthetized, but the hard palatal tissues are.

191. Local anesthetic causes cell permeability to decrease, preventing ______ from entering into the
nerve cell.

a. chloride
b. calcium
c. sodium
d. potassium

c - Sodium is prevented from entering into nerve cells when local anesthetic is used. This prevents
nerve impulses because the potassium cannot get out of the nerve, and inhibits nerve
depoliarization.

TOP © 2015 - 2023 Tests.com


291
192. What systemic condition will result in pale mucosa, angular cheilosis and glossitis?

a. Hemophelia
b. Leukemia
c. Sickle-cell disease
d. Anemia

d - Anemia in a patient would present itself as painful glossitis, pale gingiva and mucosa, and
angular cheilitis. The condition may cause abnormal bleeding in conjunction with other diseases of
the gingiva.

193. When treating a patient with a new cardiac pacemaker, which of the following precautions
should be taken?

a. Avoid the use of piezoelectric scalers


b. Avoid the use of magnetostrictive ultrasonic scalers
c. Avoid the use of all electronic scalers
d. None of the above

b - It is advised that magnetostrictive ultrasonic scalers should be avoided in the use with people
with cardiac pacemakers, even though there are shields in place to protect the pacemaker.
Piezoelectric scalers have not been shown to interfere with pacemaker devices.

194. Which condition would be treated with calcium channel blockers, causing possible gingiva
hyperplasia?

a. Muscular Dystrophy
b. Anemia
c. Cardiovascular disease
d. Diabetes

c - Cardiovascular disease is often treated with medications that are calcium channel blockers.
These prescription drugs often cause gingival hyperplasia in the oral tissues.

195. Which benign cancer originates in the epithelium?

a. Papilloma
b. Carcinoma
c. Sarcoma
d. Lipoma

a - Papilloma cancer is a benign form that originates in the epithelium tissue, including the skin and
mucous membranes. The malignant form of cancer that affects the epithelium is squamous or
basal cell carcinoma.

TOP © 2015 - 2023 Tests.com


292
196. What should be done prior to administering a breath to the unconscious patient that is not
breathing?

a. Check the airway for obstruction


b. Listen for a breath
c. Check for circulation
d. Administer chest compressions

a - Check the airway for obstruction prior to administering rescue breaths to the patient. Look
inside of the mouth and perform a finger swipe if necessary.

197. A patient experiences anaphylaxis during a dental procedure. Which of the following is NOT an
appropriate care method for this patient?

a. Terminate the treatment and activate EMS


b. Administer epinephrine
c. Place the patient in an upright position
d. Place the patient in a supine position

c - Patients should not be placed in an upright position, but instead placed in a supine position
(flat). Terminate the treatment and activate EMS, while a team member brings the emergency
office kit containing an Epi-pen (epinephrine) for administration.

198. A diabetic patient arrives having just taken their insulin, but only ate a very light lunch due to
having a dental procedure scheduled. During the appointment the patient begins experiencing
headache and shakiness. What should the practitioner do?

a. Administer nitrous oxide


b. Wait until the patient feels better to continue the treatment
c. Ask the patient to take more insulin
d. Have the patient drink a soda

d - Have the patient drink a soda or juice, or have another sugar-containing snack due to a low
level of blood sugar. After administering the sugar, monitor the patient.

199. A patient who reclines in the chair for treatment soon begins to convulse and loses
consciousness. What should the practitioner do first?

a. Sit the patient up


b. Place a prop in the mouth to prevent laceration of the tongue
c. Remove equipment from within reach of the patient
d. Activate EMS

c - Removing equipment from within the reach of the patient is the first thing that should be done

TOP © 2015 - 2023 Tests.com


293
for a patient that is experiencing a seizure in the operatory chair. This prevents the patient from
coming into contact with something that could harm them.

200. Which of the following bodily fluids is not considered to be a contamination source?

a. Urine
b. Saliva
c. Sweat
d. None of the above. All are considered sources of contamination.

c - Sweat is not regarded as a contamination source. All other bodily fluids should be considered
contaminated with blood or disease.

201. Face masks should be changed:

a. Whenever visibly soiled


b. At least twice per day
c. As often as gloves are changed
d. After each patient

d - Face masks should be changed after each patient is seen, whether or not it is visibly soiled.

202. What type of solution is shown to break down latex in gloves?

a. Alcohol-based hand sanitizer


b. Antimicrobial soap
c. Petroleum based creams
d. Lotion

c - Dental professionals should not use petroleum based creams because petroleum causes the
breakdown of latex.

203. Which type of sterilization machine is able to perform a complete sterilization cycle in less than
10 minutes?

a. Statims
b. Chemical vapor
c. Steam autoclave
d. Cold sterilization

a - Statim autoclaves are capable of performing complete sterilization cycles in under 10 minutes.
Steam is injected directly into the statim cassette and the temperature is a lower level, which
allows for quicker heating and cooling.

TOP © 2015 - 2023 Tests.com


294
204. When presenting a treatment plan to a patient, which of the following is NOT appropriate for
the dental hygienist to do upon assessing the patient?

a. Explain risks associated with not completing the treatment


b. Present a dental hygiene treatment plan and inform the patient of how many
appointments are needed
c. Prioritize patient’s periodontal health needs
d. Present a restorative treatment plan and inform the patient of how many appointments
are needed

d - Dental hygienists are not to diagnose restorative treatment needs, even if they are aware of
what treatment may be completed. The hygienist may choose to discuss possible needs that the
dentist may diagnose or plan, but hygienists are only to present dental hygiene case plans to the
patient.

205. A patient is informed of their dental treatment plan. The plan is then signed by the dentist,
hygienist and patient. This document becomes a part of the patient’s permanent file and is
called __________________.

a. Informed consent
b. Documented consent
c. Treatment plan
d. Treatment agreement

a - Informed consent is when the patient is informed of their dental treatment plan and signs the
plan acknowledging that the plan has been explained to them and the treatment is necessary. If
the patient eventually refuses to have the treatment completed there will still be documentation
that shows the treatment was reviewed with the patient, who should also be told of what possible
problems could arise if the treatment was not completed.

206. Which of the following is an essential part of a patient’s needs assessment documented by the
dental hygienist?

a. General assessment
b. Periodontal assessment
c. Dentition assessment
d. All of the above

d - All of the above are necessary for a dental hygienist to perform a comprehensive needs
assessment on a patient. General items such as medical history, intraoral and extraoral exams,
periodontal assessment and a dentition assessment need to be integrated together so as to
determine what the appropriate method of a treatment should be for the patient.

TOP © 2015 - 2023 Tests.com


295
207. Factors that determine the prognosis of a completed dental hygiene treatment plan may NOT be
influenced by:

a. Quality of existing restorations


b. Malocclusion
c. The patient’s financial situation
d. Degree of attachment loss

c - A patient’s financial situation does not affect the prognosis of a completed treatment plan.
Other health-related situations such as malocclusion, severe periodontal attachment loss and poor
or failing restorations can encourage the continuation of oral disease within the mouth.

208. Notes regarding the treatment plan, if documented in the margin of a patient’s file, should be
written in _______.

a. Pencil
b. Blue ink
c. Typed
d. Whited out and then documented in the body of the chart with ink.

b - Using ink is the appropriate method to document notes of any sort within a patient’s file.
White-out should never be used; instead a line should be drawn through any words that were
incorrect, with initials placed at the end of the line. Pencil notes are not legitimate in a legal sense
in regard to documenting patient information.

209. A new patient scheduled for a comprehensive exam and prophylaxis states on her medical
history form that she has undergone a knee replacement in the last six months. She has not
recently spoken with her primary care physician or surgeon. Which of the following would be an
appropriate part of the care plan?

a. Perform the exam and then a prophylaxis if there is a low plaque index
b. Perform the exam and then reschedule the prophylaxis to allow for antibiotic
premedication
c. Do not perform any exam and reschedule the patient to allow for antibiotic
premedication
d. Call her primary care physician to see if it is ok to perform a prophylaxis

b - The dentist may perform an intraoral exam on the patient to diagnose any care needs, but the
patient should not have a prophylactic procedure until she has been premedicated with an
antibiotic at least one hour prior to the appointment. Premedication is necessary in patients that
have had a recent joint replacement up until two years prior to receiving dental care.

TOP © 2015 - 2023 Tests.com


296
210. A patient presents with stage II Hypertension and a blood pressure rate of 165/110. The blood
pressure is then rechecked five minutes later and measures at the same level. What is the next
step of the treatment assessment or plan that should be performed in this case?

a. Perform a prophylaxis, with a written medical referral


b. Refer patient to their medical doctor for a consultation prior to prophylaxis
c. Review history of blood pressure and if it has been low at previous visits then a
prophylaxis is appropriate
d. Emergency medical referral, no dental care whatsoever

b - Patients with stage II Hypertension symptoms may only receive noninvasive care such as an
exam and x-rays, but invasive procedures should be preceded by a medical consultation with the
patient’s primary care physician. Only elective emergency care should be provided for this patient.

211. Digital palpation should not be used on ___________.

a. Hard palate
b. Alveolar ridges
c. Soft palate
d. Buccal mucosa

c - Palpitation of the soft palate is not recommended during intraoral exams as it could trigger a
gag reflex in the patient.

212. Assessing a treatment plan should be based on using all of the following areas EXCEPT:

a. Eliminating disease
b. Affordability
c. Slowing disease progression
d. Maintaining and promoting overall health

b - While affordability is a large concern of most patients, it should not be the determining factor
when presenting an effective treatment plan needs assessment to the patient. The goal of the
dental provider is to eliminate dental disease or slow it’s progression, while maintaining and
promoting oral and systemic levels of health.

213. Case presentations should:

a. Not include the length of appointments


b. Be based only on the practitioner’s diagnosis without patient preference in regard to
therapy
c. Allow the patient to decline care
d. List causative factors described in scientific, professional terminology

c - Every patient has the right to decline their treatment plan, but it should be documented so in

TOP © 2015 - 2023 Tests.com


297
the patient’s file. The case presentation should review in layman’s terminology what treatment is
needed, why, what results will occur if treatment is not completed and explain options to the
patient so that they can co-plan their treatment.

214. A new patient arrives for their dental appointment. Which of the following is most appropriate
in regard to assessing the patient?

a. Take necessary x-rays; hygienist performs intraoral and extraoral exams; perform
prophylaxis if gum health is stable
b. Take necessary x-rays; hygienist performs intraoral exam and periodontal charting; begin
prophylaxis if appropriate
c. Patient has dental needs assessed by the dentist; take necessary x-rays; hygienist
performs intraoral and extraoral exam and prophylaxis
d. Hygienist performs intraoral and periodontal exam; prophylaxis; dentist performs needs
assessment; take necessary x-rays

c - New patients should always be seen by the dentist and have the dentist diagnose x-rays to be
taken prior to any procedures such as a prophylaxis are performed on the patient. Patients of
record that have not been seen by the dentist in over a year should see the dentist prior to the
completion of any procedures.

215. A recall patient comes in for his scheduled biannual prophylaxis. During the appointment the
RDH notices that #30 has moderate bone loss and class II mobility. There are no other significant
findings and the patient is rescheduled for their next preventive care appointment with no
mention made of #30. At this appointment the tooth has 2 more mm of bone loss and class III
mobility. Which of the following statements is correct?

a. The RDH may be guilty of supervised neglect


b. The RDH should place Arestin after scaling #30 and then have the patient back for an
exam
c. The patient should have a periodontal scaling and root planing procedure
d. The patient should have a full mouth debridement

a - The RDH may be guilty of supervised neglect because she failed to inform the patient of an area
or tooth that was suffering from periodontal disease conditions. As a result the patient took no
additional oral health care steps, assuming that their oral health was in line. Further attachment
loss now may lead to the loss of the entire tooth because no preventive methods were taken and
the patient was never informed.

TOP © 2015 - 2023 Tests.com


298
216. A patient with no systemic health risks receives nonsurgical periodontal therapy. One year later
they exhibit good oral health with no bleeding on probing. The patient’s next recare
appointment should be scheduled in:

a. 3 months
b. 6 months
c. 14 months
d. Every 3-4 months for the duration of their care

b - 6-12 month recare appointments are appropriate for the periodontal patient that receives
nonsurgical care and responds well. Initial maintenance visits are performed about every 3
months, but if the patient has made it to a year post procedure and exhibits good general and oral
health then the recare appointments may be performed less frequently.

217. Prognosis of a patient’s periodontal health does NOT include:

a. Race
b. Socioeconomic status
c. Systemic health
d. Age

a - Race is not regarded as a risk factor in assessing a patient’s periodontal health. Conditions that
do affect periodontal health include socioeconomic status, age, systemic health conditions,
periodontal disease, malocclusion, appliances worn and any habits such as tobacco or alcohol use.

218. A temporary crown placed on a tooth following an RCT procedure is marked in the dental chart
as:

a. Solid blue area marked with “SSC”


b. Crown restoration marked with a “T”
c. Hashed crown in blue pencil
d. Red crown

b - Temporary crowns are marked with a “T” for temporary over the type of restoration that was
placed.

219. A diastema between #8 and 9 is marked by:

a. One vertical red line between 8/9


b. Two vertical red lines between 8/9
c. One “v” drawn between 8/9
d. Two horizontal red lines between 8/0

b - Two vertical red lines placed between teeth #8 and 9 on a dental chart indicate a diastema
between the two teeth.

TOP © 2015 - 2023 Tests.com


299
220. A patient presents with rampant caries. The RDH discusses risk factors such as diet, medication
and other areas that may cause concern. Which of the following steps is NOT part of the ideal
care plan for this patient?

a. More frequent bitewing radiographs


b. 6 month recare appointments
c. Home fluoride supplementation
d. Modification of oral hygiene practices

b - 6 month recare appointments may be too infrequent for patients that present with a high
caries rate. More frequent prophylactic appointments (such as every 3 months) can allow for
plaque removal, review of nutritional habits and fluoride application to reduce the decay rate.

221. Which of the following is NOT associated with a higher rate of tooth decay?

a. GERD
b. Frequent soda exposure
c. Xerostomia
d. Not brushing with fluoride toothpaste

d - Not brushing with a fluoride toothpaste is not considered a risk factor for caries. Risk factors
are those, which place the patient at an increased risk of tooth decay, as patients with acid reflux,
high soda intake or xerostomia would be. A patient may brush with tap water and be able to keep
their teeth clean and healthy.

222. Which diagnostic method of decay may NOT be appropriate for coronal smooth surface lesions?

a. Bitewing x-rays
b. DIAGNOdent
c. QFL
d. Clinical examination

b - DIAGNOdent diagnosis for decay is not appropriate for use on smooth coronal surface lesions.
It is for use only in suspected pit and fissure lesions on the occlusal surfaces.

223. Early childhood caries may NOT be associated with:

a. Breastfeeding
b. Use of bottled water in bottle preparation
c. Frequent juice or milk consumption
d. Putting child to bed with spill proof cup of milk

a - Breastfeeding does not place a child at a higher risk to develop early childhood caries. However,

TOP © 2015 - 2023 Tests.com


300
because teeth do erupt while many children are at a breastfeeding stage, the child’s teeth should
be cleaned immediately after nursing in order to reduce extended exposure to the milk.

224. An enlargement of the gingiva in a coronal direction with no apical migration of the junctional
epithelium is called:

a. Clinical attachment level


b. Mucogingival defect
c. Pseudopocket
d. Periodontal pocket

c - A pseudopocket or “gingival pocket” is an area where the gum tissue is enlarged coronally but
not apically, meaning there is no loss of attachment. It is a pseudo-pocket and does not imply a
sign of bone loss.

225. What classification of furcation involvement exists when the periodontal probe can be placed
into the furcation but not extend through to the other side?

a. Class I
b. Class II
c. Class III
d. Class IV

b - Class II furcation involvement is when the periodontal probe is able to reach into the furcation
area of the tooth that is probed, but it cannot extend through the furcation into the other side.
There may be some slight radiolucency in the furcation area of the radiographs.

226. When may a patient be referred to a periodontist?

a. When the hygienist decides the SCRP needs are too advanced
b. When the hygienist sees no improvement in SCRP therapy
c. When the dentist decides other therapies are not responding adequately
d. When the patient has not complied with the hygiene treatment plan of the RDH

c - Only the supervising dentist may make referrals to specialists such as a periodontist. The
hygienist is able to assess the health of the patient and provide a hygiene treatment plan to the
patient, but she may not refer him to a specialist unless the doctor states that is what should be
done.

TOP © 2015 - 2023 Tests.com


301
227. Which brushing method is a good first method for introducing to young children but not for
adults?

a. Bass method
b. Fones method
c. Charter’s method
d. Modified Stillman’s method

b - The Fones method is a good first method for introducing brushing techniques to a young child,
however it is not an efficient method for adults in regard to routine plaque removal. The Fones
method is also called the “circular” method and emphasizes plaque removal on the crown.

228. Which floss is the best option for patients with a tight contact area?

a. Waxed
b. Unwaxed
c. Tufted
d. Toothpick

a - Waxed floss is best for patients with tight interproximal contact areas, because it allows the
floss to slip more easily into the space.

229. Which device would be most efficient at removing plaque from an interproximal concavity area
with recession?

a. Superfloss
b. Electric toothbrush
c. Manual toothbrush
d. Interdental brush

d - Interdental brushes such as the Proxabrush are efficient at reaching concavities in the
interproximal areas when it comes to efficient plaque removal. They are also useful for wider
interdental spaces or orthodontic patients.

230. Rubber cup polishing should be used on a selective basis to remove:

a. Black line stain


b. Dental fluorosis
c. Amelogenesis imperfecta discoloration
d. Extrinsic tetracycline staining

a - Black line stain is the only extrinsic stain listed (there is no such thing as extrinsic tetracycline
staining). Rubber cup polishing should only be used on a selective basis and will only remove
superficial, extrinsic stains.

TOP © 2015 - 2023 Tests.com


302
231. Which stain has an etiology of ingestion of disturbance of the ameloblasts?

a. Orange
b. Brown
c. Dentinogenesis imperfecta
d. Hypoplasia

c - Dentinogenesis imperfecta is a white or brown discoloration of one or many teeth due to the
disturbance of ameloblasts during tooth formation.

232. A patient has heavy brown stain from tobacco use. Which of the following conditions would
contraindicate the use for an air polisher?

a. Diabetes
b. High blood pressure
c. Pacemaker
d. Ovesity

b - Patients with high blood pressure or those on diets that restrict their intake of sodium should
not have air polishers used on them due to the use of sodium bicarbonate powder which is used to
remove the stain.

233. Which mouthwash has an extended release that produces a therapeutic effect against bacteria?

a. Fluoride rinses
b. Phenol-containing rinses with essential oils
c. Chlorhexidine gluconate
d. Hydrogen peroxide

c - Chlorhexidine gluconate is a bactericidal agent that continues to release in the oral cavity
prolonging therapeutic effects against periodontal disease and oral bacteria.

234. Failing to remove dental prosthesis for routine cleansing may be harmful to the patient because
it can result in:

a. Excess stain buildup


b. Advanced bone loss
c. Weakening of the appliance
d. Candida albicans infections

d - Candida albicans infections (yeast infections) may occur as a result of a patient failing to
adequately clean their dental appliance on a regular basis. Stain may build up as well, but this is
not harmful to the patient as a yeast infection would be.

TOP © 2015 - 2023 Tests.com


303
235. When scaling an upper posterior implant, which of the following curette tip materials would
NOT be appropriate?

a. Stainless steel
b. Plastic
c. Graphite
d. Nylon

a - Stainless steel curettes or scalers should not be used on titanium implants as they may cause
scratching on the implant surface, resulting in bacterial accumulation.

236. Which type of electronic scaler uses air-pressure to create mechanical vibrations?

a. Magnetostrictive
b. Sonic
c. Piezoelectric
d. Air-polisher

b - Sonic scalers use air-pressure to cause mechanical vibrations used for debris removal.
Piezoelectric use expansion and contraction of crystals inside a housing unit and magnetostrictive
use magnetic strips. Air-polishers are not electric scalers.

237. Which of the following Gracey area-specific curettes would be appropriate to use on the mesial
of posterior teeth?

a. 3/4
b. 7/8
c. 11/12
d. 13/14

c - A Gracey 11/12 would be appropriate to use on the mesial of posterior teeth, as would a Gracey
15/16. A 13/14 would only be used on the distal surfaces. Other curettes listed would be for use in
the anterior teeth.

238. Which ultrasonic scaler has active surfaces only on the lateral edges?

a. Piezoelectric
b. Magnetostrictive
c. Sonic
d. Ultrasonic

a - Piezoelectric scalers use only the lateral edges as an active surface, because piezo scalers have a
linear tip motion. The face and back are not active.

TOP © 2015 - 2023 Tests.com


304
239. Which type of pocket is generated by an apical migration of the junctional epithelium?

a. Periodontal
b. Gingival
c. Suprabony
d. False

a - Periodontal pockets are the result of loss of junctional epithelium in an apical direction,
indicative of bone loss.

240. Which type of bony defect has the worst prognosis?

a. Two-wall defect
b. Three-wall defect
c. One-wall defect
d. All have the same prognosis

c - One-wall bony defects have the worst prognosis. The bone loss occurs interdentally leaving only
one surface remaining. A 3-wall defect has the best prognosis.

241. Which of the following conditions is reversible and does not cause loss of attachment fibers on
the cementum?

a. Mild periodontal disease


b. Gingivitis
c. Moderate periodontal disease
d. All periodontal diseases

b - Gingivitis is the initial stage of periodontal disease, but there is no loss of attachment and the
condition is reversible.

242. Which chronic disease appears as an erosive white area on the gingival mucosa?

a. Desquamative gingivitis
b. Pemphigoid
c. Lichen planus
d. ANUG

c - Lichen planus manifests itself as an erosive lichen planus of the gingival mucosa. There may be
erythematous areas interspersed with white striae.

TOP © 2015 - 2023 Tests.com


305
243. Which condition can be a result of an allergic reaction?

a. Pemphigus vulgaris
b. Necrotizing ulcerative gingivitis
c. Aggressive periodontitis
d. Desquamative gingivitis

d - Desquamative gingivitis may be due to a serious systemic disease, but it can also be caused by
an allergic reaction. It appears as sloughing tissues that leave red, raw areas in the mouth.

244. A patient presents with 5-7 mm pockets with red inflammation of the soft tissue. What
classification of disease is present?

a. Aggressive periodontitis
b. Moderate chronic periodontitis
c. Advanced chronic periodontitis
d. Slight chronic periodontitis

b - Moderate chronic periodontitis has pocket depths of 5-7 mm. More than 7 is considered
Advanced or aggressive, and less than 5 is only slight periodontitis.

245. What periodontal condition has gingival tissues that appear normal, with little plaque, and
usually occurs around first permanent molars and the permanent incisors?

a. Localized chronic periodontitis


b. Aggressive periodontitis
c. Juvenile periodontitis
d. Rapid progressive periodontitis

c - Juvenile periodontitis normally has no plaque, inflammation or bleeding present. Other


periodontal diseases do. Juvenile periodontitis typically affects the permanent anterior and first
molars, while other periodontal diseases typically begin manifesting themselves in the posterior
teeth.

246. What periodontal condition appears as punched out papillae covered in a pseudomembrane?

a. Periodontal abscess
b. Necrotizing ulcerative periodontitis
c. Prepubertal periodontitis
d. Advanced chronic periodontitis

b - Necrotizing ulcerative periodontitis manifests itself as “punched out” papillae and is typical in
HIV-infected patients or people with systemic immune disorders.

TOP © 2015 - 2023 Tests.com


306
247. Removal of diseased pocket epithelium during instrumentation is called _________.

a. Curettage
b. Debridement
c. Scaling
d. Root planing

a - Curettage is the removal of diseased epithelium within the periodontal pocket during
subgingival scaling. This may occur during normal curettage or done purposely.

248. The American Academy of General dentistry now advises that subgingival antibiotic therapy be
administered in periodontal pockets of _____ deep.

a. 3-5 mm
b. 4-6 mm
c. 5-6 mm
d. 7-8 mm

c - 5-6 mm pockets are now part of the new guidelines for the advised depth of periodontal
pockets that should receive antibiotic therapy during periodontal treatment. In the past, patients
would receive periodontal treatment and then possibly be administered subgingival antibiotic
therapy at a six-week evaluation appointment if needed.

249. Which site-specific antibiotic works over a two-week period?

a. Minocycline
b. Doxycycline
c. Tetracycline
d. Chlorhexidine

a - Minocycline (Arestin) inhibits protein synthesis of bacterial cells and is active to work over a
two-week period after administration.

250. What surgical intervention corrects bony defects to reshape the alveolar process?

a. Mucogingival surgery
b. Osseous surgery
c. Excisional surgery
d. Periodontal flap surgery

b - Osseous surgery reshapes the alveolar process to attain a better physiologic form around the
teeth. This may be done around defects involving periodontal disease.

TOP © 2015 - 2023 Tests.com


307
251. What incisional surgery exposes underlying bone and removes diseased tissues, including
calculus removal?

a. Excisional surgery
b. Mucogingival surgery
c. Periodongal flap surgery
d. Periodontal surgery

c - Periodontal flap surgery retracts the gingiva so that bone can be modified and diseased tissues
removed. It also allows for residual calculus removal before the flap is sewn back into place.

252. MATCH the following types of bone grafts with the source of the bone:

___ Autograft a. Cadaver bone


___ Allograft b. Synthetic bone
___ Xenograft c. Bone taken from the patient
___ Alloplast d. Cow or pig bone

c. Autograft
a. Allograft
d. Xenograft
b. Alloplast

253. What procedure involves removal of half of the tooth, including a portion of both the crown and
root?

a. Root bisection
b. Root resection
c. Root hemisection
d. Apicoectomy

c - Root hemisection is the removal of half of a tooth, including portions from both the crown and
the root. This procedure will need to be followed up with endodontic treatment.

254. What is the material of choice for dental implant fabrication?

a. Stainless steel
b. Ceramic
c. Porcelain
d. Titanium

d - Titanium is the material of choice for the fabrication of implants. The titanium has a high rate of
osseointegration with the bony tissue. Implants may sometimes be coated with ceramic coatings.

TOP © 2015 - 2023 Tests.com


308
255. Approximately _____ of bacteria that causes maloder of the breath originates in the oropharynx
region.

a. 10%
b. 25%
c. 90%
d. 100%

c - Approximately 90% of malodor bacteria reside in the oropharynx region, such as on the tongue,
periodontal conditions or around the tonsils. Systemic conditions account for the other 10%.

256. What is the most important risk factor related to periodontal disease?

a. Genetic predisposition
b. Diabetes
c. Plaque index
d. Tobacco use

d - The use of tobacco products is the most important risk factor related to periodontal disease.
The chemical irritants in tobacco products traumatize the periodontal tissues and inhibit healing.

257. Which hormonal condition has NOT been proven to increase symptoms of gingivitis?

a. Menstruation
b. Puberty
c. Pregnancy
d. Use of oral contraceptives

a - Menstruation has not been proven to have an increased risk of gingivitis symptoms. It is
actually thought that gingivitis symptoms decrease during ovulation and pre-menstruation, a time
when hormones typically peak.

258. Most periodontal pathogens are __________ and _________.

a. gram negative, nonmotile


b. gram negative, motile
c. gram positive, nonmotile
d. gram positive, motile

a - The majority of periodontal pathogens are gram negative and nonmotile, except for one or two
which are motile (but also gram negative).

TOP © 2015 - 2023 Tests.com


309
259. Which of the following clinical symptoms is a sign of occlusal trauma?

a. Widening of the PDL


b. Wear facets
c. Tilted teeth
d. All of the above

d - All of the above are signs and symptoms of occlusal trauma. Not only does trauma such as
bruxism cause wear facets, but it also widens the periodontal ligament and may cause the teeth to
shift or even become mobile.

260. The first fibers lost during periodontal disease are the periodontal ligament fibers. Transseptal
fibers become involved at the approach of moderate bone loss.

a. Both statements are true


b. Both statements are false
c. The first statement is true, the second statement is false
d. The first statement is false, the second statement is true

c - The first statement is true. PDL fibers are the first fibers lost during periodontal disease.
However, transseptal (interdental) fibers are not lost until such severe disease has occurred that
the tooth is about to be lost.

261. A narrow, slit like cleft in the marginal gingiva is called a ________ cleft.

a. Stillman’s
b. McCall’s
c. Bass
d. Recession

a - Stillman’s clefts are slit-like clefts that occur vertically in the marginal gingiva. They are usually
caused by atrophy of the facial gingiva.

262. What is the most reliable sign to detect inflammation in the periodontal pocket?

a. Erythema
b. Edema
c. Bleeding on probing
d. McCall’s festoons

c - Bleeding on probing is the most accurate sign to diagnose active inflammation in the
periodontal pocket. Healthy gums do not bleed, but unhealthy gums may appear to have no
inflammation or redness (as in smokers).

TOP © 2015 - 2023 Tests.com


310
263. What is the first sign on a radiographic assessment that there is a periodontal breakdown?

a. Horizontal bone loss


b. Vertical bone loss
c. Enlarged PDL
d. Fuzziness of the alveolar crests

d - Fuzziness of the alveolar crests as evidenced on an x-ray is the first radiographic sign that there
is a breakdown of the periodontium. As further breakdown occurs it will be noticed by vertical or
horizontal bone loss on the dental x-ray.

264. Which fluoride should not be applied to people with composite or porcelain restorations?

a. Varnish
b. Acidulated phosphate
c. Neutral sodium
d. No fluoride should be used

b - Acidulated phosphate fluoride should not be applied to patients with crowns, bridges, veneers
or composite fillings as it may cause a layer of demineralization on the tooth surface.

265. Fluoride varnish contains ______% of ______ fluoride.

a. 5; sodium
b. 2; sodium
c. 2 neutral
d. 0.4; stannous

a - Fluoride varnish uses 5% sodium fluoride that adheres onto the tooth surfaces and releases
fluoride ions for 3 to 4 hours following the application.

266. What is the optimal level of fluoride in a city water supply?

a. 0.1 ppm
b. 0.01 ppm
c. 1 ppm
d. 10 ppm

c - 1ppm is the optimal level of fluoride in a municipal water supply. This level will be less in areas
that are warmer, and more in areas that are cooler (due to the effect temperature has on the
amount of water people drink.)

TOP © 2015 - 2023 Tests.com


311
267. A lethal dose of fluoride is how many grams?

a. 4g
b. 5-10g
c. 15-20g
d. Over 25g

b - 5-10g of fluoride taken at one time is the lethal dose of fluoride. For a small child it would be
less, at approximately 0.5-1.0g.

268. What is the toxic dose of fluoride per kg for a child?

a. 1mg
b. 5mg
c. 8mg
d. 10mg

b - 5mg/kg is the toxic dose of fluoride for a child. For an adult it is 8 mg/kg. When giving a child
fluoride in the office using a foam tray, no more than 4mg of gel should be placed in the tray.

269. Which of the following is NOT true in regard to fluoride?

a. It may cause birth defects


b. Topical application does not cause fluorosis
c. Fluorosis can occur in the bone
d. Toxicity may cause cardiac arrest

a - Fluoride use does not cause birth defects. Toxicity may cause cardiac arrest, respiratory distress
and other symptoms. Heavy fluoride consumption such as found in natural water supplies can
cause fluorosis of the teeth and bones (but not when delivered topically).

270. When placing sealants, acid etch should be left on the surface of the tooth for ___________.

a. 10-15 seconds
b. 15-60 seconds
c. 60-90 seconds
d. 2 minutes

b - Depending on manufacturer recommendations, acid etch is typically left on the tooth between
15-60 seconds for proper conditioning of the dental tubules.

TOP © 2015 - 2023 Tests.com


312
271. Self-cured sealants usually set within _______.

a. 30 seconds
b. 45 seconds
c. 2 minutes
d. 5 minutes

c - 2 minutes is the most appropriate answer. Typically, self-cured sealant material will set with 1-3
minutes.

272. Which is NOT appropriate for tooth preparation when placing a sealant?

a. Air polishing
b. Rubber cup polish with prophy paste
c. Rubber cup polish with pumice
d. Prophy brush polish with pumice

b - Prophy paste should not be used when preparing a tooth for sealant preparation because the
paste contains fluoride, which will block tubules and prevent sealant retention.

273. If a tooth becomes wet due to saliva contact during sealant preparation, what is the next correct
step?

a. Use air/water syringe to re-rinse and dry the tooth


b. Use air/water syringe to dry the tooth
c. Re-etch for the full amount of time
d. Re-etch for a shorter amount of time

d - Re-etching the tooth for a shorter amount of time is enough to re-condition the dental tubules
after salivary contamination during a sealant prep. Simply drying the tooth is not enough because
saliva has come into contact with the pores of the tooth and will disrupt retention of the sealant.

274. Which is NOT true in regard to sealant material?

a. It may be clear or white


b. It may be tinted
c. It may consist of filled resins
d. It must be gently dabbed on the tooth.

d - Sealant material does not need to be dabbed onto the tooth. It may be applied with a syringe
tip or brush, and spread where it is needed to fill in the necessary pits and fissures.

TOP © 2015 - 2023 Tests.com


313
275. Light curing typically occurs within ________.

a. 15-seconds
b. 20-30 seconds
c. 45-60 seconds
d. 90-seconds

b - Sealant material typically cures within 20-30 seconds of exposure to the curing light. Always be
sure to check the sealant for under-curing after the procedure is finished.

276. Which type of sealant material will wear down naturally and does not require occlusal
adjustment?

a. Filled resin
b. Unfilled resin
c. Tinted
d. Autopolymer

b - Unfilled resin sealants do not require occlusal adjustment with a finishing bur. They will wear
down naturally. However, filled resin sealants do require occlusal adjustment because it will not
wear down through chewing or occlusal forces.

277. Which type of sealant will release fluoride, protecting the tooth by aiding in remineralization?

a. Calcium phosphate
b. Hydrophobic
c. Unfilled resin
d. No sealants release fluoride

a - Calcium phosphate, or fluoride-releasing type sealants release fluoride during the life of the
sealant and help promote remineralization, decreasing tooth decay.

278. Which nerve fibers are thought to conduct rapid hypersensitivity?

a. A-delta
b. B-delta
c. C-delta
d. D-delta

a - A-delta nerve fibers line the pulp and conduct stimuli rapidly. Stimulation comes through the
dentinal tubules and moves odontoblasts, accessing the A-delta nerves at a fast pace.

TOP © 2015 - 2023 Tests.com


314
279. Polishing the teeth for at least ____ with pumice may remove 3-4 micrometers of enamel.

a. 1 minute
b. 3 minutes
c. 5 minutes
d. 7 minutes

b - Polishing times of at least 3 minutes can cause removal of the enamel, as much as 3-4
micrometers deep. Over time a significant amount of enamel would be removed. Take care using
selective polishing.

280. Bristle brushes may be used for stain removal on:

a. Occlusal surfaces
b. Gingival margins
c. Exposed dentin
d. Facial surfaces

a - Bristle brushes are designed to use for stain removal in the occlusal surfaces of teeth. They may
be too abrasive to use near the gingival margins or on dentin, and would not work as well as a
rubber cup prophy angle on facial surfaces.

281. Once an amalgam filling is placed, polishing must wait _______ before performed on the tooth.

a. 30 minutes
b. 4 hours
c. 24 hours
d. 48 hours

c - 24 hours must lapse after an amalgam filling is placed prior to the tooth undergoing polishing,
because the amalgam must continue to set.

282. Glass ionomers are typically placed for Class V restorations and may seep fluoride out of the
final restoration for how long after placement?

a. 6 months
b. 12 months
c. 2 years
d. 5 years

c - 2 years is the approximate length of time that glass ionomer restorations seep fluoride after
placement. Teeth with glass ionomer restorations also typically have lower s. mutans on the
surface than teeth that do not.

TOP © 2015 - 2023 Tests.com


315
283. What type of cement is widely used in dentistry for bonding, temporary fillings and temporary
cement?

a. Glass ionomer
b. Zinc phosphate
c. Zinc polycarboxylate
d. Zinc oxide and eugenol

d - Zinc oxide and eugenol cement is widely used in dentistry and has many purposes including
impressions, temporary cement or fillings, bonding and even periodontal dressing.

284. What is a common material used for impression and is an irreversible hydrocolloid?

a. Silicone
b. Agar hydrocolloid
c. Alginate hydrocolloid
d. Polysulfide

c - Alginate hydrocolloid is a commonly used material for dental impressions, especially for the use
of creating study models, whitening trays and retainers. Agar is a reversible hydrocolloid.

285. What controls the set time for an alginate impression?

a. Temperature of the water


b. Temperature of the room
c. Age of the material
d. All alginate sets at the same rate

a - Temperature of the water determines how fast or slow an alginate impression sets. The warmer
the water, the shorter the set time required by the alginate.

286. What model material is used to make casts and dies of the teeth?

a. Alginate
b. Plaster
c. Stone
d. Die Stone

d - While stone can be used to make casts, die stone is for use on both casts and dies of teeth. It
requires even less water than stone and produces a stronger material than stone.

TOP © 2015 - 2023 Tests.com


316
287. Study models typically set within __________.

a. 30 minutes
b. 45-60 minutes
c. 1 1/2 − 2 hours
d. 4 hours

b - Most study models set within 45-60 minutes after being poured. This is approximately how long
it takes for the exothermic reaction to occur and for the material to harden.

288. When mixing gypsum products for study models, what ratio of powder to water is used?

a. 1:1
b. 2:3
c. 3:2
d. 2:1

a - A 1:1 ratio of powder to water is used when gypsum products are being mixed in the laboratory
for study models. Not using enough water or using too much water will make the mixture
inconsistent and unable to manipulate while pouring.

289. What is defined as the absence of prudent care that a patient should receive in the dental office?

a. Malpractice
b. Negligence
c. Fraud
d. Intentional torts

b - Negligence is when a professional does not provide prudent care to a patient in the dental
office, but is careless. It differs from Intentional torts in that actual injury or harm was not a
deliberate act; It differs from malpractice in that malpractice fails to perform up to professional
standards which results in injury.

290. Training dental personnel to perform dental hygiene duties is known as ___________.

a. Preceptorship
b. Extended duties
c. Fraud
d. On the job training

a - Preceptorship is when dental personnel are trained to perform dental hygiene duties in an
office setting. This is currently limited to only very few areas and is not widely accepted.

TOP © 2015 - 2023 Tests.com


317
291. Which of the following is NOT true regarding HIPAA?

a. Electronic health information is included in the privacy rules


b. Patient’s have access to their medical records
c. Group health plans cannot deny an application based on health status
d. It does not protect workers who lose their jobs

d - HIPAA does protect workers who lose their jobs by protecting their health insurance coverage.

292. What level of care would periodontal treatment be considered to be a part of?

a. Tertiary level of prevention


b. Primary level of prevention
c. Secondary level of prevention
d. Moderate level of prevention

c - The secondary level of prevention includes periodontal treatment as well as restorations such
as fillings and crowns. This level of prevention is for correcting disease processes that have begun
in the oral cavity.

293. What two answers describe the deliberate misdirection of the truth to deceive a person such as
a patient?

a. Defamation
b. Fraud
c. Misrepresentation
d. Malpractice

b and c - Fraud and Misrepresentation are essentially the same action, which causes deliberate
misdirection of what is true, such as causing harm to someone or representing oneself falsely.

294. When a patient is given a treatment plan and chooses not to have the treatment completed, it is
called:

a. Informed refusal
b. Informed consent
c. Refusal of treatment
d. Neglect

a - Informed refusal is when the patient has been educated and informed of their proposed
treatment plan needs and what will happen if treatment is refused, yet still refuses the treatment.
Document refusals thoroughly in the patient chart.

TOP © 2015 - 2023 Tests.com


318
295. A signature on the treatment plan agreeing to undergo the proposed dental treatment is called
______.

a. Documented treatment plan


b. A legal contract
c. Informed consent
d. Documentation

c - Informed consent is when the patient has agreed to and signed the necessary treatment plan,
as well as being informed of the risks, procedures and what will happen if treatment is not
completed.

296. What act is the actual infliction of harm to a person?

a. Assault
b. Battery
c. Negligence
d. Unintentional tort

b - Battery is the actual infliction of harm or injury to a patient, and puts them in danger.

297. What type of contract does not express all of the terms in words?

a. Implied contract
b. Express contract
c. Legal contract
d. Partial contract

a - An implied contract is a contract that has some portions of the contract which are not
expressed in terms of words.

298. What type of defamation is written?

a. Slander
b. Personal slander
c. Libel
d. Misrepresentation

c - Libel is a defamation of an individual and occurs in a written form. If defamation is verbal, it is


considered slander.

TOP © 2015 - 2023 Tests.com


319
299. Contract law is a form of ______ law.

a. Civil
b. Tort
c. Criminal
d. Judicial

a - Civil law encompasses contract law, and addresses parties during the failing or breaking of a
contract by one of the parties.

300. What judicial measure is used to determine whether a dental provider has exercised reasonable
care to his or her patient?

a. Negligent practitioner measure


b. Reasonably prudent practitioner measure
c. Malpractice measure
d. Licensed practitioner measure

b - The reasonably prudent practitioner measure is used by legal authorities to determine if a


provider has provided reasonable care for their patient or client.

301. What type of consent is given in writing or verbally?

a. Informed refusal
b. Informed consent
c. Implied consent
d. Expressed consent

d - Expressed consent may be written or verbal, but documenting the consent in writing helps to
protect the practitioner and patient at a greater level.

302. In most states a malpractice suit must be filed within ______ of the time from when the
wrongful act was committed.

a. 1 year
b. 2 years
c. 5 years
d. 10 years

b - 2 years is the typical length of time in which most states require a malpractice suit to be filed
from when the wrongful act was committed.

TOP © 2015 - 2023 Tests.com


320
303. Which government agency oversees the use of MSDS sheets?

a. OSHA
b. HIPAA
c. CDC
d. ADA

a - OSHA, the Occupational and Safety Health Administration, oversees MSDS sheets within the
dental office. These sheets are to help protect staff should occupational exposure occur.

304. According to HIPAA, all of the following statements are correct EXCEPT:

a. Patients must have access to their records upon request within at least 30 days
b. An office may charge the patient a fee for duplicating their records
c. Patients may file complaints concerning their privacy to the US Dept of Health and
Human Services
d. At least one staff member in each office must be trained on HIPAA protocols

d - All staff members in a dental office must be trained to meet HIPAA regulations concerning
patient privacy policies. Each office must document this training.

305. Which factor is one of four that would make a dental hygienist liable for negligence?

a. Not undertaking the patient for care


b. Failing to breach a duty owed to the patient
c. Harm to the patient must be proved
d. Harm does not have to be related to breach of duty

c - Harm to the patient must be proved, along with 3 other factors- undertaking the patient for
care, breaching a duty owed to the patient, and the breach of duty causing the harm. All 4 factors
must be in place prior to the hygienist being held liable for negligent care.

306. Which two considerations can spare a dentist from judgment related to negligence?

a. Patient knowingly failing to inform the dentist of a heart condition


b. Patient not taking antibiotic premedication prescribed by their physician
c. Informed consent signed by both the patient and the doctor
d. The patient having all procedures completed by the dental hygienist

a and b - If a patient knowingly fails to inform their dentist that they have a heart condition, or if
the patient knowingly does not take their antibiotic premedication prescribed by their primary
care physician prior to dental therapy, the dentist may be spared from a judgment of negligence.
In both cases the patient failed to comply with medical consultation or identify medical conditions
that the dentist should have been informed of.

TOP © 2015 - 2023 Tests.com


321
307. Parental consent must be:

a. Given for minors


b. Given for individuals that are not mentally competent
c. Written or verbal
d. All of the above

d - All of the above are correct in regards to parental consent to the care of their child, whether
they are a minor or not mentally competent. It is best if consent is documented in writing.

308. A patient that is experiencing angina should be given _________.

a. Tylenol
b. Nitroglycerin
c. Aspirin
d. NSAIDS

b - Nitroglycerin is used in prevention or to treat a patient with angina. The medication comes in a
spray or tablet form. It should be placed or sprayed under the tongue.

309. What bacterium is used when testing the effectiveness of an autoclave?

a. Bacillus stearothermophilus
b. S. Mutans
c. Lactobacillus
d. Actinomyces

a - Bacillus stearothermophilus is a gram-positive bacterium that is highly resistant to the


sterilization process. It is used in the sterilization ampules to test the effectiveness of the
sterilization process of an autoclave.

310. All of the following statements, except one, accurately describe the advantages of dry heat
sterilization. Which statement is not accurate as far as dry heat sterilization?

a. It does not corrode instruments.


b. It is used for instruments that cannot be autoclaved.
c. It is affective on oils and powders.
d. It is economical.

d - The dry heat sterilization process is long and not economical due to the restrictions on what
can and cannot be used in the process. In order to kill the most resistant spores, a dry-heat
sterilizer must bring its load to 170°C and hold it at that temperature for one hour. This method
uses a lot more energy and time than a steam autoclave. However, this is a great way to sterilize
oils and powders that cannot get wet.

TOP © 2015 - 2023 Tests.com


322
311. During CPR on an infant, what artery is used to take a pulse?

a. Carotid
b. Brachial
c. Radial
d. Femoral

b - The brachial artery is used when taking the pulse of an infant. It is located inside the upper arm,
between the elbow and the shoulder

312. What is a Naber’s probe used to measure?

a. Abfraction
b. Recession
c. Furcation
d. Periodontal pockets

c - A Naber’s probe is used to measure the extent of furcation on both the buccal and lingual of a
tooth.

313. Which tooth surface is least susceptible to plaque formation?

a. Maxillary palatal surfaces


b. Maxillary buccal posterior
c. Proximal surfaces
d. Mandibular lingual anterior

a - The maxillary teeth harbor less bacteria than the mandibular teeth. The palatal surfaces are
smooth surfaces and see activity from the tongue, which helps clear off bacteria from the smooth
surfaces.

314. What are the two most common areas for supraginvial calculus?

a. Maxillary lingual anterior and mandibular buccal posterior


b. Maxillary buccal posterior and maxillary lingual posterior
c. Maxillary buccal posterior and mandibular lingual anterior
d. Maxillary lingual posterior and mandibular lingual anterior

c - The maxillary buccal posterior and the mandibular lingual anterior are the most common areas
of supragingival calculus, because of their location to the parotid, submandibular and sublingual
glands.

TOP © 2015 - 2023 Tests.com


323
315. What type of toothbrush should be recommended to a patient with 3rd molars, rotated teeth
and furcation involvement?

a. Rubber tip
b. End-tufted
c. Interdental
d. Sulcabrush

b - The end-tuft brush has a small, round head with a few tufts of toothbrush bristles. It is designed
to be small to fit in those hard to reach areas, such as 3rd molars and furcations.

316. What is the most widely used and accepted tooth brushing technique?

a. Charters
b. Stillmans
c. Bass
d. Fones

c - The bass method or modified bass method is the most common and most widely accepted
tooth brushing technique, because it is suitable for everyone to use. Your toothbrush is placed at a
45-degree angle towards your gingival margin. This method cleans the sulcus and the cervical third
area. It can be taught to periodontal maintenance patients and regular recall patients.

317. What is the active ingredient in anti-calculus toothpaste?

a. Stannous fluoride
b. Sodium fluoride
c. Potassium nitrate
d. Pyrophosphate

d - Pyrophosphate is the ingredient in most anti-calculus toothpastes. It is designed to stop the


formation of calcium-phosphates salts from forming on your teeth.

318. All of the following are reasons why the use of hydrogen peroxide is contraindicated in regular
use, except:

a. Sponginess of gingiva
b. Root hypersensitivity
c. Staining
d. Demineralization

c - Staining is not a contraindication of hydrogen peroxide. In fact, hydrogen peroxide is a major


ingredient in teeth whitening products. However, as a mouth rinse, it is contraindicated for regular
use, due to its potential harmful effects of the gingiva and demineralization of the teeth.

TOP © 2015 - 2023 Tests.com


324
319. What is the recommended immersion time of instruments using the liquid chemical
glutaraldehyde as the solution?

a. 10 hours
b. 30 minutes
c. 5 hours
d. 1 hour

a - 10 hours is the recommended time of submersion of instruments. You would use this
technique on heat sensitive instruments. All instruments must be rinsed with sterile water prior to
their use.

320. The facial profile of a Class II occlusion is mesognathic. The facial profile of a Class I occlusion is
retrognathic.

a. Both statements are true


b. Both statements are false.
c. First state statement is true. Second statement is false.
d. First statement is false. Second statement is true.

b - Mesognathic is a Class I profile, meaning everything is properly developed and in correct


occlusion. Class II is retrognathic and means the maxilla or mandible is underdeveloped.

321. When the alveolar bone fuses directly to the root surface of a tooth:

a. Ankylosis
b. Resportion
c. Periodontal disease
d. Infraverted

a - Ankylosis is when the alveolar bone fuses directly with the root surfaces, leaving no area for
ligament space. Resorption of the cementum and root dentin is a result of ankylosis.

322. What is the term used for the phenomenon or natural tendency of teeth to move out toward the
lips?

a. Distal drift
b. Mesial drift
c. Lingual drift
d. Buccal drift

b - Mesial drift is a natural occurrence that causes our teeth to move toward our lips or to move
mesial. This is what is attributed to causing crowding. It is said to be caused by torsion in the jaw
and deflection of teeth in their tooth sockets.

TOP © 2015 - 2023 Tests.com


325
323. Which statement about bioburden is not accurate?

a. It is bacteria on a contaminated instrument.


b. Saliva is not considered bioburden.
c. Blood on an instrument is considered bioburden.
d. Saliva on a reusable prophy angle is considered bioburden.

b - Saliva, blood and any other bodily fluid is considered bioburden. The bioburden needs to be
pre-cleaned before the disinfection process. Otherwise, this can cause cross-contamination.

324. Which of the following is not considered a semi-critical instrument in regards to infection
control?

a. Dental bur
b. Dental mirror
c. Amalgam condenser
d. Headpiece

a - The dental bur is considered critical in regards to infection control, because it is used to
penetrate soft tissue or bone. Items are also considered critical if they come into contact with
blood and should be sterilized after each use.

325. As a cleaning agent, alcohol is effective against all of the following, except:

a. Bacterial spores
b. Gram-positive bacteria
c. Hepatitis B
d. Fungus

a - Alcohol is not effective against cleaning bacterial spores. Bacterial spores are eliminated by high
pressures and heat, such as the autoclave.

326. Where should intermediate disinfectants be used?

a. Dental instruments
b. Floor
c. Dental chair
d. Hand piece

c - The dental chair is considered an intermediate surface, because it is not an area of high
contamination and is able to be sterilized by the autoclave or statim.

TOP © 2015 - 2023 Tests.com


326
327. When looking at a patient’s mouth from a lateral view, the cusp of the mandibular posterior
teeth and the incisal edge of anterior teeth form a(n) __________.

a. Centric occlusion
b. Ideal occlusion
c. Malocclusion
d. Curve of Spee

d - The Curve of Spee, or Spee’s curvature, is a curved line that is formed when a patient is in
occlusion and the occlusal line curves upward towards the terminal molar.

328. Which of the following may cause an anterior open bite?

a. Thumb sucking
b. Tongue trust
c. Ankyloglossia
d. All of the above

d - All listed items can cause an open bite. The open bite can be caused by hereditary factors or by
repeated patterns, such as digit sucking or thrusting of the tongue. It can be corrected through
orthodontics.

329. A tooth that is labial to its normal position is considered to be ____________.

a. Labialverted
b. Lingualverted
c. Infraverted
d. Buccoverted

a - Labialverted teeth lay labial or towards the lips. Buccoverted is in the posterior and lays
towards the buccal or cheek of the mouth.

330. What tooth is used to determine occlusal classification if there are no first molars present?

a. Second molars
b. Canine
c. Laterals
d. Permolars

b - Canine teeth are used to determine the occlusion classification when molars are not present.
However, when determining the occlusion classification, both ideally should be observed.

TOP © 2015 - 2023 Tests.com


327
331. In terms of ergonomics, what is the correct sequence of attention when a dental hygienist
establishes her working position? Put the following in the correct order.

a. The patient in a dental chair


b. Dental hygienist’s position in relation to the dental chair
c. Dominant hand
d. The dental chair light
e. Non-Dominant hand

The correct order is: b, a, d, e, c

The correct sequence when establishing your working position is:


(1) Dental hygienist’s position in relation to the dental chair
(2) The patient in a dental chair
(3) The dental chair light
(4) Non-Dominant hand
(5) Dominant hand

332. Which of the following is not considered part of the neutral body position?

a. Head tilted 1-15 degrees


b. Tip of patients nose above the clinician’s elbow
c. Flat feet on the floor
d. Shoulders relaxed

b - The tip of the patient’s nose should be below the clinician’s elbow. This allows for relaxed
shoulders and elbows and prevents fatigue.

333. What is the correct clock seated position for the right/left hand operator?

a. 8:00-1:00/11:00-4:00
b. 9:00-2:00/12:00-5:00
c. 7:00-12:00/10:00-3:00
d. 6:00-11:00/9:00-2:00

a - The correct clock seated position for the operator allows for proper ergonomics. It prevents
fatigue and injury.

334. When an operatory uses incorrect ergonomics, he or she is at risk for what type of injury?

a. Musculoskeletal
b. Muscle
c. Skeletal
d. Nerve

TOP © 2015 - 2023 Tests.com


328
a - Musculoskeletal injuries are when the muscles and the skeletal systems are injured together. It
is caused by repeated incorrect dental hygiene ergonomics and is often seen in the neck, back and
hands/wrists of the operator.

335. There are a higher percentage of microbes in supragingival plaque. There is a lower percentage
of microbes in subgingival plaque.

a. Both statements are true.


b. Bother statements are false.
c. First statement is true. Second statement is false.
d. First statement is false. Second statement is true.

b - There is a higher percentage of microbes subgingivally, because of the shift to anaerobic and
gram negative bacteria. Supragingival plaque consists mostly of aerobic bacteria and yeast.

336. What is the duration of an acidic attack in the mouth when a person eats or drinks acidic foods?

a. 5 minutes
b. 20 minutes
c. 1 hour
d. 30 minutes

b - Immediately after eating or drinking acidic foods, your mouth is attacked by the acid. This
attack lasts for 20 minutes, until our mouth can neutralize the pH.

337. What ethnicity has a higher prevalence of periodontal disease?

a. Caucasian
b. Hispanic
c. African-American
d. Pacific Islander

c - African Americans have a higher prevalence of periodontal disease, with Hispanics coming in a
close second. It is also more prevalent in men.

338. How can the endocrine system affect the periodontal health of a patient?

a. Heart disease
b. Diabetes mellitus
c. Decrease in liver function
d. It does not affect periodontal health.

b - The endocrine system that is not controlled can cause hyperglycemia or high blood sugar levels.
This can result in an inability to fight infection or impair PMN chemotaxis. Research data suggests

TOP © 2015 - 2023 Tests.com


329
that diabetes is a risk factor for gingivitis and periodontitis.

339. Which of the following procedures can be conducted on a patient taking bisphosphonates?

a. Periodontal surgery
b. Root canal
c. Amalgam filling
d. Extraction
e. All of the above

c - Fillings can be done when a patient is on bisphosphonates. However, more invasive procedures
should be avoided. It is due to the fact that bisphosphonates inhibit osteoclast bone resorption or
necrotic tissue. It affects healing and disrupts blood supply.

340. A person that is HIV positive is more vulnerable to what type of oral disease?

a. Aggressive necrotizing periodontal disease


b. Angular chelitis
c. Cavities
d. Oral Cancer

a - HIV positive patients are more susceptible to aggressive necrotizing periodontal disease (NUG
or NUP), because of their lack of immune response and the impairment of the CD4 glycoprotien.

341. Which of the following is not a systemic risk factor for periodontal disease?

a. Diabetes type II
b. Coronary heart disease
c. Geographic tongue
d. Asthma

c - Geographic tongue is an inflammatory condition of the mucus membranes of the tongue. The
causes of this condition are unknown. It is not a systemic disease nor can it increase your risk
factor for periodontal disease.

342. Which type of tissue provides a pathway from the mouth to the vascular system for bacteria?

a. Inflamed
b. Pedunculated
c. Keratinized
d. Ulcerated

d - Ulcerated tissue provides a direct pathway to the vascular system and could cause a patient to
go into sepsis.

TOP © 2015 - 2023 Tests.com


330
343. What is the ideal angle of instrumentation for calculus removal?

a. 60-80 degrees
b. 0-40 degrees
c. 20-50 degrees
d. 40-60 degrees

a - 60-80 degrees is ideal for calculus removal. This angulation is in regards to the face of the
instrument to the tooth.

344. The two types of activation when instrumenting include wrist and digital. Digital activation is
only used when removing calculus.

a. Both statements are true.


b. Both statements are false.
c. First statement is true. Second statement is false.
d. First statement is false. Second statement is true.

c - Wrist action is used for calculus removal and digital action is used when probing, exploring and
using sonic scalers.

345. All of the following factor in stroke selection, except:

a. Position and size of gingiva


b. Pocket depth
c. Size and shape of instrument
d. Patient position

d - Your patient’s position should not be a factor in the correct stroke selection. The patient should
be repositioned or positioned in the correct way that allows for effective and ergonomically
correct instrumentation.

346. What type of stroke is used on the facial and lingual surfaces of anterior teeth?

a. Oblique
b. Horizontal
c. Vertical
d. Proximal

a - The oblique stroke is used on the facial and lingual surfaces of anterior and posterior teeth. It is
not used at the line angles.

TOP © 2015 - 2023 Tests.com


331
347. When cleaning a denture, it is recommended that a cylinder shaped denture brush be used. A
regular toothbrush can also be used.

a. Both statements are true.


b. Both statements are false.
c. First statement is true. Second statement is false.
d. First statement is false. Second statement is true.

c - It is recommended that a cylinder shaped denture brush be used on dentures when cleaning
the inner curves. A rectangular shaped denture brush is recommended for the occlusal surfaces. A
regular toothbrush would be an ineffective cleaning option due to its shape.

348. All of the following are advantages of using alkaline hypochlorite except:

a. Available as a liquid and a solid


b. Broad spectrum of antimicrobial activity
c. Removes biofilm with organic matter present
d. Removes stains

c - Alkaline hypochlorite is inactive in the presence of organic matter. If a surface you wish to
disinfect has organic matter, it needs to be wiped away and then disinfected using the instructions
of the manufacturer.

349. Which of the following is considered a secondary prevention of disease control?

a. Fluoride
b. Calculus removal
c. Mechanical/chemical biofilm control
d. Nutrition discipline

b - Calculus removal is considered secondary in the prevention of disease control. Restorations


would also be considered secondary.

350. In what order is bacteria introduced into the caries process?

a. S. mutans, lactobacilli, actinomyces


b. S. mutans, actinomyces, lactobacilli
c. Actinomyces, lactobacilli, S. mutans
d. Lactobacilli, S. mutans, actinomyces

a - S. mutans is the bacteria that is found during the developmental stage of the caries process.
Lactobacilli are usually found in the dentin during the process. Actinomyces are opportunistic and
colonize after the initial infection.

TOP © 2015 - 2023 Tests.com


332
351. Toothbrush abrasion can cause recession. Bacterium does not cause recession.

a. Both statements are true


b. Both statements are false
c. First statement is true. Second statement is false.
d. First statement is false. Second statement is true.

c - Toothbrush abrasion can cause trauma to the tissue and cause the tissue to retreat. This
exposes the root surface of the tooth and makes it susceptible to decay or sensitivity. In many
cases, bacterium is the cause of recession, as in periodontitis.

352. What drug is used to treat acute necrotizing ulcerative gingivitis?

a. Penicillin
b. Tetracycline
c. Metronidazole
d. Clindamycin

c - Metronidazole is an antibiotic used to treat bacterial infections. It is the most commonly


prescribed medication for ANUG.

353. The two main types of exudate are serous and purulent. Both contain white blood cells.

a. Both statements are true.


b. Both statements are false.
c. First statement is true. Second statement is false.
d. First statement is false. Second statement is true.

a - Both serous and purulent are types of exudate. Serous exudate has a few white blood cells and
purulent contains many white blood cells and also plasma.

354. What happens when a drug has a high first past effect?

a. The patient requires a smaller dose.


b. The patient requires a larger dose.
c. The patient should not be given the medication.
d. The patient should seek medical attention.

b - When a patient has a high first past effect, it means that the dose of the medication given was
not large enough to produce an effect. A larger dose must be given.

TOP © 2015 - 2023 Tests.com


333
355. What is the teratogenic effect?

a. Relationship between maternal drug use and congenital abnormalities


b. Hypersensitivity response to a particular drug
c. Dose related reaction to a particular drug
d. Dose related reaction that is not part of a desired therapeutic outcome

a - Teratogenic effect directly relates to the effects that drug use and medications can have on a
developing baby. Continued use can cause abnormalities. An example of this would be fetal
alcohol syndrome in newborns.

356. What is contraindicated for a patient that is under or has been under the influence of cocaine in
the last 24 hours?

a. X-rays
b. Antibiotics
c. Epinephrine
d. Nothing in contraindicated

c - Cocaine causes an epinephrine release. It would be contraindicated to give more epinephrine to


a patient with high levels of epinephrine already.

357. What does a patient known to be an intravenous drug user need to do for the dental treatment
of an infection?

a. Rinse with an anti-microbial rinse prior to dental treatment.


b. Take a premed of antibiotics.
c. Schedule an appointment when they are not using drugs.
d. Dental treatment is not advisable.

b - A known intravenous drug user has a high vascular bacterial load and will need to be
premedicated to prevent a further or more serious infection.

358. Which premed cannot be given to a patient that is allergic to penicillin?

a. Clindamycin
b. Azithromycin
c. Amoxicillin
d. Cephalexin

c - Amoxicillin is part of the penicillin family. It will cause the same allergic reaction as penicillin.

TOP © 2015 - 2023 Tests.com


334
359. Which is not a contraindication of nitrous oxide?

a. COPD
b. Pregnancy
c. Cystic Fibrosis
d. HIV

d - HIV is not a contraindication for nitrous oxide. Respiratory diseases, such as COPD and cystic
fibrosis, are contraindicated, because nitrous oxide can further respiratory complications.
Pregnancy is contra indicated, because of an elevated risk of spontaneous abortion.

360. What is camouflage therapy?

a. Moving teeth orthodontically to mask malocclusion due to poor skeletal formation


b. A skeletal surgery to help alleviate malocclusion
c. A surgery that reduces the amount of chin projection
d. A surgery that requires the mouth to be wired shut in the desire occlusion

a - When there is a skeletal deformity, the use of orthodontics can move teeth to achieve an ideal
occlusion. It is done to mask the deformity and achieve correct occlusion.

361. Which is not a common manifestation of orthodontic treatment?

a. Decalcification
b. Aphthous ulcers
c. Root resorption
d. Gingival hypoplasia

d - Gingival hypoplasia is the incomplete development of tissue. Gingival hyperplasia is the


enlargement or overgrowth of tissue, caused by plaque accumulation.

362. How does an injury occur during the x-ray process?

a. X-rays are absorbed by tissue resulting in biological damage.


b. Radiation is scattered and is absorbed by bone.
c. X-rays are absorbed by tissue causing a chemical burn.
d. No injury occurs.

a - X-rays are absorbed by tissue every time radiation is exposed. Not all of the radiation passes
through the patient onto the film and it can cause chemical changes resulting in biological damage.

TOP © 2015 - 2023 Tests.com


335
363. When talking with a new dental assistant about x-rays, you discuss the principal that exposure
should be kept to a minimum and the concept of ALARA. What does the acronym ALARA stand
for?

a. As Low As Reasonably Acquired


b. As Long As Reasonably Acquired
c. As Low As Reasonably Achievable
d. As Long as Reasonably Achievable

c -The concept of As Low As Reasonable Achievable states that all exposure to radiation should be
kept to a minimum. This is accomplished by a dental assistant and hygienist using the correct
techniques and equipment.

364. Your dentist asks you to take a periapical x-ray of a particular tooth. When exposing the x-ray,
you notice that the film is light, with little detail. What is the cause and correction?

a. The film is overexposed and the settings need to be checked and decreased.
b. Incorrect horizontal angulation and direct central ray through interproximal spaces.
c. The film was placed in the mouth backwards and the white side needs to face the PID.
d. The film was underexposed and the settings need to be checked and increased.

d - The film was underexposed and the settings need to be increased. Decreasing the setting would
result in a dark film. Placing a film backwards does result in a light film, but you would also see a
herring bone pattern from outer layer of the film.

365. #8 needs to be infiltrated for scaling and root planing. Where is the topical anesthetic placed?

a. Near the mesial of the canine tooth, for a field block


b. In the vestibule, for a posterior superior alveolar nerve block
c. In the vestibule, for an anterior superior alveolar nerve block
d. In the vestibule, near the isolated nerve branch

d - You are doing an infiltration of a specific tooth, rather than a field block or a nerve block, which
would anesthetize more teeth at one time. In this case, you would place the topical agent in the
vestibule of the specific tooth and nerve branch the dentist plans to work on.

366. When starting a new job in a dental office, you notice that the staff does not use surface barriers
and uses a chlorine based surface disinfectant. What is a disadvantage of using this type of
disinfectant?

a. Diluted solutions need to be prepared daily


b. Leaves a residual film
c. Unstable at high temperatures
d. Poor cleaning capabilities

TOP © 2015 - 2023 Tests.com


336
a - The chlorine based in-office solutions need to be prepared daily. Chlorine dioxide, part of the
chlorine family, is a poor surface cleaner. However, that is not true for the rest of the chlorine
category.

367. What type of radiography is best for examining the temporomandibular joint?

a. Panoramic
b. Bitewing
c. Cephalometric
d. None

d - It is important to note that radiography is not the best when trying to get an accurate image of
the TMJ joint. That is because there are many bony structures that obstruct the view. It is
recommended that a patient get cone beam imaging to get a more accurate image.

368. The dentist wants you to take a panoramic x-ray to evaluate a joint two dimensionally. What
type of x-ray is the panoramic?

a. Intraoral
b. Extraoral
c. Interoral
d. Periapical

b - The panoramic x-ray is considered to be an extraoral x-ray. The film is on the outside of the
mouth and exposed by rotating around the head of the patient. The patient often bites down on a
stick for stability, but nothing else is placed inside of the oral cavity.

369. When taking a full-mouth series of x-rays, a dental assistant, who is shadowing you, asks why
you have to step out of the room for each individual x-ray taken. How should you respond?

a. To protect yourself from all types of radiation exposure


b. To protect yourself from scatter radiation
c. To be overly cautious, you don’t always have to leave the room
d. To press the button, which just happens to be outside of the operatory

a - Stepping outside of the operatory while exposing x-rays protects you from all kinds of radiation,
such as primary, leakage and scatter radiation. It is important for the assistant shadowing to
understand that all dental professionals should achieve zero amounts of radiation.

TOP © 2015 - 2023 Tests.com


337
370. A chair button, which is used when positioning a patient in a dental chair, does not have a plastic
barrier. What type of surface is the chair button considered?

a. Splash surface
b. Transfer surface
c. Spatter surface
d. Touch surface

d - The patient chair button would be considered a touch surface, considering the office is not
using plastic barriers to protect the button from contamination. It could also be considered a
transfer surface if there was more than one button or key pad in the room.

371. What is the correct personal protective equipment you should wear when cleaning a treatment
room after use?

a. Latex gloves, goggles and mask


b. Goggles, mask and scrubs, face shield
c. Mask, goggles protective gown and rubber gloves
d. Non-latex gloves, protective gown, mask and goggles

c - You should always wear rubber gloves when handling contaminated materials and using
chemicals. Latex gloves can break down with certain chemicals and can easily be punctured.

372. After recording all of the periodontal pocket numbers, you check each tooth for mobility. On
#29, there is moderate mobility. What do you record in the chart?

a. The number 1
b. The number 2
c. The number 3
d. The number 4

b - 2 = moderate mobility. Every tooth has mobility, but periodontal disease and bone loss can
make certain teeth more mobile than others. 0 = normal, 1 = slight mobility, 3 = extreme mobility.

373. When performing an intraoral exam, a dentist begins evaluating a patient’s bite. She notices that
the posterior teeth are touching, but the anterior lack vertical overlap. What is this referred to?

a. Cross-bite
b. Open-bite
c. Overjet
d. Crowding

b - The patient has open-bite. This type of bite in common among thumb suckers. To correct this
type of bite, orthodontics should be considered.

TOP © 2015 - 2023 Tests.com


338
374. What type of material is alginate?

a. Irreversible Hydrocolloid
b. Reversible Hydrocolloid
c. Elastomeric
d. Polysulfide

a - Irreversible hydrocolloid aka alginate, is used when making preliminary and final impressions. It
can be used to make whitening trays and study models.

375. If a patient has an allergic reaction to sealant material, what type of allergy do they have?

a. Polymers
b. Acrylate
c. Fluoride
d. Phosphoric acid

b - Acrylate resin is the material used in sealants. It can be highly allergic to a patient with this type
of allergy. If skin contact occurs, the area exposed needs to be thoroughly wash and rinsed with
water. Acrylates can penetrate gloves. Phosphoric acid is in the etching material, not the sealant
material.

376. Once sealant material has been placed and checked for retention, you notice that one area of
sealant has a small bubble. What is the cause and solution?

a. Moisture contamination - need better isolation during placement of sealant.


b. Applied too much sealant material - sealant must be removed and replaced.
c. Vigorous scrubbing during application - add more sealant. Etch again if necessary.
d. Applied too much sealant material - needs to have bite adjusted.

c - The sealant material needs to be carefully dispensed, avoiding bubbles. If one does occur, try to
manipulate the bubbles out of the working area with explorer. If a bubble still does exist, add
sealant material to fill in the gap. If isolation was lost, etch again to add sealant.

377. What is the correct order of links in regards to the chain of infection?

a. Infectious agent, reservoir, mode of transmission, portal of entry and susceptible host
b. Infectious agent, reservoir, mode of transmission, portal of exit and susceptible host
c. Infectious agent, reservoir, portal of exit, mode of transmission, portal of entry and
susceptible host
d. Infectious agent, reservoir, mode of transmission and susceptible host

c - There are six links in the chain of infection. If one link is broken, infection cannot occur. It is
important to understand this chain in regards to infection control throughout the dental office. It

TOP © 2015 - 2023 Tests.com


339
is also important to understand the types of infections you may be exposed to in the dental office.

Refer to the following scenario to answer questions 378 - 380

A 55-year-old woman presents for her 1:00 pm dental treatment appointment. When discussing her
medical history, she states that she has been diagnosed with Type 2 diabetes and is taking an oral
medication for her diabetes.

378. What should you ask first?

a. Do you have an insulin pen with you?


b. When was your last meal?
c. When did you take your medication?
d. Do you have an inhaler?

b - Type 2 diabetes is managed through proper food intake and through oral medication. Though
both are important, asking about the patient’s last meal would help you to distinguish between
hyperglycemia and hypoglycemia and allow for proper further action.

379. The woman states that she took her oral medication, but did not eat anything before her
appointment. You also notice that she is anxious and sweating. What is the patient
experiencing?

a. Asthma Attack
b. Hyperglycemia
c. Hypoglycemia
d. Hyperventilation

c - Hypoglycemia is when a patient has low blood sugar. It can occur when a diabetic patient skips
a meal, taking too much insulin without proper food intake or exercising without adjusting their
food intake.

380. What should you do in this case?

a. Give the women a concentrated form of a sugar packet, such as cake icing.
b. Ask for the insulin pen and administer it yourself.
c. Ask for the inhaler, as she is hyperventilating.
d. Monitor and record vital signs.

a - The patient is hypoglycemic and needs to raise her insulin. Giving the patient concentrated
sugar, such as cake icing, helps the body to absorb the sugar more rapidly into the bloodstream.

TOP © 2015 - 2023 Tests.com


340
Refer to the following scenario to answer questions 381 - 384.

You are setting up the sterilization room in the morning, before patients arrive. One of your tasks
includes filling the ultrasonic cleaner and performing a test to make sure the ultrasonic cleaner is
working.

381. How do you do this?

a. Fill the chamber, turn on the ultrasonic cleaner and listen for suspicious noises.
b. Do not fill the chamber, but turn on the ultrasonic and listen for suspicious noises.
c. Fill the chamber, put instruments inside the chamber, run the ultrasonic and use a piece
of aluminum foil to detect areas of tiny pebbling.
d. Fill the chamber, run the ultrasonic and use a piece of aluminum foil to detect areas
without the tiny pebbling

d - Aluminum foil is used to detect areas where you do not see pebbling. The pebbling should be
evenly marked and throughout the entire submerged piece of foil. You perform this without
instruments to get the most accurate reading.

382. Your next job in the sterilization room is performing the daily testing on the steam autoclave.
Your autoclave is set to run 15 minutes. What temperature does the autoclave need to reach in
order to be effective in sterilization?

a. 250 degrees Fahrenheit


b. 273 degrees Fahrenheit
c. 273 degrees Celsius
d. 230 degrees Fahrenheit

a - When running in a steam autoclave for 15 or 30 minutes, it should always reach the
temperature of 250 degree Fahrenheit. However, when running on shorter cycles or a flash steam,
of three to ten minutes, you will need a temperature of 273 degrees Fahrenheit.

383. After performing the daily test on the steam autoclave, you are to run a weekly test on the
autoclave. Where do you place the biological indicator in the steam autoclave?

a. In the sterilizer near the top and run the sterilizer without instruments
b. In the sterilizer near the center and run the sterilizer without instruments
c. In the sterilizer near the bottom and run the sterilizer with instruments
d. In the sterilizer near the center and run the sterilizer with instruments

d - Placing the indicator in the center of the sterilizer with instruments allows for the most
accurate reading, as the center is the most difficult for the sterilizing agent to reach. Instruments
should be used to closely resemble everyday use.

TOP © 2015 - 2023 Tests.com


341
384. Once you finish the setup on the sterilization room, you get a phone call regarding the failure of
the autoclave’s weekly biological tests. What should you look for?

a. How many packages are in the autoclave at time of sterilization


b. Monitor cycle for correct temperature and time
c. Monitor cycle for correct temperature, pressure and time
d. Check for proper separation of packages

c - To make sure the autoclave is working properly, you need to monitor the temperature,
pressure and time that it takes for each load. Knowing how to correctly load the autoclave and to
get the proper separation is important for the sterilization process. However, when receiving a
failure regarding biological testing, you should evaluate the equipment first for proper readings.

Refer to the following scenario to answer questions 385 - 388.

A 68 year-old male presents with swelling in his upper left area. The patient states that the swelling
started overnight and has progressively worsened throughout the day. He is in significant pain.

385. You are instructed to take an intraoral picture of the tooth. What is the purpose of taking a
picture with an intraoral camera?

a. Provides magnification to better evaluate and educate the patient


b. Allows for easier access to areas that are difficult to view
c. Both a and b
d. None of the above

c - Both a and b are correct. The intraoral camera is a great tool to use in patient education. It also
allows the dentist to magnify and access areas that would not normally be as visible for evaluation.
It is not meant for diagnosis, but as an aid.

TOP © 2015 - 2023 Tests.com


342
386. The dentist has assessed the tooth through x-rays and a series of testing. She has concluded that
#14, using the Universal Numbering System, needs a root canal. How many roots does tooth #14
usually have?

a. 4
b. 3
c. 2
d. 1

b - Generally, the maxillary posterior molars have three roots. Sometimes maxillary posterior teeth
have 4 roots, as these canals are hidden on the facial or lingual side. A dentist would be able to
assess for possible hidden roots during the root canal procedure.

387. The dentist is going to do the root canal on #14 and will need to anesthetize the area. The
dentist asked you to place a topical anesthetic in the areas of injection. How long should the
topical anesthetic remain on the site of injection?

a. 15 seconds to 2 minutes
b. 10 seconds to 3 minutes
c. 1 minute to 5 minutes
d. 5 seconds to 1 minute

a - The topical solution should be placed in the areas of injection for a minimum of 15 seconds to a
maximum of 2 minutes. It is important to note the type of anesthetic you are using and follow the
instructions of placement time.

388. The dentist asked you to assemble the anesthetic syringe with septocaine. What needs to be
locked in on the syringe in order to allow aspiration?

a. Piston
b. Barrel
c. Needle adaptor
d. Harpoon

d - The harpoon needs to be locked in or engaged in order for the dentist to pull back on the
thumb ring and aspirate before administering local anesthetic to the necessary site. The harpoon is
part of the rod that pushes the anesthetic out through the needle, also known as the piston.

389. When x-rays of a patient are taken, who owns the images?

a. The patient. It is part of his dental record.


b. The patient, if he paid for them.
c. The dentist. It is part of the dentist’s dental records.
d. The insurance company, if it paid for them.

TOP © 2015 - 2023 Tests.com


343
c - Technically speaking, the dentist owns the x-rays even though they are the patient’s images and
they may have paid for them. They are part of a patient’s dental records that should be retained
indefinitely. Even though the dentist owns the images, copies can be made and released with
signed consent to another dentist or to the patient themselves.

390. The dental office you work at is doing their yearly testing on all dental x-ray equipment. The
dentist puts you in charge of managing the testing of the equipment. Though you can perform
in-office testing on all of the x-ray equipment, which of the following needs to be done by a
qualified technician?

a. Test of the output of x-rays


b. Calibration
c. Testing kilo voltage
d. Testing focal point spot

b - Calibration is done by qualified technician in intervals regulated by each state. There are many
tests a dental assistant can perform to make sure that the equipment is working properly, such as
the output of x-rays, kilo voltage and focal point spot tests.

391. What is the main concern involving infection control in radiography?

a. Cross-contamination
b. Splatter contamination
c. Transfer surface contamination
d. Saliva contamination

a - Digital radiography’s main infection control problem is cross-contamination of surfaces due to


blood and saliva. It is important to follow the CDC regulations regarding proper handling of the
digital devices.

392. A patient notes on a health history that he is allergic to latex. He also states that he breaks out in
a rash around his mouth when latex gloves are used. What class of allergic reaction is this?

a. Type I
b. Type II
c. Type III
d. Type IV

d - Type IV is often related to a contact reaction that involves the immune system. It is the
chemicals used to process the gloves that cause the allergic reaction. The rash appears 48-72 hours
after contact. Type I is the most serious of latex allergies as it involves a reaction to the proteins in
latex. A response occurs within 2-3 minutes of contact. The patient could exhibit coughing,
shortness of breath or anaphylaxis.

TOP © 2015 - 2023 Tests.com


344
393. A dental practice is to create a latex free dental office to avoid allergic reactions in those
patients allergic to latex. Patients with latex allergies do not need to be scheduled for treatment
at any particular time of the day.

a. Both statements are true.


b. Both statements are false.
c. First statement is true. Second statement is false.
d. First statement is false. Second statement is true.

b - It is impossible to create a latex free dental office, as there is latex in some of the products and
supplies that are use. It is important to understand that latex protein can be present in the air, so it
is important to schedule these patients first thing in the morning, when the proteins are at a
minimum. It is also important that all PPE and exam rooms be latex-free while the patient is in the
office, even if it not the treatment room the patient is seated in.

394. A 24-year-old woman presents to discuss teeth whitening options. After discussing options with
the patient, the dentist decides that the at-home treatment involving custom trays and peroxide
gel is the best option. What are some adverse effects that should be discussed with this patient?

a. Thermal sensitivity and tissue irritation


b. Tissue irritation and pressure sensitivity
c. Pressure sensitivity and thermal sensitivity
d. Chewing sensitivity and thermal sensitivity

a - The patient may experience thermal sensitivity that involves both hot and cold. It is
recommended that a patient use sensitivity toothpaste after teeth whitening. A patient may also
experience tissue irritation, such as redness. They may also get a gingival “burn” where the tissue
turns white from gel exposure. When this happens it is important that patient understands the
importance of not overfilling the whitening trays with gel.

395. You are updating a patient record by recording all of their previous restorations. You
accidentally write “composite filling” instead of “amalgam filling”. When correcting an entry in a
patient chart, you should:

a. Use a black pen and make a single line through the incorrect entry
b. Use a red pen and make a single line through the incorrect entry
c. Use a black pen and make a single line through the incorrect entry. Also, initial and date
d. Use a red pen and make a single line through the incorrect entry. Also, initial and date

c - Using a black pen with a single line allows another user to read the incorrect entry. Using your
initials and dating the entry allows the reader to know who made the mistake an when that
mistake was corrected.

TOP © 2015 - 2023 Tests.com


345
396. Which of the following is not a risk factor of periodontal disease?

a. Smoking
b. Diabetes
c. HIV/AIDS
d. Gingivitis

d - Gingivitis is a sign and symptom of periodontal disease, not a risk factor. Gingivitis is
inflammation in the gingival tissue and can often be reversed with good oral care. Smoking,
diabetes and HIV/AIDS are al risks factors that can contribute to periodontal disease. These risks
factors can alter the body’s response in control the bacteria that causes periodontal disease

397. A 63-year-old woman comes into the office and requests all of her current x-rays. She is going to
another office and would like to take them with her. What is the first step that should be done
when giving a patient their x-rays?

a. The patient must give a 24 hour notice.


b. The patient needs to sign a release form.
c. The patient needs to sign a waiver of liability form.
d. The patient needs to sign a release form and give a 24-hour notice.

b - Any patient that is asking to have x-rays transferred or duplicated need to sign a release form
before the x-rays can be released. An office should never give out the original x-rays for legal
reasons. However, they can release duplicates or transfer x-rays electronically to another office or
to the patient.

398. You are asked to duplicate x-rays for a patient transferring to another office. You have not
transferred to digital radiography and must duplicate the x-ray film manually. After going
through the necessary steps of duplicating and processing the x-rays, you notice that the images
are dark and cannot be used diagnostically. What most likely is the error that occurred?

a. You did not duplicate the films in a dark room. They are light sensitive.
b. You did not turn on the duplicating machine and produce the white light in order to
transfer the image.
c. When placing the duplicating paper on the x-rays, the emulsion side was facing up
instead of facing down towards the x-rays.
d. You did not process the films in the processor.

c - Often times, when a duplicated image is dark, there are two possible errors: exposure time was
too short or the duplicating paper was placed with the emulsion side up. Placing the emulsion side
down allows for accurate duplication of x-rays. Remember, when duplicating x-rays, it is the
opposite of exposing x-ray film.

TOP © 2015 - 2023 Tests.com


346
399. When duplicating x-rays, you need to be in a dark room. What type of infection control is
needed?

a. No PPE is needed
b. Latex gloves
c. Mask
d. Rubber gloves

a - No PPE is needed when duplicating x-rays, as no chemicals are involved. The duplicating
machine should be placed in an area that is considered clean and sterile. It should not be placed in
an area where contaminated chemicals or x-rays may come into contact with the machine.

Refer to the following scenario to answer questions 400 - 401.

A 9-year-old child presents for a recall examination and prophylaxis with the dentist. The child’s
mother is concerned that the child is missing a permanent tooth, as he lost the baby tooth, but has
not had a permanent tooth erupt in its place.

400. The dentist asks for a panoramic film. Looking at the panoramic film, is the child missing a
permanent tooth?

a. Yes, he is missing #7
b. Yes, he is missing number #21
c. Yes, He is missing #30
d. No, he is not missing any permanent teeth

d - He is not missing any permanent teeth. All necessary teeth are developing. Sometimes, when a
primary tooth is lost, a permanent tooth does not erupt right away. It is important to check on

TOP © 2015 - 2023 Tests.com


347
erupting periodically through x-rays and oral exams, as some children may be a case for early
orthodontics.

401. After the x-ray is processed, you notice that the x-ray is dark, especially around the areas of the
mandible? What is the error that was made?

a. The exposure time needs to be increased.


b. The exposure time, kVp and mA all need to be increased.
c. The kVp needs to be decreased.
d. The exposure time, kVp and mA all need to be decreased.

d - The panoramic x-ray was most likely taken at an adult setting and not a child setting. When the
exposure time, kVp and mA are increased, you get a darker image or increased density. By
lowering the exposure time, kVp and mA, you can decrease the density lighten the image. It is
important to know the correct setting for each patient.

402. All of the following patients have contraindications that would preclude the use of an air
polisher EXCEPT one. Which one is the exception?

a. A patient with exposed cementum


b. A patient on a sodium restricted diet
c. A patient with hypotension
d. A patient with respiratory issues

c - A patient with HYPERtension would be contraindicated for the air polisher. The polishing
solution contains sodium bicarbonate, which could cause ankyloses. It should never be used on a
person with a heart condition.

403. A 55-year-old woman presents for a comprehensive exam with x-rays. After going over the
health history with your patient, you begin to take her vital signs. After taking your patient’s
pulse, you begin to take the blood pressure with a stethoscope and manual blood pressure cuff.
The patient’s reading is 142/82. What does the first number mean in a blood pressure reading?

a. Systolic
b. Diastolic
c. Korotkoff sounds
d. Blood pressure

a - The first number is the systolic reading. It reflects the amount of pressure that is required for
the left ventricle of the heart to compress or push oxygenated blood into the blood vessels.
Diastolic is the second number and it reflects the muscle at rest.

TOP © 2015 - 2023 Tests.com


348
404. When a procedure is completed and you are disinfecting your operatory for the next patient,
what is done with the used needles from the procedure?

a. Capped and placed in a puncture resistant, red container label for hazardous material
b. Capped and placed in a puncture resistant, red container label for biohazard material
c. Uncapped and placed in a puncture resistant, blue container label for hazardous material
d. Uncapped and placed in a puncture resistant, blue container label for biohazard material

b - All items should already be capped for infection control. They are then placed in a red container
that is labeled for sharps material. The container should be puncture resistant, closable, leak proof
and labeled with a biohazard symbol.

405. Who creates the guidelines for all hazardous and infectious material in the dental office?

a. OSHA
b. CDC
c. ADA
d. HCS

b - The Center for Disease Control and Prevention is the governing body that protects the health
and safety of the people. Occupational Safety and Health Administration is to protect the health
and safety of the workers. It is important to know the difference and which governing body you
are to refer to when having questions about infection control.

406. You are about to set up the dental operatory for your first patient of the day. What should you
do before the patient is seated to reduce bacterial contamination of the dental unit waterline?

a. Suction 1 full cup of water before the first patient of the day
b. Flush waterlines for several minutes
c. Flush the waterline for 20-30 seconds
d. Nothing needs to be done, the patient can be seated

b - It is recommended that the waterlines be flushed for several minutes prior to the first patient
of the day to prevent biofilm in the waterlines. It is important that you also flush the waterlines
between patients for 20-30 seconds. This helps to eliminate material retracted from the previous
patient.

407. The office is going to conduct a test on the waterlines to ensure the safety of their patients.
When should this type of testing be completed?

a. Before scheduled maintenance, no flushing of the water lines needed


b. After scheduled maintenance, flush the water lines for at least 2 minutes before taking a
water sample
c. After schedule maintenance, no flushing of the water lines needed
d. Before scheduled maintenance, flush the water lines for at least 2 minutes before taking
a water sample

TOP © 2015 - 2023 Tests.com


349
d - When sending out water to be tested, it is important that you flush the waterlines for at least
two minutes prior to taking the sample. This allows for the most accurate sample to be tested. It is
important that you take the sample before scheduled maintenance, so they can properly make
suggestions and corrections.

408. How are safety data sheets for chemicals used in a dental office created?

a. The dental hygienist in charge is to create all safety data sheets.


b. The manufacturer of a chemical provides all safety data sheets.
c. The dentist creates the safety data sheets.
d. Both a and c only

b - The manufacturer of a chemical is required to enclose a safety data sheet with every purchase
and shipment.

409. Your dental office is considering upgrading to digital radiography and using phosphorus plates.
What type of imaging system utilizes phosphorus plates?

a. Direct
b. Indirect
c. Digitized
d. Semi-direct

b - Phosphorus plates are part of indirect imaging system. The phosphor layer is able to store the
energy of the x-ray photons. It is considered indirect, because the phosphorus plates must be
scanned into a computer in order to see the image. With digital sensors, the image is directly
placed on the computer, whether the sensor is wireless or not.

410. You have taken an x-ray and need to develop it manually. What are the five steps you need to
take after placing the x-ray in a developer?

a. Rinse film, place in fixer, time film in fixer, wash film, dry film
b. Place in fixer, time films in fixer, rinse film, wash film, dry film
c. Wash film, place in fixer, time film in fixer, rinse film, dry film
d. Dry film, rinse film, place in fixer, time film in fixer, wash film

a - After placing the x-ray in the developer, you need to rinse the film to remove all residual
developer from the film. Next you place the film in the fixer and time how long it is emerged. Next
you wash the film by using a water bath. Lastly, you dry the film before be used diagnostically.

TOP © 2015 - 2023 Tests.com


350
411. After manually processing a periapical x-ray, you notice that the film is light. What could be the
cause of a light image when you are manually processing an x-ray?

a. The developing solution is too hot


b. The developing solution is too cold
c. Excessive developing time
d. Concentrated developer solution

b - One of the reasons the image may be light is that your developer solution is too cold. Some
other causes could be: Inadequate development time, inaccurate timer or thermometer, depleted
or contaminated developer solution. Having the developing solution too hot, excessive developing
time or concentrate developer would cause a dark image.

412. The dentist requests that you take 4 bitewings on an 11-year-old patient. However, the size 2 x-
ray is too long and is causing the patient to gag. What size or type of film would you try next?

a. Size 2
b. Size 1
c. Size 0
d. Occlusal

b - You would try Size 1 film, as it is not as long horizontally, but still has the height to get all
necessary tooth and bone structure. Size 0 would work, but it is shorter vertically and horizontally.
This means you might miss needed tooth and bone structure needed for accurate diagnosis.

413. After exposing x-rays, you are going to process the films in a daylight loader. What is the first
step that you need to take, after washing and drying your hands, to ensure proper infection
control?

a. Place sterile gloves in the bottom of daylight loader


b. Place contaminated films in the bottom of the daylight loader
c. Place a paper towel in the bottom of the daylight loader
d. Use a cup to collect foil from opened x-rays

c - The first things that should be done after washing and drying your hands is to place a paper
towel down on the bottom of the daylight loader. This is where you will place a container with
contaminated films, sterile gloves and empty paper cup. This allows for proper infection control
and easy cleanup.

TOP © 2015 - 2023 Tests.com


351
414. After finishing a scaling and root planing procedure, a patient is sat into the upright position by
the dentist. As the patient begins to stand up, she falls to the floor, losing consciousness. You
are able to catch the patient and direct her to a chair. The patient comes to quickly, but is
nervous about what just happened. What do you tell the patient?

a. The patient has experienced postural hypotension.


b. The patient has experienced syncope.
c. The patient has experienced angina.
d. The patient has experience a seizure.

b - The patient has experienced syncope, which is the most frequent medical emergency in the
dental office. It happens when there is an imbalance in the distribution of blood to the brain and
to the larger vessels in the body. The possible signs and symptoms include: being in one position
to long, apprehension, fear, the sight of blood, or the sight of certain instruments.

415. As it pertains to the administration of a local anesthetic, a larger gauge needle will have _______
deflection as it advances through tissue as compared to a smaller gauge needle.

a. less
b. more
c. the same
d. None of the above.

a - Deflection is less when using a larger gauge needle. Meaning, it is less likely to bend as it enters
the tissue and allows for better accuracy. A larger gauge needle also allows for easier aspiration
and has a lower chance of needle breakage.

416. If there are large bubbles in the anesthetic cartridge, can you use it?

a. Yes, it is the result of gas during manufacturing.


b. Yes, the cartridge was frozen.
c. No, it is the result of gas during manufacturing.
d. No, the cartridge was frozen.

d - When there are large bubbles in the cartridge, it means that it was frozen. It is no longer sterile
and cannot be used.

417. When a patient has a hard time opening her mouth after receiving anesthetic during a dental
procedure, what is the patient experiencing?

a. Paresthesia
b. Anaphylaxis
c. Trismus
d. Trachyphlaxis

TOP © 2015 - 2023 Tests.com


352
c - Trismus occurs when the muscles in the oral cavity spasm, causing limited opening to the oral
cavity. This may cause pain or distress to a patient. However, it is usually temporary, not
permanent.

418. When placing anesthetic in an area of infection, the onset is _______ and may require ________
anesthetic.

a. Faster, more
b. Faster, less
c. Slower, more
d. Slower, less

c - When anesthetic is placed in an area of infection, the onset is slower due to less anesthetic
being absorbed. This means that more anesthetic may be required to complete a longer dental
procedure.

419. Vasoconstrictors are added to local anesthetic for all of the following reasons, except:

a. It constricts blood vessels.


b. It allows for a longer working time.
c. It increases blood flow.
d. It inhibits the conduction of nerves.

c - Vasoconstrictors decrease blood flow, not allowing the anesthetic to be absorbed in the blood
stream. Once it is absorbed in systemic circulation, the anesthetic action is stopped.

420. What abnormality would a dental professional observe if a patient had too much fluoride during
the formation of adult teeth?

a. Dental caries
b. Decreased amount of enamel
c. Macrodontia
d. Dental fluorosis

d - Dental fluorosis is when a patient ingests too much fluoride during the formation of enamel.
Though it is a cosmetic concern, fluorosis does not mean the teeth are more susceptible to decay.

421. A patient has a bulla. What can that be described as?

a. Loss of epithelium
b. Blister smaller than 5mm
c. Circumscribed area of color change
d. Blister larger than 5mm

TOP © 2015 - 2023 Tests.com


353
d - A bulla is a large blister that is under or in the epithelium of the skin or mucosa. If the area is
smaller than 5mm, it would be considered a vesicle.

422. What type of implant is most commonly used in dentistry?

a. Endosseous
b. Transosteal
c. Subperiosteal
d. Edentulous

a -This is due to the fact that these are placed to support dental prosthesis such as crowns, bridges
and dentures. These types of implants act as anchors to support the function of the prosthesis.

423. Match the active ingredient to the corresponding mouthwash:

a. Cetylpyridinium Chloride ____ Peridex


b. Quaternary Ammonium Compounds ____ Listerine
c. Chlorhexidine ____ Crest Pro-Health
d. Essential Oils ____ Scope

c - Chlorhexidine - Peridex
d - Essential Oils - Listerine
a - Cetylpyridinium Chloride - Crest Pro-Health
b - Quaternary Ammonium Compounds - Scope

424. What percentage of plaque is made up of water?

a. 50%
b. 60%
c. 70%
d. 80%

d - Plaque is 80% water, with the other 20% comprised of bacteria and organic/inorganic solids.

TOP © 2015 - 2023 Tests.com


354
425. Match the type/color of stain with the cause:

a. Black-line ___ Chlorhexidine


b. Green/Yellow/Orange ___ Iron compounds in saliva
c. Blue/Green ___ Chromogenic bacteria
d. Yellow-Brown ___ Industrial Worker/Copper

d - Yellow-Brown - Chlorhexidine
a - Black-line - Iron compounds in saliva
b - Green/Yellow/Orange - Chromogenic bacteria
c - Blue/Green - Industrial Worker/Copper

426. Which curette would you use to clean subgingival on the mesial, facial and lingual surface of the
posterior teeth?

a. Gracey 11/12
b. Gracey 13/14
c. Gracey 15/16
d. Gracey 17/18

a - The use of the half-moon Gracey curette is used with a modified-pen grip. Gracey curettes are
site specific due to the lateral offset of 70 degrees relative to the shank.

427. Which of the following conditions is considered ASA III?

a. Epilepsy
b. Pregnancy
c. Insulin-dependent diabetes
d. Kidney failure

c - Insulin-dependent diabetes would place a patient under an ASA III classification. You can still
treat this patient, but you will need to ask questions regarding their condition. It is also important
to note the role diabetes plays in periodontal health and vice versa.

428. The extra space created by primary canines and molars for the permanent teeth to erupt is
called:

a. Leeway space
b. Primate Space
c. Primary Space
d. Alignment Space

a - Leeway space is created by the primary teeth to allow for the larger permanent teeth to erupt.
On each side of the mandible, the leeway space created is about 2.5mm of room with about
1.5mm on each side of the maxilla.

TOP © 2015 - 2023 Tests.com


355
429. If the cone on the x-ray machine is ________ in length, the radiation absorbed dose would
________.

a. Decreased, decrease
b. Increased, stay the same
c. Increased, decrease
d. Decrease, stay the same

c - When increasing the PID or cone length on the x-ray machine, it decreases the amount of
radiation that a patient absorbs during exposure. A shorter beam means more exposure to the
patient.

430. What causes reticulation on a developed film?

a. Air bubbles in the developer solution or water bath


b. Static electricity in the air
c. Light leakage in the dark room
d. A temperature change between the developer solution and the water bath

d - Reticulation is when an arrangement of lines, resembling a net, can be seen on a developed


film. This occurs when there is a large temperature difference between developer solution and
water bath.

431. After a patient has been diagnosed with periodontal disease by a dentist and a dental hygienist
is instructed by the dentist to complete a scaling and root planning procedure on the patient,
what type of supervision is required by the dentist?

a. General supervision
b. Indirect supervision
c. Direct supervision
d. No supervision

b - Indirect supervision is required by a dentist for scaling and root planning, especially when
anesthetic is being used on a patient. A doctor must be present in the office when a hygienist is
performing any procedure on a patient.

432. When polishing with a rubber cup, an irregular shaped abrasive agent will be ______ abrasive as
compared to an abrasive agent with a regular shape.

a. less
b. more
c. equally
d. None of the above.

b - The irregular shape of a polishing agent would make it more abrasive to the tooth structure.

TOP © 2015 - 2023 Tests.com


356
Having a more regular shape would make the polishing agent smooth, less abrasive to the enamel.

433. Match the services provided by a dental hygienist with the correct term:

a. Primary Prevention ____ Discussing oral health with the patient


b. Secondary Prevention ____ Removing calculus
c. Tertiary Prevention ____ Administering a fluoride treatment
d. Educational Services ____ Scaling and Root planing
e. Therapeutic Services ____ Periodontal therapy

d - Educational Services - Discussing oral health with the patient


b - Secondary Prevention - Removing calculus
a - Primary Prevention - Administering a fluoride treatment
e - Therapeutic Services - Scaling and Root planing
c - Tertiary Prevention - Periodontal therapy

434. When a doctor performs an exam at a 6-month recall, this can be considered __________
supervision.

a. Indirect
b. Direct
c. General
d. Redirect

c - General supervision is when a doctor gives a diagnosis and authorizes a procedure. Removing
calculus and taking x-rays during a recall appointment is also considered general supervision.

435. When you notice that a patient has perimylolsis, what disease could be a contributing factor?

a. Anorexia nervosa
b. Bulimia nervosa
c. Bipolar
d. Schizophrenia

b - Bulimia nervosa is an eating disorder where a person binges on high caloric foods and then
purges through vomiting. The acid from repeated vomiting causes erosion, usually found on the
lingual of maxillary anterior teeth.

436. A patient states that he had a stroke and was hospitalized two months ago. How long does this
patient need to wait before receiving any non-emergency dental care?

a. 1 month
b. 3 months
c. 4 months
d. No wait is needed.

TOP © 2015 - 2023 Tests.com


357
c - A patient should wait at least 6 months to have any non-emergency dental procedures
completed. This is due to the high probability of second stoke during this time. Since he comes
into the office 2 months after his stroke, he has to wait 4 months.

437. All of the following are dental related risk factors for a pregnant woman except one. Which one
is the exception?

a. Gingivitis
b. Granuloma
c. Aphthous ulcer
d. Cavity

c - An aphthous ulcer is not a contagious condition, nor is it a risk factor for a pregnant woman.
Pregnant woman may be more susceptible to getting these ulcers with acid reflux and hormone
changes. Gingivitis, pregnancy granulomas and cavities could pose a risk to a pregnant woman and
baby, due to the bacteria involved. All three should be addressed by a dentist to see if further
treatment is necessary.

438. Can in-office fluoride treatments be given to an expectant mother?

a. Yes, fluoride is recommended.


b. No, it is never recommended.
c. Yes, but not in the third trimester.
d. Yes, but not in the first trimester.

a - Fluoride is deemed safe and recommended for pregnant patients, due to hormone and
bacterial imbalances.

439. What is it called when the tongue adheres to the floor of the mouth?

a. Anodontia
b. Gemination
c. Circumvallate papilla
d. Ankyloglosia

d - Ankyloglossia is when the tip of the tongue is adhered to the floor of the mouth or the anterior
portion of the mandible. This is commonly called “tongue tied”.

TOP © 2015 - 2023 Tests.com


358
440. When taking a bite-wing x-ray, you notice that the 2nd molar root tips are not in the x-ray. This is
due to an abnormal curve in the roots of the tooth. What is this abnormality called?

a. Dentinogenesis
b. Dilaceration
c. Hypodontia
d. Gemination

b - Dilacteration occurs when a root curves or bends. This can make it hard to get periapical or
bite-wing x-rays of a tooth. There is nothing clinically wrong with tooth.

441. What type of drug is Amlodipine Besylate (Norvasc) and what possible dental conditions may
occur when taking this drug?

a. Calcium channel blocker, gingival overgrowth


b. Calcium channel blocker, hypersensitivity
c. Beta-blocker, gingival over-growth
d. Beta-blocker, hypersensitivity

a - Calcium channel blockers disrupt the movement of calcium through calcium channel. This can
cause an overgrowth of gingival tissue anywhere in the oral cavity.

442. What type of retention is used when pit and fissure sealants are placed?

a. Chemical
b. Mechanical
c. Bonded
d. Fused

b - Sealants use a mechanical bond, which means it fills in the space in the grooves of the teeth
and is held there by those spaces. There is not a chemical bond between the sealant material and
the tooth.

443. Which is not a common filler for composite material?

a. Quartz
b. Silicates
c. Glass
d. Tin

d - Tin is part of an amalgam filling. Other amalgam fillers include: silver, copper and mercury.
Mercury is used as a binder for all of the materials.

TOP © 2015 - 2023 Tests.com


359
444. When polishing a composite material, what should never be used?

a. Coarse pumice
b. Tin oxide
c. Diamond
d. Zinc oxide

a - Coarse Pumice should never be used to polish composite material, because it is too abrasive to
the material. It causes the material to lose gloss. Polishes that use fine particles of uniform shape
should be used.

445. Which of the following is not an active ingredient in desensitizing toothpaste?

a. Potassium nitrate
b. Sodium citrate
c. Carbamide peroxide
d. Strontium chloride

c - Carbamide peroxide is a whitening agent that could cause tooth sensitivity. The desensitizing
agents in toothpaste would help alleviate sensitivity after whitening.

446. How many days should a pre-medicated patient wait between a scaling and root planning
appointment?

a. 5 days
b. 7 days
c. 10 days
d. 14 days

c - There should be 10 days between any dental treatment that requires a patient to pre-medicate.
This reduces the emergence of resistant bacteria. If a procedure needs to be done prior to 10 days,
the patient should take a different antibiotic to help stave off the resistant bacteria.

447. What foramen is most often mistaken as a periapical abscess?

a. Mental
b. Incisive
c. Anterior palatine
d. Nasopalatine

a - The mental foramen is the most often mistaken foramen as a periapical abscess due to its
location between the root tips of the mandibular. The angulation of your PID during x-rays and the
foramen location make this look like an abnormality.

TOP © 2015 - 2023 Tests.com


360
448. What medication does not cause xerostomia?

a. Benadryl
b. Delsym
c. Depakote
d. Pepto-Bismol

b - Delsym is an antitussive medication, which is a cough suppressant. Most medications, other


than cough suppressants, can cause dry mouth for a patient.

449. ___________ is caused by altering the neural pathways that stimulate the salivary glands.

a. Sialolithiasis
b. Xerostomia
c. Anesthetic neurotoxicity
d. Salivary gland cancer

b - Xerostomia is a side effect of many drugs. It is important to note the drugs patients are taking
and educate them on how their oral health can be affected.

450. Which two answers describe bruxism?

a. Pathological form of attrition


b. Loss of tooth structure from mastication
c. Habitual grinding of teeth
d. Loss of tooth structure from acid

a and c - Bruxism is very common among kids and adults. Most patients who experience bruxism
do not realize they are clenching or grinding their teeth, because they do it in their sleep.
However, during stressful times, people also clench or grind during the day.

451. What tooth is missing a filling on this x-ray?

a. Maxillary first molar


b. Mandibular first molar
c. Maxillary first premolar
d. Maxillary second premolar

c - The tooth that is missing a filling is tooth #12, which is a maxillary first premolar.

TOP © 2015 - 2023 Tests.com


361
452. What is abnormal about the tooth in this picture?

a. Fusion
b. Gemination
c. Primary bicuspid
d. Heart Tooth

b - When a tooth takes on a heart shape and looks like two teeth, but the child has the correct
number of primary teeth in the mouth, it is called gemination. Gemination occurs when a tooth
bud tries to divide. When two teeth fuse together, thus creating a shortage of teeth in the mouth,
this is called fusion.

453. What is the ultimate goal of scaling and root planing?

a. Decrease the colonization of bacteria


b. Gain connective tissue and reduce the size of a pocket
c. Heal gingivitis
d. Alleviate bleeding around the teeth

b - Though the job of the hygienist is to help heal infection, the ultimate goal is to allow the
connective tissue to reattach, thus creating a smaller pocket.

454. Which two answers are true regarding Kaposi’s sarcoma?

a. It often affects people with immune deficiencies, such as HIV.


b. It is a virus.
c. It is a bacterial infection.
d. It only affects males.

Both a and b - Kaposi’s sarcoma is a cancerous virus, stemming from the herpes family. It is most
often seen in those affected with suppressed immune systems, such as those whom are HIV+ or
have AIDS. This cancer causes abnormal tissue under the skin that is a purple/red color.

TOP © 2015 - 2023 Tests.com


362
455. Is it recommended that smokers use alcohol based mouth rinses?

a. Yes, to help control the bacteria in their oral cavity.


b. Yes, to prevent periodontal disease.
c. No, it puts the patient at higher risk of cancer.
d. No, they should not use any form of mouth rinse.

c - Smokers have a higher risk of oral cancer when they use an alcohol based mouth rinse. This is
due to the drying effect of the alcohol, creates a higher risk environment in the oral cavity. It is
recommended that smokers use a non-alcoholic mouth rinse.

456. What type of surgery may be necessary if the periodontal pockets are too deep for debridement
and scaling, and root planing is not advised?

a. Gingivectomy
b. Periodontal flap
c. Gingival graft
d. Gingivoplasty

b - Periodontal flap surgery is a surgery that is performed by a periodontist. It involves cutting back
the tissue to expose the teeth and bone. This allows for access for debridement.

457. When a patient has severe attrition and the cusps of molars are worn down exposing the
underlying dentin, what are the areas of wear called?

a. Decay
b. Abrasion
c. Erosion
d. Facets

d - Wear facets are often the product of severe bruxism or clenching. These areas are susceptible
to decay and can be sensitive, especially since dentin is exposed. In some cases, these areas can
be filled with composite material to help with sensitivity. It also recommended that the patient
gets a night guard.

TOP © 2015 - 2023 Tests.com


363
458. In regards to this picture, the supragingival calculus clearly extends below the gumline forming
subcalculus. Subgingival calculus is formed by gingival cervical fluid.

a. Both statements are true.


b. Both statements are false.
c. First statement is true. Second statement is false
d. First statement is false. Second statement is true.

d - Calculus is dental plaque that has undergone mineralization. Supragingival calculus is formed
by our saliva and subgingival calculus is formed by gingival cervical fluid. The picture shows
supragingival calculus and moderate gingivitis, not subcalculus.

Practice Test for the NBDHE Dental Hygienist Exam – Section 3


Community Dental Health Testlet

459. ORDER the following in accordance with Maslow’s Hierarchy of Needs as they relate to
identifying human needs and addressing them through personal or community involvement
programs.

______ a. Safety and Security


______ b. Self
______ c. Self Esteem
______ d. Physiological Needs
______ e. Actualization
______ f. Social Needs

d. Physiological Needs
a. Safety and Security
f. Social Needs
c. Self Esteem
e. Actualization
b. Self

TOP © 2015 - 2023 Tests.com


364
460. A team of dental professionals assesses the dental needs of the community and begins to form
the next step of their intervention plan to promote measures that will address these needs.
After the initial assessment, what step comes next in a community program planning routine?

a. Program financing
b. Program formulation
c. Program Implementation
d. Evaluation of effectiveness

b - Program formulation is conducted after the initial needs are assessed. The formulation will
precede implementation, financing or evaluation of the said program.

461. What is the study of the distribution and determinants of disease in a population?

a. Control study
b. Longitudinal study
c. Epidemiology
d. Pathogenic analysis

c - Epidemiology is when a disease is studied within a population. These studies will focus on how
they relate to specific cases as well as testing theories or hypothesis.

462. What type of epidemiology study focuses on testing a theory or hypothesis?

a. Analytical study
b. Descriptive study
c. Convenience
d. Cross-sectional

a - Analytical studies are a type of epidemiology study method that focuses in testing a theory or
hypothesis regarding the distribution of a disease in a population.

463. A dental hygiene program wants to conduct a study on a new antimicrobial agent for patients
with advanced periodontal disease symptoms following their scaling and root planing
procedures. What type of study is the LEAST valid due to the makeup of the group being
studied?

a. Randomized
b. Non-randomized
c. Convenience
d. Longitudinal

b - Non-randomized studies are least valid because the selection of test subjects is not
randomized. The results may be skewed, as the test subjects aren’t randomly assigned,
jeopardizing the validity of the study.

TOP © 2015 - 2023 Tests.com


365
464. A toothpaste company wants to test the efficacy of a new product. What type of study is the
most valid to prove their hypothesis?

a. Longitudinal
b. Randomized
c. Double-blind
d. Cross-sectional

c - The double-blind study would be the most valid test to perform. Neither the company or the
test participants know who is in the control or experimental group, which means there is no bias
regarding the data collection.

465. A hypothesis that was tested using a statistical test shows a p-value of 0.04. Which statement is
correct?

a. There was a significant statistical result


b. The statistical result was insignificant
c. The difference occurred by chance
d. The statistic is factual

a - If a p-value is less than 0.05 during an experiment, then there was a significant statistical result.
If it was greater than 0.05 then it was insignificant, and if it was equal to 0.05 then the difference
occurred by chance.

466. What type of test compares actual results versus independent results?

a. T-test
b. Student’s t test
c. Chi square test
d. Significance level

c - The Chi square test compares actual results versus independent results and provides a
comparison that measures the significance of observed and expected frequencies.

467. Which of the following are NOT criteria for a good index?

a. Clear
b. Simple
c. Valid
d. Quantity

d - Quantity does not qualify something as a good criteria for an index. Criteria for a good index
includes reliable, valid, simple, clear, objective, acceptable and quantifiable.

TOP © 2015 - 2023 Tests.com


366
468. An index is used to study the conditions of gingivitis in test subjects. What type of indices is
being used?

a. Irreversible index
b. Reversible index
c. Reliable index
d. Randomized index

b - A reversible index is used to study gingivitis because gingivitis is a reversible condition.

469. When conducting a double-blind study on periodontal disease patients, the examiners record
periodontal probing depths at each visit. What is necessary scoring for these measurements to
be precise?

a. Calibration
b. Standardized probing
c. Dependent variable
d. Independent variable

a - Calibration, or intraexaminer reliability, is a way to record data the same way between
examiners during a test study.

470. What index is used to measure past and present decay experiences in a population?

a. GI
b. RCI
c. DEFT
d. DMFT

d - The DMFT (decayed, missing, filled teeth) index is used to measure caries experiences in a
population. It is an irreversible index.

471. What index is used to detect early symptoms of gingivitis?

a. GI
b. PDI
c. SBI
d. DEFT

c - The SBI- sulcular bleeding index is used to detect early symptoms of gingivitis. It is a reversible
index and scores from 0-5. The GI index determines the prevalence and severity of gingivitis but it
does not detect early symptoms.

TOP © 2015 - 2023 Tests.com


367
472. A hygienist is using the PDI to measure the presence and severity of periodontal disease in one of
her patients. How does she determine the PDI score?

a. PDI = total score of teeth examined divided by number of teeth examined


b. PDI = total number of teeth examined divided by the total score of teeth examined
c. PDI = total score of teeth examined by number of teeth present
d. PDI= total number of teeth examined by number of teeth present

a - The PDI score would be tabulated using the score of the teeth examined divided by the number
of teeth that were examined. Scoring would range from 0-1 with no inflammation present to
advanced destruction.

473. A dental school aims to measure the periodontal needs of a group of patients. They decide to
choose an index that was developed by the World Health Organization to measure needs in
children and adolescents in one group, and another group of adults over 20 years of age. Which
index was selected?

a. PI
b. PDI
c. CPITN
d. OHI-S

c - CPITN is the Community Periodontal Index of Treatment Needs and was developed by the WHO
as an irreversible index to measure a group’s periodontal needs. The CPITN and PSR use the same
scoring criteria.

474. Which teeth are the Ramfjord teeth that are used to evaluate different teeth per sextant for the
use in indices?

a. 3, 9, 14, 18, 24, 30


b. 3, 8, 14, 19, 24, 30
c. 3, 8, 15, 20, 25, 31
d. 3, 9, 15, 19, 25, 31

b - The Ramfjord teeth are teeth #’s 3, 8, 14, 19, 24 and 30. There is one tooth per sextant that is
used and measured. If the tooth in that sextant is missing then there is no number recorded for
that sextant.

TOP © 2015 - 2023 Tests.com


368
475. When tabulating a PSR recording, a hygienist finds a code 4 on one of the teeth in the current
sextant of the patient’s mouth. What is the next step?

a. Record a 4 on that tooth and continue recording the codes on the other teeth in the
sextant
b. Record a 4 and examine the remaining teeth to record the next lowest number
c. Record a 4 and examine the remaining teeth in the sextant for a lower number
d. Record a 4 and do not examine the remaining teeth in the sextant

d - Once a 4 is recorded in a sextant there is no need to examine the rest of the teeth in the
sextant, because a code 4 is the highest coding used, indicating pockets of 6mm or deeper.

476. Upon inspection, the hygienist assigns a plaque index score of 2 to the patient. This score
indicates:

a. Gross soft debris within gingival pockets or margins


b. Film adheres to attached gingiva
c. Moderate soft debris within the gingival margin
d. Fair oral hygiene

c - There is moderate soft debris within the gingival margin of the patient’s teeth. Using the PI
plaque scores range from 0-3.

477. Disclosing solution is necessary for use in which indices?

a. PHP
b. PI
c. OHI-S
d. All of the above

a - The PHP (Patient Hygiene Performance) index is a reversible index that uses disclosing solution
to measure the amount of debris present.

478. What program was founded in 1997 as a way to meet the needs of children whose families
exceeded the income requirements for Medicaid but were still unable to pay for private medical
insurance?

a. CHAMPUS
b. CHIP
c. Medicare
d. IHS

b - CHIP is the Children’s Health Insurance Program and was an amendment in 1997 to cover
children whose families had too high of an income to qualify for Medicaid coverage.

TOP © 2015 - 2023 Tests.com


369
479. Which of the first amendments to social security involved providing insurance for individuals
over the age of 65, but offers little dental coverage?

a. Title XIX Medicaid


b. Title XXI Medicaid
c. Title XVIII Medicare
d. Title X Medicare

c - Title XVIII Medicare was to provide insurance for people over age 65 to cover medical needs,
although it has very limited dental coverage. This was one of the first amendments to the social
security act of 1935.

480. What type of dental insurance plan is also called a fee-for-service plan?

a. Indemnity
b. Point-of-Service
c. PPO
d. DMO

a - Indemnity plans are also called traditional or fee-for-service plans and are offered by most
insurance providers. Coverage typically range from 50-80% depending on the procedure being
performed

481. Which of the following is NOT a category of community oral health research?

a. Epidemiologic research
b. Clinical trials
c. Evaluation of community programs
d. Research in habits

d - Research in habits is not a category of community oral health research. The category that was
not listed that is the remaining of the 4 categories is Research in educational techniques and
behavioral science as they relate to oral health education.

482. What is a group observed over time to determine the natural progression of a disease?

a. Case-control
b. Cohort
c. Cross-sectional
d. Subjects

b - A Cohort is a well-defined group that is watched over time to observe a natural progression of a
condition or disease after an exposure, and no factors are controlled.

TOP © 2015 - 2023 Tests.com


370
483. Approximately what percentage of adults has experienced root caries?

a. 15%
b. 25%
c. 30%
d. 55%

b - Approximately 25% of adults over the age of 18 have had a case of root caries. The rate
increases with age.

484. What is the most common chronic childhood disease?

a. Asthma
b. Hay fever
c. Chicken pox
d. Caries

d - Dental caries is the most common chronic childhood disease, surpassing both asthma and hay
fever. It is one of the major reasons for hospitalizations in a child.

485. Which teeth are most frequently affected by caries?

a. First molars
b. Second molars
c. Both first and second molars
d. First and second bicuspids

c - Both first and second molars are the teeth most frequently affected by dental caries. These
teeth erupt at a young age and the deep grooves and fissures often develop decay if care is not
adequate.

486. What percentage of adults has experienced dental decay?

a. 25%
b. 73%
c. 85%
d. 95%

c - Approximately 85% of adults have experienced dental decay. Prevalence of decay increases
with age.

TOP © 2015 - 2023 Tests.com


371
487. Cleft lip and palates are among the most common congenital malformations in US births. Which
of the following is NOT associated with clefts?

a. Spontaneous abortion
b. Maternal fever
c. Drug consumption
d. Morning sickness

d - Morning sickness is not among the risk factors for cleft lip or palate. Other conditions that are
related to clefts include maternal influenza, oxygen deprivation, alcohol consumption and
smoking.

488. A patient exhibits white opaque areas on a tooth but it does not involve more than 30-40% of
the tooth. What classification of fluorosis is present?

a. 4
b. 2
c. 1
d. 3

b - Class 2 fluorosis is when the fluorosis is more extensive but it does not involve up to 50% of the
tooth surface.

489. A community based sealant program trains dental professionals to perform sealants in a school-
based program. Studies show that children who participate in these programs for the next 2-5
years will experience _______ fewer new decayed pit and fissure areas.

a. 25%
b. 60%
c. 85%
d. 100%

b - Nearly 60% of new decay areas on pits and fissures will be reduced for 2-5 years following
school based sealant programs. These type of programs are an efficient method to discourage
decay in school age children.

490. If a natural water supply contains a fluoridation level over ________ then the water system will
undergo a defluoridation program prior to the delivery to residents.

a. 1.8ppm
b. 5ppm
c. 2-4 ppm
d. 3.7ppm

c - Water levels with over 2-4ppm fluoride should undergo defluoridation procedures. 2ppm is a

TOP © 2015 - 2023 Tests.com


372
nonenforceable guideline while 4ppm is the maximum concentration allowed by the EPA.

491. Which cancer has a higher death rate?

a. Ovarian
b. Prostate
c. Oral
d. Lung

c - Oral cancers have a higher death rate than most popularly discussed cancers. Because many
cancers discussed in the media promote early diagnosis and intervention, these are detected
earlier and receive care. Oral cancer does not receive as much publicity and may not be noticed by
a patient who is not under the regular routine of a dentist.

492. A convenience study involves participants that are closely located to the examiner. It is
considered very reliable as it involves the general population.

a. Both statements are true


b. Both statements are false
c. The first statement is true, the second statement is false
d. The first statement is false, the second statement is true

c - A convenience study does involve participants that are close to the examiner, but it is not
considered a reliable study because it does not involve the general population, but rather a group
that is convenient to the examiner.

493. A dental indices must be able to measure consistently at different times. This is called:

a. Validity
b. Reliability
c. Quantifiable
d. Clear

b - Reliability is when the index will measure consistently no matter when the reading is taken
place. Reliability allows different examiners to collect the same data and have the same outcome
between them.

494. How many teeth are evaluated in the DMFT index?

a. 20
b. 32
c. 28
d. 24

TOP © 2015 - 2023 Tests.com


373
c - The DMFT (decayed, missing, filled teeth) index measures 28 teeth, excluding the 3rd molars and
primary teeth.

495. What index is used to measure the past and present caries experience of a population?

a. deft
b. DMFT
c. RCI
d. DTI

a - The deft index (decayed, exfoliated, filled teeth) is an irreversible index that excludes missing or
unerupted teeth.

496. How many teeth are evaluated in the deft index?

a. 20
b. 8
c. 28
d. 32

a - The deft index only evaluates 20 teeth, the primary teeth, in people ages 11 and younger.

497. Which area is NOT measured during an SBI reading?

a. mesial
b. occlusal
c. distal
d. lingual

b - Occlusal surfaces are not recorded in the SBI (sulcular bleeding index). Only distal, mesial, facial
and lingual surfaces are recorded.

498. Based on the periodontal treatment needs scale, a patient or community that needs instruction,
root debridement, elimination of plaque and has retentive margins on restorations or crowns
would be classified as what?

a. I
b. II
c. III
d. O

b - A class II periodontal treatment need would indicate that the patient or community needed
debridement, instruction and has retentive margins. The next (and highest) level also includes
possible periodontal surgical intervention with local anesthesia.

TOP © 2015 - 2023 Tests.com


374
499. What index is used to measure oral cleanliness by visual inspection for surface debris?

a. OHI-S
b. PI
c. PHP
d. PTN

a - The OHI-S (Simplified Oral Hygiene Index) examines the tooth surface for any debris or calculus,
and is used to measure oral cleanliness. It is an irreversible index.

500. What publication includes a comprehensive list of disease prevention and health promotion
objectives?

a. A National Call to Action to Promote Oral Health


b. Oral health in America
c. Healthy People 2020
d. Indian Health Services

c - Healthy People 2020 and 2010 editions contain lists of oral health objectives that help establish
priorities for community programs.

501. Which type of study requires two or more measures for a non-experimental outcome of data?

a. Descriptive
b. Case control
c. Cross-sectional
d. Cohort

d - A Cohort study requires at least 2 (or more) measures in order to perform a longitudinal,
prospective or incidence type of study.

502. The difference in a clinical trial and an epidemiologic survey is that an epidemiologic survey will:

a. Target naturally occurring samples of a population


b. Use small groups
c. Gathers data from trials
d. Uses methods that are valid and reliable

a - An epidemiologic survey will target naturally occurring samples of a population. Usually a large
sample is used, with general indices that gather data to establish underlying etiologic factors to
help develop hypothesis regarding the condition.

TOP © 2015 - 2023 Tests.com


375
503. Which of the following is not a type of sampling technique?

a. Strategic
b. Random
c. Systemic
d. Judgmental

a - Strategic is not an example of a sampling technique. There are 6 which include: Random,
Stratified, Systemic, Convenience, Judgmental and Cluster.

504. What sampling technique is most commonly used in research?

a. Stratified
b. Systemic
c. Random
d. Convenience

c - Random sampling is the most commonly used technique, because every subject is selected
independently and all subjects have an equal chance at being selected. It prevents research bias.

505. When there is constant presence of a disease within a given area, it is considered to be
__________.

a. Epidemic
b. Pandemic
c. Endemic
d. Epidemiologic

c - When a disease is endemic, there is a constant presence of the disease in a particular area or
region. An example of this would be malaria, which is localized to the tropical areas of South
America and Africa.

506. In a research study, what group does not receive treatment or intervention?

a. Experimental
b. Control
c. Discrete
d. Sample

b - The control group is a reliable baseline that does not receive the treatment or intervention of
the study. All of the results are compared against this group.

TOP © 2015 - 2023 Tests.com


376
507. All of the following, except one, are levels of government that provide community dental health
care. Which is the exception?

a. State
b. Federal
c. Regional
d. International

c - Regional is not a type or level of governmental agency that has a community dental health
program. The agencies that are part of dental health are: Local, State, Federal and International.

508. What form of research helps answer the who, what and where questions concerning disease?

a. Experimental Studies
b. Descriptive Studies
c. Incidence Study
d. Prevalence Study

b - Descriptive study helps track who is getting a disease, what they are getting and where it is
occurring. It answers the who, what and where questions related to disease. It does not answer
the how/when/why questions. Experimental studies test hypotheses to establish the cause of the
disease.

509. When calculating research data, what is the term used to describe the most frequently occurring
value in the data?

a. Mode
b. Mean
c. Median
d. Measure

a - The mode is the most frequently occurring value in research data. The median is the halfway
point in a set of research data, usually measured by dividing the set into two equal parts. The
mean is found by adding all of the values/numbers and dividing by the number of values in the set.

510. Which of the following is an accurate statement on the nature of non-parametric inferential
statistics?

a. They have a fixed number of parameters.


b. They are most useful for data measured on a quantitative scale.
c. The data it deals with is more qualitative.
d. The parameters are fixed by the model and do not grow based on the data.

c - Non-parametric inferential statistics are most useful for data measured on the nominal or
ordinal scale. Nominal or ordinal data is qualitative and although numbers may be involved, they

TOP © 2015 - 2023 Tests.com


377
are derived more subjectively than data associated with quantitative data. Techniques in non-
parametric statistics rely on data that does not belong to any particular distribution. Another
difference between non-parametric and parametric inferential statistics is that the parameters in
non-parametric statistics are more flexible and grow with the training data, where the parameters
are fixed with the model in parametric statistics. The most commonly used non-parametric test is
the chi-squared test, which is used to examine the differences between observed and expected
frequencies.

511. In statistics, the p-value describes:

a. Statistical significance
b. Different means
c. Variability in the sample groups
d. How many subjects are needed to provide significance

a - In statistics, the p-value shows statistical significance, in that it represents the probability that a
study could have come to a false conclusion. It shows the strength of the evidence about the
population in question. The smaller the p-value, the more significant the findings of the study are
considered. A small p-value (typically < 0.05) indicates strong evidence against the null hypothesis,
so it can be rejected. A large p-value (typically > 0.05) indicates weak evidence against the null
hypothesis, so its rejection fails. A p-value at or close to 0.05 is considered marginal, in that it
could go either way.

512. The following items represent the stages of learning. Put them in the correct order.

a. Awareness
b. Unawareness
c. Habit
d. Self-interest
e. Involvement
f. Action

The correct order is: b, a, d, e, f, c

The stages of learning occur in the following order:

(1) Unawareness
(2) Awareness
(3) Self-interest
(4) Involvement
(5) Action
(6) Habit

TOP © 2015 - 2023 Tests.com


378
513. What type of study involves repeated observations of the same population over a long period of
time?

a. Retrospective
b. Prospective
c. Analytical
d. Longitudinal

d - A longitudinal study often spans decades and involves the same subjects. An example of this
type of study would be the decade long study of children that studied the effects of fluoridated
water and caries reduction.

514. If you accept a null hypothesis, you are saying that, statistically, there are no significant
differences between population groups. It is tested by a statistical hypothesis test.

a. Both statements are true.


b. First statement is true. Second statement is false.
c. Second statement is false. First statement is true.
d. Both statements are false.

a - A null hypothesis means that it is opposite of the hypothesis. The null hypothesis is a default
position in the modern practice of science that there is no relationship between phenomena or
differences between groups. It is tested by using a statistical hypothesis test, which uses a null
hypothesis and alternative hypothesis.

515. Informed consent is an ongoing process. It is often discussed in a group setting.

a. Both statements are true.


b. Both statements are false.
c. First statement is true. Second statement is false.
d. First statement is false. Second statement is true.

c - Informed consent is an ongoing process, but it should not be discussed in a group setting.
Informed consent is often done in the initial stage of treatment and should be completed in a one-
on-one setting.

516. What is considered discrete data?

a. Numeric variables
b. Total time
c. Subcategories
d. Observations

a - Discrete data involves numeric variables, such as the number of patients being examined. It is
only counted in whole terms.

TOP © 2015 - 2023 Tests.com


379
517. All of the following, except one, are advantages of doing interviews in research. Which is not an
advantage of doing interviews?

a. Flexibility
b. Time
c. Complete data collected
d. Questions can be clarified

b - The biggest disadvantage of doing interviews in research is that it is very time consuming.
However, if time is not an issue, it can be a great way to get data.

518. A dental hygienist employed in a nursing home can provide routine dental care. They hold a
special license to perform these duties.

a. Both statements are true.


b. Both statements are false.
c. First statement is true. Second statement is false.
d. First statement is false. Second statement is true.

c - A job of a dental hygienist is to help provide preventative care to those who may not have the
ability or the dexterity to perform oral care themselves. A dental hygienist license is sufficient to
perform these duties in a nursing home facility and no special license is needed.

519. When using the Type III examination method, you use a tongue depressor. When using the Type
IV examination method, you would use a mouth mirror.

a. Both statements are true.


b. Both statements are false.
c. First statement is true. Second statement is false.
d. First statement is false. Second statement is true.

b - In a Type III examination, you use a mouth mirror. Type IV examinations use a tongue
depressor.

520. What type of motivation practice is used when there is the promise of a reward?

a. External
b. Internal
c. Formal
d. Informal

a - External motivation is described as motivation derived from outside of the individual. Intrinsic
motivation stems from the individual them self.

TOP © 2015 - 2023 Tests.com


380
521. Medicare provides insurance to low income families. It provides both medical and dental care.

a. Both statements are true.


b. Both statements are false.
c. First statement is true. Second statement is false.
d. First statement is false. Second statement is true.

d - Medicare provides medical and dental care, but to persons over the age of 65. Medicaid is
designed to help low income families get the proper medical and dental care they need.

522. The human papilloma virus presents with a stem like base, similar to that of a mushroom. This
can be described as ____________.

a. Pedunculated
b. Papule
c. Sessile
d. Papillary

a - Peduncle is an elongated stalk of tissue. Sessile is the opposite, meaning there is no stalk, or
lacks a peduncle. Cysts, polyps and tumors can also be pedunculated.

TOP © 2015 - 2023 Tests.com


381
Practice Test for the NBDHE Dental Hygienist Exam – Section 4
Case 1 Testlet
Age: 11 Dental History:
Sex: M This patient has not seen a dentist in 2 years as
Height: 4’11” his family moved into the area and then
Weight:110 underwent a change in dental insurance. His
B/P: 115/78 mother reports that now they have found a
dental office that they like she will bring him
Chief Complaint: every 6 months for preventive care
None. This patient is being brought to the office appointments.
for a routine dental evaluation and cleaning.
Social History:
Medical History: This patient enjoys sporting activities and plays
This patient suffers from seasonal allergies. He soccer regularly. He recently signed up to be on
has not ever had an asthma flare-up but his a flag football team at his school. He often
doctor has prescribed asthma medication to drinks sports drinks during games, but drinks
manage his cough after outdoor activities mostly water during practice.
during the cooler months.

Current Medications:
Albuterol nebulizer PRN
Claritin 5mg qd

TOP © 2015 - 2023 Tests.com


382
523. What should the hygienist recommend to this patient in regard to how his asthma medication
relates to his oral health?

a. The patient should be taking the medication daily


b. Advise the patient to rinse with fluoride daily
c. Recommend the patient clean their mouth after the use of albuterol
d. Keep an asthma inhaler handy during his dental appointments

c - It should be recommended to the patient that they clean the mouth thoroughly following the
use of albuterol, as many asthma inhalants can cause drying of the mouth as well as higher decay
rates in the teeth. The patient may want to rinse with water and brush following his treatment. An
additional fluoride rinse available over the counter would not be a bad idea, but it isn’t the primary
concern.

524. Upon inspection of the oral cavity, the patient appears to have a 3mm oval lesion just inside the
lower lip. It is flat and raw. The patient states that he had one 3 or 4 months ago as well. It is
most likely a(n):

a. Syphillic chancre
b. Apthous ulcer
c. HPV lesion
d. Apical abscess

b - Apthous ulcers or canker sores are typically flat, round or oval, and raw in appearance. Factors
like stress or sunlight can cause them to occur more frequently.

525. The patient does not have any sealants on his permanent teeth. At this time, what teeth should
the hygienist recommend placing sealants on?

a. 2, 3, 14, 15, 18, 30, 31


b. 3, 14, 18, 19, 30, 31
c. 3, 14, 19, 30
d. 3, 4, 5, 12, 13, 14, 19, 20, 21, 28, 29, 30

c - The 6 year molars, teeth 3, 14, 19 and 30 would be a good place to start with recommending
dental sealants. The 12-year molars have not all fully erupted. While sealants can be placed on
premolars as well, not all premolars have erupted so this is not an option.

526. When discussing the findings on the x-ray, the patient asks about the dark area on the distal of
#K. What should you tell him this looks like?

a. Root resorption
b. Interproximal caries
c. A failed amalgam filling
d. Periapical abscess

TOP © 2015 - 2023 Tests.com


383
b - The radiolucency in this area is typical of interproximal caries in a primary tooth. It is near the
proximal areas of #k and #14, but thankfully looks as if the decay has not reached into #14.

527. Based on the findings of tooth #K, what would be the most important piece of oral hygiene
education that should be taught at this visit?

a. The importance of maintenance visits


b. Fones method of toothbrushing
c. Sugar intake
d. Flossing technique

d - The flossing technique and regular flossing should be introduced and reviewed with the patient
to prevent interproximal decay.

528. Upon discussing the patient’s extracurricular activities, it is discovered that he does not wear
any type of tooth protection device during practice, but he does wear a stock guard during
games. What is the best advice that the hygienist could give to the patient?

a. Suggest the patient wear a guard during all practices as well as games
b. Suggest the patient only wear a custom guard during games
c. Suggest the patient continue what he is doing until a custom guard can be fabricated
d. Recommend the patient not wear a guard unless it is a custom made guard

a - Suggest the patient wear a guard during all practices as well as games, because orofacial
injuries are not specific to just organized sporting events but may occur at any time a sport is
practiced. A custom guard will be more effective than a stock guard, but wearing any guard at all
for all activities is better than none at all.

529. In regards to the patient consuming sports drinks, what type of nutritional counseling advice is
most appropriate?

a. Discuss the hydration benefits of sports drinks vs. water


b. Continue drinking lots of water during practices. Drink only one sports drink during the
game and drink only water afterward.
c. Substitute all water with sport drinks
d. Discuss types of sports drinks and which one is a more healthy option

b - Advise the patient to drink lots of water during practices. If he wants to have a sports drink
during the game, advise him to have only one and then rinse the mouth and drink only water
afterward. Discuss the benefits of water and how frequent long-term exposure of sports drinks
may contribute to decay.

TOP © 2015 - 2023 Tests.com


384
530. The patient’s mother states she is concerned that some of her son’s teeth are not coming in
straight with the other teeth. The hygienist should:

a. Discuss the occlusion and then give the patient’s mother a card for the orthodontist
b. Discuss the occlusion with the patient’s mother and then report findings to the dentist
c. Have the dentist discuss the occlusion with the patient’s mother
d. Tell the patient’s mother that her son’s occlusion will be evaluated at the next
appointment to see if it has changed

b - Hygienists may discuss the occlusion findings with the patient and parent, but they are not
allowed to refer patients to specialists such as orthodontists. The correct procedure would then
follow with the hygienist discussing the findings with the dentist so that the dentist can make the
appropriate referral or diagnose needs.

531. The patient reports occasional soreness in the posterior regions of the lower arch. What is the
likely cause?

a. Decalcification
b. Caries
c. Eruption of 12-year molars
d. Use of whitening toothpaste

c - The patient’s 12 year molars are coming very close to erupting into the oral cavity. Sporadic
discomfort is normal with tooth eruption, and as the soreness is localized to these 2 areas then
eruption would be the cause.

532. The patient asks how many baby teeth he has left to lose. The hygienist’s correct response
should be ______.

a. 5
b. 4
c. 7
d. 6

a - The patient has 5 remaining primary teeth. Teeth #A, H, J, M & R are still present in the mouth.

TOP © 2015 - 2023 Tests.com


385
Practice Test for the NBDHE Dental Hygienist Exam
Case 2 Testlet
Age: 9 Current Medications:
Sex: F None
Height: 4’4”
Weight: 68 Dental History:
B/P: 109/72 Patient has been coming into the office since
she was 3 for routine preventive care
Chief Complaint: appointments every 6 months. She has no
Patient complains of a broken tooth in her history of dental decay and had sealants placed
upper right posterior region. Her mother on her 6-year molars during the last visit.
reports that she has complained of it hurting for
the past 2 days. Social History:
Patient is in 3rd grade at a local private school.
Medical History: She enjoys reading and drawing.
The patient was hospitalized 7 months ago for a
broken arm when she fell off of a horse.

TOP © 2015 - 2023 Tests.com


386
533. What is the most likely explanation for the patient’s chief complaint of a broken tooth in the
upper right posterior?

a. Her 1st molar has developed large mesiodistal decay


b. A primary molar that had decay is resorbing but has not yet exfoliated
c. Her 2nd bicuspid has developed a fracture
d. #2 is only partially erupted through the gums

b - The patient still has 2 primary molars in this area, with #A appearing to have decayed along as
the roots undergo the natural resorption during the process of exfoliation. The patient should be
encouraged to wiggle the tooth out on her own in the fairly near future.

534. How many lower molars are shown on the patient’s x-ray?

a. 6
b. 2
c. 8
d. 10

d - There are 10 molars evident in the mandible on the patient’s x-ray. 4 primary molars, her 6-
year molars and the developing 12 year molars and wisdom teeth.

535. Upon inspection of #19, the tooth appears to have fractured and then developed decay. Which
of the following would NOT be a risk factor that could have contributed to this condition?

a. The patient’s horseback injury


b. The patient chewing on pencils and pens at school
c. Hyperfluorosis
d. Bubble gum chewing

c - Hyperfluorosis would not have contributed to an increased risk of fracture and decay. However,
if the patient sustained an injury during her horseback riding accident it could have done some
damage, which could be compounded by the fact that she chews bubble gum and pens at school.
The patient should be encouraged to wear a mouth guard even during horseback riding.

536. The dentist prescribes bitewing radiographs to check the interproximal areas of the primary and
permanent molars. How many films would this patient need?

a. 2 vertical BWX
b. 2 horizontal BWX
c. 4 vertical BWX
d. 4 horizontal BWX

b - 2 horizontal BWX would show the interproximal areas of the primary molars and permanent
first molars. Because there are no permanent second molars erupted in the mouth there is no

TOP © 2015 - 2023 Tests.com


387
need for an additional view with a 3rd and 4th film.

537. The patient’s mother complains that the patient’s adult incisors are much more yellow than the
rest of her teeth. What is a reasonable explanation?

a. Enamel fluorosis
b. Stain from a vitamin supplement
c. Natural coloration
d. Tea stain

c - Adult incisors are much more “yellow” than primary incisors, because of the large amount of
dentin inside of the tooth structure. Dentin is naturally yellow and is evident especially when the
incisors erupt next to existing primary teeth.

538. The patient exhibits some grayish intrinsic stain on teeth 7-10. What is the likely cause?

a. Fluorosis
b. Food pigment
c. Tetracycline
d. Genetic

c - Tetracycline stain is an intrinsic stain that manifests itself as a grayish discoloration of the tooth
enamel. The stain is caused from tetracycline ingestion by the mother while pregnant, or if the
child was given the medication at an early age.

539. The patient shows some moderate marginal inflammation and plaque accumulation. What type
of toothbrush should be recommended?

a. Medium bristled
b. Hard bristled electric
c. Soft bristled manual
d. Any of the above

c - A soft bristled toothbrush should always be used rather than stiffer bristles that are known to
cause conditions like gum recession and abrasion to the enamel.

540. While reviewing brushing techniques, the patient exhibits the method that she uses. She
brushes with the bristles aimed toward the gingival margin and then rolls the brush toward the
occlusal surface. What method is this?

a. Fones
b. Roll
c. Leonard
d. Charters

TOP © 2015 - 2023 Tests.com


388
b - The Roll method of brushing starts with the bristles at the gum margin and then rolls the brush
toward the incisal or occlusal surfaces.

541. During oral hygiene instruction, the hygienist discloses the patient’s teeth with an agent that
stains new and mature plaque in red and blue tints. What type of agent does she use?

a. Erythrosin
b. Disclosing tablet
c. Two-tone disclosing solution
d. Bismarck Brown

c - Two-tone disclosing solutions will stain new plaque red and mature plaque blue, that way oral
hygiene instruction can address areas in the mouth that have developed plaque buildup due to
inefficient brushing over time.

542. The patient lives in a rural area and has a well as her water source. There is no natural
fluoridation of the water in the area. The hygienist recommends a weekly supplemental dosage
of fluoride. What is the proper dosage for the patient’s age?

a. 1.0mg
b. 0.5mg
c. 0.01 mg
d. 0.25mg

a - 1.0mg of weekly supplemental fluoride is the proper dosage for people ages 6-16.

TOP © 2015 - 2023 Tests.com


389
Practice Test for the NBDHE Dental Hygienist Exam
Case 3 Testlet
Age: 8 Current Medications:
Sex: M None. Patient just completed a round of
Height: 4’ 4” amoxicillin.
Weight: 70
B/P: 110/78 Dental History:
Patient has not been seen in the office for 20
Chief Complaint: months. The patient’s mother states she had to
Patient’s mother is concerned with upper go back to work full-time and was not able to
anterior crowding and wonders if he needs to make an appointment before now.
have braces.
Social History:
Medical History: The patient is currently enrolled in 2nd grade at
This patient just completed a round of the local public elementary school. He has a pet
antibiotics for an inner ear infection. He is also dog named “Buddy.” He is enrolled in a Boy
allergic to latex. Scout group and enjoys camping on the
weekends with his den.

TOP © 2015 - 2023 Tests.com


390
543. What type of gloves should the hygienist use for treating this particular patient?

a. Standard examination gloves


b. Rubber gloves
c. Nitrile gloves
d. Surgical gloves

c - Nitrile gloves would be the appropriate choice to use on a patient with a latex allergy. Another
option that is not listed would include vinyl gloves. The other options listed contain latex and
would cause an allergic reaction.

544. Based on the appearance of the patient’s radiographs, what permanent tooth appears to be
rotated?

a. 20
b. 7
c. 31
d. 10

b - According to the patient’s radiographs, #7 is rotated. This is evidenced by the appearance of


the crown facing distally and the margin of the lingual surface visible toward #8.

545. The patient appears to exhibit good oral hygiene habits, but upon scaling interproximal areas
there is some moderate bleeding. What would be the best oral hygiene counseling to address at
this visit?

a. Fones method
b. Bass method
c. Antiseptic mouthwash
d. Flossing

d - Flossing would be the most appropriate choice of oral hygiene instruction for a patient that
exhibits interproximal bleeding, as floss is the only oral hygiene method listed above that would
remove plaque biofilm in this area.

546. The patient has no existing restorative work. In addition to the prophylaxis and fluoride
application during today’s visit, what other preventive care procedure would help prevent decay
of the permanent molars?

a. Fluoride varnish
b. Sealants
c. Bitewings
d. Exam

b - Placing sealants on the permanent molars (3, 14, 19, 30) would be an effective preventive care

TOP © 2015 - 2023 Tests.com


391
procedure that could prevent pit and fissure caries in these teeth.

547. Which tooth best exhibits the amelogenesis phase of morphodifferentiation?

a. #2
b. #16
c. #11
d. #19

b - Tooth #16 exhibits the initial enamel formation of a tooth bud. This phase is the amelogenesis
phase of morphodifferentiation where differentiated cells begin to form tooth enamel. The other
3rd molars are not yet visible.

548. Which tooth best demonstrates the appearance of a tooth in the appositional stage of tooth
formation?

a. #4
b. #6
c. #16
d. #14

a - Tooth #4 best exhibits the appearance of a tooth in the appositional stage of development. The
matrix will soon undergo maturation or calcification.

549. In addition to the patient having mild interproximal bleeding during scaling, the patient also has
moderate marginal calculus on the lingual of the lower anterior teeth. What would be an
appropriate recall schedule for this patient?

a. 3 months
b. 6 months
c. 12 months
d. No longer than 24 months

b - The ideal recall schedule for this patient should be 6 months. It is important to explain to the
child’s mother that routine preventive care appointments can help prevent large amounts of
calculus from building up and contributing to inflammation.

TOP © 2015 - 2023 Tests.com


392
550. The patient’s mother exhibits concern of her son’s crowding. When the hygienist checked the
patient’s occlusion, the mesiobuccal groove of his mandibular first molar aligned with the
mesiobuccal cusp of his maxillary first molar. He also exhibited a small overbite and crowding of
the anterior teeth. What classification of occlusion is evident?

a. Class III malocclusion


b. Normal occlusion
c. Class II malocclusion division I
d. Class I malocclusion

d - A class I malocclusion is a normal relationship between the molars with versions such as cross-
bites or overjets. The patient’s molars are in a normal class I relationship, but the slight overbite
places it into a class I malocclusion.

551. The patient admits to not bringing his toothpaste or floss with him in his backpack on overnight
Boy Scout camping trips. What oral hygiene method may be useful for him to care for his teeth
on overnight hikes?

a. Dry toothbrushing
b. Mouthwash
c. Rinsing his mouth with water
d. Brushing his teeth as soon as he gets home

a - Using a dry toothbrush, or even brushing with water alone can be useful in the removal of
plaque biofilm. The patient should be encouraged to take his toothbrush with him at the very
least, and to use it regardless of whether or not he also has toothpaste packed.

552. How many permanent teeth have erupted into the patient’s mouth?

a. 12
b. 14
c. 16
d. 10

a - The patient has 12 permanent teeth that have erupted in the mouth - #3, 7, 8, 9, 10, 14, 19, 23,
24, 25, 26 and 30.

TOP © 2015 - 2023 Tests.com


393
Practice Test for the NBDHE Dental Hygienist Exam
Case 4 Testlet
Age: 9 Dental History:
Sex: M This patient is a mouth breather and has a
Height: 4’ 6” tongue thrust. He reportedly sucked his thumb
Weight: 85 until he was 4.
B/P: 119/79
Social History:
Chief Complaint: The patient is in 4th grade at the local public
The patient’s mother reports that he has elementary school. His parents are divorced and
halitosis. he spends the weekends at his father’s house,
which lives in the next town. He has begun
Medical History: learning how to use a skateboard, and his
Patient’s mother reports he tends to have mother states he will not wear a mouth guard
allergies year-round when he goes to his father’s house with the
skateboard.
Current Medications:
Claritin 5mg qd

TOP © 2015 - 2023 Tests.com


394
553. The patient has a congenitally missing tooth. Which tooth is it?

a. 16
b. 6
c. 18
d. 32

c - #18 is a congenitally missing tooth. It is a 12-year molar and should be evident on the x-rays.
Some of the other teeth to choose answers from were 3rd molars, but because the patient is only 9
it would not be possible to know at this time if those teeth were congenitally missing or not.

554. Upon inspecting the patient’s occlusion, there is a mesial relationship of the mesiobuccal groove
of the mandibular first molar to the mesiobuccal cusp of the maxillary first molar. What
classification of occlusion is present?

a. Class I malocclusion
b. Class III malocclusion
c. Normal relationship
d. Class II malocclusion

b - A class III malocclusion includes a lower first molar that is in a mesial relationship with the
mesiobuccal groove compared to the mexiobuccal cusp of the maxillary first molar. This type of
bite usually appears as an underbite or a jaw that is too far forward.

555. The patient’s mother exhibits concern regarding breath malodor. Microorganisms in what area
of the mouth produce malodor?

a. Tonsils
b. Tongue
c. Periodontal pockets
d. Postnasal drip

b - Microorganisms on the tongue and teeth can create malodor as they produce sulfur
compounds or other foul-smelling compounds. 90% of malodor is from the oropharynx region. The
other 10% is due to systemic health.

556. Which device is appropriate for cleaning the tongue?

a. Tongue scraper
b. Manual toothbrush
c. Electric toothbrush
d. All of the above

d - All of the above options are appropriate to use on the surface of the tongue to remove odor
causing bacteria.

TOP © 2015 - 2023 Tests.com


395
557. The patient states that he cannot brush his tongue well because the bristles on his toothbrush
are all splayed out. Which of the following is NOT an area of education that the hygienist should
address?

a. Selecting the appropriate brand of toothbrush


b. Modifying the patient’s brushing technique
c. Changing the toothbrush out every 2-3 months
d. Selecting the appropriate texture of toothbrush bristle

a - The appropriate brand of toothbrush is not applicable. The patient should be educated on the
proper brushing technique so that not too much force is applied, in addition to the fact that the
patient needs to replace his toothbrush every few months or when the bristles become splayed.
Because the patient pushes hard when he brushes, it should be noted that he selects a soft
bristled brush to use.

558. When preparing the fluoride tray for a gel application, what is the maximum amount of gel that
should be placed in the fluoride tray?

a. 4ml
b. 8ml
c. 2ml
d. 12ml

a - A maximum of 2ml per arch, or 4ml total should be placed in a fluoride tray for application. A
toxic dose is 5mg/kg for children.

559. The patient exhibits a tongue thrust. What oral habit may be related to this anomaly?

a. Chewing pencils at school


b. Thumb sucking
c. Mouth breathing
d. Using a water bottle

b - Thumb sucking for an extended period of time may have caused the patient to develop a
tongue thrusting habit.

TOP © 2015 - 2023 Tests.com


396
560. When selecting a fluoride to place in the tray for the patient application, she chooses a form that
is one which should not be used on patients that have composite restorations. What type of
fluoride did she choose?

a. Neutral sodium
b. Acidulated phosphate
c. Stannous
d. Fluoride varnish

b - Acidulated phosphate fluoride should not be used on composite restorations, bridges, crowns
or veneers due to its acidity.

561. The patient has a small, 2mm, round, purple lesion just inside of the buccal mucosa. It is most
likely a(n) ____________.

a. mucocele
b. ranula
c. lipoma
d. hemangioma

d - Hemangiomas are areas that contain a benign proliferation of blood capillaries. They are
typically present at birth or as a result of trauma.

562. The patient says he does not wear a mouth guard when using his skateboard because the one he
has does not fit well. Which type of guard would provide an appropriate fit?

a. Mouth-formed protectors
b. Custom-made protectors
c. Stock protectors
d. Rigid

b - A custom made mouth guard will fit the most comfortably and snugly against the teeth as
opposed to a stock guard or mouth-formed (boil-and-bite) protector available at the store.

TOP © 2015 - 2023 Tests.com


397
Practice Test for the NBDHE Dental Hygienist Exam
Case 5 Testlet
Age: 72 Current Medications:
Sex: F Daily multivitamin
Height: 5’2” Fosamax 70mg once weekly
Weight: 109
B/P: 110/70 Dental History:
The patient’s previous dentist retired 2 years
Chief Complaint: The patient is experiencing ago. She has not had any dental problems until
some bleeding during brushing around her the recent bleeding, which caused her to go
29x31 3-unit bridge. She also reports a bad ahead and find a new dentist. She wears an
taste in her mouth and her tongue feels as if it upper and lower partial denture which seem to
is burning. not be fitting well as they used to.

Medical History: Social History:


This patient is fairly healthy. She takes a daily The patient is active in her church and conducts
multivitamin and a regular osteoporosis a ladies study at her house each week. She
medication. During her last physical exam her enjoys walking her dog and takes the Active
doctor mentioned she was slightly anemic, but Older Adults class twice a week at the YMCA.
the vitamin supplement has helped to bring her
iron levels back to where they ought to be.

TOP © 2015 - 2023 Tests.com


398
563. The patient is taking medication to control her osteoporosis. Which of the following contributes
to an increased risk of osteoporosis?

a. High intake of vitamin D


b. High body weight
c. Of African decent
d. Smoking
e. Both b and c only

d - Smoking is a risk factor associated with osteoporosis. Other factors include low body weight,
low intake of calcium, alcoholism and of white or Asian descent.

564. During initial head and neck examination, the hygienist notices inflammation and cracking at the
corners of the lips, which extend onto the facial skin. What condition has these classic
symptoms?

a. Angular cheilitis
b. Yeast infection
c. Fordyce granules
d. Chapped lips

a - Angular cheilitis is inflammation and cracking in the corners of the mouth. It is most often
caused by a candida albicans infection and may also be due to a nutritional deficiency or denture-
wear.

565. In addition to the inflammation and cracking at the corners of the patient’s lips, the hygienist
also notices her tongue is devoid of filiform papillae. What condition does the patient likely
have?

a. Extended wear of her partial


b. Geographic tongue
c. Iron deficiency anemia
d. Neutropenia

c - In iron deficiency anemia, the tongue is usually devoid of filiform papilla, causing a burning,
painful sensation. The mouth may also have dry cracks in the corners. Systemic symptoms may
include brittle nails, an enlarged spleen, pallor of skin and fatigue.

566. During oral hygiene education, the hygienist should advise the patient to:

a. Remove her partial denture and brush the underlying tissue at least once a day
b. Brush her partial denture with a separate denture brush
c. Remove her appliance overnight
d. All of the above
e. Both a and c only

TOP © 2015 - 2023 Tests.com


399
d - All of the above information is important in the daily care of removable prosthesis and the oral
tissues under them.

567. The patient has limited dexterity, which makes it difficult to clean along her fixed bridge with
anything other than a toothbrush. What oral care appliance would help with plaque control in
this area?

a. Floss threader
b. Toothpick
c. Mouthwash
d. Water flosser

d - A water flosser would provide an effective means of plaque removal for a person with limited
dexterity, under areas such as bridges. Another option available to the patient would include a
proxa-brush.

568. Tooth #22 exhibits some discoloration along the gingival margin on the distal portion of the
tooth. It is white and brown in color, localized to that particular area. Which statement is correct
regarding this area?

a. The hygienist should use selective polishing to remove the stain


b. It is due to tetracycline intake
c. It is due to embedded plaque biofilm that has caused chromogenic bacteria
d. The discoloration is intrinsic and due to dental fluorosis, therefore it cannot be removed

d - Dental fluorosis is an intrinsic stain that causes white to brown colored lesions and affects the
ameloblasts during tooth formation. It cannot be removed.

569. According to the patient’s periodontal probing depths, the patient exhibits _________________.

a. Generalized moderate bone loss


b. Localized moderate bone loss
c. Generalized severe bone loss
d. Localized severe bone loss

c - The patient has generalized severe bone loss. There are several probing depths throughout the
entire mouth that measure 7mm or deeper.

TOP © 2015 - 2023 Tests.com


400
570. What might occur to the diseased pocket epithelium during subgingival scaling the distal of #22?

a. Irrigation
b. Incidental curettage
c. Debridement
d. Calculus removal

b - Incidental curettage of diseased pocket epithelium may occur during the scaling of subgingival
areas. This occurs on the other working edge of the curette when the active blade is against the
tooth surface.

571. When probing #17, the tooth exhibited severe involvement with through and through visibility.
What grade of furcation involvement does this tooth have?

a. Grade III
b. Grade I
c. Grade IV
d. Grade II

c - Grade IV furcation is a tooth with severe furcation involvement that has through and through
visibility of the furcation. It also includes complete loss of interradicular bone.

572. Tooth #14 responds to pressure by having moderate mobility of about 2mm in a buccolingual
direction. What class of mobility is present?

a. Class II
b. Class I
c. Class III
d. Class IV

a - Class II mobility is moderate mobility in a buccolingual direction, but there is no mobility when
the tooth is compressed.

TOP © 2015 - 2023 Tests.com


401
Practice Test for the NBDHE Dental Hygienist Exam
Case 6 Testlet
Age: 68 Current Medications:
Sex: F Aricept 10 mg/day
Height: 5’1” Risperdal 4mg/day
Weight: 110
B/P: 109/69 Dental History: The patient states at her old
home she was seeing a dentist who was going
Chief Complaint: “I’m having a hard time eating to make a new partial denture for her to wear
food, because I don’t have enough teeth in my on her lower teeth. Her old partial denture has
bottom jaw.” broken.

Medical History: Patient tripped on her front Social History: The patient recently moved in
porch 6 months ago, broke her leg, and with her son and his wife. She enjoys her own
shattered her elbow. She is still undergoing private living quarters and kitchenette, while
physical therapy once a week in addition to her still being connected to the main house.
exercises at home. The patient is experiencing
early stages of dementia.

TOP © 2015 - 2023 Tests.com


402
573. Tooth #14 does not have any teeth to occlude with. What will the result likely be?

a. Advanced wear
b. Furcation involvement
c. Mobility
d. Supraeruption

d - Supraeruption of the teeth typically occurs when a tooth no longer has another tooth to
occlude with, causing it to erupt further out of the jaw.

574. What medical conditions are the patient’s prescriptions designed for?

a. Blood pressure
b. Dementia
c. Osteoporosis
d. Cholesterol

b - The patient is taking the two prescriptions for dementia. She is taking a lower dosage of Aricept
than its full dosage for advanced Alzheimer’s disease.

575. Which tooth has had root canal therapy performed on it?

a. 27
b. 26
c. 22
d. 28

a - Tooth #27 is the only tooth in the mouth to have had a root canal. This is evidenced by the
radiopacity of the nerve canal on the x-ray. The patient is missing the adjacent teeth, #26 and 28,
as noted on their clinical chart.

576. The patient has a white line-shaped lesion inside of the buccal mucosa. It is bilateral and
keratinized. What is this most appropriate name for this lesion?

a. Leukoedema
b. Linea alba
c. Leukoplakia
d. Keratosis

b - Linea alba is keratinized tissue along the occlusal plane, evident on the buccal mucosa. It is a
single line of thickness and does not require any treatment. It is simply due to occlusion forces
against the cheek.

TOP © 2015 - 2023 Tests.com


403
577. Which bridge would be the easiest for the patient to floss?

a. 2x6
b. 7x10
c. 12x14
d. They would all be equally easy to floss

c - The 12x14 bridge would be easiest for the patient to floss, as there are no abutments on either
end and it is placed on a single tooth. This would allow the floss to be brought interproximally at
the end of the bridge and then slid underneath the pontic.

578. Which of the following teeth has probing depths that reflect moderate bone loss?

a. #7
b. #13
c. #10
d. #22

b - The deepest probing depth at tooth #13 is 5mm, which is in the moderate range of bone loss.
7mm or deeper is considered severe bone loss, and 3mm or shallower is considered a healthy
pocket.

579. If the distal of tooth #31 measures 3mm from the cementoenamel junction to the top of the
marginal gingiva and has a 4mm periodontal pocket, what is the total measured bone loss?

a. 3mm
b. 4mm
c. 7mm
d. 8+mm

c - The total measured amount of bone loss is 7mm, because the pocket is 4mm deep and there is
already 3mm of recession from the CEJ to the top of the detached periodontal pocket tissue.

580. Before the patient can have a new lower partial denture made, what nutritional counseling
should the hygienist provide?

a. Advise the patient to eat a soft diet that includes nutrient-dense food
b. Encourage the patient to have a liquid diet and consume nutritional shakes or smoothies
c. Advise the patient to remove her partial denture at night so it will last longer
d. Advise the patient to have implants placed so she can function as normal as possible

a - The best nutritional counseling would be to advise the patient to have a soft diet that includes
nutrient dense foods. She may also want to have nutritional shakes, but she needs to be having
some solid food to maintain the health of the remaining teeth and physical functions.

TOP © 2015 - 2023 Tests.com


404
581. The incisal edges of teeth #7 and 11 are flat and the exposed tooth enamel below the crown is
exposed. What is the likely cause?

a. Enamel erosion
b. Toothbrush abrasion
c. Attritional wear
d. Tongue thrust

c - Attritional wear of the above mentioned crowns can be caused by premature wear of these
teeth during their occlusion against 2 of the only 3 teeth on the lower arch, most likely to the
patient not having other teeth to chew with.

582. The patient reports having some pain in her hand following her fall. All of the following would
NOT be an appropriate modified brushing method for this patient?

a. Use of an electric toothbrush


b. Using the Fones brushing method
c. Placing a large handle on the brush, such as a bike handle
d. Having someone help her brush

b - Using the Fones brushing method would not aid the patient in any way in regards to her
dexterity. However, tips such as placing a large handle on her brush for an easier grasp, using a
large handled electric toothbrush or having someone help her brush would be an effective plaque
removal method for someone with dexterity problems.

TOP © 2015 - 2023 Tests.com


405
Practice Test for the NBDHE Dental Hygienist Exam
Case 7 Testlet
Age: 47 Current Medications:
Sex: M None
Height:: 5’9”
Weight: 161 Dental History: The patient’s last dental
B/P: 140/89 appointment was over 7 years ago for
treatment on a root canal. His teeth have not
Chief Complaint: “One of my top right teeth been cleaned in over 15 years.
hurts. I have not been to the dentist in over 7
years and need to have them cleaned.” Social History: The patient is a truck driver. He
is away from home some, but is at home most
Medical History: Patient had his last physical 3 evenings. His favorite drink is Mountain Dew
years ago. He is allergic to penicillin. and he admits to drinking it 3 or 4 times a day
for the caffeine.

TOP © 2015 - 2023 Tests.com


406
583. What is the radiopacity evident on the lower right border of the mandible?

a. Ghost from jewelry worn during x-ray


b. Ghost of cervical vertebrae
c. Hyoid bone
d. Clavicle

c - The hyoid bone is the “u” or “j” shaped radiopacity that is evident in this area of all panoramic
x-rays. Depending on the positioning of the patient’s head, the hyoid may overlap onto the jaw on
the final film.

584. Based on the patient’s social history and the recurrent caries rate evident on his x-rays, what
would be the most important aspect of counseling at today’s visit?

a. Oral hygiene instruction


b. More frequent preventive care appointments
c. Nutritional counseling
d. Oral cancer screening

c - Nutritional counseling is extremely important for this patient. Based on his social history, he
consumes soda on a high frequency, which is contributing to a high rate of recurrent caries.

585. What is the projection on the distal surface of #3?

a. Amalgam overhang
b. Calculus deposit
c. Caries
d. None of the above

b - A calculus deposit is evident on the distal of #3, and is most likely visible on the mesial of #3,
although its location at the mesial appears as if it could be an amalgam overhang. Because there
are no restorations on the distal of #3 we know it must be calculus.

586. What is the radiolucency near the apex of #7?

a. Incisive fossa
b. Incisive foramen
c. Nasopalatine foramen
d. Nasopalatine cyst

a - The incisive fossa is a radiolucent area that appears around the apex of the maxillary lateral
incisors. It is due to an area of bone that is less dense than the surrounding areas.

TOP © 2015 - 2023 Tests.com


407
587. What classification of periodontal disease would this patient fall under?

a. Localized aggressive periodontitis


b. Generalized chronic severe periodontitis
c. Aggressive moderate periodontitis
d. Localized chronic moderate gingivitis

b - The patient has generalized, severe bone loss as evident by multiple pockets 5mm deep or
deeper.

588. What type of crown is placed on #19?

a. Gold
b. Stainless Steel
c. Silver
d. Porcelain fused to metal

b - A stainless steel crown is placed on #19. This can be seen through the “SSC” marked on the
dental chart, as well as observing the rough margins on the x-ray, which are typical of temporary
stainless steel crowns.

589. Which of the following is a long-term goal of nonsurgical periodontal therapy?

a. Elimination of pockets
b. Removal of hard and soft deposits
c. Definitive biofilm control instruction
d. Control of systemic risk factors

a - Elimination of periodontal pockets is a long-term goal of nonsurgical periodontal therapy. The


other options are short-term goals.

590. The hygienist prepares a solution for subgingival irrigation after the scaling and root planing
procedure. Which of the following solutions is NOT a commonly used irrigant?

a. Chlorhexidine digluconate
b. Stannous Fluoride
c. Listerine
d. Povidone-iodine

c - Listerine is not a commonly used solution for subgingival irrigation following a periodontal
procedure. The most common are 0.12% chlorhexidine, 0.4% stannous fluoride and 0.05%
povidone-iodine.

TOP © 2015 - 2023 Tests.com


408
591. When selecting the appropriate ultrasonic insert to scale the distal concavity of #3, which
ultrasonic tip would be the most effective to use?

a. Universal
b. Thin
c. Beavertail
d. After Five

d - An After Five ultrasonic tip is useful for deep areas that have furcation involvement, or
developmental concavities on the tooth.

592. Which curette would the hygienist choose to scale the distal area of #14?

a. 11/12
b. 15/16
c. 7/8
d. 13/14

d - A 13/14 curette is for scaling the distal areas of the posterior teeth on both the upper and
lower arches.

TOP © 2015 - 2023 Tests.com


409
Practice Test for the NBDHE Dental Hygienist Exam
Case 8 Testlet
Age: 37 Current Medications: None
Sex: M
Height: 6’0” Dental History: The patient’s last dental visit
Weight: 173 was 8 years ago to place the crown on tooth
B/P: 124/ 85 #19 following a root canal. Most of his fillings
are 10-20 years old. The patient does not floss
Chief Complaint: “My kids tell me that my because he finds it difficult to fit his hands into
breath stinks. I brush my teeth well every day, his mouth.
but I think I just need to have them cleaned.”
Social History: The patient is a shift supervisor
Medical History: The patient saw his doctor last at an oil refinery. He works alternating day and
month when he came down with the flu. The night
only surgery the patient has ever had was his
appendectomy 21 years ago.
shifts.

TOP © 2015 - 2023 Tests.com


410
593. As the patient demonstrates his flossing technique, which technique is improper and would
need to be addressed through oral hygiene education?

a. Wrapping the floss in a “c” shape around each tooth


b. Sliding below the gingival margin, moving up and down
c. Using unwaxed floss
d. Snapping the floss through the contacts

d - Dental floss should not be snapped through the contacts as it could result in damage to the
interdental papillae.

594. When demonstrating his tooth brushing method, the patient positions the bristles 45 degrees to
the long axis of the tooth, toward the occlusal surface of the tooth. What brushing method is
this?

a. Charter’s
b. Fones
c. Roll
d. Bass

a - Charter’s method involves placing the bristles angled upward against the long axis of the tooth,
toward the occlusal surface of the tooth, then brushing in a short back and forth motion.

595. The hygienist then demonstrates a brushing method that places the bristles at the attached
gingiva and sweeps them up apically, then sweeps over the gingiva and the tooth. What method
is this?

a. Combination
b. Leonard
c. Roll
d. Stillman’s

c - The roll method involves brushing with the bristles at the gingival margin and sweeping them
apically over the gingiva, then sweeping up over the gingiva onto the tooth.

596. What type of restoration is placed on tooth #18?

a. Amalgam
b. Composite
c. Porcelain onlay
d. Gold inlay

b - A composite MO filling is placed on #18. This is evidenced by the smoother contours on the
radiograph, and the encircled area on the patient’s chart.

TOP © 2015 - 2023 Tests.com


411
597. The patient has a green stain on the facial surface of the upper anterior teeth. Which statement
is true in regard to this type of stain?

a. The stain is intrinsic and due to chromogenic bacteria


b. The stain is extrinsic and due to chromogenic bacteria
c. The stain is intrinsic and of unknown etiology
d. The stain is extrinsic and due to food stain such as tea

b - Green stains are typically caused by chromogenic bacteria and are extrinsic. Therefore they can
be polished off of the tooth.

598. The patient has supragingival calculus that covers more than 1/3 but less than 2/3 of the
exposed teeth. What calculus rating does the hygienist assign in the patient’s record?

a. 0
b. 1
c. 2
d. 3

c - A calculus rating of 2 indicates that supragingival calculus covers more than a third, but less
than two thirds of the exposed tooth surface.

599. When the patient is probing the periodontal pockets, she uses a probe with millimeters marked
at 1, 2, 3, 5, 7 and deeper. What type of probe is this?

a. Michigan
b. Williams
c. Color Coded PSR
d. Nabor

b - The Williams probe marks mm measurements at 1, 2, 3, 5, 7, 8, 9 & 10. It is useful for


measuring the periodontium, attached tissue and oral lesions.

600. During the appointment, the hygienist needs to sharpen a different scaler for use on a particular
tooth. The face of an instrument should be at what angle to the sharpening stone?

a. 110 degrees
b. 45 degrees
c. 90 degrees
d. 180 degrees

a - 110 degrees is the proper angulation of the sharpening stone to the face of the instrument.
Then begin sharpening at the heel 1/3 of the working end, then the middle 1/3 and the toe 1/3.

TOP © 2015 - 2023 Tests.com


412
601. The dentist wants to place a site-specific therapy at the distal of #16 that uses a drug implanted
in a cord that will be placed and then removed 7-10 days later. What drug is this?

a. Arestin
b. Periostat
c. Doxycycline
d. Tetracycline

d - Tetracycline (actisite) is delivered to specific sites by being impregnated in a cord that is placed
into the pocket and then removed a week later.

602. The patient is assigned a plaque index of 2.1. This is considered:

a. Excellent
b. Good
c. Fair
d. Poor

d - A plaque index of 2.0-3.0 is considered poor.

TOP © 2015 - 2023 Tests.com


413
Practice Test for the NBDHE Dental Hygienist Exam
Case 9 Testlet
Age: 46
Sex: M health check, but his doctor advised controlling
Height: 6’4” it through diet and exercise.
Weight: 205
B/P: 134/91 Current Medications: None

Chief Complaint: “I’m packing food between a Dental History: The patient has only had one
couple of teeth on the lower right side of my filling, and he says it is over 10 years old.
mouth. It gets very tender and the gums are
swollen in that area.” Social History: This patient is a postal carrier.
He has been with the USPS for 18 years, 16 of
Medical History: The patient has no health those have been on walking the same route. He
concerns. He sees his physician every year or and his wife’s child just graduated from high
two for check-ups. His blood pressure was school. The patient dips chewing tobacco a few
mentioned as being slightly elevated at his last times each day
.

TOP © 2015 - 2023 Tests.com


414
603. Patients that use spit tobacco are prone to develop _________.

a. Black hairy tongue


b. Squamous cell carcinoma
c. Basal cell carcinoma
d. Pleomorphic adenoma

b - Squamous cell carcinoma is common near the site where tobacco is held, in people that use spit
tobacco.

604. During tobacco cessation education, the hygienist tells the patient that chewing tobacco is
associated with oral cancers. She also tells the patient that the oral lesions usually disappear
after chewing tobacco is discontinued.

a. Both statements are true


b. Both statements are false
c. The first statement is true; the second statement is false
d. The first statement is false; the second statement is true

a - Both statements are true in regards to chewing tobacco. Oral lesions often disappear after
discontinuing the use of dip. Dip is associated with oral, laryngeal and pharyngeal cancers.

605. The patient has an oval-shaped elevation in the midline of the dorsal portion of the tongue,
which is void of papilla. What condition is this known as?

a. Hemangioma
b. Pyogenic granuloma
c. Geographic tongue
d. Median rhomboid glossitis

d - Median rhomboid glossitis is the atrophy of the central papilla, and typically leaves an oval or
diamond-shape in the middle of the tongue, just before the circumvallate papilla.

606. During the scaling and root planing procedure, the hygienist observes that tooth #8 is mobile
more than normal, moving approximately 2 mm side to side, but it is not depressible. What
classification of mobility is this?

a. III
b. II
c. I
d. IV

b - Mobility that is greater than 1mm is at least a class II mobility. Once the tooth is depressible it
becomes class III, therefore this tooth is only a class II.

TOP © 2015 - 2023 Tests.com


415
607. The root on tooth #25 appears shorter than the others. The patient reports having trauma to
that tooth as a teenager. What is this condition known as?

a. Resorption
b. Recession
c. Granuloma
d. Chondroma

a - Resorption of the root causes a loss of tooth structure as it is destroyed either internally or
externally. This is typically due to trauma or caries in the pulp.

608. What type of bone loss is evident radiographically on the mesial of tooth #30?

a. Infrabony pocket
b. Horizontal bone loss
c. Vertical bone loss
d. Triangulation

c - Vertical bone loss is evident on the mesial of #30. The distal of #30 has horizontal bone loss.

609. When probing, one periodontal pocket has a base of the pocket that is apical to the alveolar
crest. What type of pocket is this?

a. Suprabony pocket
b. Supra-crestal pocket
c. Supra-alveolar pocket
d. Infrabony pocket

d - An infrabony pocket is also called an intrabony, intra-alveolar, or subcrestal pocket, and is when
the base of the pocket is apical to the alveolar crestal bone.

610. Which of the following is NOT correct in regard to the histologic changes of the periodontium in
cases of periodontal disease?

a. Vascular proliferation
b. Junctional epithelium protrudes into the connective tissue
c. Tightening of the intercellular spaces in the junctional epithelium
d. Collagenase actively breaks down connective tissue

c - The intercellular spaces of the junctional epithelium widen as pathogens invade the host
tissues.

TOP © 2015 - 2023 Tests.com


416
611. Following the patient’s periodontal scaling and root planing appointments, the dentist has the
hygienist place Arestin (Minocycline hydrochloride) in appropriate areas that have more
advanced stages of disease. This drug has an extended release for up to how many days?

a. 7 days
b. 14 days
c. 21 days
d. 30 days

c - Arestin has an extended-release of 21 days following placement. After placement the area
should not be scaled for 10 days, probed for 30 days or retreated for 3 months.

612. What method of delivery is used for Minocycline hydrochloride (Arestin) into periodontal
pockets?

a. chip
b. capsule
c. liquid
d. Injection

b - Arestin is delivered in a capsule form that is inserted into the periodontal pocket with a syringe.

TOP © 2015 - 2023 Tests.com


417
Practice Test for the NBDHE Dental Hygienist Exam
Case 10 Testlet
Age: 32 Current Medications
Sex: F Dilantin 30mg qd
Height: 5’6”
Weight: 132 Dental History: The patient enjoys going to the
B/P: 140/78 dentist. She brushes her teeth independently
each day with fluoride toothpaste.
Chief Complaint: The patient’s caregiver states
that the patient has broken off several of her Social History: The patient’s caregiver reports
teeth. She also has heavy bleeding when she that the patient has not been eating as much as
brushes around her lower canines. she used to. She also seems to be losing weight.
Otherwise she is participating in normal
Medical History: functions and activities. She does enjoy drinking
Down’s Syndrome juice.
History of seizures- The patient’s last seizure
was 42 days ago.

TOP © 2015 - 2023 Tests.com


418
613. What is the patient taking medication for?

a. Down Syndrome
b. Blood pressure
c. Seizures
d. Cholesterol

c - Dilantin is a medication used to treat epilepsy (seizures) and control them. The patient has no
other health conditions.

614. The patient has a mesh like white lesion on the buccal mucosa that cannot be wiped off.
Between the white lines the tissue appears normal. What is this condition?

a. Lichen Planus
b. White sponge nevus
c. Hyperkeratosis
d. Leukoplakia

a - Lichen Planus is a benign disorder that is typical in middle age person. It is often found on the
buccal mucosa or the dorsal surface of the tongue and it cannot be wiped off.

615. During the intra and extra oral exam on this patient, which would be typical to find?

a. Short neck
b. Dry, fissured lips
c. Protrusive, tongue
d. All of the above

d - All of the above are some of the normal features and manifestations that accompany a person
with Down syndrome.

616. Patient’s with Down syndrome have an anomaly of which chromosome?

a. 18
b. 21
c. 22
d. 23

b - Chromosome 21 is the one affected in some or all of body cells in patients with Down
syndrome. It is also called trisomy 21.

TOP © 2015 - 2023 Tests.com


419
617. What oral manifestation is NOT found in persons with Down syndrome?

a. Mandibular prognathism
b. Decreased salivary flow
c. Increased risk to periodontal disease
d. Premature tooth eruption

d - Persons that have Down syndrome typically have delayed eruption of the teeth, and may have
abnormal tooth development.

618. The patient’s last seizure was over a month ago. It lasted about 20 seconds and she experienced
a transient loss of consciousness. What type of seizure was this?

a. Generalized tonic-clonic
b. Grand mal
c. Generalized absence
d. Complex partial

c - Generalized absence seizures are also known as petit mal seizures, and only last about 5-30
seconds. A grand mal (Generalized tonic-clonic) would last 1-3 minutes, while a complex partial
seizure would cause behavioral alterations.

619. The hygienist sees plaque film at the gingival margins but no soft debris under the margin. What
plaque score would the patient be given?

a. 0
b. 2
c. 1
d. 3

c - A plaque score of I indicates that there is film at the gingival margin.

620. The patient’s oral hygiene has not been a concern in the past, yet the patient has a high rate of
recurrent tooth decay. What is a likely cause?

a. The patient has a high intake of fruit juice


b. The patient cannot floss without assistance
c. Hypercalcification of the enamel
d. Hyperfluorosis

a - Nutrutional counseling should address the patient’s high intake of fruit juices, or liquids other
than water frequently throughout the day, as this may contribute to an increased risk of recurrent
decay.

TOP © 2015 - 2023 Tests.com


420
621. Which fluoride would be contraindicated for this patient?

a. Acidulated phosphate fluoride


b. Neutral fluoride
c. Fluoride varnish
d. All fluorides are contraindicated for this patient

a - APF would be contraindicated as the low pH may cause the dissolving of glass restorations.

622. What would be a likely contributor in regard to the increase of bleeding during brushing around
the lower canine teeth?

a. Localized moderate periodontal disease


b. Rotation of the teeth
c. Irregular tooth brushing
d. Dexterity limitations of the patient

b - The rotation of #22 and 27 make them more susceptible to gingivitis as plaque removal is more
difficult to achieve. The other areas of the mouth do not have problems with bleeding, so brushing
and dexterity are not the causes. Bone loss is not evident on the x-rays.

TOP © 2015 - 2023 Tests.com


421
Practice Test for the NBDHE Dental Hygienist Exam
Case 11 Testlet
Age: 49 Dental History: The patient wore a lower partial
Sex: M denture until his accident that damaged some
Height: 6’3” of his lower right teeth as well.
Weight: 185
B/P: 123/80 Social History: The patient is receiving workers
compensation as his primary income source. His
Chief Complaint: This is the patient’s first wife has gone back to work part-time at the
dental visit since a work-related accident last high-school as a teacher’s aide. The patient is
spring. He admits that early on after the hoping to start operating a small vending
accident he did not do much to take care of his machine business at an area highway rest stop.
teeth, but he has begun being more proactive The patient was battling depression after his
when it comes to his oral health. accident but has been receiving counseling and
is doing much better. He now does many tasks
Medical History: The patient was in a work- independently around the house such as
related accident, which caused blindness in washing dishes, doing the laundry and making
both eyes. coffee.

Current Medications:
Ambien 10mg qd

TOP © 2015 - 2023 Tests.com


422
623. When escorting the patient to the operatory, what method is the best used?

a. Take the patient by the hand, or place his hand on your shoulder
b. Allow the patient to follow your voice
c. Ask the patient to have someone assist him
d. Explain the surroundings such as steps or doorways

d - The most important thing to do for visually impaired patients is to describe the surroundings or
procedures to them. The patient will let the provider know if he needs assistance.

624. When discussing home care methods, what is the best approach for a visually impaired patient?

a. Allow the patient to listen as you describe a particular method.


b. Use a “touch, tell, feel” approach.
c. Move the patient’s hands to show them what to do.
d. Use a tell and listen approach.

b - Using a “touch, feel, tell” approach is most appropriate for visually challenged patients as it
allows them to experience different sensations and better understand new concepts. Use large
educational models when possible.

625. The patient states that he feels a rough area near the gumline on the facial of #8. What is the
likely cause?

a. Exposed dentin due to gum recession


b. Heavy marginal calculus
c. Enamel hypoplasia
d. Margin of his porcelain crown

a - An area where some mild recession is present would allow the dentin to be exposed, and may
feel rougher to the patient. This would only occur at the gumline, below the CEJ.

626. What condition may affect the upper left posterior molars?

a. Supereruption
b. Localized severe bone loss
c. Inflammation of the nasal sinus
d. Mobility

a - Supereruption may occur in these teeth as there are no teeth in the opposite arch to occlude
with. If the patient is able to have a new partial fabricated, then it will help prevent further
supereruption in these teeth.

TOP © 2015 - 2023 Tests.com


423
627. What is the vertical radiopaque area near the apex of 8 and 9?

a. Median suture
b. Nasal septum
c. Nasopalatine foramen
d. Superior foramina

b - The nasal septum is the radiopaque area found here. All other options would appear as
radiolucent.

628. During subgingival exploration, the hygienist feels an extension on the distal of #2 as she pulls
up with her explorer, but does not feel it as she goes down. What is she likely feeling?

a. CEJ
b. Subgingival calculus
c. Amalgam overhang
d. Crown margin

d - Judging by the appearance of the patient’s radiograph, a poorly contoured crown is present in
this area. The hygienist is feeling the distal margin of the crown with her explorer.

629. The patient drinks coffee on a frequent basis. What type of stain would be typical?

a. Green stain
b. Black line stain
c. Brown stain
d. Orange stain

c - Brown stain is typical in coffee and tea drinkers. However, black line stain is of unknown
etiology. The two are different types of stains.

630. During the intraoral examination, the patient’s palate is found to have an irregular lesion near
the front of the hard palate that is red and has tissue sloughed away. What is the likely cause?

a. Thermal burn
b. Cyclic neutropenia
c. Pleomorphic adenoma
d. Herpes

a - A thermal burn due to a hot food is the likely cause of this lesion. Pizza often causes burns in
the roof of the mouth.

TOP © 2015 - 2023 Tests.com


424
631. The patient appears to have elongated papilla of the tongue that is brown and black in
appearance. The hygienist suspects the patient is using what as a mouthwash?

a. Chlorhexidine
b. Listerine
c. Hydrogen peroxide
d. None

c - Hydrogen peroxide is a common cause of black hairy tongue. While in smaller dosages (less
frequently) it may not cause black hairy tongue, it can when used regularly.

632. The patient has an opaque, white lesion on the buccal mucosa on the right side, near the
fractured teeth. What is the likely diagnosis of this lesion?

a. Frictional keratosis
b. Gingival hyperplasia
c. Traumatic ulcer
d. Irritation fibroma

a - Frictional keratosis has occurred due to the chronic rubbing trauma of the buccal tissues against
the fractured teeth, resulting in keratinized tissue. Treating the teeth in the lower right area will
correct this condition. If it were an irritation fibroma the tissue would be the same color of
adjacent tissues.

TOP © 2015 - 2023 Tests.com


425
Practice Test for the NBDHE Dental Hygienist Exam
Case 12 Testlet
Age: 12 symptoms since receiving medication to
Sex: F manage the condition.
Height: 5’3”
Weight: 112 Current Medications:
B/P: 103/59 Concerta 27mg qd

Chief Complaint: “The right side of my mouth Dental History: The patient has two
hurts. I can’t open it well since my soccer congenitally missing teeth. Her father was
accident this morning.” missing the same teeth.

Medical History: The patient was born with a Social History: The patient is a member of a
unilateral cleft lip which was repaired during community softball league. Last year they
her early childhood. She was diagnosed with advanced all the way to state where they came
ADHD 3 years ago, but has done well with in 3rd place.

TOP © 2015 - 2023 Tests.com


426
633. What could be causing the discomfort that the patient is experiencing?

a. Bruising from an injury the week before


b. #T preparing to exfoliate
c. Fracture of the jaw
d. Decay of #3

c - The patient has been unable to open her mouth since an accident this morning. The appearance
on the radiograph appears to show a fracture of her right jaw, which will require immediate
medical attention.

634. What permanent teeth appear to be congenitally missing?

a. #6 and 11
b. #7 and 10
c. #1 and 7
d. #11 and 32

b - The patient’s maxillary lateral incisors are missing (7 & 10). It is too early to determine whether
or not their 3rd molars are missing or not.

635. What is the opacity on the mesial of #5?

a. Retained primary canine


b. Fractured tooth cusp
c. Retained primary second molar
d. Odontoma

a - This opacity is a retained primary canine. This is visible by the location and the shape of the
opacity, which has been pushed out of the way for the permanent canine to erupt.

636. Which tooth in the upper left quadrant is rotated?

a. #5
b. #11
c. #12
d. #15

c - Tooth #12 is rotated due to the extra space in the arch, caused by other missing teeth.

TOP © 2015 - 2023 Tests.com


427
637. Which tooth shows the Amelogenesis stage of tooth development?

a. #31
b. # 16
c. #18
d. #6

b - Tooth #16 is the 3rd molar and is just beginning to show up on the radiograph, signaling that the
amelogenesis stage of tooth formation is occurring.

638. The patient was born with a bilateral cleft lip on the right side. A cleft in this area occurs when
what processes fail to fuse during fetal development?

a. Frontonasal and stomadeum


b. Frontonasal and maxillary
c. Maxillary and medial nasal
d. Medial nasal and frontonasal

c - Failure of the maxillary and medial nasal processes to fuse during fetal development will result
in cleft lip, or if severe enough, a cleft palate.

639. The patient asks why her remaining baby teeth have not fallen out yet. At what age do
mandibular second bicuspids typically erupt?

a. 9-10
b. 11-12
c. 13-14
d. 7-8

b - Mandibular second premolars typically erupt around the age of 11-12. Maxillary second
bicuspids may erupt a year sooner.

640. Which of the following is NOT true in regard to ADHD?

a. Treatment may include stimulants


b. May cause irritability and impulsiveness
c. More common in girls
d. Can make it difficult to acquire new skills

c - ADHD is more common in boys. All of the other statements are correct.

TOP © 2015 - 2023 Tests.com


428
641. What could the hygienist suggest as a way to help further protect the patient’s orofacial area
from trauma associated with playing softball?

a. Helmet use
b. Mouthguard
c. Not clenching her teeth
d. Complete her dental treatment timely

b - Using a mouth guard (or sport guard) during recreational activities can decrease the rate of
tooth fractures and concussions. Most organized sports already require protective equipment such
as helmets or pads, but many do not require mouth guards.

642. What is the round circular area around the crown of tooth #29?

a. Pseudocyst
b. Periodontal cyst
c. Dentigerous cyst
d. Primordial cyst

c - Dentigerous cysts are also called follicular cysts. They are always associated with the crown of
an unerupted tooth.

TOP © 2015 - 2023 Tests.com


429
Practice Test for the NBDHE Dental Hygienist Exam
Case 13 Testlet
Age: 29 Current Medications:
Sex: M None
Height: 6’2”
Weight: 178 Dental History: The patient has not been to the
B/P: 121/68 dentist for 2 years. He recently started a new
job and is taking advantage of having dental
Chief Complaint: “My gums bleed when I brush insurance by coming in to have a routine check-
and floss.” up and cleaning. He reports a history of grinding
his teeth. 3 months ago he had an oral piercing
Medical History: placed in the middle of his lower lip.
The patient had a spinal cord injury 6 years ago
when he fell off of an ATV. He is restricted to a Social History:
wheel chair and cannot use his lower limbs. His The patient graduated from college 2 years ago
upper body, including arms and hands, function and is now living independently. He is on a
normally. basketball team that meets once each week to
play at the community recreation center in the
next town.

TOP © 2015 - 2023 Tests.com


430
643. The patient requires some help transferring himself to the operatory chair during this
appointment. During a one person assisted transfer, where is the most appropriate place that a
hygienist should place their hands?

a. Under the patient’s elbows


b. Under the patient’s arms
c. Around the patient’s waist
d. Around the patient’s safety-waist belt

d - Patients that require some assistance transferring into and out of their chair for any reason are
best helped by utilizing a safety-waist belt.

644. Which tooth has a crown still intact and exhibits a periapical radiolucency?

a. #1
b. #14
c. #19
d. #29

d - Tooth #29 still has a large portion of the crown intact, and exhibits a periapical radiolucency,
unlike #14 which was the other tooth listed that still had a crown intact.

645. What is the long, radiolucent line that extends down through each side of the patient’s jaw, as
evident on the x-ray?

a. Mandibular canal
b. Mandibular foramen
c. Border of the mandible
d. External oblique ridge

a - The mandibular canal appears as a radiolucent line that runs down each side of the mandible
from the ramus to the mandibular foramen.

646. The patient has a bluish, 3cm bulge under the tongue. What is this called?

a. Sialolith
b. Ranula
c. Mucocele
d. Rhomboid glossitis

b - A ranula is a large 1-3cm smooth lesion that is under the tongue and is caused by the blockage
of a salivary stone.

TOP © 2015 - 2023 Tests.com


431
647. Which tooth has the doctor diagnosed as needing a porcelain crown?

a. #29
b. #30
c. #18
d. #19

c - Tooth #18 is charted as needing to have a porcelain crown placed on it. The other teeth
diagnosed with needing crowns were 3 and 14.

648. The patient is showing signs of moderate gum recession that is localized to #24 and 25. What is
the likely cause?

a. Failure to floss
b. Aggressive brushing
c. Chewing fingernails
d. Irritation from oral piercing

d - Oral piercings may cause irritation in the mouth at the area where they are located. The patient
should be advised that his piercing is contributing to gum recession in this area and should be
removed.

649. During the intraoral examination, the hygienist records a healthy probing depth that has plaque
and bleeds. What PSR score would this tooth receive?

a. 0
b. 1
c. 2
d. 3

b - A PSR score of 1 would be a healthy depth of pocket with plaque but no calculus deposits that
bleeds upon probing. If calculus were present then it would be scored higher.

650. The patient experiences occasional pain near the right Temporomandibular joint. What oral
condition could be contributing to this discomfort?

a. Crepitation
b. Bruxism
c. Acid reflux erosion
d. Failed restorations

b - A patient that grinds or experiences bruxism can place an additional strain on the TMJ area,
causing joint pain and discomfort.

TOP © 2015 - 2023 Tests.com


432
651. What would an appropriate therapy be for his TMJD symptoms?

a. Bite splint
b. Sport guard
c. Restoring missing and fractured teeth
d. Correcting his oral care habits

a - A bite splint may help relieve TMJD symptoms by preventing the muscles of mastication from
remaining clenched and strained.

652. During nutritional counseling, the patient states that he is an avid gum chewer. Because this
most likely is contributing to his caries rate, which type of gum would best be advised for this
patient?

a. Sugar-free gum
b. Xylitol containing gum
c. Bubble gum
d. No gum

b - Xylitol is known to reduce the risk of dental caries, and is found in select chewing gums.
Chewing gum with xylitol can help patients reduce the chance of developing dental decay.
However, it may want to be pointed out to this patient that chewing gum can further irritate his
TMJD.

TOP © 2015 - 2023 Tests.com


433
Practice Test for the NBDHE Dental Hygienist Exam
Case 14 Testlet
Age: 19 Current Medications:
Sex: F Insulin
Height: 5’5” Birth control pills
Weight: 165
B/P: 156/94 —SEE KAPLAN PAGE 364 Dental History: The patient comes for routine
preventive care appointments but has
Chief Complaint: The patient is here for her consistently developed dental caries as the
routine preventive care appointment. She years have gone by.
expresses concern with one tooth in the lower
right posterior that is sensitive to sweet. She Social History: The patient is in her freshman
says she also has some off and on pain on the year of university and is studying journalism.
lower teeth behind her 12 year molars. She recently joined Weight Watchers and is part
of an on-campus student nutritional support
Medical History: group that she found when taking her required
The patient is Type 2 Diabetic. physical education class. She has made an effort
to begin drinking more bottled water rather
than diet sodas when she purchases drinks at
on-campus vending machines.

TOP © 2015 - 2023 Tests.com


434
653. After the patient was seated in the operatory chair her blood pressure was recorded. Based on
the reading of 156/94, what is the most appropriate measure for the hygienist to take?

a. Recheck her in 5 minutes


b. Perform normal treatment and monitor her at the next visit
c. Perform no treatment and refer her for a medical consultation
d. Her blood pressure is normal

b - The patient’s blood pressure is slightly high, but she can be treated as normal as long as her
blood pressure is monitored at the next few future visits.

654. What classification of hypertension does the patient have?

a. Normal
b. Stage I
c. Stage II
d. Stage III

c - Stage I hypertension is for blood pressure levels at 130-139 over 80-89. Numbers higher than
that will fall under Stage II. This patients BP was 156/94.

655. The hygienist asks the patient what her last blood glucose reading was. A normal level will be:

a. 126-240 mg/dL
b. 100-125 mg/dL
c. 70-99 mg/dL
d. Less than 70 mg/dL

c - Healthy blood glucose readings are in the 70-99 mg/dL range.

656. What should be kept on hand in the dental office should a patient like this have a hypoglycemic
episode?

a. Diet soda
b. Splenda
c. Orange juice
d. Bottled water

c - Orange juice, icing or some other source of sugar should be kept on hand in the dental office
should an emergency arise when a patient happens to have a hypoglycemic episode. This allows
them to quickly get sugar back into their system and stabilize their blood sugar level.

TOP © 2015 - 2023 Tests.com


435
657. The difference between Type I and Type II diabetes is that Type II _______.

a. is non-insulin dependent
b. is more serious
c. requires daily insulin injections
d. has a juvenile onset

a - Type II diabetes is non-insulin dependent and may be controlled by dietary changes, but may
also include insulin injections.

658. The patient states that she is trying to choose healthier drinks when she is on campus between
classes and meals. What is the healthiest choice given her diabetic condition?

a. Milk
b. Diet Soda
c. Bottled water
d. Tap water

d - In most instances it is a better choice to drink tap water over bottled water. This is because
many bottled waters do not contain fluoride or have an unknown amount of fluoride. By drinking
tap water in a municipal area she can know she is getting the proper amount of fluoride. Water is
always a healthier choice over other drinks like soda, sports drinks or juices.

659. During nutritional counseling, the hygienist explains that soda is too erosive to the teeth. The
patient should drink diet soda because it is less erosive to teeth.

a. Both statements are true


b. Both statements are false
c. The first statement is true; the second statement is false
d. The first statement is false; the second statement is true

c - Both regular soda and diet soda are erosive to the teeth. Diet soda may not have the additional
sugar contributing to it, but the acidic levels of the solution can also contribute to tooth decay.

660. The patient is currently taking birth control pills. What type of medication could cause birth
control pills to fail?

a. Hyperglycemic medications
b. Blood pressure medications
c. Diet pills
d. Antibiotics

d - Antibiotic therapy may cause birth control medications to have a lapse in coverage. A
contingent method of birth control should be used if antibiotics need to be taken.

TOP © 2015 - 2023 Tests.com


436
661. Because the patient is diabetic, she is more prone to:

a. Increased saliva flow


b. Periodontal disease
c. Enamel hypercalcification
d. Pyogenic granuloma

b - People with diabetes are more susceptible to developing gingivitis and periodontal disease
conditions.

662. The patient shows mild erosion on the lingual surfaces of the maxillary anterior teeth. What is
the probable cause of this condition?

a. Increased acidic diet


b. Whitening the teeth
c. Bulimia
d. Sucking on lemons

c - Bulimia often is evident clinically by the loss of enamel (erosion) on the lingual surfaces up the
upper anterior teeth, due to the stomach acids coming into frequent contact with these teeth. The
patient should receive nutritional counseling and support.

TOP © 2015 - 2023 Tests.com


437
Practice Test for the NBDHE Dental Hygienist Exam
Case 15 Testlet
Age: 34 Current Medications:
Sex: M Procardia (for blood pressure)
Height: 5’ 10”
Weight: 149 Dental History:
B/P: 165/105 The patient has a history of fairly healthy teeth.
Chief Complaint He has only had a few cavities in the past and
has no problems with gingivitis.
Medical History:
The patient is a smoker and smokes Social History:
approximately one pack of cigarettes each day. The patient is in a bowling league and competes
He also is under the care of his physician for every Tuesday night. His son plays little league
high blood pressure. baseball, and he is an assistant coach for the
team.

TOP © 2015 - 2023 Tests.com


438
663. Based on the patient’s blood pressure reading, what is the next appropriate step?

a. Refer the patient for a medical consult


b. Have patient seek immediate medical attention
c. Retake the blood pressure in 5 minutes
d. Perform normal treatment

c - Retake the patient’s blood pressure in 5 minutes to see if it has stabilized. Many people have
elevated blood pressure when visiting the dental office. If it does not go down, do not perform any
dental treatment and refer the patient for a medical consultation.

664. The patient has enlarged, fibrous papillae in the anterior portion of the mouth. What is the likely
cause?

a. Inadequate home care


b. Elevated blood pressure
c. Drug induced
d. Tobacco use

c - Medications such as Procardia can cause gingival overgrowth, which results in fibrous and
enlarged papillae, usually in the anterior regions of the mouth.

665. Which of the patient’s teeth is supererupted?

a. 2
b. 16
c. 17
d. 32

b - Tooth #16 has supererupted due to the lack of occlusal forces from the tooth in the opposite
arch.

666. During the tobacco cessation portion of the hygienist’s assessment, she should tell the patient
that smoking is a risk factor for all of the following EXCEPT:

a. Obesity
b. Lung cancer
c. Heart disease
d. Periodontal disease

a - Smoking is not a risk factor for obesity. It is a risk factor for lung disease, chronic obstructive
lung disease, heart disease, cancers of the head and neck and periodontal disease.

TOP © 2015 - 2023 Tests.com


439
667. In addition to tobacco placing this patient at a higher risk for oral cancer, what other factors
would place him at a higher risk?

a. Alcohol use
b. HPV
c. Sun exposure
d. All of the above

d - All of the above factors could place a patient at an increased risk to develop oral cancer.

668. During the oral cancer screening, what portion of the lips is a frequent site of oral cancer and
should be checked by the hygienist?

a. Vermillion zone
b. Philtrum
c. Angular commisures
d. Labial commisures

a - The vermillion zone of the lip (the area between the skin of the face and the oral mucosa) is a
common site of oral cancer.

669. The patient’s hard palate has small red spots that are likely caused by his smoking habit. What is
this called?

a. Stomatitis
b. Ecchymosis
c. Hematoma
d. Sialometaplasia

a - Nicotine stomatitis is often seen in the hard palate of patients with a smoking habit. It appears
as small red spots across the roof of the mouth.

670. What stage of hypertension is this patient currently classified under?

a. Normal
b. Stage II
c. Stage III
d. Stage I

b - Stage II hypertension indicates an individual has a blood pressure reading greater than 140 over
90.

TOP © 2015 - 2023 Tests.com


440
671. If this patient were to go into cardiac arrest and require CPR, approximately how deep would
chest compressions need to be in order to be effective?

a. 1.5-2cm
b. 6-7.5 cm
c. 2.5-4 in
d. 1.5-2 in

d - Adult CPR requires compressions approximately 1.5-2 inches deep.

672. If the patient needed to have oxygen administered during their appointment, what color tank
would the oxygen be delivered from?

a. Blue
b. Red
c. Green
d. Orange

c - Oxygen cylinders are always green.

TOP © 2015 - 2023 Tests.com


441
Practice Test for the NBDHE Dental Hygienist Exam
Case 16 Testlet
Age: 54 Current Medications:
Sex: M None
Height: 5’11”
Weight: 172 Dental History:
B/P: 131/92 The patient had several dental implants placed
4 years ago in areas where he was missing
Chief Complaint: “I seem to get cold sores very permanent teeth.
frequently.”
Social History:
Medical History The patient is a retired US marine who now
Knee replacement - 4 months ago operates a small welding business out of his
home.

TOP © 2015 - 2023 Tests.com


442
673. The patient complains of frequent cold sores. What virus causes this condition?

a. HPV
b. HSV
c. PPV
d. HIV

b - The herpes simplex virus (HSV) causes herpetic ulcerations such as “cold sores” and “canker
sores.”

674. What preventive method does the hygienist need to use to ensure she does not contract the
virus from the patient?

a. Double glove
b. Avoid use of the ultrasonic
c. Pre procedural mouth rinse
d. None of the above

d - None of the above are correct. Any patient with a virus or communicable disease must be
treated using universal precautions - precautions that protect the provider and the patient no
matter what the health condition.

675. What condition could possibly occur if the hygienist did not use universal precautions and
contracted the herpes virus in her nail bed?

a. Headache
b. Irritability
c. Herpetic whitlow
d. Finger loss

c - Herpetic whitlow can occur in the nail of a hygienist if universal precautions are not used
around patients with the herpes simplex virus.

676. Because the patient’s knee was replaced 4 months ago, what is the necessary preventive step
for their dental treatment?

a. None, 3 months have lapsed since her joint replacement


b. Antibiotic premedication one hour before the appointment
c. Antibiotic premedication one day before the appointment
d. No dental treatment until 6 months have lapsed

b - Antibiotic premedication should be used on all patients that have had a joint replacement in
the last 6 months. Premedication is taken one hour prior to their appointment.

TOP © 2015 - 2023 Tests.com


443
677. What type of dental implants has the patient had put in place?

a. Intramucosal
b. Endosseous
c. Subperiosteal
d. Endodontic

b - The patient’s implants are endosseous - they extend into the bone. Endosseous implants make
up more than 90% of the dental implants now performed.

678. What type of scaler should be used on the dental implants to prevent scratching of the titanium
material?

a. Plastic
b. Stainless steel
c. Carbon steel
d. Metal ultrasonic tip

a - Plastic scalers must be used on dental implants to prevent damage to the implant.

679. During the extraoral assessment, the TMJ is noted to have a rough, grinding feeling upon
opening and closing. What is this condition called?

a. Deviation
b. Disk degeneration
c. TMJD
d. Crepitus

d - Crepitus or crepitation is the rough feeling felt by the practitioner as the TMJ is palpated during
opening and closing.

680. During the extraoral assessment, what should be felt for during palpation of the lymph nodes?

a. Swelling
b. Hardness
c. Enlargement
d. All of the above

d - All of the above should be checked for when palpating the lymph nodes, as well as pain and
fixed position.

TOP © 2015 - 2023 Tests.com


444
681. Which of the following is NOT something that needs to be recorded about an abnormal skin
lesion on the patient’s face?

a. Color
b. Size
c. Date of origin
d. Shape

c - The date of origin is not something that is charted in regards to an abnormal skin lesion. Things
like size, shape, texture and color should all be noted.

682. At what point in the life cycle of herpetic ulcerations should treatment be avoided?

a. Initial tingling
b. Active ulceration
c. Dried ulceration
d. Treatment should not occur during any stage of herpetic ulcerations

b - If a patient has an active ulceration that has not dried and crusted over yet, then treatment
should be avoided until it does.

TOP © 2015 - 2023 Tests.com


445
Practice Test for the NBDHE Dental Hygienist Exam
Case 17 Testlet
Age: 62 Current Medications:
Sex: F Rituxan (for RA)
Height: 5’4”
Weight: 151 Dental History:
B/P: 145/95 The patient has had extensive restorative
treatment over the past decade. She comes in
Chief Complaint: The patient is unhappy with about every 12 months to have her teeth
the way her upper front teeth appear to be cleaned despite the last hygienist’s efforts to
“longer” than the other teeth. She also encourage more frequent appointments.
complains of yellow plaque built up on the
lower teeth. She started using a new toothpaste Social History:
and reports that her teeth are sometimes The patient spends 2 days a week volunteering
sensitive. at a thrift-store that benefits the county
hospital. She would enjoy working more
Medical History: frequently, but she is not physically comfortable
The patient has rheumatoid arthritis. enough to do so. She enjoys reading books and
has a small dog, named “Tito.”

TOP © 2015 - 2023 Tests.com


446
683. What habit might be contributing to the gum recession in the area of the patient’s upper
anterior teeth?

a. Sucking lemons
b. Aggressive flossing
c. Bone health
d. Abrasive brushing

d - Abrasive brushing is a bad habit that can cause severe gum recession as well as enamel
abrasion.

684. To remove the patient’s moderate supragingival calculus, the hygienist selects an electronic
scaler that produces 25,000-50,000 vibrations per second and vibrates in a linear direction. What
type of scaler did she choose?

a. Sonic
b. Ultrasonic piezoelectric
c. Ultrasonic magnetorestrictive
d. Air polisher

b - Only piezoelectric scalers move in a linear direction. Sonic and magnetorestrictive move in an
elliptical direction.

685. What would better help prevent moderate calculus deposits on the patient’s teeth?

a. Changing toothbrushes
b. More frequent prophylaxis appointments
c. Using tartar control toothpaste
d. Nothing, it has to do with her metabolism

b - Encouraging the patient to be seen for more frequent prophylactic appointments can prevent
tartar deposits from becoming so large on her teeth.

686. Which tooth is the patient missing?

a. #4
b. #6
c. #3
d. #10

a - Tooth #4, the second bicuspid, is missing. The first molar (#3) has come forward some.

TOP © 2015 - 2023 Tests.com


447
687. What orofacial region may the patient experience some difficulties with due to her RA?

a. TMJD
b. TMJ
c. Delayed eruption
d. Decreased appetite

b - The temporomandibular joint may have some problematic symptoms due to the patient’s RA,
as RA greatly affects joints.

688. To discourage pain in the TMJ during the dental appointment, what can the hygienist do?

a. Apply a warm compress to the area


b. Have the patient premedicate with an NSAID
c. Use a bite block to help the patient keep her mouth open
d. Take frequent breaks and allows the patient to rest her jaw

d - Frequent breaks during the appointment can help the patient rest her joint, and prevent
straining it by keeping it open too long.

689. The patient has a bony hard protuberance in the midline of their palate. What is this called?

a. Torus mandibularis
b. Torus palatinus
c. Odontoma
d. Exostoses

b - Torus palatinus, or tori is a bony protuberance in the roof of the mouth.

690. When choosing a curette for instrumentation, the hygienist selects a universal curet rather than
an area specific curette. What is the difference between the two types?

a. The lower shank of the universal curette is longer.


b. The face of the area specific curette is smaller.
c. The there are two cutting edges on a universal curette.
d. The toe of the universal curette is rounder.

c - A universal curette has two cutting edges on the working face of the blade. An area specific
curette has only one working edge.

TOP © 2015 - 2023 Tests.com


448
691. During hand scaling of this patient, the hygienist should have her forearms and hands _______.

a. parallel to the floor


b. slightly lowered below the level of her elbows
c. slightly elevated above her elbows
d. where the arms are parallel to the floor and the hands are perpendicular to the floor

a - The clinician’s forearms and hands should be parallel to the floor, level with the elbows in order
to prevent muscle strain.

692. During scaling, the hygienist uses a fulcrum rest, which uses a finger from her non-dominant
hand to concentrate lateral pressure against the tooth in order to control the instrument stroke.
What type of fulcrum is this?

a. Basic intraoral
b. Advanced piggy-back
c. Advansed finger-on-finger
d. Advanced stabilized

d - Advanced stabilized fulcrums use the non-dominant hand to provide lateral pressure in order to
control instrument strokes against the tooth, to prevent slips or tissue trauma.

693. The patient reports having just recovered from shingles a few months ago. What virus causes
shingles?

a. Herpes zoster
b. Hepatitis C
c. Coxsackievirus
d. Epstein-Barr virus

a - The herpes zoster virus is a reactivation of the varicella zoster virus, which also causes
chickenpox and causes shingles typically in older adults.

TOP © 2015 - 2023 Tests.com


449
Practice Test for the NBDHE Dental Hygienist Exam
Case 18 Testlet
Age: 56 Dental History:
Sex: F The patient has had relatively healthy teeth in
Height: 5’3” the past 4 years. Prior to that she had a history
Weight: 132 of scaling and root planing associated with
B/P: 125/85 active periodontal disease. She had periodontal
maintenance visits every 3 months, then 4
Chief Complaint: months and has now extended recare visits to
The patient has no concerns at this time. She is every 6 months. The patient missed her last 2
here for her scheduled preventive care appointments due to family reasons.
appointment.
Social History:
Medical History: The patient is a sales manager at a local clothing
The patient is anemic and also has retailer. The patient’s mother was diagnosed
hyperthyroidism. She is under the care of her with cancer 18 months ago and passed away 5
physician and was last seen 4 months ago. months ago. The patient was caring for her
mother and as a result was not able to make it
Current Medications: to all of her dental appointments because she
Iron supplement had already missed so much work.
Synthroid

TOP © 2015 - 2023 Tests.com


450
694. The difference in iron-deficiency anemia and other anemia is that iron-deficiency anemia
requires _____________.

a. B12 supplementation
b. antibiotic premedication
c. blood transfusions
d. only iron supplementation

d - Iron-deficiency anemia only requires iron supplementation for its treatment. Other anemia
types may include needs such as b12 supplementation, blood transfusions, antibiotic therapy, and
bone marrow transplants.

695. Which of the following is NOT a clinical feature associated with hyperthyroidism that might be
seen in this patient?

a. Large jaws
b. Rapidly accelerated caries
c. Delayed tooth eruption
d. Osteoporosis of the alveolar bone

c - In hyperthyroidism it is typical to see accelerated tooth eruption rather than delayed eruption.

696. What features may be visible during the extraoral exam of a patient with hyperthyroidism?

a. Sunken eyes
b. Thick hair
c. Weight gain
d. Dry, cracked skin

d - Patients with hyperthyroidism typically have increased sweating and moist skin.

697. The furcation involvement at tooth #18 is visible clinically. What class of furcation involvement is
this?

a. Class V
b. Class III
c. Class IV
d. Class II

c - Class IV furcations are the highest level of furcation involvement. They are visible clinically
without probing of the furcation area. Probes will extend through the entire furcation.

TOP © 2015 - 2023 Tests.com


451
698. What type of film would be necessary to achieve a high quality view of the apex and bone
around #18?

a. Bitewing
b. Periapical
c. Panoramic
d. Digital

b - A periapical film would allow the clinician and dentist to have a high-quality view of the apex of
the tooth, and inspect the level of the surrounding bone.

699. During the intraoral assessment, the mouth mirror continues to fog up. What options can the
clinician use to prevent this?

a. Ask the patient to breathe through the nose


b. Rub the mirror along the cheek to coat with saliva
c. Use defogger or warm water to rinse the mirror
d. All of the above

d - All of the options listed above are useful to help prevent fogging of the dental mirror during an
appointment.

700. The hygienist is having a difficult time scaling a piece of subgingival calculus off of the distal of
#27. What portion of the toe on a universal curette should be used for deposit removal?

a. Lower 1/3
b. Lower 2/3
c. Upper 3/4
d. The entire toe

a - Only the lower 1/3 of the toe on a universal curette should be used for deposit removal. The
clinician should reassess their instrumentation to reflect this.

701. The patient requests nitrous oxide use during the remainder of her appointment. At what level is
the average desired effect of nitrous oxide achieved?

a. 10%
b. 60%
c. 35%
d. 55%

c - Approximately 35% nitrous oxide is optimum the desired effect on a dental patient by anxiety
as well as decreasing pain stimulation.

TOP © 2015 - 2023 Tests.com


452
702. The patient has continued using a sensitivity toothpaste to prevent hypersensitivity in areas of
recession. Which of the following ingredients is NOT an active ingredient of sensitivity
toothpastes?

a. Potassium nitrate
b. Hydrogen peroxide
c. Calcium hydroxide
d. Sodium citrate

b - Hydrogen peroxide is NOT an active ingredient of sensitivity toothpastes. It is often found in


whitening products and may actually contribute to tooth sensitivity.

703. The patient asks about irrigating around existing pockets that tend to pack food. How deep
below the gingival margin should the hygienist tell her is safe enough to place a rubber tip
irrigator?

a. Only supragingivally to the margin


b. 2mm below the margin
c. 5mm below the margin
d. As deep as comfortably possible

b - Rubber tip irrigators may be safely used by placing them up to 2mm below the margin of the
gums and irrigating periodontal pockets underneath.

704. The hygienist decides to use an air-polishing device to remove some of the tenacious stain that
has built up since the patient’s last appointment. Approximately what psi does an air pressured
polisher function at?

a. 40-100
b. 75-150
c. 150-200
d. 175-250

a - Air polishers best function around 40-100psi, and use a combination of air and powder to
remove the stain from the teeth. A water-powered polisher would use 20-60psi.

TOP © 2015 - 2023 Tests.com


453
Practice Test for the NBDHE Dental Hygienist Exam
Case 19 Testlet
Age: 16 Current Medications:
Sex: F Albuterol prn
Height: 5’7”
Weight: 145 Dental History:
B/P: 111/76 The patient has not been seen for 2 years. At
her last appointment her teeth were cleaned
Chief Complaint: ”I’m unhappy with the way my and decay was diagnosed as a small DO on #29.
teeth look. I am about to get braces. My
orthodontist told me to have my teeth cleaned Social History:
and cavities filled before he will put the braces The patient is a student at the local high school.
on my teeth.” She plays trumpet in the school band.

Medical History
The patient has asthma. Her last attack was 8
months ago.

TOP © 2015 - 2023 Tests.com


454
705. Which of the patient’s teeth appear to have an incomplete root canal?

a. #19
b. #20
c. #29
d. #30

a - Tooth #19 appears to have an incomplete root canal. There is evidence of a radiopaque
material in a portion of the canals but not the entire cavity.

706. During the extraoral assessment, the patient exhibits a butterfly-shaped rash across the face.
This is a classic feature of what condition?

a. Pemphigus vulgaris
b. Lupus
c. Erythema multiforme
d. Mono

b - Lupus clinically is seen as a butterfly configuration on the face and the nose. It is a chronic
inflammatory autoimmune disease.

707. The patient states she does not brush along the gumlines because they bleed easily. What is the
likely cause?

a. Irritation due to albuterol use


b. Periodontal disease
c. Marginal inflammation
d. Trauma

c - Marginal inflammation due to inadequate plaque removal is a common cause of bleeding,


uncomfortable gums during brushing. The patient should be educated on proper brushing
methods to reduce bleeding and gingival inflammation.

708. According to Black’s classification of caries, tooth #9 has what class of decay based on the
appearance of the radiograph?

a. II
b. IV
c. III
d. VI

c - Class III caries includes proximal areas of the incisors and canines, but does not include the
incisal edge.

TOP © 2015 - 2023 Tests.com


455
709. Which of the following appearances of gum tissue evident in the patient’s mouth would be
considered healthy?

a. Soft
b. Coral/pink
c. Shiny with thin epithelial tissue
d. Blunted

b - Healthy gums will appear coral pink in color. They are also firm, pointed in contour and have
smooth margins.

710. The patient states that she can feel a small pit in the back of her anterior teeth. What teeth are
capable of having a lingual pit?

a. Maxillary central incisors


b. Mandibular central incisors
c. Maxillary lateral incisors
d. Maxillary canines

c - Maxillary lateral incisors may have a natural lingual groove or lingual pit because they typically
have a more prominent cingulum.

711. During the intraoral assessment, the clinician sees one of the teeth has a Cusp of Carabelli.
Which tooth would this be on?

a. Mandibular first molar


b. Maxillary first molar
c. Maxillary second molar
d. Mandibular second molar

b - The cusp of Carabelli only develops on the maxillary first molar. It is the 5th lobe and found on
the mesiolingual cusp.

712. What should the hygienist tell the patient to do in order to reduce the amount of gingival
bleeding?

a. Use an antiseptic mouth rinse


b. Use an electric toothbrush
c. Brush along the gum margin
d. Rinse with warm salt water

c - Brushing along the gum margin is the only way to improve oral hygiene and reduce bleeding
caused by excess plaque. The patient should be informed that it may take several days prior to
seeing an improvement in bleeding.

TOP © 2015 - 2023 Tests.com


456
713. What supplement may help the patient maintain healthy enamel during their orthodontic
therapy?

a. Fluoride
b. Proxa-brush
c. Electric toothbrush
d. Antiseptic mouth rinse

a - Fluoride, when used in conjunction with efficient oral hygiene, can prove to be a valuable
supplement that increases the health of enamel, especially in patients with braces or poor oral
hygiene.

714. The patient has green stain on the lingual of the upper anterior teeth, typical of stain found in
persons that smoke illegal substances. What polishing method would be appropriate for the
removal of this stain?

a. Air polishing
b. Fine coarse pumice
c. Rubber cup polishing
d. Selective polishing

d - Selective polishing should be the preferred method of stain removal in all patients. Prolonged
polishing can cause the removal of enamel and tooth structure, in addition to scratching the
surface of the tooth.

715. What should the patient be advised to do after using her albuterol inhaler?

a. Rinse with water


b. Brush his teeth
c. Rest for 5 minutes
d. Do not brush his teeth for 1 hour

a - Rinsing with water after the use of albuterol can reduce the risk of tooth staining, taste
alteration, and dental caries.

TOP © 2015 - 2023 Tests.com


457
Practice Test for the NBDHE Dental Hygienist Exam
Case 20 Testlet
Age: 22 Current Medications:
Sex: F Prenatal vitamin
Height: 5’1”
Weight: 125 Dental History:
B/P: 118/74 The patient has had a few fillings in the past,
but generally possesses good overall oral
Chief Complaint: hygiene
The patient is due for a preventive care
appointment. She also states that she had a Social History:
weird growth on her gums but that it went The patient was an accountant at a credit union
away. However she is finding that she is but decided to be a stay at home mother after
experiencing some irritation on her tongue. giving birth to her child. She is currently doing
some freelance work from her home a couple of
Medical History: days each week.
The patient had a C-section 6 months ago. She
is currently breastfeeding her infant.

TOP © 2015 - 2023 Tests.com


458
716. What is the hormone-induced growth that is likely the cause of the lesion on the patient’s gums
that went away after having her baby?

a. Papillary hyperplasia
b. Fibrous hyperplasia
c. Pregnancy granuloma
d. Peripheral granuloma

c - A pregnancy granuloma, also called pregnancy tumor or pyogenic granuloma, is commonly seen
in patients that have changes in hormonal levels such as related to pregnancy.

717. What condition would cause the patient to have a map-shaped erythematous lesion with a
white border on the dorsal surface of her tongue?

a. Geographic tongue
b. Median rhomboid glossitis
c. Linea alba
d. Elongation of filiform papillae

a - Geographic tongue, also called benign migratory glossitis, is a map-shaped lesion that changes
shapes and sizes on a person’s tongue. It has a defined border and usually involves the
desquamation of filiform papilla.

718. The patient states she has increased sensitivity in her upper anterior teeth since her pregnancy.
Which of the following is the likely cause?

a. Hormonal changes
b. Rampant gum recession
c. Acid erosion
d. Homeopathic toothpaste use

c - Acid erosion due to excess vomiting or heartburn associated with her pregnancy may have
occurred, which would contribute to tooth sensitivity.

719. The patient states she has had some tissue sloughing and that is why she changed toothpastes.
What was most likely an ingredient in the last toothpaste that would have caused the sloughing?

a. Fluoride
b. SLS
c. Sodium bicarbonate
d. Xylitol

b - Sodium laurel sulphate (SLS) commonly irritates oral mucosa and is found in some toothpastes.
It is now usually replaced with lauryl sarcosinate.

TOP © 2015 - 2023 Tests.com


459
720. During oral hygiene education, the hygienist addresses care of the patient’s newborn child’s
teeth. What information would be accurate?

a. Prolonged nursing age can lead to caries


b. Prolonged nursing age can lead to inadequate jaw growth
c. Cleansing the baby’s mouth after breastfeeding is essential
d. The baby should have his first exam by 6 months of age

c - Cleansing the baby’s mouth after breastfeeding or bottle feeding can reduce the prolonged
exposure of milk on the surface of the teeth, thereby decreasing the rate of decay.

721. What should the hygienist recommend to brush the baby’s teeth with, once his first teeth erupt?

a. A washcloth
b. A soft bristled toothbrush with tap water
c. A brush with a bb sized amount of toothpaste
d. The mother’s toothbrush and tap water

b - A soft bristled toothbrush and tap water are adequate to clean the baby’s teeth. A child should
not use fluoridated toothpaste until they are old enough to expectorate and rinse.

722. At what age can she expect her baby to have his first tooth erupt?

a. 6-10 months
b. 8-13 months
c. 11-15 months
d. 12-18 months

a - Most children typically have their first tooth erupt around 6-10 months of age. The lower
central incisors are typically the first teeth to erupt.

723. What age should the patient expect her son to have his first permanent tooth erupt?

a. 5 years
b. 6 years
c. 8 years
d. 12 years

b - The first permanent teeth to erupt are usually the lower central incisors or the first molars.
These teeth typically erupt around age 6.

TOP © 2015 - 2023 Tests.com


460
724. During probing, the patient’s UR quadrant has plaque but no calculus, has no pockets deeper
than 2 mm and bleeds on probing at 2 teeth. What PSR score will be assigned?

a. 0
b. 1
c. 2
d. 3

b - A PSR of 1 will be assigned to this quadrant. If there was no bleeding or plaque it would score a
0, and if there were tartar present it would be a 2.

725. How will the hygienist conduct the intraoral examination to areas around the tongue?

a. She will ask the patient to move her tongue to the side
b. She will retract the tongue with her mirror
c. She will hold the tongue with gauze
d. She will feel by palpitation

c - Intraoral assessments around the tongue need to be conducted by gently holding the tongue
with a piece of gauze.

726. The hygienist assigns the patient a plaque score of 1. What does this mean?

a. There was no plaque


b. There was plaque at the gingival margin
c. There was plaque within the gingival margin
d. There was gross soft debris within the pockets and margins

b - A plaque score of 1 indicates that there is only plaque film along the gingival margin, but not
within it.

TOP © 2015 - 2023 Tests.com


461
Practice Test for the NBDHE Dental Hygienist Exam
Case 21 Testlet
Age: 7 Current Medications:
Sex: M None
Height: 4’3”
Weight: 65 Dental History:
B/P: 107/70 Patient just moved to the area and is a new
patient of record. He has not been to a dentist
Chief Complaint: in one year. Previous to the move, he was seen
The parents have noticed that one of the once a year by a general dentist.
permanent teeth has not erupted in the place
of the deciduous tooth. It has been about 4 Social History:
months since the child lost the tooth. Patient is in 1st grade at a local public school.
Loves playing sports and is always outside
Medical History: playing with his brother and sister.
Patient had his adenoids and tonsils removed
when he was 4 years old, due to many throat
and ear infections

TOP © 2015 - 2023 Tests.com


462
727. During the appointment, the patient receives a topical fluoride treatment. Between what ages is
fluoride most effective in preventing cavities?

a. 6 months to 14 years old


b. 6 months to 21 years old
c. 12 months to 16 years old
d. 12 months to 21 years old

a - 6 months to 14 years old is the most effective age range for fluoride, since these are the ages of
tooth development and eruption. By the age of 14 years old all permanent teeth are erupted.

728. The dentist proceeds to check on the eruption patterns of the child. When looking at the x-ray, is
the child missing a permanent tooth?

a. Yes, he is missing #7.


b. Yes, he is missing number #21.
c. Yes, He is missing #30.
d. No, he is not missing any permanent teeth.

d - He is not missing any permanent teeth. All necessary teeth are developing. Sometimes, when a
primary tooth is lost, a permanent tooth does not erupt right away. It is important to check on
eruption patterns periodically through x-rays and oral exams, as some children may be a case for
early orthodontics.

729. When looking at the same panoramic image, you see a dark, circular area distal to tooth #18.
What are you looking at?

a. Static bone cyst


b. Traumatic bone cyst
c. Tooth bud
d. Abscess

c - The dark, circular area that is distal to #18 is a tooth bud. It is specifically tooth #17 developing
into a wisdom tooth. You can see there is a dark, circular area distal to tooth #31 as well.

730. The mother is concerned about a permanent tooth missing. Your office does not have a
panoramic x-ray machine. What is the next best type of x-ray to take?

a. Bitewings
b. Periapical
c. Occlusal
d. Full-mouth series

TOP © 2015 - 2023 Tests.com


463
c - Though you could take a periapical, it would be best to take an occlusal image. When teeth are
unerupted after losing a primary tooth, impaction may be a concern. An occlusal image would give
you the best image to see where the tooth is and if it is impacted.

731. The dentist notes the child has excessive crowding and will need to be referred to a specialist.
What specialist would the dentist refer the parents and child to?

a. Orthodontist
b. Periodontist
c. Prosthodontist
d. Endodontist

a - Orthodontists specialize in the movement of teeth and aligning the jaw. Though all general
dentists are trained to be able to provide orthodontics, most dentists prefer to refer their patients
to the specialists.

TOP © 2015 - 2023 Tests.com


464
Practice Test for the NBDHE Dental Hygienist Exam
Case 22 Testlet
Age: 57 Current Medications:
Sex: M Lisinopril q.d. baby aspirin q.d.
Height: 6’0”
Weight: 205 Dental History:
B/P: 128/84 Patient has been going to the dentist on and off
throughout his life. Had scaling and root planing
Chief Complaint: 10 years ago, along with extractions of most of
After having his implants placed 6 months ago, his mandibular molars.
the patient is ready to proceed with his
removable prosthesis. Social History:
Patient has two children and recently became a
Medical History: grandfather for the first time. The patient wants
Smoker since the age of 16, but quite 2 years to be able to smile again, since he hasn’t smiled
ago. since losing his teeth many years ago.

TOP © 2015 - 2023 Tests.com


465
732. Looking at the x-ray, what type of implant was used?

a. Endosteal
b. Osseointergrated
c. Subperiosteal
d. Transosteal

a - The patient received endosteal implants. They are the most common type of implant placed.
The endosteal implant is surgically placed in the jawbone, whereas a subperiosteal implant is
placed on top of the bone. Transosteal implants are primary used in edentulous patients.
Osseointergration is the process on the bone growing around the implant.

733. Considering the dental history of this patient, what type of prosthesis is the patient likely to be
completing?

a. Denture
b. Partial denture
c. Bridge
d. Crown

b - The patient is receiving a partial denture. The implants will allow for a better, tighter fit for the
patient. Most mandibular partials can be hard to fabricate to where they are comfortable to the
patient. That is why implants are placed, to help with retention.

734. Looking at the x-ray, what is the radiopaque image in the lower left and right corner?

a. Spine
b. Vertebrae
c. Hyoid bone
d. Mandible

c - The partial image is that of the hyoid bone. The hyoid bone is u-shaped and located in the neck.
This particular bone is there to help support the tongue.

735. All of the following, except one, are instruments used in debridement around dental implants.
Which one is the exception?

a. Plastic resin
b. Graphite
c. Stainless steel
d. Gold-tipped

c - Stainless steel cannot be used in the debridement around dental implants. The material can
create micro abrasions that can collect bacteria and cause the implant to fail.

TOP © 2015 - 2023 Tests.com


466
736. What type of fixed prosthesis is being utilized in quadrant #1?

a. Implant
b. Maryland bridge
c. Cantilever Bridge
d. Crown

c - A cantilever bridge is a fixed bridge that is only supported by one adjacent tooth. These types of
bridges are not common, since have been proven to cause damage to the supporting tooth.

TOP © 2015 - 2023 Tests.com


467
Practice Test for the NBDHE Dental Hygienist Exam
Case 23 Testlet
Age: 78 Current Medications:
Sex: F donepezil q.d. galantamine extended release
Height: 5’4” q.d.
Weight: 164
B/P: 132/84 Dental History:
Patient has not been into a dentist in two years.
Chief Complaint: The patient is new to the clinic.
Husband has noticed that some of her teeth are
broken and that he noticed her bad breath Social History:
starting a couple of weeks ago. Before being diagnosed with Alzheimer’s
disease, the patient was active in the
Medical History: community and was part of a gardening club.
Patient was diagnosed with Alzheimer’s disease She raised three kids in their small town.
about 1.5 years ago. The husband has been able
to care for his wife since the diagnosis. She had
been diagnosed with Raynaud’s disease when
she was in her 30’s.

TOP © 2015 - 2023 Tests.com


468
737. The patient is very cooperative and is allowing you to look in her mouth. When trying to do your
intraoral exam, you notice that the patient has tightening of skin around her mouth, making it
hard for her to open wide. What is the cause?

a. Oral cancer
b. Scleroderma
c. Candida infection
d. Herpes virus

b - Scleroderma is the thickening and tightening of skin. When this is localized, it causes changes
mostly on the hands and face. Raynaud’s disease, which affects the small blood vessel of the
fingers, toes, ears and nose, is often the first sign of scleroderma.

738. The patient has been having a hard time brushing her teeth or forgetting to do so altogether.
Since she has many mandibular teeth that are infected beyond repair, what type of appliance
would the dentist recommend to help the patient restore chewing function?

a. Partial Denture
b. Denture
c. Implants
d. Fixed Bridges

a - A partial denture would most likely be the prosthesis the dentist would recommend. It would
be the easiest way to restore chewing function to the patient considering there are many viable
teeth in the mandibular region to support the prosthesis.

739. Which of the following is not a material used in making a partial denture and denture teeth?

a. Acrylic resin
b. Composite resin
c. Porcelain
d. Glass ionomer

d - Glass ionomers are used in prosthesis. Most partials and dentures are made of a resin material.
This makes them easy to clean and longer lasting.

740. What is the proper way to clean a removable prosthesis in the dental clinic?

a. Use water and a hard tooth brush


b. Put denture in disinfectant, clean in an ultrasonic and brush with a denture brush
c. Put denture in disinfectant and place in the ultrasonic
d. Place in ultrasonic for 10 minutes and rinse with water

b - The proper way to clean a removable appliance is by placing the prosthesis in a disinfectant
solution and then agitating it in the ultrasonic. This is affective in loosening the debris. The use of a

TOP © 2015 - 2023 Tests.com


469
denture brush is recommended, since it is designed not so scratch or dent the resin material of the
prosthesis.

741. All of the following are soft tissue considerations regarding the use of a prosthesis, except:

a. Angular cheilitis
b. Stomatitis
c. Traumatic ulcers
d. Geographic tongue

d - Geographic tongue is irregular, smooth red patches on the dorsal surface of tongue. The exact
cause of geographic tongue is unknown, but has been linked to a vitamin B deficiency. It is not
relevant to or considered a factor in the use of a prosthesis.

TOP © 2015 - 2023 Tests.com


470
Practice Test for the NBDHE Dental Hygienist Exam
Case 24 Testlet
Age: 14 Current Medications:
Sex: M None
Height: 5’9”
Weight: 140 Dental History:
B/P: 107/78 Patient has a full set of braces for over a year.
Doesn’t know when he will be getting them
Chief Complaint: removed. Pt had #14 extracted before braces
Parents would like to know if he is ready to get due to decay.
his wisdom teeth out. He is also having some
soreness near his tonsils again. Social History:
Patient is on the junior varsity football team,
Medical History: plays the quarterback position. Sips on sports
Had strep throat 6 weeks ago and finished a full drinks throughout practice and during games.
round of antibiotics prescribed by his doctor.

TOP © 2015 - 2023 Tests.com


471
742. The parents are concerned about the patient’s wisdom teeth, since some of his friends have
been getting them removed. The dentist states that he is not ready to have them removed.
What else do you notice from the panoramic x-ray regarding the patient’s wisdom teeth?

a. The patient has 3 wisdom teeth.


b. All of the patient’s wisdom teeth are extracted.
c. The patient has 2 wisdom teeth.
d. All of the wisdom have decay.

a - The patient is congenitally missing #32, which means he has only 3 wisdom teeth. It is not
uncommon for wisdom teeth to be congenitally missing.

743. When doing your intraoral exam, you notice that the patient has something lodged in the folds
near his tonsils. When the doctor does his exam, he dislodges the white, hard nodule with a
cotton swab and says that it is nothing to worry about. What most likely is the nodule the
dentist dislodged?

a. Candida
b. Tonsil stone
c. Papilloma
d. Fibroma

b - Tonsil stones are accumulations of bacteria that harden in the folds and crypts of the tonsils.
They can be covered by bacteria, which can lead to bad breath. If for some reason they cannot be
dislodged, they may need to dislodge by an ENT specialist.

744. When cleaning around the brackets, you notice that there is a white line that has formed
underneath the brackets on tooth #27. It cannot be removed. What mostly likely is the white
line?

a. Decalcification
b. Hypocalcification
c. Linea Alba
d. Fluorosis

a - Decalcification is high among teenagers, but significantly increases with those having braces.
Decalcification looks like a chalky white spot on the teeth. It is formed due to the accumulation of
plaque and poor oral hygiene.

745. What type of brushing technique should one use with braces?

a. Stillmans
b. Bass
c. Charters
d. Modified Stillmans

TOP © 2015 - 2023 Tests.com


472
c - The best brushing technique for patients with braces is the Charters technique. This is done by
placing the brush at 45-degree angle where the tooth and the tissue meet. The brush is swept
toward the apex of the tooth in a circular vibrating motion followed by a rolling stroke toward the
occlusal surface

746. A patient should wear an athletic mouth guard when playing sports. However, a patient should
not wear an athletic mouth guard if he has a full set of braces.

a. Both statements are true.


b. Both statements are false.
c. First statement is true. Second statement is false.
d. First statement is false. Second statement is true.

c - It is important to protect the teeth, especially during any type of contact sport. Braces or not, a
mouth guard should always be worn. An over-the-counter mouth guard can be purchased or it can
be specifically made for the mouth.

TOP © 2015 - 2023 Tests.com


473
Practice Test for the NBDHE Dental Hygienist Exam
Case 25 Testlet
Age: 32 Current Medications:
Sex: F Prenatal vitamins, folic acid
Height: 5’7”
Weight: 145 Dental History:
B/P: 122/82 Pt routinely comes to the dentist every 6
months and has not had periodontal issues in
Chief Complaint: the past.
“My gums are bleeding more than usual”
Social History:
Medical History: This is the patient’s second child. Has a 2-year-
Pt is 3 months pregnant. Patient is currently old son at home. Patient is a stay-at-home
having morning sickness and is vomiting every mother.
morning.

TOP © 2015 - 2023 Tests.com


474
747. When completing this periodontal evaluation, you notice generalized bleeding upon probing,
but there are not any pockets above the score of three. What most likely is the cause of this
condition?

a. Hormones
b. Poor oral hygiene
c. Trauma
d. Periodontal disease

a - Hormones during pregnancy are constantly fluctuating and leave your mouth more vulnerable
to plaque and bacteria. The best way to combat the generalized bleeding is exceptional homecare.

748. The patient states that she has been vomiting every morning due to the pregnancy. What should
you tell her to do about her tooth brushing?

a. Brush and floss your teeth immediately.


b. Wait until nighttime to brush and floss.
c. Only rinse your mouth, do not brush or floss your teeth until the 2nd trimester.
d. Wait 30 minutes after vomiting to brush and floss your teeth.

d - Vomit is highly acidic and needs time to neutralize in your mouth before you brush and floss
your teeth. Otherwise, brushing your teeth that are coated in acid could cause erosion.

749. If a pregnant patient needed scaling and root planing, when would be the best time to perform
this procedure?

a. You should not perform this procedure while a patient is pregnant


b. 1st trimester
c. 2nd trimester
d. 3rd trimester

c - The second trimester is best, because all of the fetus’s vital organs will have been developed
and there is a lower risk of cutting off blood supply. A patient that is in the supine position for long
periods of time can occlude the blood supply to the heart, leading to a loss of consciousness.

TOP © 2015 - 2023 Tests.com


475
750. When you begin to do the intra oral exam, you notice a lesion on the lower lip, which is shown in
the photo. The patient states that she had it for about 5 days, that she gets them periodically
and that each one lasts about 2 weeks. What is this lesion?

a. Herpes lesion
b. Aphthous ulcer
c. Stomatitis
d. Linea alba

b - Aphthous ulcers are usually recurrent, small round ulcers where the skin is not tightly bound to
the bone. It is not known what causes these particular ulcerations, just what may trigger them.

751. All of the following are known triggers for an aphthous ulcer, except:

a. Hormones
b. Stress
c. Vitamin C
d. Injury

c - Vitamin C could actually help fight against aphthous ulcers, since vitamin C has been known to
boost the immune system and help aid in the fight against infection.

TOP © 2015 - 2023 Tests.com


476
10 Test Preparation Tips
1. Start Studying 3 Months Before The Test - You have a lot of information to review to get prepared.
Give yourself enough time to study all of it in a relaxed state of mind. Trying to cram your study in a
month or a few weeks before the test will just create anxiety and even panic which is not conducive
to learning.

2. Outline a Study Schedule and Stick to It– You first need to find out what subjects the test covers,
then break them down into a study outline. An outline of the material will give you a birds-eye-view
of what you have to cover and allow you to plan to actually study it. Include review days throughout
the schedule where you review material you studied the month or two before. Include practice test
sessions in your schedule as well. Once you have a study schedule established, commit to it and be
disciplined. It will only help you, and give you the benefit of comprehensive study, if you actually
follow it.

3. Study Every Day for at Least One Hour – Getting prepared for a professional certification exam
takes commitment. To maintain this commitment, it is best to make it part of your regular schedule.
Plan an hour a day to study the material you have scheduled for the day.

4. Obtain a Good Study Guide – A good study guide is very important. It will give you the substance
you need to know for the test.

5. Use Flashcards – Flashcards are easy to use and can interject some fun into the study process.
Flashcards that give you a question on one side and an answer on the other are the most effective.
Use them regularly throughout your study schedule.

6. Take Untimed Practice Tests Periodically to Assess Your Knowledge of the Material – Use the
Tests.com Practice Test to find out how well you know the material. For the first couple times, do
not time yourself, but use the test simply to determine your strengths and weaknesses. Focus your
study on the areas of the exam where you had the most trouble.

7. Take a Timed Practice Test Periodically to Practice Test Taking Skills – Take the Tests.com Practice
Test using a timer setting. Determine how many questions are on your state exam and complete
that amount of questions in the allotted time. This exercise will allow you to get a sense of how fast
you need to work under time pressure.

8. Tab and Highlight your Reference Books – Depending on the test, some jurisdictions have open
book tests, allowing you to use a reference book while you take the test. Most testing rules do not
allow notes in the reference book you use, but many allow highlighting and tabbing. When you use
a reference book during a test, it is important to use it in such a way that allows you to work
efficiently and not slow you down. Place colored tabs on the pages of the book referencing the
sections, so you can turn to them quickly and not have look up page numbers in the Table of

TOP © 2015 - 2023 Tests.com


477
Contents. Highlight those sections which you believe to be important and that will be subject to
testing.

9. Meet with Friends who are Studying for the Test and have a Group Discussion - Your friends and
colleagues who are studying for the test will have different strength and weaknesses than you. You
can benefit each other by sharing information, discussing issues and asking each other questions
about the information subject to testing.

10. Don’t Study the Day or Night Before the Test – You have prepared for months. Even though you
may feel a bit anxious the day before the test, it is important that you give your brain a rest. During
the test, you must be clear of mind and able to nimbly move from question to question. If your
brain is tired and your eyes are having trouble focusing, you will put yourself at a great
disadvantage. Do not study late into the night. You know the material more than you realize. Take
the day off, go for a walk, a bike ride or see a movie.

10 Test Taking Tips


1. Get Good Rest the Night before the Test – All the study in the world will not save you if you can’t
focus your eyes and your mind is cloudy due to staying up late at night to study before the test.
Test taking is an art and you must have a clear, well rested mind to do well. An important tip, and
the first in this list for a reason, is to get a good night’s rest the day before the test.

2. Eat a Good Meal before Leaving for the Test – Tests usually last a couple of hours. They take much
concentration and mental energy. You don’t want to have your blood sugar level affect your ability
to concentrate. Eat a good meal before leaving to take the test. Stay away from foods that would
make you tired.

3. Get to the Testing Location on Time and Mentally Prepare Yourself – You do not want to get lost
on your way to the testing location or leave too late such that you miss the beginning of the exam or
even have to rush to get to your seat. You want to arrive in enough time to sit for 10 or 15 minutes
prior to the test to collect your thoughts and clear your mind. Make sure you have the address to
the testing location the day before the test, ensure you have the right directions or use a GPS
system and find out beforehand how much time it will take to get there so you know when to leave.

4. Read the Question and Understand What it is Asking – A cardinal rule of test taking is “Do not read
into the question and Answer only What is Asked.” Before you read the answers, make sure you
understand what the question is asking. Do not let yourself insert qualifications into the questions
or assume additional fact patterns.

5. Form an Answer in Your Mind before Reading the Answer Options – If an answer comes to you
before you read the answer options, and the answer that came to you matches an answer option,

TOP © 2015 - 2023 Tests.com


478
odds are that the answer option corresponding to the answer that popped into your head is the
correct answer. You know more than you realize. This is how preparation benefits you.

6. Read all of the Answers - Even though the first answer option looks right, read all of the answer
options all of the time. One of the answers is the correct choice. All the information to answer the
question is there. Read all the answer options to understand what options are available. You will
find, while one of the first top selections seems right some of the time, a bottom option will
occasionally be the right selection because it qualifies the answer in the correct way. If you just take
the first answer that seems right without reading the other answer options, you will not get the
benefit of all the information in the answer options.

7. Eliminate Obviously Wrong Answers – Some of the answer options will obviously be wrong. You
can increase the odds you will select the right answer and work more efficiently by first eliminating
obviously wrong answers.

8. Don’t get Stuck on Difficult Questions – Some questions will have difficult or complex fact patterns
that require some thought or calculation. If you find yourself getting lost in the facts or numbers, or
stuck on the answer options, such that you start feeling anxious that you are wasting time, take the
following steps: guess and register an answer, mark the question with some notation that will tell
you it was a guess, and come back to it at the end of the test, after you finished all other questions.

9. Pace Yourself - Don’t Work too Fast; Don’t Work too Slow – Time is a very important element of
test taking. Aside from the subject matter, it is the factor that most causes pressure and stress. To
obtain a good score, it is important that you have the time to read and answer all of the questions.
Tests only allow a certain amount of time per questions. Determine what that time per question is
by dividing the time by the number of questions. Pace yourself when taking the test so that you
allow yourself enough time to reasonably be able to read and answer all questions. You don’t want
to work too fast or too slow.

10. Maintain a Good Attitude during the Test – It is important to keep your composure during the test.
Having a good attitude will allow you to get through the challenging parts of an exam and avoid
becoming down or defeatist, which could slow you down or stop you altogether from finishing the
exam. Hang in there and have confidence. If you prepare for the exam following the preparation
and test taking tips discussed here, you can have confidence that you will succeed.

TOP © 2015 - 2023 Tests.com


479
Answer Bubble Sheet

TOP © 2015 - 2023 Tests.com


480
TOP © 2015 - 2023 Tests.com
481
TOP © 2015 - 2023 Tests.com
482
TOP © 2015 - 2023 Tests.com
483
TOP © 2015 - 2023 Tests.com
484
TOP © 2015 - 2023 Tests.com
485
TOP © 2015 - 2023 Tests.com
486
TOP © 2015 - 2023 Tests.com
487
TOP © 2015 - 2023 Tests.com
488
TOP © 2015 - 2023 Tests.com
489
Tests.com Format Comparison Chart
PDF
Features & Benefits Online Format
Format

New Questions / Updates YES. With the Online Format, get new question & updates in real time. NO
YES. Optionally test with instant prompts for incorrect answers &
Study Mode NO
explanations of correct ones.
Section / Topic Selection YES. Choose whole test or practice by the section. NO

YES. Optionally simulate actual test conditions. TestSIM™ is easy and flexible
NO
to use.

YES. With TestNOTES™, you have option to type notes for any question for
NO
instant reinforcement & later review.

YES. Our Focus Flash Cards™ online study system allows you to customize
NO
study by time, section(s) and more. Create/share your own too.

Focus Question Format YES. Focus on 1 question at a time. Example. NO


Customize Testing YES. Set the number of questions and time limit OR simulate your actual
NO
Preferences exam. Example.
Instant Scoring & Review YES. The Online format automatically scores your testing. NO
Reports / History / Stats YES. View results and track progress in real time. Example. NO
YES. Only the online test system format will provide you access your tests
Future Testing Needs NO
and test history from anywhere for life.
YES. Our practice exam kit was written by an expert with strong experience,
Expert Written by Leaders
education and credentials. Our test prep is so good we have a 1% return rate YES
in Test Preparation
compared to an 8% industry avg. 1 million+ tests served.
YES. Our materials are a personal assessment tool and study source
Get an Edge YES
which compliments other study materials and may give you an edge.
YES. Registered Users get service & support to pass the first time. Our
Support to Succeed YES
authors, peer reviewers, editors and tech support are on your team.
Convenience & YES. Only the Online Format system scores, reports and customizes your
NO
Study Features testing automatically and has bonus flash cards to use, create and share.

Login for the Online Format


Or Access Free Sample Questions Online

TOP © 2015 - 2023 Tests.com


490

You might also like